*NURSING > TEST BANK > Foundations of Maternal-Newborn and Women's Health Nursing, 7th Edition By: Sharon Murray, Emily McK (All)

Foundations of Maternal-Newborn and Women's Health Nursing, 7th Edition By: Sharon Murray, Emily McKinney

Document Content and Description Below

Foundations of Maternal-Newborn and Women's Health Nursing, 7th Edition By: Sharon Murray, Emily McKinney Foundations of Maternal-Newborn and Women's Health Nursing, 7th Edition By: Sharon ... Murray, Emily McKinneyFoundations of Maternal-Newborn and Women's Health Nursing, 7th Edition By: Sharon Murray, Emily McKinneyContents PART I: FOUNDATIONS FOR NURSING CARE OF THE CHILDBEARING FAMILIES 1. Maternity and Women’s Health Care Today 2. Social, Ethical and Legal Issues 3. Reproductive Anatomy and Physiology 4. Hereditary and Environmental Influences on Childbearing PART II: THE FAMILY BEFORE BIRTH 5. Conception and Prenatal Development 6. Maternal Adaptations to Pregnancy 7. Antepartum Assessment, Care and Education 8. Nutrition for Childbearing 9. Assessing the Fetus 10. Complications of Pregnancy 11. The Childbearing Family with Special Needs PART III: THE FAMILY DURING BIRTH 12. Processes of Birth 13. Pain management during Childbirth14. Intrapartum Fetal Surveillance 15. Nursing Care During Labor and Birth 16. Intrapartum Complications PART IV: THE FAMILY FOLLOWING BIRTH 17. Postpartum Adaptations and Nursing Care 18. Postpartum Maternal Complications 19. Normal Newborn: Processes of Adaptation 20. Assessment of the Normal Newborn 21. Care of the Normal Newborn 22. Infant Feeding 23. High-Risk Newborn: Complications Associated with Gestational Age and Development 24. High-Risk Newborn: Acquired and Congenital Conditions PART VI: WOMEN’S HEALTH CARE 25. Family Planning 26. Infertility 27. Women’s Health CareChapter 01: Maternity and Women‘s Health Care Today Foundations of Maternal-Newborn & Women‘s Health Nursing, 7th Edition MULTIPLE CHOICE 1. A nurse educator is teaching a group of nursing students about the history of familycentered maternity care. Which statement should the nurse include in the teaching session? a. The Sheppard-Towner Act of 1921 promoted family-centered care. b. Changes in pharmacologic management of labor prompted family-centered care. c. Demands by physicians for family involvement in childbirth increased the practice of familycentered care. d. Parental requests that infants be allowed to remain with them rather than in a nursery initiated the practice of family-centered care. ANS: D 2. Expectant parents ask a prenatal nurse educator, ―Which setting for childbirth limits the amount of parent–infant interaction?‖ Which answer should the nurse provide for these parents in order to assist them in choosing an appropriate birth setting? a. Birth center b. Home birth c. Traditional hospital birth d. Labor, birth, and recovery room ANS: C In the traditional hospital setting, the mother may see the infant for only short feeding periods, and the infant is cared for in a separate nursery. Birth centers are set up to allow an increase in parent–infant contact. Home births allow the greatest amount of parent–infant contact. The labor, birth, recovery, and postpartum room setting allows for increased parent–infant contact. DIF: Cognitive Level: Understanding OBJ: Nursing Process Step: Planning MSC: Patient Needs: Health Promotion and Maintenance 3. Which statement best describes the advantage of a labor, birth, recovery, and postpartum (LDRP) room? a. The family is in a familiar environment. b. They are less expensive than traditional hospital rooms. c. The infant is removed to the nursery to allow the mother to rest. d. The woman‘s support system is encouraged to stay until discharge. ANS: D Sleeping equipment is provided in a private room. A hospital setting is never a familiar environment to new parents. An LDRP room is not less expensive than a traditional hospital room. The baby remains with the mother at all times and is not removed to the nursery forroutine care or testing. The father or other designated members of the mother‘s support system are encouraged to stay at all times. DIF: Cognitive Level: Understanding OBJ: Nursing Process Step: Assessment MSC: Patient Needs: Health Promotion and Maintenance 4. Which nursing intervention is an independent function of the professional nurse? a. Administering oral analgesics b. Requesting diagnostic studies c. Teaching the patient perineal care d. Providing wound care to a surgical incision ANS: C Nurses are now responsible for various independent functions, including teaching, counseling, and intervening in nonmedical problems. Interventions initiated by the physician and carried out by the nurse are called dependent functions. Administrating oral analgesics is a dependent function; it is initiated by a physician and carried out by a nurse. Requesting diagnostic studies is a dependent function. Providing wound care is a dependent function; however, the physician prescribes the type of wound care through direct orders or protocol. DIF: Cognitive Level: Understanding OBJ: Nursing Process Step: Assessment MSC: Patient Needs: Safe and Effective Care Environment 5. Which response by the nurse is the most therapeutic when the patient states, ―I’m so afraid to have a cesarean birth‖? a. ―Everything will be OK.‖ b. ―Don‘t worry about it. It will be over soon.‖ c. ―What concerns you most about a cesarean birth?‖ d. ―The physician will be in later and you can talk to him.‖ ANS: C The response, ―What concerns you most about a cesarean birth‖ focuses on what the patient is saying and asks for clarification, which is the most therapeutic response. The response, ―Everything will be ok‖ is belittling the patient‘s feelings. The response, ―Don‘t worry about it. It will be over soon‖ will indicate that the patient‘s feelings are not important. The response, ―The physician will be in later and you can talk to him‖ does not allow the patient to verbalize her feelings when she wishes to do that. DIF: Cognitive Level: Application OBJ: Nursing Process Step: Implementation MSC: Patient Needs: Psychosocial Integrity 6. In which step of the nursing process does the nurse determine the appropriate interventions for the identified nursing diagnosis? a. Planning b. Evaluation c. Assessment d. InterventionANS: A The third step in the nursing process involves planning care for problems that were identified during assessment. The evaluation phase is determining whether the goals have been met. During the assessment phase, data are collected. The intervention phase is when the plan of care is carried out. DIF: Cognitive Level: Understanding OBJ: Nursing Process Step: Planning MSC: Patient Needs: Safe and Effective Care Environment 7. Which goal is most appropriate for the collaborative problem of wound infection? a. The patient will not exhibit further signs of infection. b. Maintain the patient‘s fluid intake at 1000 mL/8 hour. c. The patient will have a temperature of 98.6F within 2 days. d. Monitor the patient to detect therapeutic response to antibiotic therapy. ANS: D In a collaborative problem, the goal should be nurse-oriented and reflect the nursing interventions of monitoring or observing. Monitoring for complications such as further signs of infection is an independent nursing role. Intake and output is an independent nursing role. Monitoring a patient‘s temperature is an independent nursing role. DIF: Cognitive Level: Application OBJ: Nursing Process Step: Planning MSC: Patient Needs: Safe and Effective Care Environment 8. Which nursing intervention is written correctly? a. Force fluids as necessary. b. Observe interaction with the infant. c. Encourage turning, coughing, and deep breathing. d. Assist to ambulate for 10 minutes at 8 AM, 2 PM, and 6 PM. ANS: D Interventions might not be carried out if they are not detailed and specific. ―Force fluids‖ is not specific; it does not state how much or how often. Encouraging the patient to turn, cough, and breathe deeply is not detailed or specific. Observing interaction with the infant does not state how often this procedure should be done. Assisting the patient to ambulate for 10 minutes within a certain timeframe is specific. DIF: Cognitive Level: Application OBJ: Nursing Process Step: Planning MSC: Patient Needs: Safe and Effective Care Environment 9. The patient makes the statement: ―I’m afraid to take the baby home tomorrow.‖ Which response by the nurse would be the most therapeutic? a. ―You‘re afraid to take the baby home?‖ b. ―Don‘t you have a mother who can come and help?‖ c. ―You should read the literature I gave you before you leave.‖ d. ―I was scared when I took my first baby home, but everything worked out.‖ ANS: AThis response uses reflection to show concern and open communication. The other choices are blocks to communication. Asking if the patient has a mother who can come and assist blocks further communication with the patient. Telling the patient to read the literature before leaving does not allow the patient to express her feelings further. Sharing your own birth experience is inappropriate. DIF: Cognitive Level: Application OBJ: Nursing Process Step: Implementation MSC: Patient Needs: Psychosocial Integrity 10. The nurse is writing an expected outcome for the nursing diagnosis—acute pain related to tissue trauma, secondary to vaginal birth, as evidenced by patient stating pain of 8 on a scale of 10. Which expected outcome is correctly stated for this problem? a. Patient will state that pain is a 2 on a scale of 10. b. Patient will have a reduction in pain after administration of the prescribed analgesic. c. Patient will state an absence of pain 1 hour after administration of the prescribed analgesic. d. Patient will state that pain is a 2 on a scale of 10, 1 hour after the administration of the prescribed analgesic. ANS: D The outcome should be patient-centered, measurable, realistic, and attainable and within a specified timeframe. Patient stating that her pain is now 2 on a scale of 10 lacks a timeframe. Patient having a reduction in pain after administration of the prescribed analgesic lacks a measurement. Patient stating an absence of pain 1 hour after the administration of prescribed analgesic is unrealistic. DIF: Cognitive Level: Application OBJ: Nursing Process Step: Planning MSC: Patient Needs: Physiologic Integrity 11. Which nursing diagnosis should the nurse identify as a priority for a patient in active labor? a. Risk for anxiety related to upcoming birth b. Risk for imbalanced nutrition related to NPO status c. Risk for altered family processes related to new addition to the family d. Risk for injury (maternal) related to altered sensations and positional or physical changes ANS: D The nurse should determine which problem needs immediate attention. Risk for injury is the problem that has the priority at this time because it is a safety problem. Risk for anxiety, imbalanced nutrition, and altered family processes are not the priorities at this time. DIF: Cognitive Level: Application OBJ: Nursing Process Step: Implementation MSC: Patient Needs: Safe and Effective Care Environment 12. Regarding advanced roles of nursing, which statement related to clinical practice is the most accurate? a. Family nurse practitioners (FNPs) can assist with childbirth care in the hospital setting. b. Clinical nurse specialists (CNSs) provide primary care to obstetric patients. c. Neonatal nurse practitioners provide emergency care in the postbirth setting tohigh-risk infants. d. A certified nurse midwife (CNM) is not considered to be an advanced practice nurse. ANS: C Neonatal NPs provide care for the high-risk neonate in the birth room and in the neonatal intensive care unit, as needed. FNPs do not participate in childbirth care; however, they can take care of uncomplicated pregnancies and postbirth care outside of the hospital setting. CNSs work in hospital settings but do not provide primary care services to patients. A CNM is an advanced practice nurse who receives additional certification in the specific area of midwifery. DIF: Cognitive Level: Application OBJ: Nursing Process Step: Evaluation MSC: Patient Needs: Management of Care: Legal Rights and Responsibilities 13. Which statement is true regarding the shortage of nurses in the United States? a. There are a larger proportion of younger nurses in the workforce as compared with older nurses. b. As a result of decreased RN-to-patient ratios, there is a decrease in patient mortality in the clinical setting. c. Nursing programs are turning away qualified applicants. d. There are adequate classroom and clinical facilities for training RNs. ANS: C According to an Institute of Medicine (IOM) report, by the year 2020, 80% of new RNs should hold baccalaureate degrees. Despite this need, baccalaureate and master‘s programs are turning away qualified applicants due to an insufficient number of faculty. There are a larger proportion of older nurses in the workforce based on current research by the IOM. Increased nurse-to-patient ratios have resulted in decreased patient mortality in the clinical setting. There are currently numerous limitations of both classroom and clinical facilities necessary to train new nurses adequately. DIF: Cognitive Level: Application OBJ: Nursing Process Step: Implementation MSC: Patient Needs: Health Promotion: Teaching/Learning 14. A hospital has achieved Magnet status. Which indicators would be consistent with this type of certification? a. There is stratification of communication in a directed manner between nursing staff and administration. b. There is increased job satisfaction of nurses, with a lower staff turnover rate. c. Physicians are certified in their respective specialty areas. d. All nurses have baccalaureate degrees and certification in their clinical specialty area. ANS: B Magnet status is a certification offered by the ANCC (American Nurses Credentialing Center) in which hospitals apply based on designated criteria that consider nurse job satisfaction, staff patterns, strength, quality of nursing staff, and open communication. It is not based on physician status. Also, certification is not required for all nurses at this point. The expectation with Magnet status is that nurses will continue to expand their knowledge by earning additional degrees and certification. DIF: Cognitive Level: Application OBJ: Nursing Process Step: AssessmentMSC: Patient Needs: Health Promotion: Teaching/Learning 15. Which of the following statements highlights the nurse’s role as a researcher? a. Reading peer-reviewed journal articles b. Working as a member of the interdisciplinary team to provide patient care c. Helping patient to obtain home care postdischarge from the hospital d. Delegating tasks to unlicensed personnel to allow for more teaching time with patients ANS: A A nurse in a researcher role should look to improve her or his knowledge base by reading and reviewing evidence-based practice information as found in peer-reviewed journals. Working as a member of the interdisciplinary team to provide patient care indicates that the nurse is working as a collaborator. Helping the patient to obtain home care postdischarge from the hospital indicates that the nurse is working as a patient advocate. Delegating tasks to unlicensed personnel in order to allow for more teaching time with patients indicates that the nurse is working as a manager. DIF: Cognitive Level: Application OBJ: Nursing Process Step: Assessment MSC: Patient Needs: Health Promotion: Teaching/Learning 16. Which patient could safely be cared for by a certified nurse-midwife? a. Gravida 3, para 2, with no complications b. Gravida 1, para 0, with mild hypertension c. Gravida 2, para 1, with insulin-dependent diabetes d. Gravida 1, para 0, with borderline pelvic measurements ANS: A A certified nurse-midwife (CNM) cares for women who are at low risk for complications. The CNM would not care for a woman with hypertension. The CNM would not care for a woman with insulin-dependent diabetes. The CNM would not care for a woman with borderline pelvic measurements. DIF: Cognitive Level: Analysis OBJ: Nursing Process Step: Evaluation MSC: Patient Needs: Health Promotion and Maintenance 17. A primipara patient asks about possible support options for her during the labor process. She is apprehensive that her family members will not be prepared to assist her during this time. Which option would be most effective for this patient? a. Reassure the patient that the labor and birth staff consists of highly trained nurses who are well educated to take care of laboring patients so that should be sufficient. b. Encourage the patient to take prepared childbirth classes with her husband because that should provide the best support by a family member. c. Provide information to the patient about obtaining a doula during the labor process. d. Tell the patient that this is a normal feeling based on fear of the unknown and that it will subside once she starts the labor process. ANS: CProviding information about a doula addresses the patient‘s concern because the doula‘s designated role is to provide support during labor. Although it is true that labor and birth nurses are trained in their specialty, the patient is voicing concern for support so her feelings should not be minimized. Encouraging the patient to take prepared childbirth classes is also important; however, it does not address the patient‘s concern for support. Because this patient is a primipara, it is normal to have some anxiety over the unknown process of the labor experience but again this response minimizes the patient‘s concern. DIF: Cognitive Level: Analysis OBJ: Nursing Process Step: Planning MSC: Patient Needs: Psychologic Integrity 18. The nurse states to the newly pregnant patient, ―Tell me how you feel about being pregnant.‖ Which communication technique is the nurse using with this patient? a. Clarifying b. Paraphrasing c. Reflection d. Structuring ANS: A The nurse is attempting to follow up and check the accuracy of the patient‘s message. Paraphrasing is restating words other than those used by the patient. Reflection is verbalizing comprehension of what the patient has said. Structuring takes place when the nurse has set guidelines or set priorities. DIF: Cognitive Level: Understanding OBJ: Nursing Process Step: Analysis MSC: Patient Needs: Health Promotion and Maintenance 19. When reviewing a new patient’s birth plan, the nurse notices that the patient will be bringing a doula to the hospital during labor. What does the nurse think that this means? a. The patient will have her grandmother as a support person. b. The patient will bring a paid, trained labor support person with her during labor. c. The patient will have a special video she will play during labor to assist with relaxation. d. The patient will have a bag that contains all the approved equipment that may help with the labor process. ANS: B A doula is a trained labor support person who is employed by the mother to provide labor support. She gives physical support such as massage, helps with relaxation, and provides emotional support and advocacy throughout labor. A doula is usually not a relative of the woman. A doula is a trained labor support person. DIF: Cognitive Level: Understanding OBJ: Nursing Process Step: Assessment MSC: Patient Needs: Health Promotion and Maintenance MULTIPLE RESPONSE 1. In consideration of the historic evolution of maternity care, which treatment options were used over the past century? (Select all that apply.) a. During the nineteenth century, women of privilege were delivered by midwives in a hospital setting. b.Granny midwives received their training through a period of apprenticeship. c. The recognition of improved obstetric outcomes was related to increased usage of hygienic practices. d. A shift to hospital-based births occurred as a result of medical equipment designed to facilitate birth. e. The use of chloroform by midwives led to decreased pain during birth. ANS: B, C, D Training of granny midwives was done by apprenticeship as opposed to formal medical school training. With the advent of usage of hygienic practices, improved health outcomes were seen with regard to a decrease in sepsis. New equipment such as forceps enabled easier birth. Women of privilege in the nineteenth century delivered at home, attended by a midwife. Chloroform was used by physicians and was not available to midwives. DIF: Cognitive Level: Analysis OBJ: Nursing Process Step: Implementation MSC: Patient Needs: Health Promotion and Maintenance 2. Many communities now offer the availability of free-standing birth centers to provide care for low-risk women during pregnancy, birth, and postpartum. When counseling the newly pregnant patient regarding this option, the nurse should be aware that this type of care setting includes which advantages? (Select all that apply.) a. Staffing by lay midwives b. Equipped for obstetric emergencies c. Less expensive than acute care hospitals d. Safe, homelike births in a familiar setting e. Access to follow-up care for 6 weeks postpartum ANS: C, D, E Patients who are at low risk and desire a safe, homelike birth are very satisfied with this type of care setting. The new mother may return to the birth center for postpartum follow-up care, breastfeeding assistance, and family planning information for 6 weeks postpartum. Because birth centers do not incorporate advanced technologies into their services, costs are significantly less than in a hospital setting. The major disadvantage of this care setting is that these facilities are not equipped to handle obstetric emergencies. Should unforeseen difficulties occur, the patient must be transported by ambulance to the nearest hospital. Birth centers are usually staffed by certified nurse-midwives (CNMs). DIF: Cognitive Level: Understanding OBJ: Nursing Process Step: Planning MSC: Patient Needs: Safe and Effective Care Environment 3. The nurse is assessing a patient’s use of complementary and alternative therapies. Which should the nurse document as an alternative or complementary therapy practice? (Select all that apply.) a. Practicing yoga daily b. Drinking green tea in the morning c. Taking omeprazole (Prilosec) once a dayd. Using aromatherapy during a relaxing bath e. Wearing a lower back brace when lifting heavy objects ANS: A, B, D Complementary and alternative (CAM) therapies can be defined as those systems, practices, interventions, modalities, professions, therapies, applications, theories, and claims that are currently not an integral part of the conventional medical system in North America. Yoga is considered to be a mind–body alternative therapy. Green tea and aromatherapy are biologically based complementary therapies. Prilosec and the use of a lower back brace would be therapies consistent with those used by conventional medicine. DIF: Cognitive Level: Analysis OBJ: Nursing Process Step: Assessment MSC: Patient Needs: Health Promotion and Maintenance 4. The nurse is formulating a nursing care plan for a postpartum patient. Which actions by the nurse indicate use of critical thinking skills when formulating the care plan? (Select all that apply.) a. Using a standardized postpartum care plan b. Determining priorities for each diagnosis written c. Writing interventions from a nursing diagnosis book d. Reflecting and suspending judgment when writing the care plan e. Clustering data during the assessment process according to normal versus abnormal ANS: B, D, E Critical thinking focuses on appraisal of the way the individual thinks, and it emphasizes reflective skepticism. Determining priorities, reflecting and suspending judgment, and clustering data are actions that indicate the use of critical thinking. Using a standardized care plan and writing interventions from a nursing diagnosis book do not show that reflection about the patient‘s individual care is being done. DIF: Cognitive Level: Application OBJ: Nursing Process Step: Planning MSC: Patient Needs: Physiologic Integrity 5. The RN is delegating tasks to the unlicensed assistive personnel (UAP). Which tasks can the nurse delegate? (Select all that apply.) a. Teaching the patient about breast care b. Assessment of a patient‘s lochia and perineal area c. Assisting a patient to the bathroom for the first time after birth d. Vital signs on a postpartum patient who delivered the night before e. Assisting a postpartum patient to take a shower on the second postpartum day ANS: D, E Nurses must be aware that they remain legally responsible for patient assessments and must make the critical judgments necessary to ensure patient safety when delegating tasks to unlicensed personnel. The nurse cannot delegate assessment, teaching, or evaluation. The twotasks that the nurse can delegate are vital signs on a stable postpartum patient and assisting a stable postpartum patient on the second postpartum day to take a shower. DIF: Cognitive Level: Application OBJ: Nursing Process Step: Implementation MSC: Patient Needs: Safe and Effective Care Environment Chapter 02: Social, Ethical, and Legal Issues Foundations of Maternal-Newborn & Women‘s Health Nursing, 7th Edition MULTIPLE CHOICE 1. During which phase of the cycle of violence does the batterer become contrite and remorseful? a. Battering b. Honeymoon c. Tension-building d. Increased drug taking ANS: B During the honeymoon phase, the battered person wants to believe that the battering will never happen again, and the batterer will promise anything to get back into the home. During the battering phase, violence actually occurs, and the victim feels powerless. During the tensionbuilding phase, the batterer becomes increasingly hostile, swears, threatens, throws things, and pushes the battered person. Often, the batterer increases the use of drugs during the tensionbuilding phase. DIF: Cognitive Level: Understanding OBJ: Nursing Process Step: Assessment MSC: Patient Needs: Psychosocial Integrity 2. The United States ranks poorly in terms of worldwide infant mortality rates. Which factor has the greatest impact on decreasing the mortality rate of infants? a. Providing more women‘s shelters b. Ensuring early and adequate prenatal care c. Resolving all language and cultural differences d. Enrolling pregnant women in the Medicaid program by their eighth month of pregnancy ANS: B Because preterm infants form the largest category of those needing expensive intensive care, early pregnancy intervention is essential for decreasing infant mortality. The women in shelters have the same difficulties in obtaining health care as other poor people, particularly lack of transportation and inconvenient clinic hours. Language and cultural differences are not infant mortality issues but must be addressed to improve overall health care. Medicaid provides health care for poor pregnant women, but the process may take weeks to take effect. The eighth month is too late to apply and receive benefits for this pregnancy. DIF: Cognitive Level: Understanding OBJ: Nursing Process Step: Assessment MSC: Patient Needs: Health Promotion and Maintenance3. The nurse is planning a teaching session for staff on ethical theories. Which situation best reflects the Deontologic theory? a. Approving a physician-assisted suicide b. Supporting the transplantation of fetal tissue and organs c. Using experimental medications for the treatment of AIDS d. Initiating resuscitative measures on a 90-year-old patient with terminal cancer ANS: D In the Deontologic theory, life must be maintained at all costs, regardless of quality of life. Approving a physician-assisted suicide, supporting the transplantation of fetal tissue and organs, and using experimental medications for the treatment of AIDS are examples of a utilitarian model. DIF: Cognitive Level: Application OBJ: Nursing Process Step: Planning MSC: Patient Needs: Psychosocial Integrity 4. Which step of the nursing process is being used when the nurse decides whether an ethical dilemma exists? a. Analysis b. Planning c. Evaluation d. Assessment ANS: A When a nurse uses the collected data to determine whether an ethical dilemma exists, the data are being analyzed. Planning is done after the data have been analyzed. Evaluation occurs once the outcome has been achieved. Assessment is the data collection phase. DIF: Cognitive Level: Understanding OBJ: Nursing Process Step: Evaluation MSC: Patient Needs: Safe and Effective Care Environment: Coordinated Care 5. The nurse is interviewing a patient who is 6-weeks pregnant. The patient asks the nurse, ―Why is elective abortion considered such an ethical issue?‖ Which response by the nurse is most appropriate? a. Abortion requires third-party consent. b. The U.S. Supreme Court ruled that life begins at conception. c. Abortion law is unclear about a woman‘s constitutional rights. d. There is a conflict between the rights of the woman and the rights of the fetus. ANS: D Elective abortion is an ethical dilemma because two opposing courses of action are available. Abortion does not require third-party consent. The Supreme Court has not ruled on when life begins. Abortion laws are clear concerning a woman‘s constitutional rights. DIF: Cognitive Level: Application OBJ: Nursing Process Step: Implementation MSC: Patient Needs: Safe and Effective Care Environment 6. At the present time, which agency governs surrogate parenting?a. State law b. Federal law c. Individual court decision d. Protective child services ANS: C Each surrogacy case is decided individually in a court of law. Surrogate parenting is not governed by either state or federal law. Protective child services do not make decisions related to surrogacy. DIF: Cognitive Level: Understanding OBJ: Nursing Process Step: Assessment MSC: Patient Needs: Health Promotion and Maintenance 7. Which patient will most likely seek prenatal care? a. A 15-year-old patient who tells her friends, ―I just don‘t believe that I am pregnant‖ b. A 28-year-old who is in her second pregnancy and abuses drugs and alcohol c. A 20-year-old who is in her first pregnancy and has access to a free prenatal clinic d. A 30-year-old who is in her fifth pregnancy and delivered her last infant at home with the help of her mother and sister ANS: C The patient who acknowledges the pregnancy early, has access to health care, and has no reason to avoid health care is most likely to seek prenatal care. Being in denial regarding the pregnancy will prevent a patient from seeking health care. Patients who abuse substances are less likely to seek health care. Some women see pregnancy and birth as a natural occurrence and do not seek health care. DIF: Cognitive Level: Understanding OBJ: Nursing Process Step: Assessment MSC: Patient Needs: Health Promotion and Maintenance 8. A medical-surgical nurse is asked to float to a women‘s health unit to care for patients who are scheduled for therapeutic abortions. The nurse refuses to accept this assignment and expresses her personal beliefs as being incongruent with this medical practice. The nursing supervisor states that the unit is short-staffed and the nurse is familiar with caring for postoperative patients. In consideration of legal and ethical practices, can the nursing supervisor enforce this assignment? a. The staff nurse has the responsibility of accepting any assignment that is made while working for a health care unit, so the nursing supervisor is within his or her rights to enforce this assignment. b. Because the unit is short-staffed, the staff nurse should accept the assignment to provide care by benefit of her or his experience to patients who need care. c. The staff nurse has expressed a legitimate concern based on his or her feelings; the nursing supervisor does not have the authority to enforce this assignment. d.The nursing supervisor should emphasize that this assignment requires care of a surgical patient for which the staff nurse is adequately trained and should therefore enforce the assignment. ANS: C The Nurse Practice Act allows nurses to refuse assignments that involve practices that they have expressed as being opposed to their religious, cultural, ethical, and/or moral values. Although the nursing supervisor has a right to arrange assignments, the supervisor, if made aware of a potential bias or limitation, must act accordingly and accept the nurse‘s position. This should be upheld regardless of staffing limitations and independent of persuasive efforts to make the nurse feel guilty for her or his stated beliefs. DIF: Cognitive Level: Analysis OBJ: Nursing Process Step: Implementation MSC: Patient Needs: Safe Effective Care: Ethical Practice/Assignment, Delegation and Supervision 9. With regard to an obstetric litigation case, a nurse working in labor and birth is found to be negligent. Which intervention performed by the nurse indicates that a breach of duty has occurred? a. The nurse did not document fetal heart tones (FHR) during the second stage of labor. b. The patient was only provided ice chips during the labor period, which lasted 8 hours. c. The nurse allowed the patient to use the bathroom rather than a bedpan during the first stage of labor. d. The nurse asked family members to leave the room when she prepared to do a pelvic exam on the patient. ANS: A A breach of duty has occurred when a nurse or health care provider fails to provide treatment relative to the standard of care. In this case, documentation of FHR during the second stage of labor is a recognized standard of care. Providing ice chips to laboring patients is within the standard of care. The time period of 8 hours is not excessive. A patient without any risk factors can use the bathroom and be ambulatory during the first stage of labor. Asking family members to leave during a vaginal exam helps maintain patient privacy. DIF: Cognitive Level: Analysis OBJ: Nursing Process Step: Implementation MSC: Patient Needs: Safe Effective Care: Legal Rights and Responsibilities 10. A nurse is working with an active labor patient who is in preterm labor and has been designated as high risk. The patient is very apprehensive and asks the nurse, ―Is everything going to be all right?‖ The nurse replies, ―Yes, everything will be okay.‖ Following delivery via an emergency cesarean birth, the newborn undergoes resuscitation and does not survive. The patient is distraught over the outcome and blames the nurse for telling her that everything would be okay. Which ethical principle did the nurse violate? a. Autonomy b. Fidelity c. Beneficence d. Accountability ANS: BIn this type of situation, the nurse (and/or health care provider) cannot make statements or promises that cannot be kept. Telling the patient that everything will be okay is not based on the accuracy of medical diagnosis and should not be conveyed to the patient. The other ethical principles of autonomy (self-determination), beneficence (greatest good), and accountability (accepting responsibility) do not apply in this situation. DIF: Cognitive Level: Analysis OBJ: Nursing Process Step: Implementation MSC: Patient Needs: Safe Effective Care: Legal Rights and Responsibilities 11. A nurse is working in the area of labor and birth. Her assignment is to take care of a gravida 1 para 0 woman who presents in early labor at term. Vaginal exam reflects the following: 2 cm, cervix posterior, –1 station, and vertex with membranes intact. The patient asks the nurse if she can break her water so that her labor can go faster. The nurse‘s response, based on the ethical principle of nonmaleficence, is which of the following? a. Tell the patient that she will have to wait until she has progressed further on the vaginal exam and then she will perform an amniotomy. b. Have the patient write down her request and then call the physician for an order to implement the amniotomy. c. Instruct the patient that only a physician or certified midwife can perform this procedure. d. Give the patient an enema to stimulate labor. ANS: C The ethical principle of nonmaleficence conveys the concept that one should avoid risk taking or harm to others. The procedure of amniotomy is performed by a physician and/or certified nurse midwife. It is not in the scope of practice of an RN, so option C validates that the nurse is upholding this ethical principle. Options A and B are not within the scope of practice. The use of an enema as a labor stimulant is no longer considered necessary during labor. DIF: Cognitive Level: Analysis OBJ: Nursing Process Step: Implementation MSC: Patient Needs: Safe Effective Care: Legal Rights and Responsibilities 12. A nurse working in a labor and birth unit is asked to take care of two high-risk patients in the labor and birth suite: a 34 weeks‘ gestation 28-year-old gravida 3, para 2 in preterm labor and a 40-year-old gravida 1, para 0 who is severely preeclamptic. The nurse refuses this assignment telling the charge nurse that based on individual patient acuity, each patient should have oneon-one care. Which ethical principle is the nurse advocating? a. Accountability b. Beneficence c. Justice d. Fidelity ANS: B In this situation, the patients are each exhibiting significant high-risk conditions and should receive individual nursing care. The nurse is advocating the principle of beneficence in that she is trying to do the ―greatest good or the least harm‖ to improve patient outcomes. The other ethical principles do not apply in this situation. DIF: Cognitive Level: Analysis OBJ: Nursing Process Step: Implementation MSC: Patient Needs: Safe Effective Care: Legal Rights and Responsibilities13. A charge nurse is working on a postpartum unit and discovers that one of the patients did not receive AM care during her shift assessment. The charge nurse questions the nurse assigned to provide care and finds out that the nurse thought ―the patient should just do it by herself because she will have to do this at home.‖ On further questioning of the nurse, it is determined that the rest of her assigned patients were provided AM care. The assigned nurse has violated which ethical principle? a. Justice b. Truth c. Confidentiality d. Autonomy ANS: A The ethical principle of justice indicates that all patients should be treated equally and fairly. In this case, the charge nurse ascertained that the AM care was not equally applied to all the nurse‘s assigned patients. The other ethical principles do not apply to this situation. DIF: Cognitive Level: Analysis OBJ: Nursing Process Step: Implementation MSC: Patient Needs: Safe Effective Care: Legal Rights and Responsibilities 14. A nurse is entering information on the patient‘s electronic health record (EHR) and is called to assist in an emergency situation with regard to another patient in the labor and birth suite. The nurse rushes to the scene to assist; however, she leaves the chart open on the computer screen. The emergent patient situation is resolved satisfactorily, and the nurse comes back to the computer entry screen to complete charting. At the end of the shift, the nurse manager asks to speak with the nurse and tells her that she is concerned with what happened today on the unit because there was a breach in confidentiality. Which response by the nurse indicates that she understands the nurse manager‘s concerns? a. The nurse acknowledges that she should have made sure that her patient was safe before assisting with the emergency. b. The nurse states that she should have logged out of the EHR prior to attending to the emergency. c. The nurse indicates that the unit was understaffed. d. The nurse indicates that the she changed her password following the clinical emergency to maintain confidentiality. ANS: B With the use of electronic health records, it is necessary to take all steps to maintain confidentiality and limit access to nonhealth care personnel. In an emergent care situation, the nurse should have logged out of the system to maintain confidentiality. Although it is important to make sure that one‘s patient is safe, there is no information here to suggest that there were any safety issues applicable to her assigned patient. The staffing of the unit should not affect confidentiality. Changing the password for logging in to a system is an option for clinical practice but does not affect the situation as described. DIF: Cognitive Level: Analysis OBJ: Nursing Process Step: Implementation MSC: Patient Needs: Safe Effective Care: Legal Rights and Responsibilities 15. A nurse is admitting a patient to the labor and birth unit in early labor that was sent to the facility following a checkup with her health care provider in the office. The patient is a gravida 1,para 0, and is at term. No health issues are discerned from the initial assessment, and the nurse prepares to initiate physician orders based on standard procedures. Which action by the nurse manager is warranted in this situation? a. No action is indicated because the nurse is acting within the scope of practice. b. The nurse manager should intervene and ask the nurse to clarify admission orders directly with the physician. c. The nurse manager should review standard procedures with the nurse to validate that orders are being carried out accurately. d. The nurse manger should review the admission procedure with the nurse. ANS: A Standard procedures are often used in labor and birth settings because they are based on physician-directed orders that apply to general admissions. The nurse is acting appropriately since the patient was sent directly to the unit, by the health care provider. The nurse manager does not have to intervene at this point. There is no additional need to review standard procedures or the admission process with the nurse at this time. There is no evidence that the nurse needs additional training and/or does not have the prerequisite knowledge to admit the patient. DIF: Cognitive Level: Application OBJ: Nursing Process Step: Implementation MSC: Patient Needs: Health Promotion and Maintenance 16. A nurse who works in the emergency department (ED) is assigned to a patient who is experiencing heavy vaginal bleeding at 12 weeks‘ gestation. An ultrasound has confirmed the absence of a fetal heart rate, and the patient is scheduled for a dilation and evacuation of the pregnancy. The nurse refuses to provide any further care for this patient based on moral principles. What is the nurse manager‘s initial response to the nurse? a. ―I recall you sharing that information in your interview. I will arrange for another nurse to take report on this patient.‖ b. ―Because we are shorthanded today, you have to continue to provide care. There is no one else available to provide care for this patient.‖ c. ―I understand your point of view. You were hired to work here in the ED so you had to know this situation was possible.‖ d. ―Abandonment is a serious issue. I have to advise you to continue to provide care for this patient.‖ ANS: A Nurses do not have to provide care if the care is in violation of their moral, ethical, or religious principles. It is the responsibility of the nurse to share these views at the time of the initial interview. Disclosing beliefs that would affect the care of patients at the point of care and refusing to provide care is unethical on behalf of the nurse. The manager cannot force the nurse to provide care if the nurse‘s principles were shared at the time of the initial interview. It is the manager‘s responsibility to disclose the type of care delivered in the department at the time of the interview. Threats of abandonment are unwarranted at this time. DIF: Cognitive Level: Application OBJ: Nursing Process Step: Implementation MSC: Patient Needs: Health Promotion and Maintenance17. The nurse is providing care to a patient who was just admitted to the labor and birth unit in active labor at term. The patient informed the nurse, ―I have not received any prenatal care because I cannot afford to go to the doctor. And, this is my third baby, so I know what to expect.‖ What is the nurse‘s primary concern when developing the patient‘s plan of care? a. Low birth weight b. Oligohydramnios c. Gestational diabetes d. Gestational hypertension ANS: A Due to adverse living conditions, poor health care, and inadequate nutrition, infants born to lowincome women are more likely to begin life with problems such as low birth weight. Oligohydramnios is a condition where there is too little amniotic fluid and is not directly correlated with poverty. While gestational diabetes and gestational hypertension are associated with poverty, they can be seen during any pregnancy. This patient is in active labor and the primary concern at this time is the fetus. DIF: Cognitive Level: Application OBJ: Nursing Process Step: Planning MSC: Patient Needs: Health Promotion and Maintenance 18. A nurse is reviewing evidence-based teaching and learning principles. Which situation is most conducive to learning with patients of other cultures? a. An auditorium is being used as a classroom for 300 students. b. A teacher who speaks very little Spanish is teaching a class of Hispanic students. c. A class is composed of students of various ages and educational backgrounds. d. An Asian nurse provides nutritional information to a group of pregnant Asian women. ANS: D A patient‘s culture influences the learning process; thus a situation that is most conducive to learning is one in which the teacher has knowledge and understanding of the patient‘s cultural beliefs. A large class is not conducive to learning. It does not allow questions, and the teacher cannot see nonverbal cues from the students to ensure understanding. The ability to understand the language in which teaching is done determines how much the patient learns. Patients for whom English is not their primary language may not understand idioms, nuances, slang terms, informal usage of words, or medical terms. The teacher should be fluent in the language of the student. Developmental levels and educational levels influence how a person learns best. For the teacher to present the information in the best way, the class should be at the same level. DIF: Cognitive Level: Application OBJ: Nursing Process Step: Planning MSC: Patient Needs: Psychosocial Integrity 19. The nurse is teaching a parenting class to new parents. Which statement should the nurse include in the teaching session about the characteristics of a healthy family? a. Adults agree on the majority of basic parenting principles. b. The parents and children have rigid assignments for all the family tasks. c. Young families assume total responsibility for the parenting tasks, refusing any assistance.d. The family is overwhelmed by the significant changes that occur as a result of childbirth. ANS: A Adults in a healthy family communicate with each other, so there is minimal discord in areas such as discipline and sleep schedules. Healthy families remain flexible in their role assignments. Members of a healthy family accept assistance without feeling guilty. Healthy families can tolerate irregular sleep and meal schedules, which are common during the months after childbirth. DIF: Cognitive Level: Application OBJ: Nursing Process Step: Implementation MSC: Patient Needs: Health Promotion and Maintenance 20. A patient who is 6 months pregnant has sought medical attention, saying she fell down the stairs. Which scenario would cause an emergency department nurse to suspect that the woman has been battered? a. She avoids making eye contact and is hesitant to answer questions. b. The woman and her partner are having an argument that is loud and hostile. c. The woman has injuries on various parts of her body that are in different stages of healing. d. Examination reveals a fractured arm and fresh bruises. Her husband asks her about her pain. ANS: C The battered woman often has multiple injuries in various stages of healing. It is more normal for the woman to have a flat affect. A loud and hostile argument is not always an indication of battering. Often the batterer will be attentive and refuse to leave the woman‘s bedside. DIF: Cognitive Level: Analysis OBJ: Nursing Process Step: Assessment MSC: Patient Needs: Psychosocial Integrity 21. Which situation is most representative of an extended family? a. It includes adoptive children. b. It is headed by a single-parent. c. It contains children from previous marriages. d. It is composed of children, parents, and grandparents living in the same house. ANS: D An extended family is defined as a family having members from three generations living under the same roof. A family with adoptive children is a nuclear family. A single-parent family is headed by a single parent. A blended family is one that contains children from previous marriages. DIF: Cognitive Level: Understanding OBJ: Nursing Process Step: Assessment MSC: Patient Needs: Health Promotion and Maintenance 22. The nurse is reviewing the principles of family-centered care with a primiparous patient. Which patient statement will the nurse need to correct? a. ―Remaining focused on my family will help benefit me and my baby.‖ b. ―Most of the time, childbirth is uncomplicated and a healthy event for the family.‖ c. ―Because childbirth is normal, after my baby‘s birth our family dynamics will not change.‖d. ―With correct information, I am able to make decisions regarding my health care while I am pregnant.‖ ANS: C The birth of an infant alters family relationships and structures; family dynamics will change with the birth of an infant. Childbirth is usually a normal and healthy event. Given professional support and guidance, the pregnant woman is able to make decisions about her prenatal care. Maintaining a focus on family or other support can benefit a woman as she seeks to maintain her health throughout pregnancy. DIF: Cognitive Level: Application OBJ: Nursing Process Step: Evaluation MSC: Patient Needs: Health Promotion and Maintenance 23. Which issue is a major concern among members of lower socioeconomic groups? a. Practicing preventive health care b. Meeting health needs as they occur c. Maintaining an optimistic view of life d. Maintaining group health insurance for their families ANS: B Because of their economic uncertainty, lower socioeconomic groups place more emphasis on meeting the needs of the present rather than on future goals. Lower socioeconomic groups may value health care but generally cannot afford preventive health care. They may struggle for basic needs and often do not see a way to improve their situation. It is difficult to maintain optimism. Lower socioeconomic groups usually do not have group health insurance. DIF: Cognitive Level: Understanding OBJ: Nursing Process Step: Assessment MSC: Patient Needs: Health Promotion and Maintenance 24. While teaching an Asian patient regarding prenatal care, the nurse notes that the patient refuses to make eye contact. Which is the most likely cause for this behavior? a. A submissive attitude b. Lack of understanding c. Embarrassment about the subject d. Cultural beliefs about eye contact ANS: D The nurse must understand that making eye contact means different things in different cultures. The nurse should have a basic understanding of normal responses of various cultures within her community. Asians believe that eye contact shows disrespect, not submission. Many Asian women may nod and smile during patient teaching; however, this does not indicate understanding. They are responding that they heard you; therefore validation of information is important. Concerns regarding modesty are more common among Muslim women. DIF: Cognitive Level: Understanding OBJ: Nursing Process Step: Assessment MSC: Patient Needs: Psychosocial Integrity 25. The nurse in labor and birth is caring for a Muslim patient during the active phase of labor. The nurse notes that the patient quickly draws away when touched. Which intervention should the nurse implement? a.Ask the charge nurse to reassign you to another patient. b. Assume that she does not like you and decrease your time with her. c. Continue to touch her as much as you need to while providing care. d. Limit touching to a minimum because physical contact may not be acceptable in her culture. ANS: D Touching is an important component of communication in various cultures; however, if the patient appears to find it offensive, the nurse should respect her cultural beliefs and limit touching her. Asking the charge nurse to reassign you could be offensive to the patient. A Muslim‘s response to touch does not reflect like or dislike. By continuing to touch her, the nurse is showing disrespect for the patient‘s cultural beliefs. DIF: Cognitive Level: Application OBJ: Nursing Process Step: Implementation MSC: Patient Needs: Psychosocial Integrity 26. Which patient may require more help and understanding when integrating the newborn into the family? a. A primipara from an upper income family b. A primipara who comes from a large family c. A multipara (gravida 2) who has a supportive husband and mother d. A multipara (gravida 6) who has two children younger than 3 years ANS: D Pregnancy tasks are more complex for the multipara (gravida 6), and she may need special assistance to integrate the infant into the family structure. A primipara from an upper income family has the financial resources to assist her with daily care of the home. This leaves her free to concentrate on the newborn‘s needs. The primipara with a large support system has help available to her. The multipara (gravida 2) who has a supportive husband and mother has a support system to assist with integrating the infant into the family structure. DIF: Cognitive Level: Analysis OBJ: Nursing Process Step: Assessment MSC: Patient Needs: Health Promotion and Maintenance 27. A patient arrives to the clinic 2 hours late for her prenatal appointment. This is the third time she has been late. What is the nurse‘s best action in response to this patient‘s tardiness? a. Ask the patient if she has a way to tell the time. b. Ask the patient if she is deliberately being late for her appointments. c. Determine if the patient wants this baby and if this is her way of acting out. d. Determine if the patient arrives after the start time for other types of appointments. ANS: D Time orientation is viewed differently by other cultures. Native Americans, Middle Easterners, Hispanics, and American Eskimos tend to emphasize the moment rather than the future. This causes conflicts in the health care setting, in which tests or appointments are scheduled at particular times. If a woman does not place the same importance on keeping appointments, she may encounter anger and frustration in the health care setting. Asking if she has a way to tell time does not get to the potential root of the problem. Asking if she is deliberately late isinconsiderate and nontherapeutic. Although her action may be an acting-out behavior, there are other considerations that must be considered first. DIF: Cognitive Level: Application OBJ: Nursing Process Step: Assessment MSC: Patient Needs: Health Promotion and Maintenance MULTIPLE RESPONSE 1. The clinic nurse often cares for patients who are considering an abortion. Which responsibilities does this nurse have in regard to this issue? (Select all that apply.) a. Informing the patient about pro-life options b. Informing the patient about pro-choice support groups c. Being informed about abortion from a legal standpoint d. Being informed about abortion from an ethical standpoint e. Recognizing that this issue may result in confusion for the patient ANS: C, D, E Nurses have several responsibilities while caring for patients who request a termination of pregnancy. First, the nurse must be informed about the complexity of the abortion issue from a legal and an ethical standpoint and know the regulations and laws in their state. Second, the nurse must recognize that for many patients abortion is an ethical dilemma that results in confusion, ambivalence, and personal distress. Informing the patient regarding pro-life options or pro-choice support groups would not be appropriate because it is the patient‘s decision and these interventions show bias on the nurse‘s part. DIF: Cognitive Level: Analysis OBJ: Nursing Process Step: Evaluation MSC: Patient Needs: Health Promotion and Maintenance 2. A couple asks the nurse about the procedure for surrogate parenting. Which correct responses should the nurse provide for this couple? (Select all that apply.) a. Donated embryos can be implanted into the surrogate mother. b. The surrogate mother needs to have carried one previous birth to term. c. You both need to be infertile to be eligible for surrogate parenting. d. Conception can take place outside the surrogate mother‘s body and then implanted. e. The surrogate mother can be inseminated artificially with sperm from the intended father. ANS: A, D, E In surrogate parenting, conception may take place outside the body using ova and sperm from the couple that wishes to become parents. These embryos are then implanted into the surrogate mother, or the surrogate mother may be inseminated artificially with sperm from the intended father. Donated embryos may also be implanted into a surrogate mother. The couple does not need to be infertile. The surrogate parent does not need to have previously carried a pregnancy to term. DIF: Cognitive Level: Application OBJ: Nursing Process Step: Implementation MSC: Patient Needs: Health Promotion and Maintenance 3. Which actions by the nurse indicate compliance with the Health Insurance Portability and Accountability Act (HIPAA)? (Select all that apply.) a.The nurse posts an update about a patient on Facebook. b. The nurse gives the report to the oncoming nurse in a private area. c. The nurse gives information about the patient‘s status over the phone to the patient‘s friend. d. The nurse logs off any computer screen showing patient data before leaving the computer unattended. e. The nurse puts any documentation with the patient‘s information in the shred bin at the hospital before leaving for the day. ANS: B, D, E HIPAA regulations provide consumers with significant power over their records, including the right to see and correct their records, the application of civil and criminal penalties for violations of privacy standards, and protection against deliberate or inadvertent misuse or disclosure. Discussions about a patient with other professionals should be restricted to those who need to know and should occur in a private location. Nurses must take care to avoid violating patient confidentiality when using electronic patient data formats. For example, nurses must promptly log off terminals when finished so that unauthorized individuals cannot gain access to the system. Shredding documentation with patient identifiers should be done before leaving the hospital. Discussing a patient‘s status in any online forum is a violation of HIPAA. Giving information to a patient‘s friend over the phone, without the patient‘s consent, is a violation of HIPAA. DIF: Cognitive Level: Application OBJ: Nursing Process Step: Implementation MSC: Patient Needs: Safe and Effective Care Environment 4. In some Middle Eastern and African cultures, female genital mutilation (female cutting) is a prerequisite for marriage. Women who now live in North America need care from nurses who are knowledgeable about the procedure and comfortable with the abnormal appearance of their genitalia. When caring for this patient, the nurse can formulate a diagnosis with the understanding that the patient may be at risk for which of the following? (Select all that apply.) a. Infection b. Laceration c. Hemorrhage d. Obstructed labor e. Increased signs of pain response ANS: A, B, C, D The patient is at risk for infection, laceration, hemorrhage, and obstructed labor. Female genital mutilation, cutting, or circumcision involves removal of some or all of the external female genitalia. The labia majora are often stitched together over the vaginal and urethral openings as part of this practice. Enlargement of the vaginal opening may be performed before or during the birth. The woman is unlikely to give any verbal or nonverbal signs of pain. This lack of response does not indicate lack of pain. In fact, pelvic examinations are likely to be very painful because the introitus is so small, and inelastic scar tissue makes the area especially sensitive. A pediatric speculum may be necessary, and the patient should be made as comfortable as possible. DIF: Cognitive Level: Analysis OBJ: Nursing Process Step: DiagnosisMSC: Patient Needs: Psychosocial Integrity 5. A Vietnamese patient who speaks little English is admitted to the labor and birth unit in early labor. The nurse plans to use an interpreter during an initial assessment. Which should the nurse plan to implement with regard to using an interpreter? (Select all that apply.) a. Face the interpreter when speaking. b. Listen carefully to what the patient says. c. Speak slowly and smile when appropriate. d. Plan to use a male interpreter, even if a female interpreter is available. e. Ask the interpreter to explain exactly what is said as much as possible, instead of paraphrasing. ANS: B, C, E The nurse planning to use an interpreter should listen carefully to what the patient says. The nurse should speak slowly and smile when appropriate. Ask the interpreter to explain exactly what is said instead of paraphrasing. It is preferable to use a trained female interpreter when one is available instead of a male interpreter. The nurse should directly face the patient when speaking. DIF: Cognitive Level: Application OBJ: Nursing Process Step: Planning MSC: Patient Needs: Psychosocial Integrity Chapter 03: Reproductive Anatomy and Physiology Foundations of Maternal-Newborn & Women‘s Health Nursing, 7th Edition MULTIPLE CHOICE 1. A postpartum patient who has had a vaginal birth asks the nurse, ―I was wondering if my cervix will return to its previous shape before I had the baby?‖ Which is the best response by the nurse? a. The cervix will now have a slit-like shape. b. The cervix will be round and smooth after healing occurs. c. The cervix will remain 50% effaced now that you have had a baby. d. The cervix will be slightly dilated to 2 cm for about 6 months. ANS: A After vaginal birth, the external os has an irregular slit-like shape and may have tags of scar tissue. The external os of a childless woman is round and smooth; however, after a vaginal birth it will be round and smooth. During labor, the cervix effaces (thins) and dilates (opens) to allow passage of the fetus. Once the baby is born, the cervix will close and return to close to 100% effacement. DIF: Cognitive Level: Application OBJ: Nursing Process Step: Implementation MSC: Patient Needs: Health Promotion and Maintenance 2. The school nurse is conducting health education classes for a group of adolescents. Which statement best describes a secondary sexual characteristic? a. Maturation of ova b. Production of spermc. Female breast development d. Secretion of gonadotropin-releasing hormone ANS: C A secondary sexual characteristic is one not directly related to reproduction, such as development of the characteristic female body form. Maturation of ova is directly related to reproduction and is a primary sexual characteristic. Production of sperm is directly related to reproduction and is a primary sexual characteristic. Secretion of hormones is directly related to reproduction and is a primary sexual characteristic. DIF: Cognitive Level: Understanding OBJ: Nursing Process Step: Implementation MSC: Patient Needs: Health Promotion and Maintenance 3. Which 16-year-old female patient is most likely to experience secondary amenorrhea? a. 5 ft 2 in, 130 lb b. 5 ft 9 in, 180 lb c. 5 ft 7 in, 96 lb d. 5 ft 4 in, 125 lb ANS: C Due to her height and low body weight, this adolescent is at risk of developing secondary amenorrhea. Secondary amenorrhea occurs in women who are thin and have a low percentage of body fat. Fat is necessary to make the sex hormones that stimulate ovulation and menstruation. The other patients are of sufficient height and weight to promote sex hormone production. DIF: Cognitive Level: Application OBJ: Nursing Process Step: Assessment MSC: Patient Needs: Physiologic Integrity 4. Which characteristic best describes the levator ani? a. Division of the fallopian tube b. Collection of three pairs of muscles c. Imaginary line that divides the true pelvis and false pelvis d. Basin-shaped structure at the lower end of the spine ANS: B The levator ani is a collection of three pairs of muscles that support internal pelvic structures and resist increases in intraabdominal pressure. The fallopian tube divisions are the interstitial portion, isthmus, ampulla, and infundibulum. The linea terminalis is the imaginary line that divides the false from the true pelvis. The basin-shaped structure at the lower end of the spine is the bony pelvis. DIF: Cognitive Level: Remembering OBJ: Nursing Process Step: Assessment MSC: Patient Needs: Health Promotion and Maintenance 5. The nurse is describing the size and shape of the nonpregnant uterus to a patient. Which is an accurate description? a. The nonpregnant uterus is the size and shape of a pear. b.The nonpregnant uterus is the size and shape of a cantaloupe. c. The nonpregnant uterus is the size and shape of a grapefruit. d. The non-pregnant uterus is the size and shape of a large orange. ANS: A The nonpregnant uterus is about 7.5 5 2.5 cm, which is close to the size and shape of a pear. A cantaloupe would be too large and is the wrong shape for the uterus. A grapefruit is too large for the nonpregnant uterus; the uterus is larger at the upper end and tapers down. An orange may be the appropriate size, but it is not the appropriate shape. DIF: Cognitive Level: Application OBJ: Nursing Process Step: Implementation MSC: Patient Needs: Health Promotion and Maintenance 6. If a woman‘s menstrual cycle began on June 2, on which date should ovulation mostly likely have occurred? a. June 10 b. June 16 c. June 29 d. July 5 ANS: B June 16 would be 18 days into the cycle; ovulation should have occurred at this point. June 10 would just be 8 days into the cycle and too early for ovulation. Ovulation occurs about 12 to 14 days after the beginning of the next menstrual period in a 28-day cycle; ovulation normally occurs about 14 days before the beginning of the next period. June 29 is at the end of the cycle. July 5 would be 27 days into the cycle and about time for the next period. DIF: Cognitive Level: Application OBJ: Nursing Process Step: Assessment MSC: Patient Needs: Health Promotion and Maintenance 7. A patient states, ―My breasts are so small. I don‘t think I will be able to breastfeed my baby.‖ Which is the nurse‘s best response? a. ―It may be difficult but you should try anyway.‖ b. ―You can always supplement with formula.‖ c. ―All women have about the same amount of glandular tissue necessary to secrete milk.‖ d. ―The ability to produce breast milk depends on increased levels of estrogen and progesterone.‖ ANS: C All women have 15 to 20 lobes arranged around and behind the nipple and areola. These lobes, not the size of the breast, are responsible for milk production. The size of the breasts does not ensure success or failure in breastfeeding. Supplementation decreases the production of breast milk by decreasing stimulation. Stimulation of the breast, not the size of the breast, brings about milk production. Increased levels of estrogen decrease the production of milk by affecting prolactin. DIF: Cognitive Level: Application OBJ: Nursing Process Step: Implementation MSC: Patient Needs: Physiologic Integrity 8. The nurse is explaining the function of the male‘s cremaster muscle to a group of nursing students. Which statement accurately describes the function of the cremaster muscle?a. Assists with transporting sperm b. Aids in temperature control of the testicles c. Aids in voluntary control of excretion of urine d. Entraps blood in the penis to produce an erection ANS: B One cremaster muscle is attached to each testicle. Its function is to bring the testicle closer to the body to warm it or allow it to fall away from the body to cool it, thus promoting normal sperm production. Seminal fluid assists with transporting sperm. The urinary meatus aids in controlling the excretion of urine. Entrapment of the blood in the penis is a result of the spongy tissue. DIF: Cognitive Level: Understanding OBJ: Nursing Process Step: Assessment MSC: Patient Needs: Physiologic Integrity 9. A newly pregnant patient asks the nurse, ―What is a false pelvis?‖ Which statement by the nurse will best explain this anatomy to the patient? a. It is the total anterior portion of the pelvis. b. It is considered to be the lower portion of the pelvis. c. It provides support for the internal organs and the upper part of the body. d. It is the narrowest part of the pelvis through which a fetus will pass during birth. ANS: C The linea terminalis, also called the pelvic brim or iliopectineal line, is an imaginary line that divides the upper, or false, pelvis from the lower, or true, pelvis. The false pelvis provides support for the internal organs and upper part of the body. The false pelvis is the upper portion, not the total anterior portion. The lower portion of the pelvis is the true pelvis, which is most important during childbirth because it has the narrowest portion through which the fetus will pass during childbirth. DIF: Cognitive Level: Understanding OBJ: Nursing Process Step: Assessment MSC: Patient Needs: Health Promotion and Maintenance 10. The clinic nurse is reviewing breastfeeding with a pregnant patient. Which hormone will the nurse explain is responsible for milk production after the birth of the placenta? a. Pitocin b. Prolactin c. Estrogen d. Progesterone ANS: B During pregnancy, high levels of estrogen and progesterone produced by the placenta stimulate growth of the alveoli and ductal system to prepare them for lactation. Prolactin secretion by the anterior pituitary gland stimulates milk production during pregnancy; however, this effect is inhibited by estrogen and progesterone produced by the placenta. Inhibiting effects of estrogen and progesterone stop when the placenta is expelled after birth, and active milk productionoccurs in response to the infant‘s suckling while breastfeeding. Pitocin is the hormone that causes the let-down reflex during breastfeeding. DIF: Cognitive Level: Understanding OBJ: Nursing Process Step: Assessment MSC: Patient Needs: Health Promotion and Maintenance 11. Which hormonal effect is noted during the menstrual cycle? a. Luteinizing hormone (LH) and follicle-stimulating hormone (FSH) secretion rise during the ovulatory phase. b. A negative feedback mechanism is exhibited by the anterior pituitary gland and ovaries. c. The posterior pituitary gland secretes LH. d. Estrogen secretion enhances FSH secretion. ANS: A Levels of LH and FSH rise dramatically during the ovulatory phase and are known as the LH surge prior to ovulation. A positive feedback mechanism occurs with regard to the menstrual cycle. The anterior pituitary gland secretes LH. Estrogen secretion minimizes FSH secretion. DIF: Cognitive Level: Application OBJ: Nursing Process Step: Assessment MSC: Patient Needs: Physiologic Integrity: Reduction of Risk Potential/System-Specific Assessments 12. A female patient who has gone through puberty and started menstruating without any problems has developed cessation of periods after 2 years of normal cycles. Which finding would indicate a possible cause for this occurrence? a. Lag in development of secondary sexual characteristics b. Overproduction of androgenic hormones c. Negative pregnancy test d. Clinical diagnosis of primary amenorrhea ANS: B An overproduction of androgenic hormones may cause the development of secondary amenorrhea. This patient has progressed through puberty, which would indicate that there is no problem with the development of secondary sexual characteristics. If the patient had a positive pregnancy test, then menstruation would stop. These signs and symptoms indicate the occurrence of secondary amenorrhea. DIF: Cognitive Level: Application OBJ: Nursing Process Step: Assessment MSC: Patient Needs: Health Promotion and Maintenance 13. On speculum examination of the cervix, it is found to be round and smooth. These findings suggest that the patient a. is a multipara. b. has had previous vaginal deliveries. c. is nulliparous. d. is a gravida 1, para 0. ANS: CThese findings indicate that the patient has never been pregnant and she would be classified as nulliparous. The other findings indicate that the patient is a multipara, has had vaginal deliveries, or is a gravida 1, para 0. These all refer to a patient with a positive history of pregnancy. DIF: Cognitive Level: Application OBJ: Nursing Process Step: Assessment MSC: Patient Needs: Health Promotion and Maintenance: Techniques of Physical Assessment 14. Which statement with regard to reproductive anatomy and physiology is inaccurate? a. Female patients who are past puberty and sexually active can become pregnant even if they have not had a menstrual cycle. b. Puberty symptoms are more prominent in males than females. c. Females enter puberty earlier than their male counterparts. d. Secondary sexual characteristics develop during puberty. ANS: B Puberty symptoms are less prominent in developing males than females. The other statements are correct. DIF: Cognitive Level: Application OBJ: Nursing Process Step: Assessment MSC: Patient Needs: Health Promotion and Maintenance 15. The nurse is reviewing normal female development with a mother of a 10-year-old daughter. The mother states, ―I noticed that my daughter developed breast buds about a year ago. When do you think she will start her menstrual cycle?‖ What is the nurse‘s best response? a. ―In about a year.‖ b. ―Likely any time now.‖ c. ―Does your daughter know what to expect?‖ d. ―It is impossible to predict when she will start her cycle.‖ ANS: A Menarche occurs about 2 to 2.5 years after breast development. Asking the mother if her daughter knows what to expect is a vague response that does not answer the mother‘s question. DIF: Cognitive Level: Understanding OBJ: Nursing Process Step: Implementation MSC: Patient Needs: Health Promotion and Maintenance 16. The middle school nurse is reviewing the phases of the endometrial cycle with a group of female students. Which statement by a student will the nurse need to correct? a. ―The proliferative phase occurs when the ovum is maturing.‖ b. ―The expulsion phase occurs when the ovum is discharged from the ovary.‖ c. ―The secretory phase occurs during the second half of the menstrual cycle.‖ d. ―The menstrual phase occurs after the levels of estrogen and progesterone fall.‖ ANS: BThe menstrual cycle has only three phases: proliferative, secretory, and menstrual. Occurrences of each of the three phases have been described. There is no expulsion phase in the menstrual cycle. DIF: Cognitive Level: Understanding OBJ: Nursing Process Step: Evaluation MSC: Patient Needs: Health Promotion and Maintenance MULTIPLE RESPONSE 1. A young female patient comes to the health unit at school to discuss her irregular periods. In providing education regarding the female reproductive cycle, the nurse describes the regular and recurrent changes related to the ovaries and the uterine endometrium. Although this is generally referred to as the menstrual cycle, the ovarian cycle includes which phases? (Select all that apply.) a. Follicular b. Ovulatory c. Luteal d. Proliferative e. Secretory ANS: A, B, C The follicular phase is the period during which the ovum matures. It begins on day 1 and ends around day 14. The ovulatory phase occurs near the middle of the cycle, about 2 days before ovulation. After ovulation and under the influence of the luteinizing hormone, the luteal phase corresponds with the last 12 days of the menstrual cycle. The proliferative and secretory phases are part of the endometrial cycle. The proliferative phase takes place during the first half of the ovarian cycle when the ovum matures. The secretory phase occurs during the second half of the cycle when the uterus is prepared to accept the fertilized ovum. These are followed by the menstrual phase if fertilization does not occur. DIF: Cognitive Level: Understanding OBJ: Nursing Process Step: Assessment MSC: Patient Needs: Health Promotion and Maintenance 2. The school nurse is conducting health education classes for a group of adolescent girls. Select the actions of the estrogen hormone that the nurse should include in the lessons. (Select all that apply.) a. Stimulates contractions during birth b. Relaxes pelvic ligaments during pregnancy c. Stimulates the endometrium before ovulation d. Stimulates growth of uterus during pregnancy e. Stimulates the let-down reflex during breastfeeding ANS: B, C, D The hormone estrogen relaxes pelvic ligaments during pregnancy, stimulates the endometrium before ovulation, and stimulates the growth of the uterus during pregnancy. Oxytocin stimulates contractions during pregnancy and stimulates the let-down reflex during breastfeeding. DIF: Cognitive Level: Application OBJ: Nursing Process Step: Implementation MSC: Patient Needs: Health Promotion and MaintenanceCOMPLETION 1. Delayed onset of menstruation or primary amenorrhea is considered if the girl‘s periods have not begun by which age in years? Record your answer in a whole number. ANS: 16 Delayed onset of menstruation is called primary amenorrhea if the girl‘s periods have not begun within 2 years after the onset of breast development or by age 16, or if the girl is more than 1 year older than her mother or sisters were when their menarche occurred. DIF: Cognitive Level: Understanding OBJ: Nursing Process Step: Assessment MSC: Patient Needs: Health Promotion and Maintenance Chapter 04: Hereditary and Environmental Influences on Childbearing Foundations of Maternal-Newborn & Women‘s Health Nursing, 7th Edition MULTIPLE CHOICE 1. A clinic nurse is planning a teaching session for childbearing-age female patients. Which information should the nurse include in the teaching session with regard to avoiding exposing a fetus to teratogens? a. Eliminate use of acne medications. b. Immunizations should be updated during the first trimester of pregnancy. c. Use of saunas and hot tubs during pregnancy should be during the winter months only. d. Alcoholic beverages can be consumed in the first and third trimesters of pregnancy. ANS: A Elimination of nontherapeutic drugs is the best action to avoid teratogen exposure. Acne medication is not essential during pregnancy. Immunizations for diseases such as rubella are contraindicated during pregnancy. Use of saunas and hot tubs are not recommended because maternal hyperthermia is a significant teratogen. Alcohol is an environmental substance known to be teratogenic and should not be consumed during pregnancy. DIF: Cognitive Level: Application OBJ: Nursing Process Step: Planning MSC: Patient Needs: Health Promotion and Maintenance 2. The parents of a child with a karyotype of 47,XY,+21 ask the nurse what this means. Which is the most accurate response by the nurse? a. This karyotype is for a normal male. b. This karyotype is for a normal female. c. This karyotype is for a male with Down syndrome. d. This karyotype is for a female with Turner‘s syndrome. ANS: C This child is male because his sex chromosomes are XY. He has one extra copy of chromosome 21 (for a total of 47, instead of 46), resulting in Down syndrome. A normal female would have 46 chromosomes and XX for the sex chromosomes. A normal male would have 46 chromosomes. A female with Turner‘s syndrome would have 45 chromosomes; the sex chromosome would have just one X. DIF: Cognitive Level: Application OBJ: Nursing Process Step: Implementation MSC: Patient Needs: Physiologic Integrity3. People who have two copies of the same abnormal autosomal dominant gene are generally a. mildly affected with the disorder. b. infertile and unable to transmit the gene. c. carriers of the trait but not affected with the disorder. d. more severely affected by the disorder than people with one copy of the gene. ANS: D People who have two copies of an abnormal gene are usually more severely affected by the disorder because they have no normal gene to compensate and maintain normal function. Those mildly affected with the disorder will have only one copy of the abnormal gene. Infertility may or may not be caused by chromosomal defects. A carrier of a trait has one recessive gene. DIF: Cognitive Level: Understanding OBJ: Nursing Process Step: Assessment MSC: Patient Needs: Physiologic Integrity 4. An infant is born with blood type AB. The father is type A and the mother is type B. The father asks why the baby has a blood type different from that of the parents. The nurse‘s answer should be based on the knowledge that a. both A and B blood types are dominant. b. types A and B are recessive when linked together. c. the baby has a mutation of the parents‘ blood types. d. type A is recessive and links more easily with type B. ANS: A Types A and B are equally dominant, and the baby can thus inherit one from each parent. Both types A and B are dominant, not recessive. The infant has inherited both blood types from the parents and this is not a mutation. Both blood types A and B are equally dominant. DIF: Cognitive Level: Application OBJ: Nursing Process Step: Assessment MSC: Patient Needs: Physiologic Integrity 5. Which statement regarding multifactorial disorders is correct? a. They may not be evident until later in life. b. They are usually present and detectable at birth. c. The disorders are characterized by multiple defects. d. Secondary defects are rarely associated with them. ANS: B Multifactorial disorders result from an interaction between a person‘s genetic susceptibility and environmental conditions that favor development of the defect. They are characteristically present and detectable at birth. They are usually single isolated defects, although the primary defect may cause secondary defects. Secondary defects can occur with multifactorial disorders. DIF: Cognitive Level: Remembering OBJ: Nursing Process Step: Assessment MSC: Patient Needs: Health Promotion and Maintenance 6. Which information should the nurse include when discussing the prenatal diagnosis of genetic disorders with an expectant couple?a. The diagnosis may be slow and could be inconclusive. b. A comprehensive evaluation will result in an accurate diagnosis. c. Common disorders can be quickly diagnosed through blood tests. d. Diagnosis can be obtained promptly through most hospital laboratories. ANS: A Even the best efforts at diagnosis do not always yield the information needed to counsel the patient. The process may require many visits over several weeks. Some tests must be sent to special laboratories, which take additional time. Despite a comprehensive evaluation, a diagnosis may never be established. At this time there are no rapid result blood tests available to diagnose genetic disorders. Some tests must be sent to a special laboratory, which requires a longer waiting period for results. DIF: Cognitive Level: Application OBJ: Nursing Process Step: Implementation MSC: Patient Needs: Health Promotion and Maintenance 7. A patient tells the nurse at a prenatal interview that she has quit smoking, and only has one glass of wine with dinner. Which response by the nurse will be most helpful in promoting a lifestyle change? a. ―Those few things won‘t cause any trouble. Good for you.‖ b. ―You need to do a lot better than that. You are still hurting your baby.‖ c. ―Here are some pamphlets for you to study. They will help you to find more ways to improve.‖ d. ―You have made some good progress toward having a healthy baby. Let‘s talk about the changes you have made.‖ ANS: D Praising her for making positive changes is an effective technique for motivating a patient. She still has risk factors to alter for optimal outcome, and a gentle maneuver to help her see these for herself will be most likely to succeed. Alcohol consumption is still a major risk factor and needs to be addressed in a positive, nonjudgmental manner. The statement, ―You need to do a lot better‖ is belittling to the patient; she will be less likely to confide in the nurse. The nurse is not acknowledging the efforts that the patient has already accomplished by offering pamphlets; those accomplishments need to be praised to motivate the patient to continue. DIF: Cognitive Level: Application OBJ: Nursing Process Step: Implementation MSC: Patient Needs: Health Promotion and Maintenance 8. A 35-year-old patient has an amniocentesis performed to identify whether her baby has a chromosomal defect. Which statement indicates that the patient understands the situation? a. ―The doctor will tell me if I should have an abortion when the test results come back.‖ b. ―When all the lab results come back, my husband and I will make a decision about the pregnancy.‖ c. ―My mother must not find out about all this testing. If she does, she will think I‘m having an abortion.‖ d.―I know there are support groups for parents who have a baby with birth defects, but we have plenty of insurance to cover what we need.‖ ANS: B The final decision about genetic testing and the future of the pregnancy lies with the patient. The patient will involve only those people whom she chooses. An amniocentesis is done to detect chromosomal defects; many women have this done to prepare and educate themselves for the baby‘s arrival. The woman should also be assured that her care is confidential. Insurance will help cover expenses; however, a child with birth defects also takes a toll on the emotional, physical, and social aspects of the parents‘ lives. Support groups are extremely important for parents of a baby with birth defects. DIF: Cognitive Level: Analysis OBJ: Nursing Process Step: Evaluation MSC: Patient Needs: Health Promotion and Maintenance 9. Which characteristic is related to Down syndrome? a. Up-slanting eyes b. Abnormal genitalia c. Bleeding tendency d. Edema of extremities ANS: A Up-slanting eyes, wide short fingers, and low-set ears are often seen in infants with Down syndrome. Bleeding tendency, edema of extremities, and abnormal genitalia are not characteristics of Down syndrome. DIF: Cognitive Level: Understanding OBJ: Nursing Process Step: Assessment MSC: Patient Needs: Physiologic Integrity 10. Which question posed by the nurse will most likely promote the sharing of sensitive information during a genetic counseling interview? a. ―What kind of defects or diseases seem to run in the family?‖ b. ―How many people in your family are mentally retarded or handicapped?‖ c. ―Did you know that you can always have an abortion if the fetus is abnormal?‖ d. ―Are there any members of your family who have learning or developmental problems?‖ ANS: D The nurse should probe gently using layperson-oriented terminology, such as learning problems rather than defects or diseases. Some individuals may not be aware of which diseases are genetically linked and may not answer the question accurately. ―How many people in your family are mentally retarded or handicapped?‖ assumes that there are genetic problems that resulted in retardation in the family. Some individuals may find these terms offensive. ―Did you know that you can always have an abortion if the fetus is abnormal?‖ is taking the decision away from the parents. They are seeking counseling to prevent problems, not to find out what to do if there is a problem. DIF: Cognitive Level: Application OBJ: Nursing Process Step: Implementation MSC: Patient Needs: Health Promotion and Maintenance 11. Testing for the cause of anomalies in a stillborn infant is underway. The mother angrily asks the nurse how long these tests are going to take. The nurse should understand that this mother isa. exhibiting normal grief behavior. b. trying to place blame on someone. c. being impatient and unreasonable. d. feeling guilty and blaming herself. ANS: A Grief after a fetal loss may initially be expressed as anger. The mother is not placing blame; she is in the anger stage of the grieving process. The mother is not being impatient or unreasonable. The mother is expressing anger as an initial stage of grief, not blaming herself or feeling guilty. DIF: Cognitive Level: Application OBJ: Nursing Process Step: Implementation MSC: Patient Needs: Psychosocial Integrity 12. While obtaining a genetic history from a female patient, you note that there is a family history of a genetic disease on the maternal side; however, no evidence of symptomatology in the patient or the patient‘s children, two girls. Which observation can you make related to genetic expression? a. Autosomal dominant expression is observed. b. X-linked dominant trait is observed. c. More information is needed to determine the answer. d. Autosomal recessive expression is observed and both the children will be carriers of the disease process. ANS: C Because we have no information about the father and/or paternal side, the other stated options do not apply. If an autosomal dominant expression were present in the family history, the patient would be symptomatic. X-linked recessive traits are more common than X-linked dominant traits and, again, the patient and children are not symptomatic. Although an autosomal recessive expression may be present, we cannot predict that the children will be carriers. There is a 25% chance of being affected. DIF: Cognitive Level: Analysis OBJ: Nursing Process Step: Diagnosis MSC: Patient Needs: Physiologic Integrity: Pathophysiology 13. A couple is undergoing genetic counseling and are very concerned about the possibility of having a child with a birth defect as a result of a strong family history on both sides of the family. Which statement made by the nurse is evidence of therapeutic communication? a. ―It is important to ask other members of your family for any information they can provide that will help obtain more insight into the health history.‖ b. ―Given what you have told me, there is little that anyone can do to improve outcomes.‖ c. ―Although you may feel that you have no options, I can‘t really discuss these matters as only the physician can provide you with information.‖ d. ―Do you have all your forms filled out correctly? This will make the review easier to accomplish. ‖ ANS: AHaving as much information as possible will help analyze potential health outcomes. It also shows that the nurse is taking the patients‘ concerns seriously. Telling the patients that there is little anyone can do does not provide any comfort or hope and is therefore self-defeating. Although the patients may have limited options based on their genetic and medical history, it is important to provide support and not defer all communication to the physician. The nurse must be able to provide support and counseling to patients. It is important to have completed forms, but asking patients about them does not address their psychological concerns. DIF: Cognitive Level: Analysis OBJ: Nursing Process Step: Planning MSC: Patient Needs: Psychosocial Integrity: Therapeutic Communication 14. A patient presents with curly hair and blue eyes. These findings are consistent with a. phenotype. b. genotype. c. dominant alleles. d. recessive traits. ANS: A Curly hair is considered to be a dominant trait, whereas blue eyes are considered to be a recessive trait. Observation of characteristics is noted as phenotype. Genotype identifies the genetic makeup of traits. DIF: Cognitive Level: Application OBJ: Nursing Process Step: Assessment MSC: Patient Needs: Physiologic Integrity: Reduction of Risk Potential 15. The nurse is working with a patient to obtain information necessary for genetic counseling. Which tool will be used to obtain this information? a. Braden scale b. Genogram c. Chorionic villus sampling (CVS) d. Serum protein electrophoresis ANS: B When obtaining information with regard to genetic counseling, it is important to obtain a family history using a genogram or pedigree as the clinical tool. The use of this diagram provides information for maternal and paternal histories and allows for the interpretation of significance based on findings of age, death, and medical history. A Braden scale is used to assess problems with skin leading to potential breakdown. CVS is a diagnostic procedure used during pregnancy to obtain genetic information about the fetus. Serum protein electrophoresis is a lab test used to determine immunoglobulin levels. DIF: Cognitive Level: Application OBJ: Nursing Process Step: Planning MSC: Patient Needs: Physiologic Integrity: Reduction of Risk Potential 16. The nurse receives report on an infant whose analysis indicates 47 total chromosomes, with the abnormality noted at chromosome 21. Which additional assessments will the nurse include when evaluating the infant? a. Cleft palate b. Protruding tonguec. Extra fingers or toes (polydactyly) d. Intellectual developmental delay ANS: B Trisomy 21 is associated with a number of notable physical characteristics, including wide-set eyes, flat bridge of the nose, protruding tongue, short neck, small chin, poor muscle tone, and space between the great and second toes. An infant with trisomy 21 tends to be short in stature and developmentally delayed, but two characteristics will become more noticeable as the child gets older. Cleft palate and polydactyly are more common with trisomy 13. DIF: Cognitive Level: Application OBJ: Nursing Process Step: Assessment MSC: Patient Needs: Health Promotion and Maintenance 17. The patient indicates to the clinic nurse that she is trying to become pregnant. The clinic nurse reviews the patient‘s chart and notes the following laboratory values: Blood type O, RPR nonreactive, rubella non-immune, HCT 35%. Which laboratory value is most concerning to the nurse? a. HCT 35% b. Blood type O c. RPR nonreactive d. Rubella non-immune ANS: D Rubella non-immune indicates that the patient does not have immunity against rubella and is therefore susceptible to the infection. Exposure to rubella, or German measles, in the first trimester is associated with fetal congenital anomalies. The patient requires a rubella immunization and must not become pregnant within the next 28 days. Although a HCT of 35% is low, women of childbearing age may have mild anemia associated with menstruation. A blood type of O indicates that the patient will require Rho GAM; however, it is not of concern in the preconception period. A nonreactive RPR indicates that the patient has not been exposed to syphilis. DIF: Cognitive Level: Synthesis OBJ: Nursing Process Step: Analysis MSC: Patient Needs: Health Promotion and Maintenance MULTIPLE RESPONSE 1. The clinic nurse is reviewing charts on prenatal patients. Which patient histories indicate that a referral to a genetic counselor is warranted? (Select all that apply.) a. A father who is aged 35 b. A patient having a first baby at age 30 c. A family history of unexplained stillbirths d. A patient with a family history of birth defects e. A patient who is a carrier of an X-linked disorder ANS: C, D, E Reasons for a referral to a genetic counselor include family history of unexplained stillbirths, family history of birth defects, and a woman who is a carrier of an X-linked disorder. Pregnantwomen who will be 35 years of age or older when the infant is born and men who father children after age 40 constitute reasons for referral to a genetic counselor. The patient who is 30 and the father who is aged 35 would not warrant a referral to a genetic counselor. DIF: Cognitive Level: Application OBJ: Nursing Process Step: Evaluation MSC: Patient Needs: Health Promotion and Maintenance 2. The nurse is teaching prenatal patients about avoiding substances or conditions that can harm the fetus. Which should the nurse include in the teaching session? (Select all that apply.) a. Elimination of use of alcohol b. Avoidance of supplemental folic acid replacement c. Stabilization of blood glucose levels in a diabetic patient with insulin d. Avoidance of nonurgent radiologic procedures during the pregnancy e. Avoidance of maternal hyperthermia to temperatures of 37.8C (100F) or higher ANS: A, C, D, E The best action is for the pregnant woman to eliminate use of nontherapeutic drugs and substances such as alcohol. A woman who has diabetes should try to keep her blood glucose levels normal and stable before and during pregnancy for the best possible fetal outcomes. Nonurgent radiologic procedures may be done during the first 2 weeks after the menstrual period begins, before ovulation occurs. Exposure to temperatures of 37.8C (100F) or higher is not advised for the pregnant patient. Folic acid supplements should be taken. All women of childbearing age should take at least 0.4 mg (400 mcg) of folic acid daily before and after conception because this has been found to reduce the incidence of neural tube defects by 50% to 70%. DIF: Cognitive Level: Application OBJ: Nursing Process Step: Implementation MSC: Patient Needs: Health Promotion and Maintenance 3. The nurse is explaining genetics to a group of nursing students. Which are autosomal recessive disorders that the nurse should discuss during the teaching session? (Select all that apply.) a. Hemophilia b. Cystic fibrosis c. Sickle cell disease d. Turner‘s syndrome e. Phenylketonuria (PKU) disease ANS: B, C, E Cystic fibrosis, sickle cell disease, and PKU disease are autosomal recessive disorders. Hemophilia and Turner‘s syndrome are X-linked genetic disorders. DIF: Cognitive Level: Application OBJ: Nursing Process Step: Implementation MSC: Patient Needs: Health Promotion and Maintenance COMPLETION 1. Two healthy parents who carry the same abnormal autosomal recessive gene have what percentage chance of having a child affected with the disorder caused by this gene? Record your answer as a whole number. %ANS: 25 Two healthy parents who carry the same abnormal autosomal recessive gene have a 25% chance of having a child affected with the disorder caused by this gene. Unaffected parents are carriers of the abnormal autosomal recessive trait. Children of carriers have a 25% (1 in 4) chance of receiving both copies of the defective gene and thus having the disorder. DIF: Cognitive Level: Application OBJ: Nursing Process Step: Evaluation MSC: Patient Needs: Health Promotion and Maintenance Chapter 05: Conception and Prenatal Development Foundations of Maternal-Newborn & Women‘s Health Nursing, 7th Edition MULTIPLE CHOICE 1. An expectant father asks the nurse, ―Which part of the mature sperm contains the male chromosome?‖ What is the correct response by the nurse? a. X-bearing sperm b. The tail of the sperm c. The head of the sperm d. The middle portion of the sperm ANS: C The head of the sperm contains the male chromosomes that will join the chromosomes of the ovum. If an X-bearing sperm fertilizes the ovum, the baby will be female. The tail of the sperm helps propel the sperm toward the ovum. The middle portion of the sperm supplies energy for the tail‘s whip-like action. DIF: Cognitive Level: Application OBJ: Nursing Process Step: Implementation MSC: Patient Needs: Health Promotion and Maintenance 2. One of the assessments performed in the birth room is checking the umbilical cord for blood vessels. Which finding is considered to be within normal limits? a. One artery and one vein b. Two veins and one artery c. Two arteries and one vein d. Two arteries and two veins ANS: C The umbilical cord contains two arteries and one vein to transport blood between the fetus and the placenta. Any option other than two arteries and one vein is considered abnormal and requires further assessment. Two veins and one artery is abnormal and may indicate an anomaly. Two arteries instead is a normal finding; this infant would require further assessment for anomalies due to the finding of two veins. DIF: Cognitive Level: Understanding OBJ: Nursing Process Step: Assessment MSC: Patient Needs: Health Promotion and Maintenance 3. What is the purpose of the ovum‘s zona pellucida? a. Prevents multiple sperm from fertilizing the ovum b.Stimulates the ovum to begin mitotic cell division c. Allows the 46 chromosomes from each gamete to merge d. Makes a pathway for more than one sperm to reach the ovum ANS: A Fertilization causes the zona pellucida to change its chemical composition so that multiple sperm cannot fertilize the ovum. Mitotic cell division begins when the nuclei of the sperm and ovum unite. Each gamete (sperm and ovum) has only 23 chromosomes; there will be 46 chromosomes when they merge. Once sperm has entered the ovum, the zona pellucida changes to prevent other sperm from entering. DIF: Cognitive Level: Understanding OBJ: Nursing Process Step: Assessment MSC: Patient Needs: Health Promotion and Maintenance 4. The nurse is explaining the process of cell division during the preembryonic period to a group of nursing students. Which statement best describes the characteristics of the morula? a. Fertilized ovum before mitosis begins b. Double layer of cells that becomes the placenta c. Flattened, disk-shaped layer of cells within a fluid-filled sphere d. Solid ball composed of the first cells formed after fertilization ANS: D The morula is so named because it resembles a mulberry. It is a solid ball of 12 to 16 cells that develops after fertilization. The fertilized ovum is called the zygote. The placenta is formed from two layers of cells—the trophoblast, which is the other portion of the fertilized ovum, and the decidua, which is the portion of the uterus where implantation occurs. The flattened, diskshaped layer of cells is the embryonic disk; it will develop into the body. DIF: Cognitive Level: Understanding OBJ: Nursing Process Step: Implementation MSC: Patient Needs: Health Promotion and Maintenance 5. The upper uterus is the best place for the fertilized ovum to implant due to which anatomical adaptation? a. Maternal blood flow is lower. b. Placenta attaches most firmly. c. Uterine endometrium is softer. d. Developing baby is best nourished. ANS: D The uterine fundus is richly supplied with blood and has the thickest endometrium, both of which promote optimum nourishment of the fetus. The blood supply is rich in the fundus, which allows for optimal nourishment of the fetus. If the placenta attaches too deeply, it does not easily detach. Softness is not a concern with implantation; attachment and nourishment are the major concerns. DIF: Cognitive Level: Understanding OBJ: Nursing Process Step: Assessment MSC: Patient Needs: Health Promotion and Maintenance 6. Some of the embryo‘s intestines remain within the umbilical cord during the embryonic period because thea. intestines need this time to grow until week 15. b. nutrient content of the blood is higher in this location. c. abdomen is too small to contain all the organs while they are developing. d. umbilical cord is much larger at this time than it will be at the end of pregnancy. ANS: C The abdominal contents grow more rapidly than the abdominal cavity, so part of their development takes place in the umbilical cord. By 10 weeks, the abdomen is large enough to contain them. The intestines remain within the umbilical cord only until about week 10. Blood supply is adequate in all areas; intestines stay in the umbilical cord for about 10 weeks because they are growing faster than the abdomen. Intestines begin their development within the umbilical cord, but only because the liver and kidneys occupy most of the abdominal cavity, not because of the size of the umbilical cord. DIF: Cognitive Level: Understanding OBJ: Nursing Process Step: Assessment MSC: Patient Needs: Health Promotion and Maintenance 7. A patient who is 16 weeks pregnant with her first baby asks how long it will be before she feels the baby move. Which is the nurse‘s best answer? a. ―You should have felt the baby move by now.‖ b. ―The baby is moving, but you can‘t feel it yet.‖ c. ―Some babies are quiet and you don‘t feel them move.‖ d. ―Within the next month you should start to feel fluttering sensations.‖ ANS: D Maternal perception of fetal movement (quickening) usually begins between 17 and 20 weeks after conception. Because this is her first pregnancy, movement is felt toward the later part of the 17 to 20 weeks. ―The baby is moving, but you can‘t feel it yet‖ may be alarming to the woman. ―Some babies are quiet and you don‘t feel them move‖ is a true statement; the fetus‘ movements are not strong enough to be felt until 17 to 20 weeks; however, this statement does not answer the woman‘s concern. Fetal movement should be felt between 17 and 20 weeks; if movement is not perceptible by the end of that time, further assessment will be necessary. DIF: Cognitive Level: Application OBJ: Nursing Process Step: Implementation MSC: Patient Needs: Health Promotion and Maintenance 8. Which statement best describes the changes that occur during the fetal period of development? a. Maturation of organ systems b. Development of basic organ systems c. Resistance of organs to damage from external agents d. Development of placental oxygen–carbon dioxide exchange ANS: A During the fetal period, the body systems grow in size and mature in function to allow independent existence after birth. Basic organ systems are developed during the embryonicperiod. The organs are always at risk for damage from external sources; however, the older the fetus, the more resistant will be the organs. The greatest risk is when the organs are developing. The placental system is complete by week 12, but that is not the best description of the fetal period. DIF: Cognitive Level: Understanding OBJ: Nursing Process Step: Assessment MSC: Patient Needs: Health Promotion and Maintenance 9. An expectant mother says to the nurse, ―When my sister‘s baby was born, it was covered in a cheese-like coating. What is the purpose of this coating?‖ The correct response by the nurse is to explain that the purpose of vernix caseosa is to a. regulate fetal temperature. b. protect the fetal skin from amniotic fluid. c. promote normal peripheral nervous system development. d. allow the transport of oxygen and nutrients across the amnion. ANS: B Prolonged exposure to amniotic fluid during the fetal period could result in breakdown of the skin without the protection of the vernix caseosa. The amniotic fluid aids in maintaining fetal temperature. Normal peripheral nervous system development is dependent on the nutritional intake of the mother. The amnion is the inner membrane that surrounds the fetus. It is not involved in the oxygen and nutrient exchange. DIF: Cognitive Level: Application OBJ: Nursing Process Step: Implementation MSC: Patient Needs: Health Promotion and Maintenance 10. An expectant mother, diagnosed with oligohydramnios, asks the nurse what this condition means for the baby. Which statement should the nurse provide for the patient? a. Oligohydramnios can cause poor fetal lung development. b. Oligohydramnios means that the fetus is excreting excessive urine. c. Oligohydramnios could mean that the fetus has a gastrointestinal blockage. d. Oligohydramnios is associated with fetal central nervous system abnormalities. ANS: A Because an abnormally small amount of amniotic fluid restricts normal lung development, the fetus may have poor fetal lung development. Oligohydramnios may be caused by a decrease in urine secretion. Excessive amniotic fluid production may occur when the gastrointestinal tract prevents normal ingestion of amniotic fluid. Excessive amniotic fluid production may occur when the fetus has a central nervous system abnormality. DIF: Cognitive Level: Application OBJ: Nursing Process Step: Implementation MSC: Patient Needs: Health Promotion and Maintenance 11. The nurse is conducting a staff in-service on multifetal pregnancy. Which statement regarding dizygotic twin development should the nurse include in the teaching session? a. Dizygotic twins arise from two fertilized ova and are the same sex. b. Dizygotic twins arise from a single fertilized ovum and are always of the same sex. c. Dizygotic twins arise from two fertilized ova and may be the same sex or different sexes.d. Dizygotic twins arise from a single fertilized ovum and may be the same sex or different sexes. ANS: C Dizygotic twins arise from two ova that are fertilized by different sperm. They may be the same or different gender, and they may not have similar physical traits. Monozygotic twins are always the same sex. A single fertilized ovum that produces twins is called monozygotic. Dizygotic twins are from two fertilized ova and may or may not be the same sex. DIF: Cognitive Level: Application OBJ: Nursing Process Step: Implementation MSC: Patient Needs: Health Promotion and Maintenance 12. An infant is diagnosed with fetal anemia. Which information would support this clinical diagnosis? a. Presence of excess maternal hormones b. Maternal blood type O-negative, Rh-negative, and infant blood type O-negative, Rh-negative c. Passive immunity d. Rh-negative mother and Rh-positive baby ANS: C Passive immunity provides temporary protection to the baby based on the transfer of maternal antibodies. Maternal hormones would not lead to a clinical diagnosis of fetal anemia. These blood types and Rh factors are the same; therefore, no antibodies will be created. In this situation, an Rh-negative mother and Rh-positive baby will result in stimulation of antibodies that will stimulate a reaction leading to hemolysis. DIF: Cognitive Level: Analysis OBJ: Nursing Process Step: Assessment MSC: Patient Needs: Physiologic Integrity/Reduction of Risk Potential 13. The nurse is explaining the function of the placenta to a pregnant patient. Which statement indicates to the nurse that further clarification is necessary? a. ―My baby gets oxygen from the placenta.‖ b. ―The placenta functions to help excrete waste products.‖ c. ―The nourishment that I take in passes through the placenta.‖ d. ―The placenta helps maintain a stable temperature for my baby.‖ ANS: D Amniotic fluid and not the placenta helps with thermoregulation. The remaining statements are correct regarding placental function. DIF: Cognitive Level: Application OBJ: Nursing Process Step: Evaluation MSC: Patient Needs: Health Promotion and Maintenance 14. The nurse is assessing a newborn immediately after birth. After assigning the first Apgar score of 9, the nurse notes two vessels in the umbilical cord. What is the nurse‘s next action? a. Assess for other abnormalities of the infant. b. Note the assessment finding in the infant‘s chart. c. Notify the health care provider of the assessment finding. d.Call for the neonatal resuscitation team to attend the infant immediately. ANS: A The normal finding in the umbilical cord is two arteries and one vein. Two vessels may indicate other fetal anomalies. Notation of the finding is the appropriate next step when the finding is expected. The health care provider will need to be notified; however, the infant is the nurse‘s primary concern and must be assessed for abnormalities first. The initial Apgar score is 9, indicating no signs of distress or need of resuscitation. DIF: Cognitive Level: Application OBJ: Nursing Process Step: Assessment MSC: Patient Needs: Health Promotion and Maintenance 15. A pregnant patient asks the nurse how her baby gets oxygen to breathe. What is the nurse‘s best response? a. ―Oxygen-rich blood is delivered through the umbilical vein to the baby.‖ b. ―Take lots of deep breaths because the baby gets all of its oxygen from you.‖ c. ―You don‘t need to be concerned about your baby getting enough oxygen.‖ d. ―The baby‘s lungs are not mature enough to actually breathe, so don‘t worry.‖ ANS: A Oxygen-rich blood travels from the mother‘s circulatory system to the placenta and from the placenta to the umbilical vein (veins carry blood to the heart). From the vein, most of the oxygenated blood travels to the fetal liver or the inferior vena cava. Taking deep breaths can temporarily increase oxygenation but can also lead to increased carbon dioxide retention and dizziness. The patient is asking a normal fetal developmental question often asked by pregnant women. Fetal lungs reach maturity by 37 weeks of gestation, but fetal breathing movements are common. Oxygen transport across lung tissue occurs with the first breath. DIF: Cognitive Level: Application OBJ: Nursing Process Step: Implementation MSC: Patient Needs: Health Promotion and Maintenance MULTIPLE RESPONSE 1. Which physical characteristics decrease as the fetus nears term? (Select all that apply.) a. Vernix caseosa b. Lanugo c. Port wine stain d. Brown fat e. Eyebrows or head hair ANS: A, B Both vernix caseosa and lanugo decrease as the fetus reaches term. Port wine stain is a birthmark and, if present, will be exhibited at or shortly after birth. Brown fat in the fetus will be maintained in order to maintain core temperature. Eyebrows and head hair increase as the fetus nears term. DIF: Cognitive Level: Application OBJ: Nursing Process Step: Assessment MSC: Patient Needs: Health Promotion and Maintenance: Techniques of Physical Assessment 2. Along with gas exchange and nutrient transfer, the placenta produces many hormones necessary for normal pregnancy, including which of the following? (Select all that apply.) a.Insulin b. Estrogen c. Progesterone d. Testosterone e. Human chorionic gonadotropin (hCG) ANS: B, C, E HCG causes the corpus luteum to persist and produce the necessary estrogens and progesterone for the first 6 to 8 weeks. Estrogens cause enlargement of the woman‘s uterus and breasts and growth of the ductal system in the breasts and, as term approaches, plays a role in the initiation of labor. Progesterone causes the endometrium to change, providing early nourishment. Progesterone also protects against spontaneous abortion by suppressing maternal reactions to fetal antigens and reduces unnecessary uterine contractions. Other hormones produced by the placenta include hCT, hCA, and a number of growth factors. Insulin and testosterone are not secreted by the placenta. DIF: Cognitive Level: Understanding OBJ: Nursing Process Step: Assessment MSC: Patient Needs: Health Promotion and Maintenance 3. The nurse is planning a prenatal class on fetal development. Which characteristics of prenatal development should the nurse include for a fetus of 24 weeks, based on fertilization age? (Select all that apply.) a. Ear cartilage firm b. Skin wrinkled and red c. Testes descending toward the inguinal rings d. Surfactant production nears mature levels e. Fetal movement becoming progressively more noticeable ANS: B, C, E A fetus of 24 weeks, based on fertilization age, will have wrinkled and red skin, testes descending toward inguinal rings, and the fetal movement becoming progressively more noticeable. Surfactant production nearing the mature levels does not occur until 32 weeks and ear cartilage is not firm until 38 weeks. DIF: Cognitive Level: Application OBJ: Nursing Process Step: Planning MSC: Patient Needs: Health Promotion and Maintenance 4. The nurse is explaining fetal circulation to a group of nursing students. Which information should be included in the teaching session? (Select all that apply.) a. After birth the ductus venosus remains open, but the other shunts close. b. The foramen ovale shunts blood from the right atrium to the left atrium. c. The ductus venosus shunts blood from the liver to the inferior vena cava. d. The ductus arteriosus shunts blood from the right ventricle to the left ventricle. ANS: B, CThe foramen ovale shunts oxygenated blood from the right atrium to the left atrium, bypassing the lungs. The ductus venosus shunts oxygenated blood from the liver to the inferior vena cava. All shunts close after birth. The ductus arteriosus shunts blood from the right ventricle to the aorta. DIF: Cognitive Level: Application OBJ: Nursing Process Step: Implementation MSC: Patient Needs: Health Promotion and Maintenance 5. A nurse is conducting prenatal education classes for a group of expectant parents. Which information should the nurse include in her discussion of the purpose of amniotic fluid? (Select all that apply.) a. Cushions the fetus b. Protects the skin of the fetus c. Provides nourishment for the fetus d. Allows for buoyancy for fetal movement e. Maintains a stable temperature for the fetus ANS: A, D, E The amniotic fluid provides cushioning for the fetus against impacts to the maternal abdomen. It provides a stable temperature and allows room and buoyancy for fetal movement. Vernix caseosa, the cheeselike coating on the fetus, provides skin protection. The placenta provides nourishment for the fetus. DIF: Cognitive Level: Application OBJ: Nursing Process Step: Implementation MSC: Patient Needs: Health Promotion and Maintenance Chapter 06: Maternal Adaptations to Pregnancy Foundations of Maternal-Newborn & Women‘s Health Nursing, 7th Edition MULTIPLE CHOICE 1. During vital sign assessment of a pregnant patient in her third trimester, the patient complains of feeling faint, dizzy, and agitated. Which nursing intervention is most appropriate? a. Have the patient stand up and retake her blood pressure. b. Have the patient sit down and hold her arm in a dependent position. c. Have the patient turn to her left side and recheck her blood pressure in 5 minutes. d. Have the patient lie supine for 5 minutes and recheck her blood pressure on both arms. ANS: C Blood pressure is affected by positioning during pregnancy. The supine position may cause occlusion of the vena cava and descending aorta. Turning the pregnant woman to a lateral recumbent position alleviates pressure on the blood vessels and quickly corrects supine hypotension. Pressures are significantly higher when the patient is standing. This would cause an increase in systolic and diastolic pressures. The arm should be supported at the same level of the heart. The supine position may cause occlusion of the vena cava and descending aorta, creating hypotension. DIF: Cognitive Level: Analysis OBJ: Nursing Process Step: Implementation MSC: Patient Needs: Physiologic Integrity2. A pregnant woman has come to the emergency department with complaints of nasal congestion and epistaxis. Which is the correct interpretation of these symptoms by the health care provider? a. Nasal stuffiness and nosebleeds are caused by a decrease in progesterone. b. These conditions are abnormal. Refer the patient to an ear, nose, and throat specialist. c. Estrogen relaxes the smooth muscles in the respiratory tract, so congestion and epistaxis are within normal limits. d. Estrogen causes increased blood supply to the mucous membranes and can result in congestion and nosebleeds. ANS: D As capillaries become engorged, the upper respiratory tract is affected by the subsequent edema and hyperemia, which causes these conditions, seen commonly during pregnancy. Progesterone is responsible for the heightened awareness of the need to breathe in pregnancy. Progesterone levels increase during pregnancy. The patient should be reassured that these symptoms are within normal limits. No referral is needed at this time. Relaxation of the smooth muscles in the respiratory tract is affected by progesterone. DIF: Cognitive Level: Analysis OBJ: Nursing Process Step: Planning MSC: Patient Needs: Physiologic Integrity 3. While providing education to a primiparous patient regarding the normal changes of pregnancy, what is an important information for the nurse to share regarding Braxton Hicks contractions? a. These contractions may indicate preterm labor. b. These are contractions that never cause any discomfort. c. Braxton Hicks contractions only start during the third trimester. d. These occur throughout pregnancy, but you may not feel them until the third trimester. ANS: D Throughout pregnancy, the uterus undergoes irregular contractions called Braxton Hicks contractions. During the first two trimesters, the contractions are infrequent and usually not felt by the woman until the third trimester. Braxton Hicks contractions do not indicate preterm labor. Braxton Hicks contractions can cause some discomfort, especially in the third trimester. Braxton Hicks contractions occur throughout the whole pregnancy. DIF: Cognitive Level: Application OBJ: Nursing Process Step: Implementation MSC: Patient Needs: Health Promotion and Maintenance 4. What is the physiologic reason for vascular volume increasing by 40% to 60% during pregnancy? a. Prevents maternal and fetal dehydration b. Eliminates metabolic wastes of the mother c. Provides adequate perfusion of the placenta d. Compensates for decreased renal plasma flowANS: C The primary function of increased vascular volume is to transport oxygen and nutrients to the fetus via the placenta. Preventing maternal and fetal dehydration is not the primary reason for the increase in volume. Assisting with pulling metabolic wastes from the fetus for maternal excretion is one purpose of the increased vascular volume. Renal plasma flow increases during pregnancy. DIF: Cognitive Level: Understanding OBJ: Nursing Process Step: Assessment MSC: Patient Needs: Physiologic Integrity 5. Physiologic anemia often occurs during pregnancy due to a. inadequate intake of iron. b. the fetus establishing iron stores. c. dilution of hemoglobin concentration. d. decreased production of erythrocytes. ANS: C When blood volume expansion is more pronounced and occurs earlier than the increase in red blood cells, the woman will have physiologic anemia, which is the result of dilution of hemoglobin concentration rather than inadequate hemoglobin. Inadequate intake of iron may lead to true anemia. If the woman does not take an adequate amount of iron, true anemia may occur when the fetus pulls stored iron from the maternal system. There is increased production of erythrocytes during pregnancy. DIF: Cognitive Level: Understanding OBJ: Nursing Process Step: Assessment MSC: Patient Needs: Physiologic Integrity 6. Which finding is a positive sign of pregnancy? a. Amenorrhea b. Breast changes c. Fetal movement felt by the woman d. Visualization of fetus by ultrasound ANS: D The only positive signs of pregnancy are auscultation of fetal heart tones, visualization of the fetus by ultrasound, and fetal movement felt by the examiner. Amenorrhea is a presumptive sign of pregnancy. Breast changes are a presumptive sign of pregnancy. Fetal movement is a presumptive sign of pregnancy. DIF: Cognitive Level: Understanding OBJ: Nursing Process Step: Assessment MSC: Patient Needs: Health Promotion and Maintenance 7. A patient in her first trimester complains of nausea and vomiting. The patient asks, ―Why is this happening?‖ What is the nurse‘s best response? a. ―It is due to an increase in gastric motility.‖ b. ―It may be due to changes in hormones.‖ c. ―It is related to an increase in glucose levels.‖ d.―It is caused by a decrease in gastric secretions.‖ ANS: B Nausea and vomiting are believed to be caused by increased levels of hormones, decreased gastric motility, and hypoglycemia. Gastric motility decreases during pregnancy. Glucose levels decrease in the first trimester. Gastric secretions decrease, but this is not the main cause of nausea and vomiting. DIF: Cognitive Level: Application OBJ: Nursing Process Step: Implementation MSC: Patient Needs: Physiologic Integrity 8. The patient has just learned that she is pregnant and overhears the gynecologist saying that she has a positive Chadwick‘s sign. When the patient asks the nurse what this means, how would the nurse respond? a. ―Chadwick‘s sign signifies an increased risk of blood clots in pregnant women because of a congestion of blood.‖ b. ―That sign means the cervix has softened as the result of tissue changes that naturally occur with pregnancy.‖ c. ―This means that a mucus plug has formed in the cervical canal to help protect you from uterine infection.‖ d. ―This sign occurs normally in pregnancy, when estrogen causes increased blood flow in the area of the cervix.‖ ANS: D Increasing levels of estrogen cause hyperemia (congestion with blood) of the cervix, resulting in the characteristic bluish purple color that extends to include the vagina and labia. This discoloration, referred to as Chadwick‘s sign, is one of the earliest signs of pregnancy. Although Chadwick‘s sign occurs with hyperemia (congestion with blood), the sign does not signify an increased risk of blood clots. The softening of the cervix is called Goodell‘s sign, not Chadwick‘s sign. Although the formation of a mucus plug protects from infection, it is not called Chadwick‘s sign. DIF: Cognitive Level: Application OBJ: Nursing Process Step: Assessment MSC: Patient Needs: Health Promotion and Maintenance 9. An expected change during pregnancy is a darkly pigmented vertical midabdominal line. The nurse recognizes this alteration as a. epulis. b. linea nigra. c. melasma. d. striae gravidarum. ANS: B The linea nigra is a dark pigmented line from the fundus to the symphysis pubis. Epulis refers to gingival hypertrophy. Melasma is a different kind of dark pigmentation that occurs on the face. Striae gravidarum (stretch marks) are lines caused by lineal tears that occur in connective tissue during periods of rapid growth. DIF: Cognitive Level: Knowledge OBJ: Nursing Process Step: Assessment MSC: Patient Needs: Health Promotion and Maintenance10. What is the best explanation that the nurse can provide to a patient who is concerned that she has ―pseudoanemia‖ of pregnancy? a. Have her write down her concerns and tell her that you will ask the physician to respond once the lab results have been evaluated. b. Tell her that this is a benign self-limiting condition that can be easily corrected by switching to a high-iron diet. c. Inform her that because of the pregnancy, her blood volume has increased, leading to a substantial dilution effect on her serum blood levels, and that most women experience this condition. d. Contact the physician and get a prescription for iron pills to correct this condition. ANS: C Providing factual information based on physiologic mechanisms is the best option. Although having the patient write down her concerns is reasonable, the nurse should not refer this conversation to the physician but rather address the patient‘s specific concerns. Switching to a high-iron diet will not correct this condition. This physiologic pattern occurs during pregnancy as a result of hemodilution from excess blood volume. Iron medication is not indicated for correction of this condition. There is no need to contact the physician for a prescription. DIF: Cognitive Level: Application OBJ: Nursing Process Step: Implementation MSC: Patient Needs: Physiologic Integrity: Physiologic Adaptation 11. Which physiologic finding is consistent with normal pregnancy? a. Systemic vascular resistance increases as blood pressure decreases. b. Cardiac output increases during pregnancy. c. Blood pressure remains consistent independent of position changes. d. Maternal vasoconstriction occurs in response to increased metabolism. ANS: B Cardiac output increases during pregnancy as a result of increased stroke volume and heart rate. Systemic vascular resistance decreases while blood pressure remains the same. Maternal blood pressure changes in response to patient positioning. In response to increased metabolism, maternal vasodilation is seen during pregnancy. DIF: Cognitive Level: Analysis OBJ: Nursing Process Step: Assessment MSC: Patient Needs: Physiologic Integrity: Physiologic Adaptation 12. A pregnant woman complains of frequent heartburn. The patient states that she has never had these symptoms before and wonders why this is happening now. The most appropriate response by the nurse is to a. examine her dietary intake pattern and tell her to avoid certain foods. b. tell her that this is a normal finding during early pregnancy and will resolve as she gets closer to term. c. explain to the patient that physiologic changes caused by the pregnancy make her more likely to experience these types of symptoms. d.refer her to her health care provider for additional testing because this is an abnormal finding. ANS: C The presentation of heartburn is a normal abnormal finding that can occur in pregnant woman because of relaxation of the lower esophageal sphincter as a result of the physiologic effects of pregnancy. Although foods may contribute to the heartburn, the patient is asking why this presentation is occurring, so the nurse should address the cause first. It is independent of gestation. There is no need to refer to the physician at this time because this is a normal abnormal finding. There is no evidence of complications ensuing from this presentation. DIF: Cognitive Level: Analysis OBJ: Nursing Process Step: Implementation MSC: Patient Needs: Physiologic Integrity: Physiologic Adaptation 13. Which physiologic adaptation of pregnancy may lead to increased constipation during the pregnancy? a. Increased emptying time in the intestines b. Abdominal distention and bloating c. Decreased absorption of water d. Decreased motility in the intestines ANS: D Decreased motility in the intestines leading to increased water absorption would cause constipation. Increased emptying time in the intestines leads to increased nutrient absorption. Abdominal distention and bloating are a result of increased emptying time in the intestines. Decreased absorption of water would not cause constipation. DIF: Cognitive Level: Application OBJ: Nursing Process Step: Assessment MSC: Patient Needs: Physiologic Integrity: Physiologic Adaptation 14. Which physiologic findings related to gallbladder function may lead to the development of gallstones during pregnancy? a. Decrease in alkaline phosphatase levels compared with nonpregnant women b. Increase in albumin and total protein as a result of hemodilution c. Hypertonicity of gallbladder tissue d. Prolonged emptying time ANS: D Prolonged emptying time is seen during pregnancy and may lead to the development of gallstones. In pregnancy, there is a twofold to fourfold time increase in alkaline phosphatase levels as compared with those in nonpregnant woman. During pregnancy, a decrease in albumin level and total protein is seen as a result of hemodilution. Gallbladder tissue becomes hypotonic during pregnancy. DIF: Cognitive Level: Application OBJ: Nursing Process Step: Assessment MSC: Patient Needs: Physiologic Integrity: Physiologic Adaptation 15. Which of these findings would indicate a potential complication related to renal function during pregnancy? a. Increase in glomerular filtration rate (GFR) b. Increase in serum creatinine levelc. Decrease in blood urea nitrogen (BUN) d. Mild proteinuria ANS: B With pregnancy, one would expect the serum creatinine and BUN levels to decrease. An elevation in the serum creatinine level should be investigated. With pregnancy, the GFR increases because of increased renal blood flow and is thus a normal expected finding. A decrease in the blood urea nitrogen level and mild proteinuria is expected findings in pregnancy. DIF: Cognitive Level: Application OBJ: Nursing Process Step: Assessment MSC: Patient Needs: Physiologic Integrity: Physiologic Adaptation 16. A pregnant woman notices that she is beginning to develop dark skin patches on her face. She denies using any different type of facial products as a cleansing solution or makeup. What would the priority nursing intervention be in response to this situation? a. Refer the patient to a dermatologist for further examination. b. Ask the patient if she has been eating different types of foods. c. Take a culture swab and send to the lab for culture and sensitivity (C&S). d. Let the patient know that this is a common finding that occurs during pregnancy. ANS: D This condition is known as chloasma or melasma (mask of pregnancy) and is a result of pigmentation changes relative to hormones. It can be exacerbated by exposure to the sun. There is no need to refer to a dermatologist. Intake of foods is not associated with exacerbation of this process. There is no need for a C&S to be taken. The patient should be assured that this is a normal finding of pregnancy. DIF: Cognitive Level: Application OBJ: Nursing Process Step: Implementation MSC: Patient Needs: Physiologic Integrity: Physiologic Adaptation 17. A patient reports to the clinic nurse that she has not had a period in over 12 weeks, she is tired, and her breasts are sore all of the time. The patient‘s urine test is positive for hCG. What is the correct nursing action related to this information? a. Ask the patient if she has had any nausea or vomiting in the morning. b. Schedule the patient to be seen by a health care provider within the next 4 weeks. c. Send the patient to the maternity screening area of the clinic for a routine ultrasound. d. Determine if there are any factors that might prohibit her from seeking medical care. ANS: D The patient has presumptive and probable indications of pregnancy. However, she has not sought out health care until late in the first or early in the second trimester. The nurse must assess for barriers to seeking health care, physical or emotional, because regular prenatal care is key to a positive pregnancy outcome. Asking if the patient has nausea or vomiting will only add to the list of presumptive signs of pregnancy, and this information will not add to the assessment data to determine whether the patient is pregnant. The patient needs to see a health care provider before the next 4 weeks because she is late in seeking early prenatal care. Ultrasound testing must be prescribed by a health care provider. DIF: Cognitive Level: Analysis OBJ: Nursing Process Step: AnalysisMSC: Patient Needs: Health Promotion and Maintenance 18. Which comment made by a patient in her first trimester indicates ambivalent feelings? a. ―My body is changing so quickly.‖ b. ―I haven‘t felt well since this pregnancy began.‖ c. ―I‘m concerned about the amount of weight I‘ve gained.‖ d. ―I wanted to become pregnant, but I‘m scared about being a mother.‖ ANS: D Ambivalence refers to conflicting feelings. Expressing a concern about being a mother indicates possible ambivalent feelings. Not feeling well since the pregnancy began does not reflect conflicting feelings. The woman is trying to confirm the pregnancy when she is stating the rapid changes to her body. She is not expressing conflicting feelings. By expressing concerns over gaining weight, which is normal, the woman is trying to confirm the pregnancy. DIF: Cognitive Level: Analysis OBJ: Nursing Process Step: Assessment MSC: Patient Needs: Health Promotion and Maintenance 19. A patient who is 7 months pregnant states, ―I‘m worried that something will happen to my baby.‖ Which is the nurse‘s best response? a. ―Your baby is doing fine.‖ b. ―Tell me about your concerns.‖ c. ―There is nothing to worry about.‖ d. ―The doctor is taking good care of you and your baby.‖ ANS: B Encouraging the patient to discuss her feelings is the best approach. The nurse should not disregard or belittle the patient‘s feelings. Responding that your baby is doing fine disregards the patient‘s feelings and treats them as unimportant. Responding that there is nothing to worry about does not answer the patient‘s concerns. Saying that the doctor is taking good care of you and your baby is belittling the patient‘s concerns. DIF: Cognitive Level: Application OBJ: Nursing Process Step: Implementation MSC: Patient Needs: Psychosocial Integrity 20. What is the term for the step in maternal role attainment that relates to the woman giving up certain aspects of her previous life? a. Fantasy b. Grief work c. Role playing d. Looking for a fit ANS: B The woman experiences sadness as she realizes that she must give up certain aspects of her previous self and that she can never go back. This is called grief work. Fantasies allow the woman to try on a variety of possibilities or behaviors. This usually deals with how the child will look and the characteristics of the child. Role playing involves searching for opportunities toprovide care for infants in the presence of another person. Looking for a fit is when the woman observes the behaviors of mothers and compares them with her own expectations. DIF: Cognitive Level: Understanding OBJ: Nursing Process Step: Assessment MSC: Patient Needs: Health Promotion and Maintenance 21. An expectant patient in her third trimester reports that she developed a strong tie to her baby from the beginning and now is really in tune to her baby‘s temperament. The nurse interprets this as the development of which maternal task of pregnancy? a. Learning to give of herself b. Developing attachment with the baby c. Securing acceptance of the baby by others d. Seeking safe passage for herself and her baby ANS: B Developing a strong tie in the first trimester and progressing to be in tune is the process of commitment, attachment, and interconnection with the infant. This stage begins in the first trimester and continues throughout the neonatal period. Learning to give of herself is the task that occurs during pregnancy as the woman allows her body to give space to the fetus. She continues with giving to others in the form of food and presents. Securing acceptance of the baby is a process that continues throughout pregnancy as the woman reworks relationships. Seeking safe passage is the task that ends with birth. During this task, the woman seeks health care and carries out cultural practices. DIF: Cognitive Level: Analysis OBJ: Nursing Process Step: Evaluation MSC: Patient Needs: Health Promotion and Maintenance 22. Which situation best describes a man trying on fathering behaviors? a. Reading books on newborn care b. Spending more time with his siblings c. Coaching a little league baseball team d. Exhibiting physical symptoms related to pregnancy ANS: C Coaching a little league baseball team shows interaction with children and assuming the behavior and role of a father. This best describes a man trying on the role of being a father. Men do not normally read information that is provided in advance. The nurse should be prepared to present information after the baby is born, when it is more relevant. The man will normally seek closer ties with his father. Exhibiting physical symptoms related to pregnancy is called couvade. DIF: Cognitive Level: Understanding OBJ: Nursing Process Step: Assessment MSC: Patient Needs: Health Promotion and Maintenance 23. A 36-year-old divorcee with a successful modeling career finds out that her 18-year-old daughter is expecting her first child. Which is a major factor in determining how this woman will respond to becoming a grandmother? a. Her age b. Her career c.Being divorced d. Age of the daughter ANS: A Age is a major factor in determining the emotional response of prospective grandparents. Young grandparents may not be happy with the stereotype of grandparents as being old. Career responsibilities may have demands that make the grandparents not as accessible but are not a major factor in determining the woman‘s response to becoming a grandmother. Being divorced is not a major factor that determines the adaptation of grandparents. The age of the daughter is not a major factor that determines the adaptation of grandparents. The age of the grandparent is a major factor. DIF: Cognitive Level: Understanding OBJ: Nursing Process Step: Assessment MSC: Patient Needs: Health Promotion and Maintenance 24. Which comment made by a new mother to her own mother is most likely to encourage the grandmother‘s participation in the infant‘s care? a. ―Could you help me with the housework today?‖ b. ―The baby is spitting up a lot. What should I do?‖ c. ―I know you are busy, so I‘ll get John‘s mother to help me.‖ d. ―The baby has a stomachache. I‘ll call the nurse to find out what to do.‖ ANS: B Looking to the grandmother for advice encourages her to become involved in the care of the infant. Housework does not encourage the grandmother to participate in the infant‘s care. Getting John‘s mother to help and calling the nurse about advice excludes the grandmother. DIF: Cognitive Level: Analysis OBJ: Nursing Process Step: Evaluation MSC: Patient Needs: Health Promotion and Maintenance 25. Which comment made by a new mother exhibits understanding of her toddler‘s response to a new sibling? a. ―I can‘t believe he is sucking his thumb again.‖ b. ―He is being difficult and I don‘t have time to deal with him.‖ c. ―When we brought the baby home, we made Michael stop sleeping in the crib.‖ d. ―My husband is going to stay with the baby so I can take Michael to the park tomorrow.‖ ANS: D It is important for a mother to seek time alone with her toddler to reassure him that he is loved. It is normal for a child to regress when a new sibling is introduced into the home. The toddler may have feelings of jealousy and resentment toward the new baby taking attention away from him. Frequent reassurance of parental love and affection is important. Changes in sleeping arrangements should be made several weeks before the birth so the child does not feel displaced by the new baby. DIF: Cognitive Level: Analysis OBJ: Nursing Process Step: Evaluation MSC: Patient Needs: Health Promotion and Maintenance 26. An expectant couple asks the nurse about intercourse during pregnancy and whether it is safe for the baby. What information should the nurse provide?a. Intercourse is safe until the third trimester. b. Safer sex practices should be used once the membranes rupture. c. Intercourse should be avoided if any spotting from the vagina occurs afterward. d. Intercourse and orgasm are often contraindicated if a history of or signs of preterm labor are present. ANS: D Uterine contractions that accompany orgasm can stimulate labor and would be problematic if the woman is at risk for or has a history of preterm labor. Intercourse can continue as long as the pregnancy is progressing normally. Rupture of the membranes may require abstaining from intercourse. Safer sex practices are always recommended. Some spotting can normally occur as a result of the increased fragility and vascularity of the cervix and vagina during pregnancy. DIF: Cognitive Level: Application OBJ: Nursing Process Step: Implementation MSC: Patient Needs: Health Promotion and Maintenance 27. A pregnant patient comes into the medical clinic stating that her family and friends are telling her that she is always talking about the pregnancy and nothing else. She is concerned that something is wrong with her. What psychological behavior is she exhibiting? a. Antepartum obsession b. Ambivalence c. Uncertainty d. Introversion ANS: D The patient is exhibiting behaviors associated with introversion and/or narcissism. These are normal findings during pregnancy as long as they do not become obsessive to the exclusion of everything else. The patient is talking about the pregnancy but there is no evidence that it is affecting her perception of reality and/or ability to perform ADLs. It is normal for pregnant women to focus on the self as being of prime importance in their life initially during the pregnancy. Some women may feel ambivalent about their pregnancy, which is a normal reaction. However, this patient‘s behavior does not support this finding. Some women react with uncertainty at the news of being pregnant, which is a normal reaction. However, this patient‘s behavior does not support this finding. DIF: Cognitive Level: Application OBJ: Nursing Process Step: Assessment MSC: Patient Needs: Psychosocial Integrity 28. A patient relates a story of how her boyfriend is feeling her aches and pains associated with her pregnancy. She is concerned that her boyfriend is making fun of her concerns. How would you respond to this patient statement? a. Tell her not to worry because it is natural for her boyfriend to make her feel better by identifying with her pregnancy. b. Refer the patient to a psychologist for counseling to deal with this problem because it is clearly upsetting her. c.Explain that her boyfriend may be experiencing couvade syndrome and that this is a normal finding seen with male partners. d. Ask the patient specifically to define her concerns related to her relationship with her boyfriend and suggest methods to stop this type of behavior by her significant other. ANS: C Provide factual information that will help reduce stress and modify acceptance. Telling her not to worry does not address the possibility that her boyfriend may be experiencing couvade syndrome. The patient is expressing concern but does not have all the facts related to couvade syndrome and requires education, rather than referral. Couvade syndrome is not an abnormal condition and should be treated with acceptance and understanding. DIF: Cognitive Level: Application OBJ: Nursing Process Step: Implementation MSC: Patient Needs: Psychosocial Integrity MULTIPLE RESPONSE 1. The nurse is assessing a patient in her 37th week of pregnancy for the psychological responses commonly experienced as birth nears. Which psychological responses should the nurse expect to evaluate? (Select all that apply.) a. The patient is excited to see her baby. b. The patient has not started to prepare the nursery for the new baby. c. The patient expresses concern about how to know if labor has started. d. The patient and her spouse are concerned about getting to the birth center in time. e. The patient and her spouse have not discussed how they will share household tasks. ANS: A, C, D As birth nears, the expectant patient will express a desire to see the baby. Most pregnant patients are concerned with their ability to determine when they are in labor. Many couples are anxious about getting to the birth facility in time for the birth. As birth nears, a nesting behavior occurs, which means getting the nursery ready. Not preparing the nursery at this stage is not a response that the nurse should expect to assess. Negotiation of tasks is done during this stage. Discussion regarding the division of household chores is not a response that the nurse should expect to assess at this stage. DIF: Cognitive Level: Analysis OBJ: Nursing Process Step: Evaluation MSC: Patient Needs: Psychosocial Integrity 2. The nurse is teaching a pregnant patient about signs of possible pregnancy complications. Which should the nurse include in the teaching plan? (Select all that apply.) a. Report watery vaginal discharge. b. Report puffiness of the face or around the eyes. c. Report any bloody show when you go into labor. d. Report visual disturbances, such as spots before the eyes. e. Report any dependent edema that occurs at the end of the day. ANS: A, B, DWatery vaginal discharge could mean that the membranes have ruptured. Puffiness of the face or around the eyes and visual disturbances may indicate preeclampsia or eclampsia. These three signs should be reported. Bloody show as labor starts may mean the mucus plug has been expelled. One of the earliest signs of labor may be bloody show, which consists of the mucus plug and a small amount of blood. This is a normal occurrence. Up to 70% of women have dependent edema during pregnancy. This is not a sign of a pregnancy complication. DIF: Cognitive Level: Application OBJ: Nursing Process Step: Implementation MSC: Patient Needs: Physiologic Integrity 3. Which findings are presumptive signs of pregnancy? (Select all that apply.) a. Quickening b. Amenorrhea c. Ballottement d. Goodell‘s sign e. Chadwick‘s sign ANS: A, B, E Quickening, amenorrhea, and Chadwick‘s sign are presumptive signs of pregnancy. Ballottement and Goodell‘s sign are probable signs of pregnancy. DIF: Cognitive Level: Understanding OBJ: Nursing Process Step: Assessment MSC: Patient Needs: Health Promotion and Maintenance COMPLETION 1. The capacity of the uterus in a term pregnancy is how many times its prepregnant capacity? Record your answer as a whole number. times ANS: 500 The prepregnant capacity of the uterus is about 10 mL, and it reaches 5000 mL (5 L) by the end of the pregnancy, which reflects a 500-fold increase. DIF: Cognitive Level: Understanding OBJ: Nursing Process Step: Assessment MSC: Patient Needs: Physiologic Integrity Chapter 07: Antepartum Assessment, Care, and Education Foundations of Maternal-Newborn & Women‘s Health Nursing, 7th Edition MULTIPLE CHOICE 1. Which suggestion is most helpful for the pregnant patient who is experiencing heartburn? a. Drink plenty of fluids at bedtime. b. Eat only three meals a day so the stomach is empty between meals. c. Drink coffee or orange juice immediately on arising in the morning. d. Use Tums or Rolaids to obtain relief, as directed by the health care provider. ANS: D Antacids high in calcium (e.g., Tums, Rolaids) can provide temporary relief. Fluids overstretch the stomach and may precipitate reflux when lying down. Instruct the patient to eat five or six small meals per day rather than three full meals. Coffee and orange juice stimulate acid formation in the stomach and may need to be eliminated from the diet.DIF: Cognitive Level: Understanding OBJ: Nursing Process Step: Implementation MSC: Patient Needs: Physiologic Integrity 2. What is the rationale for a woman in her first trimester of pregnancy to expect to visit her health care provider every 4 weeks? a. Problems can be eliminated. b. She develops trust in the health care team. c. Her questions about labor can be answered. d. The conditions of the expectant mother and fetus can be monitored. ANS: D This routine allows for monitoring maternal health and fetal growth and ensures that problems will be identified early. All problems cannot be eliminated because of prenatal visits; however, they can be identified early. Developing a trusting relationship should be established during these visits, but that is not the primary reason. Most women do not have questions concerning labor until the last trimester of the pregnancy. DIF: Cognitive Level: Understanding OBJ: Nursing Process Step: Assessment MSC: Patient Needs: Health Promotion and Maintenance 3. Which advice to the patient is one of the most effective methods for preventing venous stasis? a. Sit with the legs crossed. b. Rest often with the feet elevated. c. Sleep with the foot of the bed elevated. d. Wear elastic stockings in the afternoon. ANS: B Elevating the feet and legs improves venous return and prevents venous stasis. Sitting with the legs crossed will decrease circulation in the legs and increase venous stasis. Elevating the legs at night may cause pressure on the diaphragm and increase breathing problems. Elastic stockings should be applied before lowering the legs in the morning. DIF: Cognitive Level: Application OBJ: Nursing Process Step: Planning MSC: Patient Needs: Physiologic Integrity 4. What is the gravida and para for a patient who delivered triplets 2 years ago and is now pregnant again? a. 2, 3 b. 1, 2 c. 2, 1 d. 1, 3 ANS: C She has had two pregnancies (gravida 2); para refers to the outcome of the pregnancy rather than the number of infants from that pregnancy. She is pregnant now, so that would make her a gravida 2. She is para 1 because she had one pregnancy that progressed to the age of viability.DIF: Cognitive Level: Analysis OBJ: Nursing Process Step: Assessment MSC: Patient Needs: Health Promotion and Maintenance 5. A patient, gravida 2, para 1, comes for a prenatal visit at 20 weeks of gestation. Her fundus is palpated 3 cm below the umbilicus. This finding is a. appropriate for gestational age. b. a sign of impending complications. c. lower than normal for gestational age. d. higher than normal for gestational age. ANS: C By 20 weeks, the fundus should reach the umbilicus. The fundus should be at the umbilicus at 20 weeks, so 3 cm below the umbilicus is an inappropriate height and needs further assessment. This is lower than expected at this date. It may be a complication, but it may also be because of incorrect dating of the pregnancy. DIF: Cognitive Level: Analysis OBJ: Nursing Process Step: Assessment MSC: Patient Needs: Health Promotion and Maintenance 6. Which complaint made by a patient at 35 weeks of gestation requires additional assessment? a. Abdominal pain b. Ankle edema in the afternoon c. Backache with prolonged standing d. Shortness of breath when climbing stairs ANS: A Abdominal pain at 35 weeks gestation may indicate preeclampsia, or abruptio placentae. Ankle edema in the afternoon is a normal finding at this stage of the pregnancy. Backaches while standing is a normal finding in the later stages of pregnancy. Shortness of breath is an expected finding at 35 weeks. DIF: Cognitive Level: Analysis OBJ: Nursing Process Step: Assessment MSC: Patient Needs: Physiologic Integrity 7. A gravida 1 patient at 32 weeks of gestation reports that she has severe lower back pain. What should the nurse‘s assessment include? a. Palpation of the lumbar spine b. Exercise pattern and duration c. Observation of posture and body mechanics d. Ability to sleep for at least 6 hours uninterrupted ANS: C Correct posture and body mechanics can reduce lower back pain caused by increasing lordosis. Pregnancy should not cause alterations in the spine. Any assessment for malformation should be done early in pregnancy. Certain exercises can help relieve back pain. Rest is important for overall well-being; however, the primary concern related to back pain is a thorough evaluation of posture and body mechanics.DIF: Cognitive Level: Application OBJ: Nursing Process Step: Assessment MSC: Patient Needs: Health Promotion and Maintenance 8. Which laboratory result would be a cause for concern if exhibited by a patient at her first prenatal visit during the second month of her pregnancy? a. Rubella titer, 1:6 b. Platelets, 300,000/mm3 c. White blood cell count, 6000/mm3 d. Hematocrit 38%, hemoglobin 13 g/dL ANS: A A rubella titer of less than 1:8 indicates a lack of immunity to rubella, a viral infection that has the potential to cause teratogenic effects on fetal development. Arrangements should be made to administer the rubella vaccine after birth during the postpartum period because administration of rubella, a live vaccine, would be contraindicated during pregnancy. Women receiving the vaccine during the postpartum period should be cautioned to avoid pregnancy for 3 months. The lab values for WBCs, platelets, and hematocrit/hemoglobin are within the expected range for pregnant women. DIF: Cognitive Level: Analysis OBJ: Nursing Process Step: Assessment MSC: Patient Needs: Physiologic Integrity 9. A patient in her third trimester of pregnancy is asking about safe travel. Which statement should the nurse provide regarding safe travel during pregnancy? a. ―Only travel by car during pregnancy.‖ b. ―Avoid use of the seat belt during the third trimester.‖ c. ―You can travel by plane until your 38th week of gestation.‖ d. ―If you are traveling by car stop to walk every 1 to 2 hours.‖ ANS: D Car travel is safe during normal pregnancies. Suggest that the woman stop to walk every 1 to 2 hours so she can empty her bladder. Walking also helps decrease the risk of thrombosis that is elevated during pregnancy. Seat belts should be worn throughout the pregnancy. Instruct the woman to fasten the seat belt snugly, with the lap belt under her abdomen and across her thighs and the shoulder belt in a diagonal position across her chest and above the bulge of her uterus. Travel by plane is generally safe up to 36 weeks if there are no complications of the pregnancy, so only traveling by car is an inaccurate statement. DIF: Cognitive Level: Application OBJ: Nursing Process Step: Assessment MSC: Patient Needs: Health Promotion and Maintenance 10. When a pregnant woman develops ptyalism, which guidance should the nurse provide? a. Chew gum or suck on lozenges between meals. b. Eat nutritious meals that provide adequate amounts of essential vitamins and minerals. c. Take short walks to stimulate circulation in the legs and elevate the legs periodically. d. Use pillows to support the abdomen and back during sleep.ANS: A Some women experience ptyalism, or excessive salivation. The cause of ptyalism may be decreased swallowing associated with nausea or stimulation of the salivary glands by the ingestion of starch. Small frequent meals and use of chewing gum and oral lozenges offer limited relief for some women. All other options include recommendations for pregnant women; however, they do not address ptyalism. DIF: Cognitive Level: Application OBJ: Nursing Process Step: Implementation MSC: Patient Needs: Physiologic Integrity: Basic Care and Comfort 11. When documenting a patient encounter, which term will the nurse use to describe the woman who is in the 28th week of her first pregnancy? a. Multigravida b. Multipara c. Nullipara d. Primigravida ANS: D A primigravida is a woman pregnant for the first time. A multigravida has been pregnant more than once. A nullipara is a woman who has never been pregnant or has not completed a pregnancy of 20 weeks or more. A primipara has delivered one pregnancy of at least 20 weeks. A multipara has delivered two or more pregnancies of at least 20 weeks. DIF: Cognitive Level: Knowledge OBJ: Nursing Process Step: Implementation MSC: Patient Needs: Health Promotion and Maintenance 12. You are performing assessments for an obstetric patient who is 5 months pregnant with her third child. Which finding would cause you to suspect that the patient was at risk? a. Patient states that she doesn‘t feel any Braxton Hicks contractions like she had in her prior pregnancies. b. Fundal height is below the umbilicus. c. Cervical changes, such as Goodell‘s sign and Chadwick‘s sign, are present. d. She has increased vaginal secretions. ANS: B Based on gestational age (20 weeks), the fundal height should be at the umbilicus. This finding is abnormal and warrants further investigation about potential risk. With subsequent pregnancies, multiparas may not perceive Braxton Hicks contractions as being evident compared with their initial pregnancy. Cervical changes such as Goodell‘s and Chadwick‘s signs should be present and are considered a normal finding. Increased vaginal secretions are normal during pregnancy as a result of increased vascularity. DIF: Cognitive Level: Application OBJ: Nursing Process Step: Assessment MSC: Patient Needs: Physiologic Integrity: Reduction of Risk Potential 13. Determine the obstetric history of a patient in her fifth pregnancy who has had two spontaneous abortions in the first trimester, one infant at 32 weeks‘ gestation, and one infant at 38 weeks‘ gestation. a. G5 T1 P2 A2 L 2 b.G5 T1 P1 A1 L2 c. G5 T0 P2 A2 L2 d. G5 T1 P1 A2 L2 ANS: D This patient is in her fifth pregnancy, which is G5, she had one viable term infant (between 38 and 42 weeks‘ gestation), which is T1, she had one viable preterm infant (between 20 and 37 weeks‘ gestation), which is P1, two spontaneous abortions (before 20 weeks‘ gestation), which is A2, and she has two living children, which is L2. DIF: Cognitive Level: Application OBJ: Nursing Process Step: Analysis MSC: Patient Needs: Health Promotion and Maintenance 14. Use Nägele‘s rule to determine the EDD (estimated day of birth) for a patient whose last menstrual period started on April 12. a. February 19 b. January 19 c. January 21 d. February 7 ANS: B Nägele‘s rule subtracts 3 months from the month of the last menstrual period (month 4 – 3 = January) and adds 7 days to the day that the last menstrual period started (April 12 + 7 days = April 19), so the correct answer is January 19 of the following calendar year. DIF: Cognitive Level: Application OBJ: Nursing Process Step: Analysis MSC: Patient Needs: Health Promotion and Maintenance 15. Which of the patient health behaviors in the first trimester would the nurse identify as a risk factor in pregnancy? a. Sexual intercourse two or three times weekly b. Moderate exercise for 30 minutes daily c. Working 40 hours a week as a secretary in a travel agency d. Relaxing in a hot tub for 30 minutes a day, several days a week ANS: D Pregnant women should avoid activities that might cause hyperthermia. Maternal hyperthermia, particularly during the first trimester, may be associated with fetal anomalies. She should not be in a hot tub for more than 10 minutes at less than 100F. Sexual intercourse is generally safe for the healthy pregnant woman; moderate exercise during pregnancy can strengthen muscles, reduce backache and stress, and provide a feeling of well-being; working during pregnancy is acceptable as long as the woman is not continually on her feet or exposed to environmental toxins and industrial hazards. DIF: Cognitive Level: Analysis OBJ: Nursing Process Step: Assessment MSC: Patient Needs: Physiologic Integrity 16. A patient who smokes one pack of cigarettes daily has a positive pregnancy test. The nurse will explain that smoking during pregnancy increases the risk of which condition? a.Congenital anomalies b. Death before or after birth c. Neonatal hypoglycemia d. Neonatal withdrawal syndrome ANS: B Smoking during pregnancy increases the risk for spontaneous abortion, low birth weight, abruptio placentae, placenta previa, preterm birth, perinatal mortality, and SIDS. Smoking does not appear to cause congenital anomalies, hypoglycemia, or withdrawal syndrome. DIF: Cognitive Level: Understanding OBJ: Nursing Process Step: Implementation MSC: Patient Needs: Physiologic Integrity: Reduction of Risk Potential 17. A patient with an IUD in place has a positive pregnancy test. When planning care, the nurse will base decisions on which anticipated action? a. A therapeutic abortion will need to be scheduled since fetal damage is inevitable. b. Hormonal analyses will be done to determine the underlying cause of the false-positive test result. c. The IUD will need to be removed to avoid complications such as miscarriage or infection. d. The IUD will need to remain in place to avoid injuring the fetus. ANS: C Pregnancy with an intrauterine device (IUD) in place is unusual; however, it can occur and cause complications such as spontaneous abortion and infection. A therapeutic abortion is not indicated unless infection occurs. DIF: Cognitive Level: Understanding OBJ: Nursing Process Step: Planning MSC: Patient Needs: Physiologic Integrity: Reduction of Risk Potential 18. The health care provider reports that the primigravida‘s fundus can be palpated at the umbilicus. Which priority question will the nurse include in the patient‘s assessment? a. ―Have you noticed that it is easier for you to breathe now?‖ b. ―Would you like to hear the baby‘s heartbeat for the first time?‖ c. ―Have you felt a fluttering sensation in your lower pelvic area yet?‖ d. ―Have you recently developed any unusual cravings, such as for chalk or dirt?‖ ANS: C Quickening is the first maternal sensation of fetal movement and is often described as a fluttering sensation. Quickening is detected at approximately 20 weeks in the primigravida and as early as 16 weeks in the multigravida. The fundus is at the umbilicus at 20 weeks‘ gestation. Lightening is associated with descent of the fetal head into the maternal pelvis and is associated with improved lung expansion. Lightening occurs approximately 2 weeks before birth in the primipara. Fetal heart tones can be detected by Doppler as early as 9 to 12 weeks of gestation. Pica is the craving for nonnutritive substances such as chalk, dirt, clay, or sand. It can develop at any time during pregnancy. It can be associated with malnutrition and the health care provider should monitor the patient‘s hematocrit/hemoglobin, zinc, and iron levels. DIF: Cognitive Level: Application OBJ: Nursing Process Step: AnalysisMSC: Patient Needs: Health Promotion and Maintenance 19. The nurse is scheduling the next appointment for a healthy primigravida currently at 28 weeks gestation. When will the nurse schedule the next prenatal visit? a. 1 week b. 2 weeks c. 3 weeks d. 4 weeks ANS: B From 29 to 36 weeks, routine prenatal assessment is every 2 weeks. If the pregnancy is high risk, the patient will see the health care provider more frequently. DIF: Cognitive Level: Understanding OBJ: Nursing Process Step: Implementation MSC: Patient Needs: Health Promotion and Maintenance 20. Which information is covered by early pregnancy classes offered in the first and second trimesters? a. Methods of pain relief b. The phases and stages of labor c. Coping with common discomforts of pregnancy d. Prebirth and postbirth care of a patient having a cesarean birth ANS: C Early pregnancy classes focus on the first two trimesters and cover information on adapting to pregnancy, dealing with early discomforts, and understanding what to expect in the months ahead. Methods of pain relief are discussed in a childbirth preparation class. The phases and stages of labor are usually covered in a childbirth preparation class. Cesarean birth preparation classes discuss prebirth and postbirth of a patient having a cesarean birth. DIF: Cognitive Level: Understanding OBJ: Nursing Process Step: Assessment MSC: Patient Needs: Health Promotion and Maintenance 21. Which is the method of childbirth that helps prevent the fear-tension-pain cycle by using slow abdominal breathing in early labor and rapid chest breathing in advanced labor? a. Bradley b. Lamaze c. Leboyer d. Dick-Read ANS: D The Dick-Read method helps prevent the fear-tension-pain cycle by using slow abdominal breathing in early labor and rapid chest breathing in advanced labor. The Lamaze method involves concentration and conditioning to help the woman respond to contractions with relaxation to decrease pain. Viewing childbirth as a traumatic experience, the Leboyer method uses decreased light and noise to help the newborn adapt to extrauterine life more easily. TheBradley method teaches women to use abdominal muscles to increase relaxation and breath control; it emphasizes avoidance of all medications and interventions. DIF: Cognitive Level: Understanding OBJ: Nursing Process Step: Assessment MSC: Patient Needs: Health Promotion and Maintenance 22. Which type of cutaneous stimulation involves massage of the abdomen? a. Imagery b. Effleurage c. Mental stimulation d. Thermal stimulation ANS: B Effleurage is massage usually performed on the abdomen during contractions. Imagery exercises enhance relaxation by teaching the woman to imagine herself in a relaxing setting. Mental stimulation is a group of methods to decrease pain by increasing mental stimulation. Thermal stimulation decreases pain by using applications of heat and cold. DIF: Cognitive Level: Understanding OBJ: Nursing Process Step: Assessment MSC: Patient Needs: Physiologic Integrity 23. What does a birth plan help the parents accomplish? a. Avoidance of an episiotomy b. Determining the outcome of the birth c. Assuming complete control of the situation d. Taking an active part in planning the birth experience ANS: D The birth plan helps the woman and her partner look at the available options and plan the birth experience to meet their personal needs. A birth plan cannot dictate the need for or avoidance of an episiotomy. The outcome of the birth is not an absolute determinant. A birth plan does not assume complete control of the situation; it allows for expanding communication. Parents who prepare a birth plan should be educated that flexibility is essential as each labor and delivery is unique and may present unexpected complications. DIF: Cognitive Level: Understanding OBJ: Nursing Process Step: Assessment MSC: Patient Needs: Health Promotion and Maintenance 24. A relaxation technique that can be used during the childbirth experience to decrease maternal pain perception is a. using increased environmental stimulation as a method of distraction. b. restricting family and friends from visiting during the labor period to keep the patient focused on breathing techniques. c. medicating the patient frequently to reduce pain perception. d. assisting the patient in breathing methods aimed at taking control of pain perception based on the contraction pattern. ANS: DRelaxation techniques are aimed at incorporating mind and body activities to maintain control over pain. Additional environmental stimuli may have the opposite effect and increase patient anxiety, which will affect pain perception. Restricting visitors may have the opposite effect, leading to increased anxiety because of isolation. Medicating a patient may not decrease pain perception but may place the patient at risk for adverse reactions and/or complications of pregnancy related to medications. DIF: Cognitive Level: Analysis OBJ: Nursing Process Step: Planning MSC: Patient Needs: Psychosocial Integrity: Therapeutic Communication 25. Which technique would provide the best pain relief for a pregnant woman with an occiput posterior position? a. Neuromuscular disassociation b. Effleurage c. Psychoprophylaxis d. Sacral pressure ANS: D The use of sacral pressure may provide relief for patients who are experiencing back labor. The presentation of the fetus in a posterior position indicates this. Neuromuscular dissociation is used as a conditioned response to affect pain relief based on the mother tensing one group of muscles and focusing on releasing tension in the rest of her body. Effleurage is the process of using circular massage to effect pain relief. Psychoprophylaxis is another name for the Lamaze method of prepared childbirth. DIF: Cognitive Level: Analysis OBJ: Nursing Process Step: Planning MSC: Patient Needs: Psychosocial Integrity: Sensory Perceptual Alterations 26. The labor nurse is reviewing breathing techniques with a primiparous patient admitted for induction of labor. When is the best time to encourage the laboring patient to use slow, deep chest breathing with contractions? a. During labor, when she can no longer talk through contractions b. During the first stage of labor, when the contractions are 3 to 4 minutes apart c. Between contractions, during the transitional phase of the first stage of labor d. Between her efforts to push to facilitate relaxation between contractions ANS: A Focused breathing techniques should not be used in labor until they are actually needed, which is usually when the woman can no longer walk and talk during a contraction. If breathing techniques are used too early, the woman tends to move through the different techniques too quickly, and she may stop using them. In addition, the use of the more complex breathing patterns in latent labor may increase fatigue. Women should be encouraged to adapt the techniques to their own comfort and needs. Breathing deeply between contractions or pushing can increase the possibility of carbon dioxide retention and make the patient dizzy. DIF: Cognitive Level: Application OBJ: Nursing Process Step: Planning MSC: Patient Needs: Health Promotion and Maintenance 27. In a prenatal education class, the nurse is reviewing the importance of using relaxation techniques during labor. Which patient statement will the nurse need to correct? a.―We will practice relaxation techniques only in a quiet setting so I can focus.‖ b. ―Relaxation is important during labor because it will help me conserve my energy.‖ c. ―If I relax in between contractions, my baby will get more oxygen during labor.‖ d. ―My partner and I will practice relaxation throughout the remainder of my pregnancy.‖ ANS: A Relaxation exercises must be practiced frequently to be useful during labor. Couples begin practice sessions in a quiet, comfortable setting. Later, they practice in other places that simulate the noise and unfamiliar setting of the hospital. The ability to relax during labor is an important component of coping effectively with childbirth. Relaxation conserves energy, decreases oxygen use, and enhances other pain relief techniques. Women learn exercises to help them recognize and release tension. The labor partner assists the woman by providing feedback during exercise sessions and labor. DIF: Cognitive Level: Application OBJ: Nursing Process Step: Evaluation MSC: Patient Needs: Health Promotion and Maintenance MULTIPLE RESPONSE 1. A pregnant patient reports that she works in a long-term care setting and is concerned about the impending flu season. She asks about receiving the flu vaccine. As the nurse, you are aware that some immunizations are safe to administer during pregnancy, whereas others are not. Which vaccines could this patient receive? (Select all that apply.) a. Tetanus b. Varicella c. Influenza d. Hepatitis A and B e. Measles, mumps, rubella (MMR) ANS: A, C, D Inactivated vaccines such as those for tetanus, hepatitis A, hepatitis B, and influenza are safe to administer to women who have a risk for contracting or developing the disease. Immunizations with live virus vaccines such as MMR, varicella (chickenpox), or smallpox are contraindicated during pregnancy because of the possible teratogenic effects on the fetus. DIF: Cognitive Level: Application OBJ: Nursing Process Step: Evaluation MSC: Patient Needs: Health Promotion and Maintenance 2. The nurse is planning care for a patient in her first trimester of pregnancy. The patient is experiencing nausea and vomiting. Which interventions should the nurse plan to share with this patient? (Select all that apply.) a. Suck on hard candy. b. Take prenatal vitamins in the morning. c. Try some herbal tea to relieve the nausea. d. Drink fluids frequently but separate from meals. e.Eat crackers or dry cereal before arising in the morning. ANS: A, D, E A patient experiencing nausea and vomiting should be taught to suck on hard candy, drink fluids frequently but separately from meals, and eat crackers, dry toast, or dry cereal before arising in the morning. Prenatal vitamins should be taken at bedtime because they may increase nausea if taken in the morning. Before taking herbal tea, the patient should check with her health care provider. DIF: Cognitive Level: Application OBJ: Nursing Process Step: Planning MSC: Patient Needs: Physiologic Integrity 3. Which factors contribute to the presence of edema in the pregnant patient? (Select all that apply.) a. Diet consisting of processed foods b. Hemoconcentration c. Increase in colloid osmotic pressure d. Last trimester of pregnancy e. Decreased venous return ANS: A, D, E Processed foods, which are high in sodium content, can contribute to edema formation. As the pregnancy progresses, because of the weight of the uterus, compression takes place, leading to decreased venous return and an increase in edema formation. A decrease in colloid osmotic pressure would contribute to edema formation and fluid shifting. Hemodilution would also lead to edema formation. DIF: Cognitive Level: Analysis OBJ: Nursing Process Step: Assessment MSC: Patient Needs: Physiologic Integrity: Physiologic Adaptation 4. The prenatal nurse educator is teaching couples the technique of applying sacral pressure during labor. Which should be included in the teaching session? (Select all that apply.) a. The technique can be combined with heat to the area. b. A jiggling motion should be used while applying the pressure. c. Tennis balls may be used to apply the pressure to the sacral area. d. The pressure against the sacrum should be intermittent during the contraction. e. The hand may be moved slowly or remain positioned directly over the sacrum. ANS: A, C, E Sacral pressure can be combined with thermal stimulation to increase effectiveness. The hand may be moved slowly over the area or remain positioned directly over the sacrum, but pressure should be continuous and firm throughout the contraction. Care should be taken not to jiggle the woman, which may be irritating. DIF: Cognitive Level: Application OBJ: Nursing Process Step: Implementation MSC: Patient Needs: Physiologic Integrity COMPLETION1. Calculate the estimated date of birth (EDD) in October using Nägele‘s rule for a patient whose last normal menstrual period (LNMP) began on January 1. Record your answer as a whole number. ANS: 8 Nägele‘s rule is often used to establish the EDD. This method involves subtracting 3 months from the date that the LNMP began, adding 7 days, and then correcting the year, if appropriate. Subtracting 3 months from January 1 gives you the month of October and adding 7 days = October 8. DIF: Cognitive Level: Analysis OBJ: Nursing Process Step: Evaluation MSC: Patient Needs: Health Promotion and Maintenance Chapter 08: Nutrition for Childbearing Foundations of Maternal-Newborn & Women‘s Health Nursing, 7th Edition MULTIPLE CHOICE 1. When planning a healthy diet with a pregnant patient, what should the nurse‘s first action be? a. Teach the patient about MyPlate. b. Review the patient‘s current dietary intake. c. Instruct the patient to limit the intake of fatty foods. d. Caution the patient to avoid large doses of vitamins, especially those that are fat-soluble. ANS: B The first action should be to assess the patient‘s current dietary pattern and practices because instruction should be geared to what she already knows and does. Teaching the food guide MyPlate is important but not the first action when planning a diet with a pregnant patient. Limiting intake of fatty foods is important in a pregnant patient‘s diet; however, not the first action. Caution regarding about excessive fat-soluble vitamins is important; however, not the first action. DIF: Cognitive Level: Application OBJ: Nursing Process Step: Planning MSC: Patient Needs: Physiologic Integrity 2. The nurse is meeting with a patient with an elevated BMI regarding an optimal diet for pregnancy. Which major source of nutrients should be a significant component of this patient‘s diet? a. Fats b. Fiber c. Simple sugars d. Complex carbohydrates ANS: D Complex carbohydrates supply the pregnant woman with vitamins, minerals, and fiber. Fats provide 9 calories in each gram, in contrast to carbohydrates and proteins, which provide only 4 calories in each gram. Fiber is supplied primarily by complex carbohydrates. The most common simple carbohydrate is table sugar, which is a source of energy but does not provide any nutrients. DIF: Cognitive Level: Application OBJ: Nursing Process Step: ImplementationMSC: Patient Needs: Health Promotion and Maintenance 3. In order to increase the absorption of iron by a pregnant patient, which beverage should an iron preparation be given with? a. Tea b. Milk c. Coffee d. Orange juice ANS: D Vitamin C source may increase the absorption of iron and would be the optimal choice. Tannin in the tea reduces the absorption of iron. The calcium and phosphorus in milk decrease iron absorption. Decreased intake of caffeine is recommended during pregnancy. DIF: Cognitive Level: Application OBJ: Nursing Process Step: Implementation MSC: Patient Needs: Physiologic Integrity 4. When should iron supplementation during a normal pregnancy begin? a. Before pregnancy b. In the first trimester c. In the third trimester d. In the second trimester ANS: D Vitamin supplements should be prescribed in the second trimester, when the need for iron is increased. Healthy young women do not usually need iron supplementation for their diets. Morning sickness in the first trimester increases the routine side effects of iron supplements. The iron supplements may continue to be prescribed in the third trimester and during the postpartum period. DIF: Cognitive Level: Understanding OBJ: Nursing Process Step: Assessment MSC: Patient Needs: Physiologic Integrity 5. A patient in her fifth month of pregnancy asks the nurse, ―How many more calories should I be eating daily?‖ What is the correct response by the nurse? a. 180 more calories a day b. 340 more calories a day c. 452 more calories a day d. 500 more calories a day ANS: B The increased nutritional needs of pregnancy can be met with an additional 340 calories per day. 180 calories are not enough to meet the increased nutritional needs of pregnancy. 452 calories are more than the recommended calories for pregnancy at this gestation. A patient in her third trimester would increase her energy intake by 452 calories per day. 500 calories are more than the recommended calories for pregnancy. DIF: Cognitive Level: Application OBJ: Nursing Process Step: ImplementationMSC: Patient Needs: Health Promotion and Maintenance 6. A pregnant patient‘s diet may not meet her need for folate. Which food choice is an excellent source of this nutrient? a. Chicken b. Cheese c. Potatoes d. Green leafy vegetables ANS: D Sources of folate include green leafy vegetables, whole grains, fruits, liver, dried peas, and beans. Chicken is a good source of protein, but poor in folate. Cheese is an excellent source of calcium, but poor in folate. Potatoes contain carbohydrates and vitamins but are poor in folate. DIF: Cognitive Level: Understanding OBJ: Nursing Process Step: Evaluation MSC: Patient Needs: Health Promotion and Maintenance 7. A pregnant patient asks the nurse if she can double her prenatal vitamin dose because she does not like to eat vegetables. What is the nurse‘s response regarding the danger of taking excessive vitamins? a. Increases caloric intake b. Has toxic effects on the fetus c. Increases absorption of all vitamins d. Promotes development of pregnancy-induced hypertension (PIH) ANS: B The use of vitamin supplements in addition to food may increase the intake of some nutrients to doses much higher than the recommended amounts. Overdoses of some vitamins have been linked to fetal defects. Vitamin supplements do not contain calories. Vitamin supplements do not have better absorption than natural vitamins and minerals. There is no relationship between vitamin supplements and PIH. DIF: Cognitive Level: Application OBJ: Nursing Process Step: Implementation MSC: Patient Needs: Physiologic Integrity 8. The nurse is conducting a prenatal nutrition education class for a group of nursing students. Which statement best describes the condition known as pica? a. Iron-deficiency anemia b. Intolerance to milk products c. Ingestion of nonfood substances d. Episodes of anorexia and vomiting ANS: C The practice of eating substances not normally thought of as food is called pica. Clay, dirt, and solid laundry starch are the substances most commonly ingested. Pica may produce irondeficiency anemia if proper nutrition is decreased. Intolerance to milk products is termed lactose intolerance. Pica is not related to anorexia and vomiting.DIF: Cognitive Level: Application OBJ: Nursing Process Step: Implementation MSC: Patient Needs: Physiologic Integrity 9. Which effect is a common response to both smoking and cocaine use in the pregnant patient? a. Vasoconstriction b. Increased appetite c. Increased metabolism d. Changes in insulin metabolism ANS: A Both smoking and cocaine use cause vasoconstriction, which results in impaired placental blood flow to the fetus. Smoking and cocaine use do not increase appetite, change insulin metabolism, or increase metabolism. DIF: Cognitive Level: Understanding OBJ: Nursing Process Step: Assessment MSC: Patient Needs: Health Promotion and Maintenance: Physiologic Integrity 10. Which is the most important reason for evaluating the pattern of weight gain in pregnancy? a. Prevents excessive adipose tissue deposits b. Determines cultural influences on the woman‘s diet c. Assesses the need to limit caloric intake in obese women d. Identifies potential nutritional problems or complications of pregnancy ANS: D Deviations from the recommended pattern of weight gain may indicate nutritional problems or developing complications. Excessive adipose tissue may occur with excess weight gain but is not the reason for monitoring the weight gain pattern. The pattern of weight gain is not affected by cultural influences. It is important to monitor the pattern of weight gain for the developing complications. DIF: Cognitive Level: Understanding OBJ: Nursing Process Step: Assessment MSC: Patient Needs: Physiologic Integrity 11. A pregnant patient would like to know which foods, other than dairy products, contain the most calcium. Which food group would the nurse recommend? a. Legumes b. Lean meat c. Whole grains d. Yellow vegetables ANS: A Although dairy products contain the greatest amount of calcium, it can also be found in legumes, nuts, dried fruits, and some dark green leafy vegetables. Lean meats are rich in protein and phosphorus. Whole grains are rich in zinc and magnesium. Yellow vegetables are rich in vitamin A. DIF: Cognitive Level: Application OBJ: Nursing Process Step: ImplementationMSC: Patient Needs: Physiologic Integrity 12. To determine cultural influences on a patient‘s diet, what is the nurse‘s primary action? a. Evaluate the patient‘s weight gain during pregnancy. b. Assess the socioeconomic status of the patient. c. Discuss the four food groups with the patient. d. Identify the food preferences and methods of food preparation common to the patient‘s culture. ANS: D Understanding the patient‘s food preferences and how she prepares food will assist the nurse in determining whether the patient‘s culture is adversely affecting her nutritional intake. Evaluating a patient‘s weight gain during pregnancy should be included for all patients, not just for those who are culturally different. The socioeconomic status of the patients may alter the nutritional intake, but not the cultural influence. Teaching the food groups to the patient should come after assessing food preferences. DIF: Cognitive Level: Application OBJ: Nursing Process Step: Assessment MSC: Patient Needs: Physiologic Integrity 13. In teaching a pregnant adolescent about nutrition, what should the nurse include in the care plan? a. Determine the weight gain needed to meet adolescent growth and add 35 lb. b. Suggest that she does not eat at fast food restaurants to avoid foods of poor nutritional value. c. Realize that most adolescents are unwilling to make dietary changes during pregnancy. d. Emphasize the need to eliminate common teen snack foods because they are too high in fat and sodium. ANS: A Adolescents should gain in the upper range of the recommended weight gain. They also need to gain weight that would be expected for their own normal growth. Adolescents are willing to make changes; however, they still need to be like their peers. Eliminating fast foods will make her appear different from her peers. She should be taught to choose foods that add needed nutrients. Changes in the diet should be kept at a minimum and snacks should be included. Snack foods can be included in moderation and other foods added to make up for the lost nutrients. DIF: Cognitive Level: Application OBJ: Nursing Process Step: Implementation MSC: Patient Needs: Health Promotion and Maintenance 14. The traditional diet of Asian women includes little meat and few dairy products and may be low in calcium and iron. The nurse can assist a patient increase her intake of these foods by which action? a. Suggest that she eat more tofu, bok choy, and broccoli. b. Suggest that she eat more hot foods during pregnancy. c. Emphasize the need for increased milk intake during pregnancy. d. Tell her husband that she must increase her intake of fruits and vegetables for the baby‘s sake.ANS: A The diet should be improved by increasing foods acceptable to the woman. These foods are common in the Asian diet and are good sources of calcium and iron. Pregnancy is considered hot; therefore the woman would eat cold foods. Because milk products are not part of this woman‘s diet, it should be respected and other alternatives offered. Also, lactose intolerance is common. Fruits and vegetables are cold foods and should be included in the diet. In regard to the family dynamics, however, the husband does not dictate to the wife in this culture. DIF: Cognitive Level: Application OBJ: Nursing Process Step: Implementation MSC: Patient Needs: Physiologic Integrity 15. Which patient would require additional calories and nutrients? a. A 36-year-old female gravida 2, para 1, in her first trimester of pregnancy b. An 18-year-old female who delivered a 7-lb baby and is bottle feeding c. A 23-year-old female who had a cesarean birth and is bottle feeding d. A 20-year-old female who had a vaginal birth 5 months ago and is breastfeeding ANS: D A patient who is breastfeeding will require more calories and nutrients than women who are pregnant. The type of birth has no impact on nutrient intake. A patient who is bottle-feeding does not require additional calories. DIF: Cognitive Level: Application OBJ: Nursing Process Step: Assessment MSC: Patient Needs: Physiologic Integrity: Basic Care and Comfort 16. A patient postdelivery is concerned about getting back to her prepregnancy weight as soon as possible. She had only gained 15 lb during her pregnancy. Which assessment factor would be of concern at her 6-week postpartum checkup? a. Patient has lost 30 lb during the 6-week period prior to her scheduled checkup. b. Patient states that she is eating healthy and limiting intake of processed foods. c. Patient relates increased consumption of fruits and vegetables in her diet postbirth. d. Patient has resumed her usual exercise pattern of walking around the neighborhood for 10 minutes each night. ANS: A Although a certain amount of weight loss is expected in the postpartum period, the fact that the reported weight loss is double the amount of weight gained during the pregnancy places the patient at risk for malnutrition. Further inquiry is needed. Limiting the intake of processed foods is a healthy dietary alternative to decreasing sodium intake. Increases in fruits and vegetables are a healthy dietary alternative to decrease possible occurrence of hypertension. An exercise program is part of a healthy nutrition approach. DIF: Cognitive Level: Application OBJ: Nursing Process Step: Assessment MSC: Patient Needs: Health Promotion and Maintenance 17. Which clinical finding is associated with inadequate maternal weight gain during pregnancy? a. Prolonged labor b. Preeclampsia c.Gestational diabetes d. Low-birth-weight infant ANS: D Inadequate maternal weight gain during pregnancy can manifest in the birth of a low-birthweight infant. Prolonged labor and gestational diabetes are associated with excess weight gain during pregnancy. Preeclampsia is based on maternal hypertension, proteinuria, and edema states. DIF: Cognitive Level: Application OBJ: Nursing Process Step: Assessment MSC: Patient Needs: Health Promotion and Maintenance 18. A pregnant patient arrives for her first prenatal visit at the clinic. She informs the nurse that she has been taking an additional 400 mcg of folic acid prior to becoming pregnant. Based on the patient‘s history, she has reached 8 weeks‘ gestation. Which recommendation would the nurse provide regarding folic acid supplementation? a. Have the patient continue to take 400 mcg folic acid throughout her pregnancy. b. Tell the patient that she no longer has to take additional folic acid because it will be included in her prenatal vitamins. c. Have the patient increase her folic acid intake to 1000 mcg throughout the rest of her pregnancy. d. Schedule the patient to go for an AFP (alpha-fetoprotein) test. ANS: B Prenatal vitamins include adequate folic acid supplementation, so patients should not take additional supplementation as long as they continue their prenatal vitamins. During pregnancy, the recommendation is to increase the folic acid intake to 600 mcg. 1000 mcg of folic acid would be an excessive dose. The AFP test should be done at 15 to 18 weeks‘ gestation. This is not clinically indicated because the patient is at 8 weeks‘ gestation. DIF: Cognitive Level: Application OBJ: Nursing Process Step: Planning MSC: Patient Needs: Health Promotion and Maintenance 19. Which patient is most at risk for a low-birth-weight infant? a. 22-year-old, 60 inches tall, normal prepregnant weight b. 18-year-old, 64 inches tall, body mass index is <18.5 c. 30-year-old, 78 inches tall, prepregnant weight is 15 lb above the norm d. 35-year-old, 75 inches tall, total weight gain in previous pregnancies was 33 lb ANS: B The patient who has a low prepregnancy weight is associated with preterm labor and low-birthweight infants. Women who are underweight should gain more during pregnancy to meet the needs of pregnancy as well as their own need to gain weight; patients who have a normal prepregnancy weight, who start pregnancy overweight, or who have a history of excessive weight gain in pregnancy are not at risk for low-birth-weight infants. DIF: Cognitive Level: Analysis OBJ: Nursing Process Step: Analysis MSC: Patient Needs: Health Promotion and Maintenance 20. Changes in the diet of the pregnant patient who has phenylketonuria would include a.adding foods high in vitamin C. b. eliminating drinks containing aspartame. c. restricting protein intake to <20 g a day. d. increasing caloric intake to at least 1800 cal/day. ANS: B Use of aspartame by women with phenylketonuria can result in fetal brain damage because these women lack the enzyme to metabolize aspartame. Adding vitamin C, restricting protein, and increasing caloric intake are not necessary for the pregnant patient with phenylketonuria. DIF: Cognitive Level: Analysis OBJ: Nursing Process Step: Analysis MSC: Patient Needs: Health Promotion and Maintenance 21. When explaining the recommended weight gain to your patient, the nurse‘s teaching should include which statement? a. ―All pregnant women need to gain a minimum of 25 to 35 lb.‖ b. ―The fetus, amniotic fluid, and placenta require 15 lb of weight gain.‖ c. ―Weight gain in pregnancy is based on the patient‘s prepregnant body mass index.‖ d. ―More weight should be gained in the first and second trimesters and less in the third.‖ ANS: C Recommendations for weight gain in pregnancy are based on the woman‘s prepregnancy weight for her height (body mass index). Depending on the prepregnant weight, recommendation for weight gain may be more or less than 25 to 35 lb. The combination of the fetus, amniotic fluid, and placenta averages about 11 lb in the patient who has a normal BMI. Less weight should be gained in the first trimester, when the fetus needs fewer nutrients for growth, and more in the third trimester, when fetal growth is accelerated. DIF: Cognitive Level: Application OBJ: Nursing Process Step: Implementation MSC: Patient Needs: Health Promotion and Maintenance 22. Which patient has correctly increased her caloric intake from her recommended pregnancy intake to the amount necessary to sustain breastfeeding in the first 6 postpartum months? a. From 1800 to 2200 calories per day b. From 2000 to 2500 calories per day c. From 2200 to 2530 calories per day d. From 2500 to 2730 calories per day ANS: C The increased calories necessary for breastfeeding are 500, with 330 calories coming from increased caloric intake and 170 calories from maternal stores. An increase of 230 calories is insufficient for breastfeeding. An increase of 400 and 500 calories is above the recommended amount. DIF: Cognitive Level: Analysis OBJ: Nursing Process Step: Analysis MSC: Patient Needs: Health Promotion and Maintenance23. A pregnant woman of normal weight enters her 13th week of pregnancy. If the patient eats and exercises as directed, what will the nurse anticipate as the ongoing weight gain for the remaining trimesters? a. 0.3 lb every week b. 1 lb every week c. 1.8 lb every week d. 2 lb every week ANS: B After the first 12 weeks (first trimester), the pregnant woman should gain 0.35 to 0.5 kg (0.8 to 1 lb) per week for the remainder of the pregnancy. DIF: Cognitive Level: Knowledge OBJ: Nursing Process Step: Assessment MSC: Patient Needs: Health Promotion and Maintenance 24. A patient with a BMI of 32 has a positive pregnancy test. What is the maximum number of pounds that the nurse will advise the patient gain during the pregnancy? a. 20 lb b. 25 lb c. 28 lb d. 40 lb ANS: A The weight gain for obese women is 5 to 9 kg (11 to 20 lb). A BMI of 30 or higher categorizes the patient as obese. The other options refer to minimal or maximal weight gain for patients in other BMI categories. DIF: Cognitive Level: Application OBJ: Nursing Process Step: Implementation MSC: Patient Needs: Health Promotion and Maintenance 25. A pregnant patient with significant iron-deficiency anemia is prescribed iron supplements. The patient explains to the nurse that she cannot take iron because it makes her nauseous. What is the best response by the nurse? a. ―Iron will be absorbed more readily if taken with orange juice.‖ b. ―It is important to take this drug regardless of this side effect.‖ c. ―Taking the drug with milk may decrease your symptoms.‖ d. ―Try taking the iron at bedtime on an empty stomach.‖ ANS: D Iron taken at bedtime may be easier to tolerate. All the answers are true statements; however, only the option that states that iron taken at bedtime may be easier to tolerate addresses both optimal absorption of iron and alleviation of nausea, which will not be noticeable during sleep. It is true that taking iron with milk will decrease the symptoms; however, it will also decrease absorption. DIF: Cognitive Level: Application OBJ: Nursing Process Step: Implementation MSC: Patient Needs: Physiologic Integrity: Pharmacologic Therapies26. Which guidance related to a healthy diet during pregnancy will the nurse provide to a patient in her 1st trimester? a. ―Every day you need to have at least 6 ounces of protein from sources such as meat, fish, eggs, beans, nuts, soybean products, and tofu.‖ b. ―High-dose vitamin A supplements will promote optimal vision while preventing a common cause of blindness in neonates.‖ c. ―Meals such as sushi with a cold deli salad made with raw sprouts combine high-fiber foods with protein sources to meet multiple nutritional needs.‖ d. ―Vitamin and mineral supplements can meet your nutrient needs if you have inadequate intake because of nausea or a sensation of fullness.‖ ANS: A Protein sources include meat, poultry, fish, eggs, legumes (e.g., beans, peas, lentils), nuts, and soybean products such as tofu. Pregnant women need 6 to 6.5 oz of protein daily. Vitamin A can cause fetal anomalies of the bones, urinary tract, and central nervous system when taken in high doses. Pregnant women should avoid raw fish and foods such as cold deli salads and raw sprouts. Supplements do not generally contain protein and calories and may lack many necessary nutrients; therefore they cannot serve as food substitutes. DIF: Cognitive Level: Application OBJ: Nursing Process Step: Implementation MSC: Patient Needs: Health Promotion and Maintenance 27. For the pregnant patient who is a vegan, what combination of foods will the nurse advise to meet the nutritional needs for all essential amino acids? a. Eggs and beans b. Fruits and vegetables c. Grains and legumes d. Vitamin and mineral supplements ANS: C Combining incomplete plant proteins with other plant foods that have complementary amino acids allows intake of all essential amino acids. Dishes that contain grains (e.g., wheat, rice, corn) and legumes (e.g., garbanzo, navy, kidney, or pinto beans, peas, peanuts) are combinations that provide complete proteins. Eggs are not consumed by vegans. Fruits and vegetables alone will not provide the essential amino acids. Vitamin and mineral supplements do not provide amino acids. DIF: Cognitive Level: Application OBJ: Nursing Process Step: Implementation MSC: Patient Needs: Health Promotion and Maintenance 28. A pregnant patient has lactose intolerance. Which recommendation will the nurse provide to best help the patient meet dietary needs for calcium? a. Add foods such as nuts, dried fruit, and broccoli to the diet. b. Consume dairy products but take an over-the-counter anti-gas product. c. Increase the intake of dark leafy vegetables, such as spinach and chard. d.Use powdered milk instead of liquid forms of milk. ANS: A Calcium is present in legumes, nuts, dried fruits, and broccoli, so these foods can be added to increase calcium intake. Although dark leafy vegetables contain calcium, they also contain oxalates that decrease the availability of calcium. Powdered milk contains lactase, similar to the nondehydrated varieties. Milk products should be avoided by patients with lactose intolerance. Adequate calcium may be obtained from food and supplements. Some patients may be able to tolerate lactose free dairy products. DIF: Cognitive Level: Understanding OBJ: Nursing Process Step: Implementation MSC: Patient Needs: Health Promotion and Maintenance 29. The nurse is reviewing a list of foods high in folic acid with a patient who is considering becoming pregnant. The nurse determines that the patient understands the teaching when the patient states she will include which list of foods in her diet? a. Peaches, yogurt, and tofu b. Strawberries, milk, and tuna c. Asparagus, lemonade, and chicken breast d. Spinach, orange juice, and fortified bran flakes ANS: D Prepregnant, the recommendation for folic acid is 800 mcg. Foods high in folic acid are dark green leafy vegetables, legumes (beans, peanuts), orange juice, asparagus, spinach, and fortified cereal and pasta. In the United States, folic acid is added to orange juice and wheatbased products. DIF: Cognitive Level: Application OBJ: Nursing Process Step: Evaluation MSC: Patient Needs: Health Promotion and Maintenance 30. A patient at 8 weeks‘ gestation complains to the nurse, ―I feel sick almost every morning. And I throw up at least two or three times a week.‖ What is the nurse‘s best guidance for this patient? a. ―Do you like cheese?‖ b. ―Try eating four meals a day instead of three meals a day.‖ c. ―Try eating peanut butter on whole wheat bread right before going to bed.‖ d. ―If you can eat enough throughout the day, you don‘t have to worry about being sick.‖ ANS: C Eating a bedtime protein snack helps maintain glucose levels throughout the night. Cheese is high in fat and can aggravate nausea. Small and frequent meals is the optimal recommendation. Four meals a day would not be ideal for a patient experiencing nausea, she needs to eat more frequently. Consumption is not the patient‘s stated concern—it is the nausea and vomiting. DIF: Cognitive Level: Analysis OBJ: Nursing Process Step: Analysis MSC: Patient Needs: Health Promotion and Maintenance MULTIPLE RESPONSE 1. The nurse is teaching a patient taking prenatal vitamins how to avoid constipation. Which should the nurse plan to include in the teaching session? (Select all that apply.) a. Advise taking a daily laxative for constipation.b. Recommend a diet high in fruits and vegetables. c. Encourage an increase in fluid consumption during the day. d. Increase the intake of whole grains and whole grain products. e. Suggest increasing the intake of dairy products, especially cheeses. ANS: B, C, D Common sources of dietary fiber include fruits and vegetables (with skins when possible— apples, strawberries, pears, carrots, corn, potatoes with skins, and broccoli), whole grains, and whole grain products—whole wheat bread, bran muffins, bran cereals, oatmeal, brown rice, and whole wheat pasta. Increased intake of fluids can help prevent constipation. A pregnant patient should not take a daily laxative unless prescribed by her health care provider. Increased intake of dairy products, especially cheese, may increase constipation. DIF: Cognitive Level: Application OBJ: Nursing Process Step: Planning MSC: Patient Needs: Health Promotion and Maintenance 2. The nurse is teaching a breastfeeding patient about substances to avoid while she is breastfeeding. Which substances should the nurse include in the teaching session? (Select all that apply.) a. Caffeine b. Alcohol c. Omega-6 fatty acids d. Appetite suppressants e. Polyunsaturated omega-3 fatty acids ANS: A, B, D Foods high in caffeine should be limited. Infants of mothers who drink more than two or three cups of caffeinated coffee or the equivalent each day may be irritable or have trouble sleeping. Although the relaxing effect of alcohol was once thought to be helpful to the nursing mother, the deleterious effects of alcohol are too important to consider this suggestion appropriate today. An occasional single glass of an alcoholic beverage may not be harmful, but larger amounts may interfere with the milk-ejection reflex and may be harmful to the infant. Nursing mothers should avoid appetite suppressants, which may pass into the milk and harm the infant. The long-chain polyunsaturated omega-3 and omega-6 fatty acids are present in human milk. Therefore they should be included in the mother‘s diet during lactation. DIF: Cognitive Level: Application OBJ: Nursing Process Step: Planning MSC: Patient Needs: Health Promotion and Maintenance 3. The nurse is advising a lactose-intolerant pregnant patient about calcium intake. Which calcium sources are approximately equivalent to 1 cup of milk? (Select all that apply.) a. cup yogurt b. 1 cup of sherbet c. oz of hard cheese d.cups of ice cream e. cup of low-fat cottage cheese ANS: A, C, D Calcium sources approximately equivalent to 1 cup of milk include cup yogurt, ounce of hard cheese, and cups of ice cream. It takes 3 cups of sherbet and cups of low-fat cottage cheese to equal the calcium equivalent of 1 cup of milk. DIF: Cognitive Level: Application OBJ: Nursing Process Step: Planning MSC: Patient Needs: Health Promotion and Maintenance 4. The nurse is teaching a pregnant patient about food safety during pregnancy and lactation. Which statements by the patient indicate she understood the teaching? (Select all that apply.) a. ―I will limit my intake of shrimp to 12 oz a week.‖ b. ―I will avoid the soft cheeses made with unpasteurized milk.‖ c. ―I plan to continue to pack my bologna sandwich for lunch.‖ d. ―I am glad I can still go to the sushi bar during my pregnancy.‖ e. ―I will not eat any swordfish or shark while I am pregnant or nursing.‖ ANS: A, B, E Statements that indicate the patient understood the teaching are limiting shrimp to 12 oz a week, avoiding soft cheeses, and not eating any swordfish. A bologna sandwich should be avoided unless it is reheated until steaming hot. Raw or undercooked fish should be avoided. DIF: Cognitive Level: Analysis OBJ: Nursing Process Step: Evaluation MSC: Patient Needs: Health Promotion and Maintenance Chapter 09: Assessing the Fetus Foundations of Maternal-Newborn & Women‘s Health Nursing, 7th Edition MULTIPLE CHOICE 1. A pregnant patient‘s biophysical profile score is 8. The patient asks the nurse to explain the results. What is the nurse‘s most appropriate response? a. ―The test results are within normal limits.‖ b. ―Immediate birth by cesarean birth is being considered.‖ c. ―Further testing will be performed to determine the meaning of this score.‖ d. ―An obstetric specialist will evaluate the results of this profile and, within the next week, will inform you of your options regarding birth.‖ ANS: A The normal biophysical score ranges from 8 to 10 points if the amniotic fluid volume is adequate. A normal score allows conservative treatment of high-risk patients. Birth can be delayed if fetal well-being is an issue. Scores less than 4 would be investigated, and birth could be initiated sooner than planned. This score is within normal range, and no further testing is required at this time. The results of the biophysical profile are usually available immediately after the procedure is performed.DIF: Cognitive Level: Application OBJ: Nursing Process Step: Implementation MSC: Patient Needs: Physiologic Integrity 2. Which analysis of maternal serum is the best predictor of chromosomal abnormalities in the fetus? a. Biophysical profile b. Multiple-marker screening c. Lecithin-to-sphingomyelin ratio d. Blood type and crossmatch of maternal and fetal serum ANS: B Maternal serum can be analyzed for abnormal levels of alpha-fetoprotein (AFP), human chorionic gonadotropin (hCG), inhibin A, and estriol. The multiple-marker screening may predict chromosomal defects in the fetus. The biophysical profile is used to evaluate fetal status during the antepartum period. Five variables are used; however, none are concerned with chromosomal problems. The lecithin-to-sphingomyelin ratio is used to determine fetal lung maturity. The blood type and crossmatch will not predict chromosomal defects in the fetus. DIF: Cognitive Level: Understanding OBJ: Nursing Process Step: Assessment MSC: Patient Needs: Physiologic Integrity 3. The clinic nurse is obtaining a health history on a newly pregnant patient. Which is an indication for fetal diagnostic procedures if present in the health history? a. Maternal diabetes b. Weight gain of 25 lb c. Maternal age older than 30 years d. Previous infant weighing more than 3000 g at birth ANS: A Diabetes is a risk factor in pregnancy because of possible impairment of placental perfusion. Excessive weight gain is an indication for testing. Normal weight gain is 25 to 35 lb. A maternal age older than 35 years is an indication for testing. Having had another infant weighing more than 4000 g is an indication for testing. DIF: Cognitive Level: Analysis OBJ: Nursing Process Step: Evaluation MSC: Patient Needs: Physiologic Integrity 4. What point in the pregnancy is the most accurate time to determine gestational age through ultrasound? a. First trimester b. Second trimester c. Third trimester d. No difference in accuracy among the trimesters ANS: AGestational age determination by ultrasonography is increasingly less accurate after the first trimester. Gestational age determination is best done in the first trimester. There is a difference in trimesters when doing a gestational age ultrasonography. DIF: Cognitive Level: Understanding OBJ: Nursing Process Step: Assessment MSC: Patient Needs: Physiologic Integrity 5. The primary reason for evaluating alpha-fetoprotein (AFP) levels in maternal serum is to determine whether the fetus has which condition? a. Hemophilia b. Sickle cell anemia c. A neural tube defect d. Abnormal lecithin-to-sphingomyelin ratio ANS: C An open neural tube allows a high level of AFP to seep into the amniotic fluid and enter the maternal serum. Hemophilia is a genetic defect and is best detected with chromosomal studies, such as chorionic villus sampling or amniocentesis. Sickle cell anemia is a genetic defect and is best detected with chromosomal studies such as chorionic villus sampling or amniocentesis. L/S ratios are determined with an amniocentesis and are usually performed in the third trimester. DIF: Cognitive Level: Understanding OBJ: Nursing Process Step: Assessment MSC: Patient Needs: Physiologic Integrity 6. When is the earliest interval that chorionic villus sampling (CVS) can be performed during pregnancy? a. 4 weeks b. 8 weeks c. 10 weeks d. 14 weeks ANS: C CVS is normally performed between 10 and 13 weeks gestation. The fetal villus tissue can be analyzed directly for chromosomal, metabolic, or DNA abnormalities. It is too early to perform CVS at 4 or 8 weeks of pregnancy. The test can no longer be performed a 14 weeks gestation. Results are available within 24 to 48 hours. DIF: Cognitive Level: Understanding OBJ: Nursing Process Step: Assessment MSC: Patient Needs: Physiologic Integrity 7. Which aspect of fetal diagnostic testing is most important to expectant parents? a. Safety of the fetus b. Duration of the test c. Cost of the procedure d. Physical discomfort caused by the procedure ANS: AAlthough all of these are considerations, parents are usually most concerned about the safety of the fetus. Parents are concerned about the duration of the test; however, it is not the greatest concern. The cost of the procedure is important to parents, especially those without third-party payers; but again, this is not the greatest concern. With adequate preparation for the procedure by the nurse physical discomfort can be allayed. DIF: Cognitive Level: Understanding OBJ: Nursing Process Step: Assessment MSC: Patient Needs: Physiologic Integrity 8. The nurse‘s role in diagnostic testing is to provide which of the following? a. Advice to the couple b. Information about the tests c. Reassurance about fetal safety d. Assistance with decision making ANS: B The nurse should provide the couple with all necessary information regarding a procedure so that the couple can make an informed decision. The nurse‘s role is to inform, not to advice. Ensuring fetal safety is not possible with all the diagnostic tests. To offer this is to give false reassurance to the parents. The nurse can inform the couple about potential problems so they can make an informed decision. Decision making should always lie with the couple involved. The nurse should provide information so that the couple can make an informed decision. DIF: Cognitive Level: Understanding OBJ: Nursing Process Step: Assessment MSC: Patient Needs: Physiologic Integrity 9. Which factors should be considered a contraindication for transcervical chorionic villus sampling? a. Rh-negative mother b. Gestation less than 15 weeks c. Maternal age younger than 35 years d. Positive for group B Streptococcus ANS: D Maternal infection is a risk with this procedure, and it is contraindicated if the patient has an active infection in the cervix, vagina, or pelvic area. This procedure is done between 10 and 12 weeks. This procedure is usually done for women older than 35; however, if the woman is at high risk for fetal anomalies, her age is not a contraindication. The procedure can still be performed; however, Rh sensitization may occur if the mother is Rh-negative. Rho(D) immune globulin can be administered following the procedure. DIF: Cognitive Level: Analysis OBJ: Nursing Process Step: Assessment MSC: Patient Needs: Physiologic Integrity 10. What is the purpose of amniocentesis for a patient hospitalized at 34 weeks of gestation with pregnancy-induced hypertension? a. Determine if a metabolic disorder exists. b. Identify the sex of the fetus. c.Identify abnormal fetal cells. d. Determine fetal lung maturity. ANS: D During the third trimester, amniocentesis is most often performed to determine fetal lung maturity. In cases of pregnancy-induced hypertension, preterm birth may be necessary because of changes in placental perfusion. The test is done in the early portion of the pregnancy if a metabolic disorder is genetic. Amniocentesis is done early in the pregnancy to do genetic studies and determine the sex. Identification of abnormal cells is done during the early portion of the pregnancy. DIF: Cognitive Level: Analysis OBJ: Nursing Process Step: Assessment MSC: Patient Needs: Physiologic Integrity 11. What does optimal nursing care after an amniocentesis include? a. Pushing fluids by mouth b. Monitoring uterine activity c. Placing the patient in a supine position for 2 hours d. Applying a pressure dressing to the puncture site ANS: B A risk with amniocentesis is the onset of spontaneous contractions. Hydration is important; however, the woman has not been NPO, so this should not be a problem. The supine position may decrease uterine blood flow; the side-lying position is preferred. Pressure dressings are not necessary. DIF: Cognitive Level: Application OBJ: Nursing Process Step: Implementation MSC: Patient Needs: Physiologic Integrity 12. What is the term for a nonstress test in which there are two or more fetal heart rate accelerations of 15 or more beats per minute (BPM) with fetal movement in a 20-minute period? a. Positive b. Negative c. Reactive d. Nonreactive ANS: C The nonstress test (NST) is reactive (normal) when there are two or more fetal heart rate accelerations of at least 15 BPM (each with a duration of at least 15 seconds) in a 20-minute period. A positive result is not used with an NST. The contraction stress test (CST) uses positive as a result term. A negative result is not used with an NST. The CST uses negative as a result term. A nonreactive result means that the heart rate did not accelerate during fetal movement. DIF: Cognitive Level: Analysis OBJ: Nursing Process Step: Evaluation MSC: Patient Needs: Physiologic Integrity 13. What is the purpose of initiating contractions in a contraction stress test (CST)? a. Increase placental blood flow. b. Identify fetal acceleration patterns.c. Determine the degree of fetal activity. d. Apply a stressful stimulus to the fetus. ANS: D The CST involves recording the response of the fetal heart rate to stress induced by uterine contractions. The CST records the fetal response to stress. It does not increase placental blood flow. The NST looks at fetal heart accelerations with fetal movements. The NST and biophysical profiles look at fetal movements. DIF: Cognitive Level: Understanding OBJ: Nursing Process Step: Assessment MSC: Patient Needs: Physiologic Integrity 14. A biophysical profile is performed on a pregnant patient. The following assessments are noted: nonreactive stress test (NST), three episodes of fetal breathing movements (FBMs), limited gross movements, opening and closing of hang indicating the presence of fetal tone, and adequate amniotic fluid index (AFI) meeting criteria. Which answer would be the correct interpretation of this test result? a. A score of 10 would indicate that the results are equivocal. b. A score of 8 would indicate normal results. c. A score of 6 would indicate that birth should be considered as a possible treatment option. d. A score of 9 would indicate reassurance. ANS: B The biophysical profile is used to assess fetal well-being. Five categories of assessment are used in this combination test: fetal monitoring NST, evaluation of FBMs, gross movements, fetal tone, and calculation of the amniotic fluid index (AFI). A maximum of 2 points is used if criteria are met successfully in each category; thus a score in the range of 8 to 10 indicates a normal or reassuring finding. A score of 6 provides equivocal results and further testing or observation is necessary. A score of 4 or less requires immediate intervention, and birth may be warranted. The provided assessments indicate a score of 8 as the only area that has not met the stated criteria in the NST. DIF: Cognitive Level: Analysis OBJ: Nursing Process Step: Evaluation MSC: Patient Needs: Physiologic Integrity: Reduction of Risk Potential/Diagnostic Tests 15. In preparing a pregnant patient for a nonstress test (NST), which of the following should be included in the plan of care? a. Have the patient void prior to being placed on the fetal monitor because a full bladder will interfere with results. b. Maintain NPO status prior to testing. c. Position the patient for comfort, adjusting the tocotransducer belt to locate fetal heart rate. d. Have an infusion pump prepared with oxytocin per protocol for evaluation. ANS: C The nurse must adjust the tocotransducer to find the best location to pick up and record the fetal heart rate. Positioning the patient for comfort during testing is a prime concern. Although a full bladder may affect patient comfort, it will not interfere with testing results. NPO status is not required for an NST. Instead, a pregnant patient should maintain her normal nutritional intake toprovide energy to herself and the fetus. An infusion pump with oxytocin is required for a contraction stress test (CST). DIF: Cognitive Level: Analysis OBJ: Nursing Process Step: Planning MSC: Patient Needs: Physiologic Integrity: Reduction of Risk Potential/Diagnostic Tests 16. The results of a contraction stress test (CST) are positive. Which intervention is necessary based on this test result? a. Repeat the test in 1 week so that results can be trended based on this baseline result. b. Contact the health care provider to discuss birth options for the patient. c. Send the patient out for a meal and repeat the test to confirm that the results are valid. d. Ask the patient to perform a fetal kick count assessment for the next 30 minutes and then reassess the patient. ANS: B A positive CST test is an abnormal finding, and the provider should be notified so that birth options can be initiated. A positive CST indicates possible fetal compromise. Intervention should not be delayed by 1 week and results do not have to be trended. Because this is an abnormal result, there is no need to repeat the test. Sending the patient out for a meal will delay treatment options and may interfere with possible birth interventions if anesthesia is needed. Fetal kick count assessment is not needed at this time and will further delay treatment interventions for this abnormal result, which indicates fetal compromise. DIF: Cognitive Level: Analysis OBJ: Nursing Process Step: Implementation MSC: Patient Needs: Physiologic Integrity: Physiologic Adaptation/Unexpected Response to Therapies 17. A pregnant patient has received the results of her triple-screen testing and it is positive. She provides you with a copy of the test results that she obtained from the lab. What would the nurse anticipate as being implemented in the patient‘s plan of care? a. No further testing is indicated at this time because results are normal. b. Refer to the physician for additional testing. c. Validate the results with the lab facility. d. Repeat the test in 2 weeks and have the patient return for her regularly scheduled prenatal visit. ANS: B Additional genetic testing is indicated to provide the patient with treatment options. A positive result on a triple-screen test is considered to be an abnormal finding so the patient should be referred to the physician for additional genetic testing. Validation of the test with a lab facility is not necessary because the patient provided you with a copy of the test results. There is no need to repeat the clinical test because the findings have already been determined. DIF: Cognitive Level: Analysis OBJ: Nursing Process Step: Implementation MSC: Patient Needs: Physiologic Integrity: Physiologic Adaptation/Unexpected Response to Therapies 18. A pregnant woman is scheduled to undergo chorionic villus sampling (CVS) based on genetic family history. Which medication does the nurse anticipate will be administered? a. Magnesium sulfate b.Prostaglandin suppository c. RhoGAM if the patient is Rh-negative d. Betamethasone ANS: C CVS can increase the likelihood of Rh sensitization if a woman is Rh-negative. There is no indication for magnesium sulfate because it is used to stop preterm labor. There is no indication for administration of a prostaglandin suppository. Betamethasone is given to pregnant women in preterm labor to improve fetal lung maturity. DIF: Cognitive Level: Analysis OBJ: Nursing Process Step: Planning MSC: Patient Needs: Physiologic Integrity: Pharmacologic and Parenteral Therapies 19. For which patient would an L/S ratio of 2:1 potentially be considered abnormal? a. A 38-year-old gravida 2, para 1, who is 38 weeks‘ gestation b. A 24-year-old gravida 1, para 0, who has diabetes c. A 44-year-old gravida 6, para 5, who is at term d. An 18-year-old gravida 1, para 0, who is in early labor at term ANS: B Even though an L/S ratio of 2:1 is typically considered to be a normal finding to validate fetal lung maturity prior to 38 weeks‘ gestation, the result may not be accurate in determining fetal lung maturity if a patient is diabetic. DIF: Cognitive Level: Analysis OBJ: Nursing Process Step: Assessment MSC: Patient Needs: Physiologic Integrity: Reduction of Risk Potential 20. Which complication could occur as a result of percutaneous umbilical blood sampling (PUBS)? a. Postdates pregnancy b. Fetal bradycardia c. Placenta previa d. Uterine rupture ANS: B PUBS is an invasive test whereby a needle is inserted into the umbilical cord to obtain blood as the basis for diagnostic testing with the guidance of ultrasound technology. The most common complication is fetal bradycardia, which is temporary. PUBS has no effect on extending the gestation of pregnancy, the development of placenta previa, or uterine rupture. DIF: Cognitive Level: Analysis OBJ: Nursing Process Step: Planning MSC: Patient Needs: Physiologic Integrity: Reduction of Risk Potential 21. A newly pregnant patient tells the nurse that she has irregular periods and is unsure of when she got pregnant. Scheduling an ultrasound is a standing prescription for the patient‘s health care provider. When is the best time for the nurse to schedule the patient‘s ultrasound? a. Immediately b. In 2 weeksc. In 4 weeks d. In 6 weeks ANS: A An embryo can be seen about 5 to 6 weeks after the last menstrual period. At this time the crown–rump length (CRL) of the embryo is the most reliable measure of gestational age. Fetal viability is confirmed by observation of fetal heartbeat, which is visible when the CRL of the embryo is 5 mm. DIF: Cognitive Level: Application OBJ: Nursing Process Step: Planning MSC: Patient Needs: Health Promotion and Maintenance 22. The nurse is reviewing the procedure for alpha-fetoprotein (AFP) screening with a patient at 16 weeks‘ gestation. The nurse determines that the patient understands the teaching when she states that will be collected for the initial screening process? a. Urine b. Blood c. Saliva d. Amniotic fluid ANS: B Initial screening is completed with blood. AFP can be detected in amniotic fluid; however, that procedure is more costly and invasive. Procedures progress from least invasive to most invasive. DIF: Cognitive Level: Understanding OBJ: Nursing Process Step: Evaluation MSC: Patient Needs: Health Promotion and Maintenance 23. A patient at 36 weeks gestation is undergoing a nonstress (NST) test. The nurse observes the fetal heart rate baseline at 135 beats per minute (bpm) and four nonepisodic patterns of the fetal heart rate reaching 160 bpm for periods of 20 to 25 seconds each. How will the nurse record these findings? a. NST positive, nonreassuring b. NST negative, reassuring c. NST reactive, reassuring d. NST nonreactive, nonreassuring ANS: C The presence of at least three accelerations of at least 15 beats, over at least 15 seconds, over a duration of at least 20 minutes, is considered reactive and reassuring. Nonreactive testing reveals no or fewer accelerations over the same or longer period. The NST test is not recorded as positive or negative. DIF: Cognitive Level: Analysis OBJ: Nursing Process Step: Analysis MSC: Patient Needs: Health Promotion and Maintenance MULTIPLE RESPONSE 1. Which clinical conditions are associated with increased levels of alpha fetoprotein (AFP)? (Select all that apply.) a.Down syndrome b. Molar pregnancy c. Twin gestation d. Incorrect gestational age assessment of a normal fetus—estimation is earlier in the pregnancy e. Threatened abortion ANS: C, D, E Elevated APF levels are seen in multiple gestations, underestimation of fetal age, and threatened abortion. Decreased levels are seen in Down syndrome and a molar pregnancy. DIF: Cognitive Level: Analysis OBJ: Nursing Process Step: Diagnosis MSC: Patient Needs: Physiologic Integrity: Physiologic Adaptation/Pathophysiology 2. Transvaginal ultrasonography is often performed during the first trimester. A 6-week-gestation patient expresses concerns over the necessity for this test. The nurse should explain that this diagnostic test may be necessary to determine which of the following? (Select all that apply.) a. Multifetal gestation b. Bicornuate uterus c. Presence and location of pregnancy d. Amniotic fluid volume e. Presence of ovarian cysts ANS: A, B, C, E A bicornuate uterus, multifetal gestation, presence of ovarian cysts, and presence and location of pregnancy can be determined by transvaginal ultrasound in the first trimester of pregnancy. This procedure is also used for estimating gestational age, confirming fetal viability, identifying fetal abnormalities or chromosomal defects, and identifying the maternal abnormalities mentioned, as well as fibroids. Amniotic fluid volume is assessed during the second and third trimesters. Conventional ultrasound would be used. DIF: Cognitive Level: Analysis OBJ: Nursing Process Step: Planning MSC: Patient Needs: Physiologic Integrity 3. A woman who is 36 weeks pregnant asks the nurse to explain the vibroacoustic stimulator (VAS) test. Which should the nurse include in the response? (Select all that apply.) a. The test is invasive. b. The test uses sound to elicit fetal movements. c. The test may confirm nonreactive nonstress test results. d. The test can only be performed if contractions are present. e. Vibroacoustic stimulation can be repeated at 1-minute intervals up to three times. ANS: B, C, E Also referred to as VAS or acoustic stimulation, the vibroacoustic stimulator (similar to an electronic larynx) is applied to the maternal abdomen over the area of the fetal head. Vibrationand sound are emitted for up to 3 seconds and may be repeated. A fetus near term responds by increasing the number of gross body movements, which can be easily seen and felt. The procedure can confirm reassuring NST findings and shorten the length of time necessary to obtain NST data. The test is noninvasive and contractions do not need to be present to perform the test. DIF: Cognitive Level: Application OBJ: Nursing Process Step: Implementation MSC: Patient Needs: Physiologic Integrity 4. The nurse is instructing a patient on how to perform kick counts. Which information should the nurse include in the teaching session? (Select all that apply.) a. Use a clock or timer when performing kick counts. b. Your bladder should be full before performing kick counts. c. Notify your health care provider if you have not felt movement in 24 hours. d. Protocols can provide a structured timetable for concentrating on fetal movements. e. You should lie on your side, place your hands on the largest part of the abdomen, and concentrate on the number of movements felt. ANS: A, D, E The nurse should instruct the patient to lie on her side, place her hands on the largest part of her abdomen, and concentrate on fetal movements. She should use a clock or timer and record the number of movements felt during that time. Protocols are not essential; however, they may give the patient a more structured timetable for when to concentrate on fetal movements. The bladder does not need to be full for kick counts; it is better to have the patient empty her bladder before beginning the assessment of fetal movements. Further evaluation is recommended if the patient feels no movements in 12 hours; 24 hours is too long before notifying the health care provider. DIF: Cognitive Level: Application OBJ: Nursing Process Step: Implementation MSC: Patient Needs: Physiologic Integrity 5. The nurse is preparing a patient for a nonstress test (NST). Which interventions should the nurse plan to implement? (Select all that apply.) a. Ensure that the patient has a full bladder. b. Plan approximately 15 minutes for the test. c. Have the patient sit in a recliner with the head elevated 45 degrees. d. Apply electronic monitoring equipment to the patient‘s abdomen. e. Instruct the patient to press an event marker every time she feels fetal movement. ANS: C, D, E The patient may be seated in a reclining chair or have her head elevated at least 45 degrees. The nurse applies external electronic monitoring equipment to the patient‘s abdomen to detect the fetal heart rate and any contractions. The woman may be given an event marker to press each time she senses movement. Before the NST, the patient should void and her baseline vital signs should be taken. The NST takes about 40 minutes, allowing for most fetal sleep-wake cycles, although the fetus may show a reassuring pattern more quickly or need more time to awaken and become active. Fifteen minutes would not allow enough time to complete the test.DIF: Cognitive Level: Application OBJ: Nursing Process Step: Implementation MSC: Patient Needs: Physiologic Integrity Chapter 10: Complications of Pregnancy Foundations of Maternal-Newborn & Women‘s Health Nursing, 7th Edition MULTIPLE CHOICE 1. A patient with preeclampsia is being treated with bed rest and intravenous magnesium sulfate. The drug classification of this medication is a a. diuretic. b. tocolytic. c. anticonvulsant. d. antihypertensive. ANS: C Anticonvulsant drugs act by blocking neuromuscular transmission and depress the central nervous system to control seizure activity. Diuresis is a therapeutic response to magnesium sulfate. A tocolytic drug slows the frequency and intensity of uterine contractions but is not used for that purpose in this scenario. Decreased peripheral blood pressure is a therapeutic response (side effect) of the anticonvulsant magnesium sulfate. DIF: Cognitive Level: Understanding OBJ: Nursing Process Step: Assessment MSC: Patient Needs: Physiologic Integrity 2. Which clinical intervention is the only known cure for preeclampsia? a. Magnesium sulfate b. Delivery of the fetus c. Antihypertensive medications d. Administration of aspirin (ASA) every day of the pregnancy ANS: B Delivery of the infant is the only known intervention to halt the progression of preeclampsia. Magnesium sulfate is one of the medications used to treat but not cure preeclampsia. Antihypertensive medications are used to lower the dangerously elevated blood pressures in preeclampsia and eclampsia. Low doses of aspirin (81 mg/day) have been administered to women at high risk for developing preeclampsia. This intervention appears to have little benefit. DIF: Cognitive Level: Understanding OBJ: Nursing Process Step: Assessment MSC: Patient Needs: Physiologic Integrity 3. The clinic nurse is performing a prenatal assessment on a pregnant patient at risk for preeclampsia. Which clinical sign would not present as a symptom of preeclampsia? a. Edema b. Proteinuria c. Glucosuria d. HypertensionANS: C Glucose into the urine is not one of the three classic symptoms of preeclampsia. The first sign noted by the pregnant patient is rapid weight gain and edema of the hands and face. Proteinuria usually develops later than the edema and hypertension. The first indication of preeclampsia is usually an increase in the maternal blood pressure. DIF: Cognitive Level: Application OBJ: Nursing Process Step: Assessment MSC: Patient Needs: Physiologic Integrity 4. Which intrapartal assessment should be avoided when caring for a patient with HELLP syndrome? a. Abdominal palpation b. Venous sample of blood c. Checking deep tendon reflexes d. Auscultation of the heart and lungs ANS: A Palpation of the abdomen and liver could result in a sudden increase in intraabdominal pressure, leading to rupture of the subcapsular hematoma. Assessment of heart and lungs is performed on every patient. Checking reflexes is not contraindicated. Venous blood is checked frequently to observe for thrombocytopenia. DIF: Cognitive Level: Application OBJ: Nursing Process Step: Implementation MSC: Patient Needs: Physiologic Integrity 5. A nurse is explaining to the nursing students working on the antepartum unit how to assess for edema. Which edema assessment score indicates edema of the lower extremities, face, hands, and sacral area? a. +1 b. +2 c. +3 d. +4 ANS: C Edema of the extremities, face, and sacral area is classified as +3 edema. Edema classified as +1 indicates minimal edema of the lower extremities. Marked edema of the lower extremities is +2 edema. Generalized massive edema (+4) includes the accumulation of fluid in the peritoneal cavity. DIF: Cognitive Level: Application OBJ: Nursing Process Step: Implementation MSC: Patient Needs: Physiologic Integrity 6. Which maternal condition always necessitates delivery by cesarean birth? a. Partial abruptio placentae b. Total placenta previa c. Ectopic pregnancy d. EclampsiaANS: B In total placenta previa, the placenta completely covers the cervical os. The fetus would die if a vaginal birth occurred. If the patient has stable vital signs and the fetus is alive, a vaginal birth can be attempted. If the fetus has already expired, a vaginal birth is preferred. The most common ectopic pregnancy is a tubal pregnancy, which is usually detected and treated in the first trimester. Labor can be safely induced if the eclampsia is under control. DIF: Cognitive Level: Understanding OBJ: Nursing Process Step: Assessment MSC: Patient Needs: Physiologic Integrity 7. Spontaneous termination of a pregnancy is considered to be an abortion if a. the pregnancy is less than 20 weeks. b. the fetus weighs less than 1000 g. c. the products of conception are passed intact. d. there is no evidence of intrauterine infection. ANS: A An abortion is the termination of pregnancy before the age of viability (20 weeks). The weight of the fetus is not considered because some fetuses of an older age may have a low birth weight. A spontaneous abortion may be complete or incomplete. A spontaneous abortion may be caused by many problems, one being intrauterine infection. DIF: Cognitive Level: Understanding OBJ: Nursing Process Step: Assessment MSC: Patient Needs: Health Promotion and Maintenance 8. An abortion when the fetus dies but is retained in the uterus is called a. inevitable. b. missed. c. incomplete. d. threatened. ANS: B A missed abortion refers to a dead fetus being retained in the uterus. An inevitable abortion means that the cervix is dilating with the contractions. An incomplete abortion means that not all the products of conception were expelled. With a threatened abortion, the patient has cramping and bleeding but not cervical dilation. DIF: Cognitive Level: Understanding OBJ: Nursing Process Step: Assessment MSC: Patient Needs: Physiologic Integrity 9. A placenta previa when the placental edge just reaches the internal os is called a. total. b. partial. c. low-lying. d. marginal. ANS: DA placenta previa that does not cover any part of the cervix is termed marginal. With a total placenta previa, the placenta completely covers the os. With a partial previa, the lower border of the placenta is within 3 cm of the internal cervical os but does not completely cover the os. A complete previa is termed total. The placenta completely covers the internal cervical os. DIF: Cognitive Level: Understanding OBJ: Nursing Process Step: Assessment MSC: Patient Needs: Physiologic Integrity 10. Which finding would indicate concealed hemorrhage in abruptio placentae? a. Bradycardia b. Hard boardlike abdomen c. Decrease in fundal height d. Decrease in abdominal pain ANS: B Concealed hemorrhage occurs when the edges of the placenta do not separate. The formation of a hematoma behind the placenta and subsequent infiltration of the blood into the uterine muscle results in a very firm, boardlike abdomen. The patient will have shock symptoms that include tachycardia. The fundal height will increase as bleeding occurs. Abdominal pain may increase significantly. DIF: Cognitive Level: Analysis OBJ: Nursing Process Step: Assessment MSC: Patient Needs: Physiologic Integrity 11. The priority nursing intervention when admitting a pregnant patient who has experienced a bleeding episode in late pregnancy is to a. monitor uterine contractions. b. assess fetal heart rate and maternal vital signs. c. place clean disposable pads to collect any drainage. d. perform a venipuncture for hemoglobin and hematocrit levels. ANS: B Assessment of the fetal heart rate (FHR) and maternal vital signs will assist the nurse in determining the degree of the blood loss and its effect on the patient and fetus. Monitoring uterine contractions is important; however, not the top priority. It is important to assess future bleeding, but the top priority is patient and fetal well-being. The most important assessment is to check patient and fetal well-being. The blood levels can be obtained later. DIF: Cognitive Level: Application 12. A patient with preeclampsia is admitted complaining of pounding headache, visual changes, and epigastric pain. Nursing care is based on the knowledge that these signs indicate a. gastrointestinal upset. b. effects of magnesium sulfate. c. anxiety caused by hospitalization. d. worsening disease and impending convulsion. ANS: DHeadache and visual disturbances are caused by increased cerebral edema. Epigastric pain indicates distention of the hepatic capsules and often warns that a convulsion is imminent. Gastrointestinal upset is not an indication as severe as the headache and visual disturbance. She has not yet been started on magnesium sulfate as a treatment. The signs and symptoms do not describe anxiety. DIF: Cognitive Level: Analysis OBJ: Nursing Process Step: Evaluation MSC: Patient Needs: Physiologic Integrity 13. Rh incompatibility can occur if the patient is Rh-negative and the a. fetus is Rh-negative. b. fetus is Rh-positive. c. father is Rh-positive. d. father and fetus are both Rh-negative. ANS: B For Rh incompatibility to occur, the mother must be Rh-negative and her fetus Rh-positive. If the fetus is Rh-negative, the blood types are compatible and no problems should occur. The father‘s Rh factor is a concern only as it relates to the possible Rh factor of the fetus. If the fetus is Rhnegative, the blood type with the mother is compatible. The father‘s blood type does not enter into the problem. DIF: Cognitive Level: Understanding OBJ: Nursing Process Step: Assessment MSC: Patient Needs: Physiologic Integrity 14. In which situation would a dilation and curettage (D&C) be indicated? a. Complete abortion at 8 weeks b. Incomplete abortion at 16 weeks c. Threatened abortion at 6 weeks d. Incomplete abortion at 10 weeks ANS: D D&C is carried out to remove the products of conception from the uterus and can be performed safely until week 14 of gestation. If all the products of conception have been passed (complete abortion), a D&C is not necessary. If the pregnancy is still viable (threatened abortion), a D&C is not indicated. DIF: Cognitive Level: Understanding OBJ: Nursing Process Step: Assessment MSC: Patient Needs: Physiologic Integrity 15. Which data found on a patient‘s health history would place her at risk for an ectopic pregnancy? a. Ovarian cyst 2 years ago b. Recurrent pelvic infections c. Use of oral contraceptives for 5 years d. Heavy menstrual flow of 4 days‘ duration ANS: BInfection and subsequent scarring of the fallopian tubes prevent normal movement of the fertilized ovum into the uterus for implantation. Ovarian cysts do not cause scarring of the fallopian tubes. Oral contraceptives do not increase the risk for ectopic pregnancies. Heavy menstrual flow of 4 days‘ duration will not cause scarring of the fallopian tubes, which is the main risk factor for ectopic pregnancies. DIF: Cognitive Level: Understanding OBJ: Nursing Process Step: Assessment MSC: Patient Needs: Physiologic Integrity 16. Which finding on a prenatal visit at 10 weeks might suggest a hydatidiform mole? a. Blood pressure of 120/80 mm Hg b. Complaint of frequent mild nausea c. Fundal height measurement of 18 cm d. History of bright red spotting for 1 day weeks ago ANS: C The uterus in a hydatidiform molar pregnancy is often larger than would be expected on the basis of the duration of the pregnancy. A patient with a molar pregnancy may have early-onset, pregnancy-induced hypertension. Nausea increases in a molar pregnancy because of the increased production of human chorionic gonadotropin (hCG). The history of bleeding is normally described as being of a brownish color. DIF: Cognitive Level: Analysis OBJ: Nursing Process Step: Assessment MSC: Patient Needs: Health Promotion and Maintenance 17. Which routine nursing assessment is contraindicated for a patient admitted with suspected placenta previa? a. Determining cervical dilation and effacement b. Monitoring FHR and maternal vital signs c. Observing vaginal bleeding or leakage of amniotic fluid d. Determining frequency, duration, and intensity of contractions ANS: A Vaginal examination of the cervix may result in perforation of the placenta and subsequent hemorrhage. Monitoring FHR and maternal vital signs is a necessary part of the assessment for this patient. Monitoring for bleeding and rupture of membranes is not contraindicated with this patient. Monitoring contractions is not contraindicated with this patient. DIF: Cognitive Level: Analysis OBJ: Nursing Process Step: Assessment MSC: Patient Needs: Physiologic Integrity 18. A laboratory finding indicative of DIC is one that shows a. decreased fibrinogen. b. increased platelets. c. increased hematocrit. d. decreased thromboplastin time. ANS: ADIC develops when the blood-clotting factor thromboplastin is released into the maternal bloodstream as a result of placental bleeding. Thromboplastin activates widespread clotting, which uses the available fibrinogen, resulting in a decreased fibrinogen level. The platelet count will decrease. The hematocrit may decrease if bleeding is pronounced. The thromboplastin time is prolonged. DIF: Cognitive Level: Analysis OBJ: Nursing Process Step: Assessment MSC: Patient Needs: Physiologic Integrity 19. Which assessment in a patient diagnosed with preeclampsia who is taking magnesium sulfate would indicate a therapeutic level of medication? a. Drowsiness b. Urinary output of 20 mL/hour c. Normal deep tendon reflexes d. Respiratory rate of 10 to 12 breaths per minute ANS: C Magnesium sulfate is administered for preeclampsia to reduce the risk of seizures from cerebral irritability. Hyperreflexia (deep tendon reflexes above normal) is a symptom of cerebral irritability. If the dosage of magnesium sulfate is effective, reflexes should decrease to normal or slightly below normal levels. Drowsiness is another sign of CNS depression from magnesium toxicity. A urinary output of 20 mL/hour is inadequate output. A respiratory rate of 10 to 12 breaths per minute is too slow and could be indicative of magnesium toxicity. DIF: Cognitive Level: Analysis OBJ: Nursing Process Step: Assessment MSC: Patient Needs: Physiologic Integrity 20. A patient taking magnesium sulfate has a respiratory rate of 10 breaths per minute. In addition to discontinuing the medication, which action should the nurse take? a. Increase the patient‘s IV fluids. b. Administer calcium gluconate. c. Vigorously stimulate the patient. d. Instruct the patient to take deep breaths. ANS: B Calcium gluconate reverses the effects of magnesium sulfate. Increasing the patient‘s IV fluids will not reverse the effects of the medication. Stimulation will not increase the respirations. Deep breaths will not be successful in reversing the effects of the magnesium sulfate. DIF: Cognitive Level: Application OBJ: Nursing Process Step: Implementation MSC: Patient Needs: Physiologic Integrity 21. A 32-year-old primigravida is admitted with a diagnosis of ectopic pregnancy. Nursing care is based on which of the following? a. Hemorrhage is the primary concern. b. She will be unable to conceive in the future. c. Bed rest and analgesics are the recommended treatment. d.A D&C will be performed to remove the products of conception. ANS: A Severe bleeding occurs if the fallopian tube ruptures. If the tube must be removed, the patient‘s fertility will decrease; however, she will be able to achieve a future pregnancy. The recommended treatment is to remove the pregnancy before hemorrhage occurs. A D&C is done on the inside of the uterine cavity. The ectopic is located within the tubes. DIF: Cognitive Level: Understanding OBJ: Nursing Process Step: Planning MSC: Patient Needs: Physiologic Integrity 22. A patient who was pregnant had a spontaneous abortion at approximately 4 weeks‘ gestation. At the time of the miscarriage, it was thought that all products of conception were expelled. Two weeks later, the patient presents at the clinic office complaining of ―crampy‖ abdominal pain and a scant amount of serosanguineous vaginal drainage with a slight odor. The pregnancy test is negative. Vital signs reveal a temperature of 100F, with blood pressure of 100/60 mm Hg, irregular pulse 88 beats/minute (bpm), and respirations, 20 breaths per minute. Based on this assessment data, what does the nurse anticipate as a clinical diagnosis? a. Ectopic pregnancy b. Uterine infection c. Gestational trophoblastic disease d. Endometriosis ANS: B The patient is exhibiting signs of uterine infection, with elevated temperature, vaginal discharge with odor, abdominal pain, and blood pressure and pulse manifesting as shock-trended vitals. Because the pregnancy test is negative, an undiagnosed ectopic pregnancy and gestational trophoblastic disease are ruled out. There is no supportive evidence to indicate a clinical diagnosis of endometriosis at this time; however, it is more likely that this is an infectious process that must be aggressively treated. DIF: Cognitive Level: Analysis OBJ: Nursing Process Step: Diagnosis MSC: Patient Needs: Physiologic Integrity/Physiologic Adaptation 23. A patient with no prenatal care delivers a healthy male infant via the vaginal route, with minimal blood loss. During the labor period, vital signs were normal. At birth, significant maternal hypertension is noted. When the patient is questioned, she relates that there is history of heart disease in her family; but, that she has never been treated for hypertension. Blood pressure is treated in the hospital setting and the patient is discharged. The patient returns at her scheduled 6-week checkup and is found to be hypertensive. Which type of hypertension is the patient is exhibiting? a. Pregnancy-induced hypertension (PIH) b. Gestational hypertension c. Preeclampsia superimposed on chronic hypertension d. Undiagnosed chronic hypertension ANS: D Even though the patient has no documented prenatal care or medical history, she does relate a family history that is positive for heart disease. Additionally, the patient‘s blood pressure increased following birth and was treated in the hospital and resolved. Now the patient appearsat the 6-week checkup with hypertension. Typically, gestational hypertension resolves by the end of the 6-week postpartum period. The fact that this has not resolved is suspicious for undiagnosed chronic hypertension. There is no evidence to suggest that the patient was preeclamptic prior to the birth. DIF: Cognitive Level: Analysis OBJ: Nursing Process Step: Diagnosis MSC: Patient Needs: Physiologic Integrity/Physiologic Adaptation 24. A high-risk labor patient progresses from preeclampsia to eclampsia. Aggressive management is instituted, and the fetus is delivered via cesarean birth. Which finding in the immediate postoperative period indicates that the patient is at risk of developing HELLP syndrome? a. Platelet count of 50,000/mcL b. Liver enzyme levels within normal range c. Negative for edema d. No evidence of nausea or vomiting ANS: A HELLP syndrome is characterized by Hemolysis, Elevated Liver enzyme levels, and a Low platelet count. A platelet count of 50,000/mcL indicates thrombocytopenia. DIF: Cognitive Level: Analysis OBJ: Nursing Process Step: Assessment MSC: Patient Needs: Physiologic Integrity/Pathophysiology 25. As the triage nurse in the emergency room, you are reviewing results for the high-risk obstetric patient who is in labor because of traumatic injury experienced as a result of a motor vehicle accident (MVA). You note that the Kleihauer–Betke test is positive. Based on this information, you anticipate that a. immediate birth is required. b. the patient should be transferred to the critical care unit for closer observation. c. RhoGAM should be administered. d. a tetanus shot should be administered. ANS: A A positive Kleihauer-Betke test indicates that fetal bleeding is occurring in the maternal circulation. This is a serious complication and, because the patient is a trauma victim, it is highly likely that she is experiencing an abruption. Therefore the patient should be delivered as quickly as possible to improve outcomes. There is no evidence to support that RhoGAM should be administered, because we have no information related to Rh factor and/or blood type. Similarly, a tetanus shot is not indicated at this time because there is no evidence of penetrating trauma. The patient should be transferred to the obstetric area for birth, not the critical care unit setting. DIF: Cognitive Level: Analysis OBJ: Nursing Process Step: Evaluation MSC: Patient Needs: Physiologic Integrity: Medical Emergencies 26. A patient who had premature rupture of the membranes (PROM) earlier in the pregnancy at 28 weeks returns to the labor unit 1 week later complaining that she is now in labor. The labor and birth nurse performs the following assessments. The vaginal exam is deferred until the physician is in attendance. The patient is placed on electronic fetal monitoring (EFM) and a baseline FHR of 130 bpm is noted. No contraction pattern is observed. The patient is then transferred to the antepartum unit for continued observation. Several hours later, the patientcomplains that she does not feel the baby move. Examination of the abdomen reveals a fundal height of 34 cm. Muscle tone is no different from earlier in the hospital admission. The patient is placed on the EFM and no fetal heart tones are observed. What does the nurse suspect is occurring? a. Placental previa b. Active labor has started c. Placental abruption d. Hidden placental abruption ANS: D The patient‘s signs and symptoms indicate that a hidden abruption is occurring. Fundal height has increased and there is an absence of fetal heart tones. This is a medical emergency and the physician should be contacted to come directly to the unit for intervention and imminent birth. DIF: Cognitive Level: Analysis OBJ: Nursing Process Step: Evaluation MSC: Patient Needs: Physiologic Integrity: Medical Emergencies 27. What is the priority nursing intervention for the patient who has had an incomplete abortion? a. Methylergonovine (Methergine), 0.2 mg IM b. Preoperative teaching for surgery c. Insertion of IV line for fluid replacement d. Positioning of patient in left side-lying position ANS: C Initial treatment of an incomplete abortion should be focused on stabilizing the patient‘s cardiovascular state. Methylergonovine would be administered after surgical treatment, preoperative teaching is not a priority until the patient is stabilized, and the left side-lying position provides no benefit to the patient in this situation. DIF: Cognitive Level: Application OBJ: Nursing Process Step: Implementation MSC: Patient Needs: Safe and Effective Care Environment: Management of Care 28. Which finding in the assessment of a patient following an abruption placenta could indicate a major complication? a. Urine output of 30 mL in 1 hour b. Blood pressure of 110/60 mm Hg c. Bleeding at IV insertion site d. Respiratory rate of 16 breaths per minute ANS: C DIC is a life-threatening defect in coagulation that may occur following abruptio placentae. DIC allows excess bleeding from any vulnerable area such as IV sites, incisions, gums, or nose. A urine output of 30 mL in 1 hour, blood pressure of 110/60 mm Hg, and respiratory rate of 16 breaths per minute are normal findings in a postpartum patient. DIF: Cognitive Level: Analysis OBJ: Nursing Process Step: AssessmentMSC: Patient Needs: Physiologic Integrity/Physiologic Adaptation 29. Which assessment by the nurse would differentiate a placenta previa from an abruptio placentae? a. Saturated perineal pad in 1 hour b. Pain level 0 on a scale of 0 to 10 c. Cervical dilation at 2 cm d. Fetal heart rate at 160 bpm ANS: B The classic sign of placenta previa is the sudden onset of painless uterine bleeding, whereas abruptio placentae results in abdominal pain and uterine tenderness; heavy bleeding, cervical dilation, and fetal heart rate of 160 bpm could be associated with both conditions. DIF: Cognitive Level: Analysis OBJ: Nursing Process Step: Analysis MSC: Patient Needs: Safe and Effective Care Environment: Management of Care 30. A blood-soaked peripad weighs 900 g. The nurse would document a blood loss of mL. a. 1800 b. 450 c. 900 d. 90 ANS: C One g equals 1 mL of blood. DIF: Cognitive Level: Application OBJ: Nursing Process Step: Evaluation MSC: Patient Needs: Health Promotion and Maintenance 31. Which intervention is the priority for the patient diagnosed with an intact tubal pregnancy? a. Assessment of pain level b. Administration of methotrexate c. Administration of Rh immune globulin d. Explanation of the common side effects of the treatment plan ANS: B The goal of medical management of an intact tube is to preserve the tube and improve the chance of future fertility. Methotrexate (a folic acid antagonist) is used to inhibit cell division and stop growth of the embryo. Assessment of pain level, administration of Rh immune globulin, and explaining common side effects of the treatment plan should be implemented in conjunction with or soon after treatment with methotrexate has begun. DIF: Cognitive Level: Analysis OBJ: Nursing Process Step: Analysis MSC: Patient Needs: Safe and Effective Care Environment: Management of Care 32. Which finding in the exam of a patient with a diagnosis of threatened abortion would change the diagnosis to inevitable abortion? a.Presence of backache b. Rise in hCG level c. Clear fluid from vagina d. Pelvic pressure ANS: C Clear fluid from the vagina indicates rupture of the membranes. Abortion is usually inevitable (cannot be stopped) when the membranes rupture, the presence of backache and pelvic pressure are common symptoms in threatened abortion, and a rise in the hCG level is consistent with a viable pregnancy. DIF: Cognitive Level: Analysis OBJ: Nursing Process Step: Analysis MSC: Patient Needs: Physiologic Integrity/Physiologic Adaptation 33. What should the nurse recognize as evidence that the patient is recovering from preeclampsia? a. 1+ protein in urine b. 2+ pitting edema in lower extremities c. Urine output >100 mL/hour d. Deep tendon reflexes +2 ANS: C Rapid reduction of the edema associated with preeclampsia results in urinary output of 4 to 6 L/day as interstitial fluids shift back to the circulatory system. 1+ protein in urine and 2+ pitting edema in lower extremities are signs of continuing preeclampsia. Deep tendon reflexes are not a reliable sign, especially if the patient has been treated with magnesium. DIF: Cognitive Level: Analysis OBJ: Nursing Process Step: Analysis MSC: Patient Needs: Physiologic Integrity/Physiologic Adaptation 34. Fraternal twins are delivered by your Rh-negative patient. Twin A is Rh-positive and twin B is Rh-negative. Prior to administering Rho(D) immune globulin (RhoGAM), the nurse should determine the results of the a. direct Coombs test of twin A. b. direct Coombs test of twin B. c. indirect Coombs test of the mother. d. transcutaneous bilirubin level for both twins. ANS: C Administration of RhoGAM is based on the results of the indirect Coombs test on the patient. A negative result confirms that the mother has not been sensitized by the positive Rh factor of twin A and that RhoGAM is indicated. A direct Coombs test is a diagnostic test used to determine maternal antibodies in fetal blood and to guide treatment of the newborn when Rh and ABO incompatibilities occur. Transcutaneous bilirubin is a noninvasive measure to determine the level of bilirubin in a newborn. DIF: Cognitive Level: Analysis OBJ: Nursing Process Step: Analysis MSC: Patient Needs: Physiologic Integrity/Pharmacologic and Parenteral Therapies35. For the patient who delivered at 6:30 AM on January 10, Rho(D) immune globulin (RhoGAM) must be administered prior to a. 6:30 AM on January 13. b. 6:30 PM on January 13. c. 6:30 PM on January 14. d. 6:30 AM on January 15. ANS: A Rho(D) immune globulin (RhoGAM) must be administered within 72 hours after the birth of an Rh-positive infant. 6:30 PM on January 13, 6:30 PM on January 14, and 6:30 AM on January 15 do not fall within the established timeframe. DIF: Cognitive Level: Application OBJ: Nursing Process Step: Implementation MSC: Patient Needs: Physiologic Integrity/Pharmacologic and Parenteral Therapies 36. The labor and birth nurse is reviewing the risk factors for placenta previa with a group of nursing students. The nurse determines that the students understood the discussion when they identify which patient being at the highest risk for developing a placenta previa? a. Female fetus, Mexican-American, primigravida b. Male fetus, Asian-American, previous preterm birth c. Male fetus, African-American, previous cesarean birth d. Female fetus, European-American, previous spontaneous abortion ANS: C The rate of placenta previa is increasing. It is more common in older women, multiparous women, women who have had cesarean births, and women who had suction curettage for an induced or spontaneous abortion. It is also more likely to recur if a woman has had a placenta previa. African or Asian ethnicity also increases the risk. Cigarette smoking and cocaine use are personal habits that add to a woman‘s risk for a previa. Previa is more likely if the fetus is male. The Mexican-American primipara has no risk factors for developing a placenta previa. The Asian-American multipara has two risk factors for developing a previa. The African-American multipara has three risk factors for developing a previa. The European-American multigravida has one risk factor for developing a placenta previa. DIF: Cognitive Level: Synthesis OBJ: Nursing Process Step: Analysis MSC: Patient Needs: Health Promotion and Maintenance 37. A labor and birth nurse receives a call from the laboratory regarding a preeclamptic patient receiving an IV infusion of magnesium sulfate. The laboratory technician reports that the patient‘s magnesium level is 7.6 mg/dL. What is the nurse‘s priority action? a. Stop the infusion of magnesium. b. Assess the patient‘s respiratory rate. c. Assess the patient‘s deep tendon reflexes. d. Notify the health care provider of the magnesium level. ANS: BThe therapeutic serum level for magnesium is 4 to 8 mg/dL although it is elevated in terms of normal lab values. Adverse reactions to magnesium sulfate usually occur if the serum level becomes too high. The most important is CNS depression, including depression of the respiratory center. Magnesium is excreted solely by the kidneys, and the reduced urine output that often occurs in preeclampsia allows magnesium to accumulate to toxic levels in the woman. Frequent assessment of serum magnesium levels, deep tendon reflexes, respiratory rate, and oxygen saturation can identify CNS depression before it progresses to respiratory depression or cardiac dysfunction. Monitoring urine output identifies oliguria that would allow magnesium to accumulate and reach excessive levels. Discontinue magnesium if the respiratory rate is below 12 breaths per minute, a low pulse oximeter level (<95%) persists, or deep tendon reflexes are absent. Additional magnesium will make the condition worse. DIF: Cognitive Level: Analysis OBJ: Nursing Process Step: Assessment MSC: Patient Needs: Health Promotion and Maintenance 38. Which factor is most important in diminishing maternal, fetal, and neonatal complications in a pregnant patient with diabetes? a. Evaluation of retinopathy by an ophthalmologist b. The patient‘s stable emotional and psychological status c. Degree of glycemic control before and during the pregnancy d. Total protein excretion and creatinine clearance within normal limits ANS: C The occurrence of complications can be greatly diminished by maintaining normal blood glucose levels before and during the pregnancy. Even nonpregnant diabetics should have an annual eye examination. Assessing a patient‘s emotional status is helpful. Coping with a pregnancy superimposed on preexisting diabetes can be very difficult for the whole family; however, it is not the top priority. Baseline renal function is assessed with a 24-hour urine collection and does not diminish the patient‘s risk for complications. DIF: Cognitive Level: Understanding OBJ: Nursing Process Step: Assessment MSC: Patient Needs: Health Promotion and Maintenance 39. Which major neonatal complication is carefully monitored after the birth of the infant of a diabetic mother? a. Hypoglycemia b. Hypercalcemia c. Hypoinsulinemia d. Hypobilirubinemia ANS: A The neonate is at highest risk for hypoglycemia because fetal insulin production is accelerated during pregnancy to metabolize excessive glucose from the mother. At birth, the maternal glucose supply stops, and the neonatal insulin exceeds the available glucose, leading to hypoglycemia. Hypocalcemia is associated with preterm birth, birth trauma, and asphyxia, all common problems of the infant of a diabetic mother. Because fetal insulin production is accelerated during pregnancy, the neonate shows hyperinsulinemia. Excess erythrocytes are broken down after birth, releasing large amounts of bilirubin into the neonate‘s circulation, which results in hyperbilirubinemia.DIF: Cognitive Level: Understanding OBJ: Nursing Process Step: Planning MSC: Patient Needs: Health Promotion and Maintenance 40. Which factor is known to increase the risk of gestational diabetes mellitus? a. Previous birth of large infant b. Maternal age younger than 25 years c. Underweight prior to pregnancy d. Previous diagnosis of type 2 diabetes mellitus ANS: A Prior birth of a large infant suggests gestational diabetes mellitus. A patient younger than 25 is not at risk for gestational diabetes mellitus. Obesity (>90 kg [198 lb]) creates a higher risk for gestational diabetes. The person with type 2 diabetes mellitus already is a diabetic and will continue to be so after pregnancy. Insulin may be required during pregnancy because oral hypoglycemia drugs are contraindicated during pregnancy. DIF: Cognitive Level: Understanding OBJ: Nursing Process Step: Assessment MSC: Patient Needs: Health Promotion and Maintenance 41. Which disease process improves during pregnancy? a. Epilepsy b. Bell‘s palsy c. Rheumatoid arthritis d. Systemic lupus erythematosus (SLE) ANS: C Although the reason is unclear, marked improvement is seen with rheumatoid arthritis in pregnancy. Unfortunately relapse occurs within 36 months postpartum. With epilepsy, the effect of pregnancy is variable and unpredictable. Seizures may increase, decrease, or remain the same. Bell‘s palsy was thought to be the result of infection by a virus three times more common during pregnancy and generally occurring in the third trimester. The patient with SLE can have a normal pregnancy but must be treated as high risk because 50% of all births will be premature. Pregnancy can exacerbate SLE. DIF: Cognitive Level: Understanding OBJ: Nursing Process Step: Assessment MSC: Patient Needs: Health Promotion and Maintenance 42. Nursing intervention for pregnant patients with diabetes is based on the knowledge that the need for insulin is a. varied depending on the stage of gestation. b. increased throughout pregnancy and the postpartum period. c. decreased throughout pregnancy and the postpartum period. d. should not change because the fetus produces its own insulin. ANS: AInsulin needs decrease during the first trimester, when nausea, vomiting, and anorexia are a factor. Insulin needs increase during the second and third trimesters, when the hormones of pregnancy create insulin resistance in maternal cells. Insulin needs change during pregnancy. DIF: Cognitive Level: Understanding OBJ: Nursing Process Step: Implementation MSC: Patient Needs: Physiologic Integrity 43. Which form of heart disease in women of childbearing years usually has a benign effect on pregnancy? a. Cardiomyopathy b. Mitral valve prolapse c. Rheumatic heart disease d. Congenital heart disease ANS: B Mitral valve prolapse is a benign condition that is usually asymptomatic. Cardiomyopathy produces congestive heart failure during pregnancy. Rheumatic heart disease can lead to heart failure during pregnancy. Some congenital heart diseases will produce pulmonary hypertension or endocarditis during pregnancy. DIF: Cognitive Level: Understanding OBJ: Nursing Process Step: Assessment MSC: Patient Needs: Physiologic Integrity 44. Which instructions should the nurse include when teaching a pregnant patient with Class II heart disease? a. Advise her to gain at least 30 lb. b. Instruct her to avoid strenuous activity. c. Inform her of the need to limit fluid intake. d. Explain the importance of a diet high in calcium. ANS: B Activity may need to be limited so that cardiac demand does not exceed cardiac capacity. Weight gain should be kept at a minimum with heart disease. Iron and folic acid are important to prevent anemia. Fluid intake is necessary to prevent fluid deficits. Fluid intake should not be limited during pregnancy. The patient may also be put on a diuretic. DIF: Cognitive Level: Understanding OBJ: Nursing Process Step: Implementation MSC: Patient Needs: Physiologic Integrity 45. Anti-infective prophylaxis is indicated for a pregnant patient with a history of mitral valve stenosis related to rheumatic heart disease because the patient is at risk of developing a. hypertension. b. postpartum infection. c. bacterial endocarditis. d. upper respiratory infections. ANS: CBecause of vegetations on the leaflets of the mitral valve and the increased demands of pregnancy, the patient is at greater risk of bacterial endocarditis. Pulmonary hypertension may occur with mitral valve stenosis, but anti-infective medications will not prevent it from occurring. Women with cardiac problems must be observed for possible infections during the postpartum period but are not given prophylactic antibiotics to prevent them. Women are not put on prophylactic antibiotics to prevent upper respiratory infections. DIF: Cognitive Level: Understanding OBJ: Nursing Process Step: Planning MSC: Patient Needs: Physiologic Integrity 46. A patient, who delivered her third child yesterday, has just learned that her two school-age children have contracted chickenpox. What should the nurse tell her? a. Her two children should be treated with acyclovir before she goes home from the hospital. b. The baby will acquire immunity from her and will not be susceptible to chickenpox. c. The children can visit their mother and baby in the hospital as planned but must wear gowns and masks. d. She must make arrangements to stay somewhere other than her home until the children are no longer contagious. ANS: D Varicella (chickenpox) is highly contagious. Although the baby inherits immunity from the mother, it would not be safe to expose either the mother or the baby. Acyclovir is used to treat varicella pneumonia. The baby is already born and has received the immunity. If the mother never had chickenpox, she cannot transmit the immunity to the baby. Varicella infection occurring in a newborn may be life-threatening. DIF: Cognitive Level: Application OBJ: Nursing Process Step: Implementation MSC: Patient Needs: Safe and Effective Care Environment 47. A patient has a history of drug use and is screened for hepatitis B during the first trimester. Which action is most appropriate? a. Practice respiratory isolation. b. Plan for retesting during the third trimester. c. Discuss the recommendation to bottle feed her baby. d. Anticipate administering the vaccination for hepatitis B as soon as possible. ANS: B A person who has a history of high-risk behaviors should be rescreened during the third trimester. Hepatitis B is transmitted through blood. The first trimester is too early to discuss feeding methods with a woman in the high-risk category. The vaccine may not have time to affect a person with high-risk behaviors. DIF: Cognitive Level: Application OBJ: Nursing Process Step: Planning MSC: Patient Needs: Health Promotion and Maintenance 48. A patient has tested HIV-positive and has now discovered that she is pregnant. Which statement indicates that she understands the risks of this diagnosis? a. ―I know I will need to have an abortion as soon as possible.‖ b. ―Even though my test is positive, my baby might not be affected.‖c. ―My baby is certain to have AIDS and die within the first year of life.‖ d. ―This pregnancy will probably decrease the chance that I will develop AIDS.‖ ANS: B The fetus is likely to test positive for HIV in the first 6 months, until the inherited immunity from the mother wears off. Many of these babies will convert to HIV-negative status. With the newer drugs, the risk for infection of the fetus has decreased. Also, the life span of an infected newborn has increased. The pregnancy will increase the chance of converting. DIF: Cognitive Level: Analysis OBJ: Nursing Process Step: Evaluation MSC: Patient Needs: Physiologic Integrity 49. Examination of a newborn in the birth room reveals bilateral cataracts. Which disease process in the maternal history would likely cause this abnormality? a. Rubella b. Cytomegalovirus (CMV) c. Syphilis d. HIV ANS: A Transmission of congenital rubella causes serious complications in the fetus that may manifest as cataracts, cardiac defects, microcephaly, deafness, intrauterine growth restriction (IUGR), and developmental delays. DIF: Cognitive Level: Application OBJ: Nursing Process Step: Assessment MSC: Patient Needs: Physiologic Integrity/Physiologic Adaptation 50. Which postpartum patient requires further assessment? a. G4 P4 who has had four saturated pads during the last 12 hours b. G1 P1 with Class II heart disease who complains of frequent coughing c. G2 P2 with gestational diabetes whose fasting blood sugar level is 100 mg/dL d. G3 P2 postcesarean patient who has active herpes lesions on the labia ANS: B Frequent coughing may be a sign of congestive heart failure in the postpartum patient with heart disease. Four saturated pads in a 4-hour period is acceptable postpartum blood loss, a fasting blood sugar is a normal value, and the patient with identified active herpes does not require further assessment. DIF: Cognitive Level: Analysis OBJ: Nursing Process Step: Analysis MSC: Patient Needs: Health Promotion and Maintenance 51. The nurse is reviewing the instructions given to a patient at 24 weeks‘ gestation for a glucose challenge test (GCT). The nurse determines that the patient understands the teaching when she makes which statement? a. ―I have to fast the night before the test.‖ b. ―I will drink a sugary solution containing 100 g of glucose.‖ c.―I will have blood drawn at 1 hour after I drink the glucose solution.‖ d. ―I should keep track of my baby‘s movements between now and the test.‖ ANS: C A GCT is administered between 24 and 28 weeks of gestation, often to low- and high-risk antepartum patients. Fasting is not necessary for a GCT, and the woman is not required to follow any pretest dietary instructions. The woman should ingest 50 g of oral glucose solution, and 1 hour later a blood sample is taken. Fetal surveillance with kick counts is an ongoing evaluation for pregnant women; they should contact their health care provider if there is a noticeable decrease in fetal movement. DIF: Cognitive Level: Application OBJ: Nursing Process Step: Evaluation MSC: Patient Needs: Health Promotion and Maintenance 52. The labor nurse is admitting a patient in active labor with a history of genital herpes. On assessment, the patient reports a recent outbreak, and the nurse verifies lesions on the perineum. What is the nurse‘s next action? a. Ask the patient when she last had anything to eat or drink. b. Take a culture of the lesions to verify the involved organism. c. Ask the patient if she has had unprotected sex since her outbreak. d. Use electronic fetal surveillance to determine a baseline fetal heart rate. ANS: A A cesarean birth is recommended for women with active lesions in the genital area, whether recurrent or primary, at the time of labor. The patient‘s dietary intake is needed to prepare for surgery. This patient is in active labor and the fetus is at risk for infection if the membranes rupture. The health care provider needs to be notified, and a cesarean birth needs to be performed as soon as possible. There is no need to validate the infection because the patient is well aware of the symptoms of an active infection. Although transmission to sexual partners is valid information, it is not necessary information in an urgent situation such as depicted in this scenario. Electronic fetal surveillance is the standard of care. DIF: Cognitive Level: Synthesis OBJ: Nursing Process Step: Assessment MSC: Patient Needs: Health Promotion and Maintenance MULTIPLE RESPONSE 1. The nurse is monitoring a patient with severe preeclampsia who is on IV magnesium sulfate. Which signs of magnesium toxicity should the nurse monitor for? (Select all that apply.) a. Cool, clammy skin b. Altered sensorium c. Pulse oximeter reading of 95% d. Respiratory rate of less than 12 breaths per minute e. Absence of deep tendon reflexes ANS: B, D, E Signs of magnesium toxicity include the following: Respiratory rate of less than 12 breaths per minute (hospitals may specify a rate <14 breaths per minute)Maternal pulse oximeter reading lower than 95% Absence of deep tendon reflexes Sweating, flushing Altered sensorium (confused, lethargic, slurred speech, drowsy, disoriented) Hypotension Serum magnesium value above the therapeutic range of 4 to 8 mg/dL Cold, clammy skin and a pulse oximeter reading of 95% would not be signs of toxicity. DIF: Cognitive Level: Analysis OBJ: Nursing Process Step: Evaluation MSC: Patient Needs: Physiologic Integrity 2. The rate of obesity in the United States has reached epidemic proportions. Morbidity and mortality for both the mother and baby are increased in these circumstances. The nurse caring for the patient with an elevated BMI should be cognizant of these potential complications and plan care accordingly. Significant risks include (Select all that apply.) a. Breech presentation b. Ectopic pregnancy c. Birth defects d. Venous thromboembolism e. Postpartum anemia ANS: C, D, E Maternal complications associated with pregnancy include: Gestational diabetes, preeclampsia, venous thromboembolism, Caesarean delivery, wound infection, respiratory complications, preterm birth, birth trauma and postpartum anemia. Obese women also have an increased risk of spontaneous abortions and stillbirth. Complications for infants of obese mothers have an increased risk of neural tube defects, hydrocephaly, cardiovascular defects, macrosomia, hypoglycemia, and birth injuries from shoulder dystocia. DIF: Cognitive Level: Analysis OBJ: Nursing Process Step: Planning MSC: Patient Needs: Physiologic Integrity COMPLETION 1. What is the value of the main line fluid rate for your patient, whose total fluid intake is ordered at 150 mL/hour and who is also being given magnesium sulfate at 1 g/hour (1 g = 25 mL/hour) IV piggyback and pitocin at 15 mU/minute (l mU/minute = 1 mL/hour) IV piggyback. ANS: 110 The rate of infusion of magnesium sulfate (25 mL/hour) and pitocin (15 mL/hour) equals 40 mL/hour. Subtracting the 40 mL from the total ordered of 150 mL leaves 110 mL of main line fluid to be infused per hour. DIF: Cognitive Level: Application OBJ: Nursing Process Step: Implementation MSC: Patient Needs: Physiologic Integrity/Pharmacologic and Parenteral TherapiesChapter 11: The Childbearing Family with Special Needs Foundations of Maternal-Newborn & Women‘s Health Nursing, 7th Edition MULTIPLE CHOICE 1. A pregnant patient who abuses cocaine admits to exchanging sex for her drug habit. This behavior puts her at a greater risk for a. postmature birth. b. sexually transmitted diseases. c. hypotension and vasodilation. d. depression of the central nervous system. ANS: B Sex acts exchanged for drugs place the woman at increased risk for sexually transmitted diseases because of having multiple partners and lack of protection. Premature delivery of the infant is one of the most common problems associated with cocaine use during pregnancy. Cocaine causes hypertension and vasoconstriction. Cocaine is a central nervous system stimulant. DIF: Cognitive Level: Understanding OBJ: Nursing Process Step: Assessment MSC: Patient Needs: Health Promotion and Maintenance 2. Which factor is a major barrier to health care for adolescent mothers? a. Health care workers have a positive attitude. b. The hospital or clinic is within walking distance of the girl‘s home. c. Seeing a different nurse and/or health care provider at every visit. d. The institution is open days, evenings, and Saturday by special arrangement. ANS: C Whenever possible, the teen should be scheduled to see the same nurses and practitioners for continuity of care. A positive attitude of the health care providers is important in teen pregnancy care. If the hospital or clinic were within walking distance of the girl‘s home, it would prevent the teen from missing appointments due to transportation problems. If the institution were open days, evenings, and Saturday by special arrangement, this would be helpful for teens who work, go to school, or have other time of day restrictions. Scheduling conflicts are a major barrier to health care. DIF: Cognitive Level: Understanding OBJ: Nursing Process Step: Assessment MSC: Patient Needs: Psychosocial Integrity 3. In planning sex education classes for the middle school age group, more emphasis should be placed on a. how to set limits for sexual behavior. b. the inaccuracy of information from peers. c. the use of oral contraceptives to prevent unwanted pregnancy. d. the use of condoms to prevent sexually transmitted diseases as well as pregnancy. ANS: ASetting limits for sexual behavior is particularly important for younger teenagers who may be pressured to become sexually active before they are physically and emotionally ready. Oral contraceptives are not the preferred method of birth control for teenagers because they forget to take them, and they do not protect against STIs. The use of condoms is appropriate and an important concept to discuss but should not be the emphasis. DIF: Cognitive Level: Understanding OBJ: Nursing Process Step: Planning MSC: Patient Needs: Health Promotion and Maintenance 4. Which action should the nurse take when counseling a teenaged patient who has decided to relinquish her baby for adoption? a. Question her about her feelings regarding adoption. b. Tell her she can always change her mind about adoption. c. Affirm her decision while acknowledging her maturity in making it. d. Ask her if anyone is coercing her into the decision to relinquish her baby. ANS: C A supportive affirming approach by the nurse will strengthen the patient‘s resolve and help her appreciate the significance of the event. It is important for the nurse to support and affirm the decision the patient has made. This will strengthen the patient‘s resolve to follow through. Later the patient should be given an opportunity to express her feelings. Telling her that she can always change her mind about adoption should not be an option after the baby is born and placed with the adoptive parents. It is important that the teenager be treated as an adult, with the assumption that she is capable of making an important decision on her own. DIF: Cognitive Level: Application OBJ: Nursing Process Step: Implementation MSC: Patient Needs: Psychosocial Integrity 5. A patient who is older than 35 years may have difficulty achieving pregnancy because a. prepregnancy medical attention is lacking. b. personal risk behaviors influence fertility. c. contraceptives have been used for an extended period of time. d. the ovaries may be affected by the normal aging process. ANS: D Once the mature woman decides to conceive, a delay in becoming pregnant may occur because of the normal aging of the ovaries. Prepregnancy medical care is available and encouraged. The older adult participates in fewer risk behaviors than the younger adult. The problem is the age of the ovaries, not the past use of contraceptives. Fertility begins to decline at age 32 and decreases more rapidly by age 37. DIF: Cognitive Level: Understanding OBJ: Nursing Process Step: Assessment MSC: Patient Needs: Physiologic Integrity 6. Which health concern is most likely to be an issue for the older mother? a. Nutrition and diet planning b. Exercise and fitness c. Having enough rest and sleepd. Effective contraceptive methods ANS: C The woman who delays childbearing may have unique concerns, one of which is having less energy than younger mothers. The older mother is better off financially and can afford better nutrition. Information about exercise and fitness is readily available. The older mother usually has more financial means to search out effective contraceptive methods. DIF: Cognitive Level: Understanding OBJ: Nursing Process Step: Assessment MSC: Patient Needs: Psychosocial Integrity 7. Which is the most dangerous effect on the fetus of a patient who smokes cigarettes while pregnant? a. Intrauterine growth restriction b. Genetic changes and anomalies c. Extensive central nervous system damage d. Fetal addiction to the substance inhaled ANS: A The major consequences of smoking tobacco during pregnancy are low-birth-weight infants, prematurity, and increased perinatal loss. Cigarettes normally will not cause genetic changes or extensive central nervous system damage. Addiction is not a normal concern with the neonate. DIF: Cognitive Level: Understanding OBJ: Nursing Process Step: Assessment MSC: Patient Needs: Health Promotion and Maintenance 8. A patient at 24 weeks of gestation reports that she has a glass of wine with dinner every evening. Which rationale should the nurse provide this patient regarding the necessity to eliminate alcohol intake? a. The fetus is placed at risk for altered brain growth. b. The fetus is at risk for severe nervous system injury. c. The patient will be at risk for abusing other substances as well. d. A daily consumption of alcohol indicates a risk for alcoholism. ANS: A The brain grows most rapidly in the third trimester and is most vulnerable to alcohol exposure during this time. The major concerns are mental retardation, learning disabilities, high activity level, and short attention span. The risk to the patient for abusing other substances is not the major risk for the infant. It has not been proven that daily consumption of alcohol indicates a risk for alcoholism. DIF: Cognitive Level: Application OBJ: Nursing Process Step: Implementation MSC: Patient Needs: Psychosocial Integrity 9. Which of the following is an example of healthy grieving? a. The mother exhibits an absence of crying or expression of feelings. b. The parents do not mention the baby in conversation with family members. c. The mother asks that the baby be taken away from the delivery area quickly.d. While holding the baby, the mother says to her husband, ―He has your eyes and nose.‖ ANS: D Attachment behaviors are necessary for healthy grieving. Absence of crying and not mentioning the baby may be signs of denial. By not seeing the baby, attachment and therefore healthy grieving will not occur. DIF: Cognitive Level: Understanding OBJ: Nursing Process Step: Assessment MSC: Patient Needs: Psychosocial Integrity 10. A patient has delivered twins. The first twin was stillborn, and the second is in the intensive care nursery, recovering quickly from respiratory distress. The patient is crying softly and says, ―I wish my baby could have lived.‖ What is the most therapeutic response? a. ―How soon do you plan to have another baby?‖ b. ―Don‘t be sad. At least you have one healthy baby.‖ c. ―I have a friend who lost a twin and she‘s doing just fine now.‖ d. ―I am so sorry about your loss. Would you like to talk about it?‖ ANS: D The nurse should recognize the woman‘s grief and its significance. Asking her about plans for another baby is denying the loss of the other infant. Pointing out the health of another baby is belittling her feelings. Stating that the nurse has a friend who lost a twin is denying the loss of the infant and her grief and belittling her feelings. DIF: Cognitive Level: Application OBJ: Nursing Process Step: Implementation MSC: Patient Needs: Psychosocial Integrity 11. Which action is the most appropriate nursing measure when a baby has an unexpected defect at birth? a. Remove the baby from the delivery area immediately. b. Inform the parents immediately that something is wrong. c. Tell the parents that the baby has to go to the nursery immediately. d. Explain the defect and show the baby to the parents as soon as possible. ANS: D Parents experience less anxiety when they are told about the defect as early as possible and are allowed to touch and hold the baby. The parents should be able to touch and hold the baby as soon as possible. The nurse should not take the baby away; this would raise anxiety levels of the parents. They should be told about the defect and allowed to see the baby. DIF: Cognitive Level: Application OBJ: Nursing Process Step: Planning MSC: Patient Needs: Psychosocial Integrity 12. Which environment can assist a pregnant teen to achieve the task of establishing a stable identity? a. Home schooling b. Alternative education program c. School-based mothers‘ programd. Continuing mainstream high school classes ANS: C A school-based mothers‘ program that provides peer support is important. Home schooling, alternative education, and continuing mainstream high school classes would not provide as much peer support. DIF: Cognitive Level: Analysis OBJ: Nursing Process Step: Planning MSC: Patient Needs: Psychosocial Integrity 13. Which complication of adolescent pregnancy should the nurse plan to monitor? a. Anemia b. Placenta previa c. Abruptio placenta d. Incompetent cervix ANS: A Adolescent pregnancies are at increased risk for anemia, nutritional deficiencies, pregnancyassociated hypertension, HIV and other STDs, short interval until next pregnancy, and depression. They do not have a higher incidence of placenta previa, abruptio placentae, or incompetent cervix. DIF: Cognitive Level: Application OBJ: Nursing Process Step: Planning MSC: Patient Needs: Physiologic Integrity 14. The nurse is seeing a 17-year-old female in the clinic for complaints of acne. The nurse plans on taking advantage of this teachable moment with the teen. Which topics will the nurse include in the teen‘s teaching plan? a. Smoking habits, folic acid intake, and heart disease b. Hyperlipidemia, distracted driving, and menstrual history c. Sexual activity, contraception, and screening for violence d. Optimum weight, hypothyroidism, and sexually transmitted diseases ANS: C All the topics mentioned are worthy of discussion. Sexual activity, contraception, and screening for violence have priority related to the age and gender of the patient. Adolescents are seen by health care providers for a variety of reasons before they become pregnant. Counseling to improve health for a future pregnancy should be offered to them during any health care visit. Smoking cessation, attaining optimum weight, folic acid intake, and screening for violence are topics that should be discussed with all young women so that any future pregnancy has the most positive outcome. DIF: Cognitive Level: Analysis OBJ: Nursing Process Step: Planning MSC: Patient Needs: Health Promotion and Maintenance 15. A patient has just acknowledged that she is 20 weeks pregnant and confides to the nurse that she has a daily heroin habit. The nurse discusses treatment options for the patient. Which patient statement requires follow-up? a. ―My plan is to visit the outpatient clinic daily for treatment.‖ b.―I will see my health care provider at least every 2 weeks.‖ c. ―My baby will not have to go through withdrawal when I take methadone.‖ d. ―With oral methadone, my baby and I are at decreased risk of infection.‖ ANS: C Pregnant women who use heroin are often prescribed an alternative drug such as methadone, a synthetic opiate. Methadone can be taken orally once daily and is long-acting, providing consistent blood levels to decrease the adverse fetal effects of wide swings in blood levels found with heroin use. Methadone also reduces the risk of infections from contaminated needles and drug-seeking behavior, such as prostitution. At therapeutic levels, it does not produce the euphoria or sedation of heroin and allows the woman to have a relatively normal lifestyle. The woman who receives a daily dose of methadone in a drug treatment program is more likely to receive prenatal care. However, the newborn must withdraw from methadone after birth. DIF: Cognitive Level: Application OBJ: Nursing Process Step: Evaluation MSC: Patient Needs: Health Promotion and Maintenance 16. Which data in the patient‘s history should the nurse recognize as being pertinent to a possible diagnosis of postpartum depression? a. Previous depressive episode b. Unexpected operative birth c. Ambivalence during the first trimester d. Second pregnancy in a 3-year period ANS: A A personal or family history of depression or other mental illness is a risk factor for postpartum depression. An operative birth, ambivalence during the first trimester, and two pregnancies in 3 years are not risk factors for postpartum depression. DIF: Cognitive Level: Analysis OBJ: Nursing Process Step: Analysis MSC: Patient Needs: Psychosocial Integrity MULTIPLE RESPONSE 1. Which nursing diagnoses may apply to the childbearing family with special needs? (Select all that apply.) a. Risk for spiritual distress b. Risk for injury c. Readiness for enhanced nutrition d. Ineffective breathing pattern e. Situational low self-esteem ANS: A, B, E A childbearing family with special needs may be at risk to develop spiritual distress, experience injury, and exhibit situational low self-esteem. There are no supportive data to hypothesize an ineffective breathing pattern and/or readiness for enhanced nutrition. DIF: Cognitive Level: Application OBJ: Nursing Process Step: Nursing Diagnosis MSC: Patient Needs: Health Promotion and Maintenance2. Many teens wait until the second or even third trimester to seek prenatal care. The nurse should understand that the reasons behind this delay include which of the following? (Select all that apply.) a. Continuing to deny the pregnancy b. Uncertainty about where to go for care c. Lack of realization that they are pregnant d. A desire to gain control over their situation e. Wanting to hide the pregnancy as long as possible ANS: A, B, C, E Denying the pregnancy, uncertainty about where to go for care, lack of realization of pregnancy, and wanting to hide the pregnancy are all valid reasons for the teen to delay seeking prenatal care. A desire to gain control is not a reason to delay seeking health care. DIF: Cognitive Level: Analysis OBJ: Nursing Process Step: Diagnosis MSC: Patient Needs: Psychosocial Integrity 3. Which characteristics of fetal alcohol syndrome (FAS) should the nurse expect to assess in affected neonates? (Select all that apply.) a. Hydrocephaly b. Low activity c. Epicanthal folds d. Short palpebral fissures e. Flat midface, with a low nasal bridge ANS: C, D, E Common facial anomalies associated with FAS include microcephaly, short palpebral fissures (the openings between the eyelids), epicanthal folds, flat midface with a low nasal bridge, indistinct philtrum (groove between the nose and upper lip), and a thin upper lip. Microcephaly is present, not hydrocephaly. Central nervous system impairment includes a high activity level, not a low one. DIF: Cognitive Level: Understanding OBJ: Nursing Process Step: Assessment MSC: Patient Needs: Physiologic Integrity 4. Which congenital defects in a newborn are associated with long-term parenting problems? (Select all that apply.) a. Polydactyl b. Cleft lip and palate c. Ventral septal defect d. Ambiguous genitalia ANS: B, DAlthough any defect in a newborn produces extreme concern and anxiety, certain defects are associated with long-term parenting problems. Accepting an infant with facial or genital anomalies is particularly difficult for the family and community. Polydactyl and ventral septal defects are reparable, with good outcomes. DIF: Cognitive Level: Analysis OBJ: Nursing Process Step: Evaluation MSC: Patient Needs: Psychosocial Integrity Chapter 12: Processes of Birth Foundations of Maternal-Newborn & Women‘s Health Nursing, 7th Edition MULTIPLE CHOICE 1. The nurse is explaining the physiology of uterine contractions to a group of nursing students. Which statement best explains the maternal-fetal exchange of oxygen and waste products during a contraction? a. Little to no affect b. Increases as blood pressure decreases c. Diminishes as the spiral arteries are compressed d. Continues except when placental functions are reduced ANS: C During labor contractions, the maternal blood supply to the placenta gradually stops as the spiral arteries supplying the intervillous space are compressed by the contracting uterine muscle. The exchange of oxygen and waste products is affected by contractions. The exchange of oxygen and waste products decreases. The maternal blood supply to the placenta gradually stops with contractions. DIF: Cognitive Level: Application OBJ: Nursing Process Step: Implementation MSC: Patient Needs: Physiologic Integrity 2. The nurse is directing an unlicensed assistive personnel (UAP) to obtain maternal vital signs between contractions. Which statement is the appropriate rationale for assessing maternal vital signs between contractions rather than at another interval? a. Vital signs taken during contractions are inaccurate. b. During a contraction, assessing fetal heart rate is the priority. c. Maternal blood flow to the heart is reduced during contractions. d. Maternal circulating blood volume increases temporarily during contractions. ANS: D During uterine contractions, blood flow to the placenta temporarily stops, causing a relative increase in the mother‘s blood volume, which in turn temporarily increases blood pressure and slows the pulse. Vital signs are altered by contractions but are considered accurate for a period of time. It is important to monitor the fetal response to contractions, but the question is concerned with the maternal vital signs. Maternal blood flow is increased during a contraction. DIF: Cognitive Level: Application OBJ: Nursing Process Step: Implementation MSC: Patient Needs: Safe and Effective Care Environment 3. Uncontrolled maternal hyperventilation during labor results in a. metabolic acidosis.b. metabolic alkalosis. c. respiratory acidosis. d. respiratory alkalosis. ANS: D Rapid deep respirations cause the laboring woman to lose carbon dioxide through exhalation, resulting in respiratory alkalosis. Hyperventilation does not cause respiratory acidosis, metabolic acidosis, or metabolic alkalosis. DIF: Cognitive Level: Understanding OBJ: Nursing Process Step: Assessment MSC: Patient Needs: Physiologic Integrity 4. Which mechanism of labor occurs when the largest diameter of the fetal presenting part passes the pelvic inlet? a. Extension b. Engagement c. Internal rotation d. External rotation ANS: B Engagement occurs when the presenting part fully enters the pelvic inlet. Extension occurs when the fetal head meets resistance from the tissues of the pelvic floor and the fetal neck stops under the symphysis. This causes the fetal head to extend. Internal rotation occurs when the fetus enters the pelvic inlet. The rotation allows the longest fetal head diameter to conform to the longest diameter of the maternal pelvis. External rotation occurs after the birth of the head. The head then turns to the side so the shoulders can internally rotate and are positioned with their transverse diameter in the anteroposterior diameter of the pelvic outlet. DIF: Cognitive Level: Understanding OBJ: Nursing Process Step: Assessment MSC: Patient Needs: Health Promotion and Maintenance 5. The laboring patient asks the nurse how the labor contractions cause the cervix to dilate. The nurse responds that labor contractions facilitate cervical dilation by a. promoting blood flow to the cervix. b. contracting the lower uterine segment. c. enlarging the internal size of the uterus. d. pulling the cervix over the fetus and amniotic sac. ANS: D Effective uterine contractions pull the cervix upward at the same time the fetus and amniotic sac are pushed downward. Blood flow decreases to the uterus during a contraction. The contractions are stronger at the fundus. The internal size becomes smaller with the contractions; this helps push the fetus down. DIF: Cognitive Level: Application OBJ: Nursing Process Step: Implementation MSC: Patient Needs: Physiologic Integrity 6. Pregnant patients can usually tolerate the normal blood loss associated with childbirth because of which physiologic adaptation to pregnancy?a. A higher hematocrit b. Increased leukocytes c. Increased blood volume d. A lower fibrinogen level ANS: C Women have a significant increase in blood volume during pregnancy. After birth, the additional circulating volume is no longer necessary. The hematocrit decreases with pregnancy due to the higher fluid volume. Leukocyte levels increase during labor; however, that is not the reason for the toleration of blood loss. Fibrinogen levels increase with pregnancy. DIF: Cognitive Level: Understanding OBJ: Nursing Process Step: Assessment MSC: Patient Needs: Physiologic Integrity 7. The nurse is assessing the duration of a patient‘s labor contractions. Which method does the nurse implement to assess the duration of labor contractions? a. Assess the strongest intensity of each contraction. b. Assess uterine relaxation between two contractions. c. Assess from the beginning to the end of each contraction. d. Assess from the beginning of one contraction to the beginning of the next. ANS: C Duration of labor contractions is the average length of contractions from beginning to end. Assessing the strongest intensity of each contraction assesses the strength or intensity of the contractions. Assessing uterine relaxation between two contractions is the interval of the contraction phase. Assessing from the beginning of one contraction to the beginning of the next is the frequency of the contractions. DIF: Cognitive Level: Application OBJ: Nursing Process Step: Assessment MSC: Patient Needs: Physiologic Integrity 8. Which physiologic event is the key indicator of the commencement of true labor? a. Bloody show b. Cervical dilation and effacement c. Fetal descent into the pelvic inlet d. Uterine contractions every 7 minutes ANS: B The conclusive distinction between true and false labor is that contractions of true labor cause progressive change in the cervix. Bloody show can occur before true labor. Fetal descent can occur before true labor. False labor may have contractions that occur this frequently but is usually inconsistent. DIF: Cognitive Level: Understanding OBJ: Nursing Process Step: Assessment MSC: Patient Needs: Physiologic Integrity 9. Which factor ensures that the smallest anterior-posterior diameter of the fetal head enters the pelvis?a. Station b. Flexion c. Descent d. Engagement ANS: B The anterior-posterior diameter of the head varies with how much it is flexed. In the most favorable situation, the head is fully flexed and the anterior-posterior diameter is the suboccipitobregmatic, averaging 9.5 cm. The station is the relationship of the fetal presenting part to the level of the ischial spine. Descent is the moving of the fetus through the birth canal. Engagement occurs when the largest diameter of the fetal presenting part has passed the pelvic outlet. DIF: Cognitive Level: Understanding OBJ: Nursing Process Step: Assessment MSC: Patient Needs: Health Promotion and Maintenance 10. An increase in urinary frequency and leg cramps after the 36th week of pregnancy are an indication of a. lightening. b. breech presentation. c. urinary tract infection. d. onset of Braxton-Hicks contractions. ANS: A As the fetus descends toward the pelvic inlet near the end of pregnancy, increased pelvic pressure occurs, resulting in greater urinary frequency and more leg cramps. Breech presentation does not cause urinary frequency and leg cramps. A urinary tract infection may cause urinary frequency but with burning and would not cause leg cramps. Braxton-Hicks contractions are irregular and mild and occur throughout the pregnancy. DIF: Cognitive Level: Understanding OBJ: Nursing Process Step: Assessment MSC: Patient Needs: Health Promotion and Maintenance 11. A patient just delivered her baby via the vaginal route. The patient asks the nurse why the baby‘s head is not round, but oval. Which explanation should the nurse provide the patient? a. This results from molding. b. This results from lightening. c. This results from the fetal lie. d. This results from the fetal presentation. ANS: A The sutures and fontanels allow the bones of the fetal head to move slightly, changing the shape of the fetal head so it can adapt to the size and shape of the pelvis. Lightening is the descent of the fetus toward the pelvic inlet before labor. Lie is the relationship of the long axis of the fetus to the long axis of the mother. Presentation is the fetal part that first enters the pelvic outlet.DIF: Cognitive Level: Application OBJ: Nursing Process Step: Implementation MSC: Patient Needs: Physiologic Integrity 12. A patient whose cervix is dilated to 6 cm is considered to be in which phase of labor? a. Latent phase b. Active phase c. Second stage d. Third stage ANS: B The active phase of labor is characterized by cervical dilation of 5 to 6 cm. Historically, the latent phase is from the beginning of true labor until 3 cm of cervical dilation. Recent research has suggested that the latent phase be considered to last up until 5 to 6 cm. dilated. The second stage of labor begins when the cervix is completely dilated until the birth of the baby. The third stage of labor is from the birth of the baby until the expulsion of the placenta. DIF: Cognitive Level: Understanding OBJ: Nursing Process Step: Assessment MSC: Patient Needs: Physiologic Integrity 13. The nurse is assessing a patient in the active phase of labor. What should the nurse expect during this phase? a. The patient is sociable and excited. b. The patient is requesting pain medication. c. The patient begins to experience the urge to push. d. The patient experiences loss of control and irritability. ANS: B During the active phase of labor, contraction intensity and discomfort increase to the point where women often request pain medication. Sociability and excitability occur during the latent phase. The urge to push occurs at the end of the transition phase or the second stage of labor. Loss of control and irritability occur during the transition phase of labor. DIF: Cognitive Level: Application OBJ: Nursing Process Step: Planning MSC: Patient Needs: Physiologic Integrity 14. A laboring patient asks the nurse how she will know that the contraction is at its peak. The nurse explains that the contraction peaks during which stage of measurement? a. The acme b. The interval c. The increment d. The decrement ANS: A The acme is the peak or period of greatest strength during the middle of a contraction cycle. The interval is the period between the end of the contraction and the beginning of the next. The increment is the beginning of the contraction until it reaches the peak. The decrement occurs after the peak until the contraction ends.DIF: Cognitive Level: Application OBJ: Nursing Process Step: Implementation MSC: Patient Needs: Physiologic Integrity 15. A patient in labor presents with a breech presentation. The nurse understands that a breech presentation is associated with a. more rapid labor. b. a high risk of infection. c. maternal perineal trauma. d. umbilical cord compression. ANS: D The umbilical cord can compress between the fetal body and maternal pelvis when the body has been born but the head remains within the pelvis. Breech presentation is not associated with a more rapid labor. There is no higher risk of infection with a breech birth. There is no higher risk for perineal trauma with a breech birth. Most breech presentations are now delivered by caesarean birth. DIF: Cognitive Level: Understanding OBJ: Nursing Process Step: Assessment MSC: Patient Needs: Physiologic Integrity 16. The primary difference between the labor of a nullipara and that of a multipara is a. total duration of labor. b. level of pain experienced. c. amount of cervical dilation. d. sequence of labor mechanisms. ANS: A Multiparas usually labor more quickly than nulliparas, making the total duration of their labor shorter. The level of pain is individual to the woman, not the number of labors she has experienced. Cervical dilation is the same for all labors. The sequence of labor mechanisms is the same with all labors. DIF: Cognitive Level: Analysis OBJ: Nursing Process Step: Evaluation MSC: Patient Needs: Physiologic Integrity 17. Which maternal factor may inhibit fetal descent during labor? a. A full bladder b. Decreased peristalsis c. Rupture of membranes d. Reduction in internal uterine size ANS: A A full bladder may inhibit fetal descent because it occupies space in the pelvis needed by the fetal presenting part. Peristalsis does not influence fetal descent. Rupture of membranes will assist in the fetal descent. Contractions will reduce the internal uterine size to assist fetal descent. DIF: Cognitive Level: Understanding OBJ: Nursing Process Step: AssessmentMSC: Patient Needs: Physiologic Integrity 18. Which assessment finding would cause a concern for a patient who had delivered vaginally? a. Estimated blood loss (EBL) of 500 mL during the birth process b. White blood cell count of 28,000 mm3 postbirth c. Patient complains of fingers tingling d. Patient complains of thirst ANS: C A patient‘s complaint of fingers tingling may represent respiratory alkalosis due to hyperventilation breathing patterns during labor. As such it requires intervention by the nurse to have the patient slow breathing down and restore normal carbon dioxide levels. DIF: Cognitive Level: Application OBJ: Nursing Process Step: Assessment MSC: Patient Needs: Physiologic Integrity/Reduction of Risk Potential 19. On admission to the labor and birth unit, a 38-year-old female, gravida 4, para 3, at term in early labor is found to have a transverse lie on vaginal examination. What is the priority intervention at this time? a. Perform a vaginal exam to denote progress. b. Notify the health care provider. c. Initiate parenteral therapy. d. Apply oxygen via nasal cannula at 8 L/minute. ANS: B A transverse lie is considered to be an abnormal presentation so the physician should be notified and the process of a Caesarean section as the birth method should be initiated. The information provided relative to transverse lie was found on vaginal exam. At this point, the priority is to prepare for a surgical birth because assessment data also indicate that the patient is in early labor; thus a vaginal birth is not imminent. Although initiating parenteral therapy will be required, it is not the priority at this time. Application of oxygen is not required because there is no evidence of fetal or maternal distress. DIF: Cognitive Level: Analysis OBJ: Nursing Process Step: Implementation MSC: Patient Needs: Safe and Effective Care Environment 20. Which assessment finding indicates that cervical dilation and/or effacement has occurred? a. Onset of irregular contractions b. Cephalic presentation at 0 station c. Bloody mucus drainage from vagina d. Fetal heart tones (FHTs) present in the lower right quadrant ANS: C Cervical dilation and/or effacement results in loss of the mucus plug as well as rupture of small capillaries in the cervix; irregular contractions, cephalic presentation, and FHTs in the lower right quadrant do not indicate the onset of cervical ripening. DIF: Cognitive Level: Analysis OBJ: Nursing Process Step: AnalysisMSC: Patient Needs: Health Promotion and Maintenance 21. If a notation on the patient‘s health record states that the fetal position is LSP, this indicates that the a. head is in the right posterior quadrant of the pelvis. b. head is in the left anterior quadrant of the pelvis. c. buttocks are in the left posterior quadrant of the pelvis. d. buttocks are in the right upper quadrant of the abdomen. ANS: C LSP explains the position of the fetus in the maternal pelvis. L = left side of the pelvis, S = sacrum (fetus is in breech presentation), P = posterior quadrants of the pelvis. When the head is in the right posterior quadrant of the pelvis, the position is ROP. When the head is in the left anterior quadrant of the pelvis, the position is ROA. When the buttocks are in the upper quadrant of the abdomen, the position would be ROA, ROP, LOA, LOP, LOT, or ROT. DIF: Cognitive Level: Analysis OBJ: Nursing Process Step: Analysis MSC: Patient Needs: Health Promotion and Maintenance 22. To determine if the patient is in true labor, the nurse would assess for changes in a. cervical dilation. b. amount of bloody show. c. fetal position and station. d. pattern of uterine contractions. ANS: A Cervical changes are the only indication of true labor and are used to determine true and false labor. Changes in the amount of bloody show, fetal position and station, and pattern of uterine contractions are unreliable indicators of true labor. DIF: Cognitive Level: Analysis OBJ: Nursing Process Step: Implementation MSC: Patient Needs: Physiologic Integrity 23. The health care provider for a laboring patient makes the following entry into the patient‘s record: 3/50%/. What instruction will the nurse implement with the patient? a. ―You will need to remain in bed attached to the electronic fetal monitor.‖ b. ―Breathe with me slowly, in through your nose and out through your mouth.‖ c. ―I will begin the administration of 1000 mL of IV fluid so you can have an epidural.‖ d. ―Your partner will need to change into scrub attire to attend the imminent birth.‖ ANS: B This patient is in the latent phase of the first stage of labor. Use slow, deep chest breathing patterns early in labor to conserve energy for the upcoming process. There is no mention in the stem that the membranes are ruptured, which may prohibit the patient from ambulating. Ambulating during early labor uses gravity to facilitate fetal descent. This is desired because the head is at 1 station. Epidural placement during early labor may slow down the labor process andshould be delayed. There is no indication that birth is imminent because the patient is only 3 cm dilated. DIF: Cognitive Level: Analysis OBJ: Nursing Process Step: Implementation MSC: Patient Needs: Health Promotion and Maintenance 24. The examiner indicates to the labor nurse that the fetus is in the left occiput anterior (LOA) position. To facilitate the labor process, how will the nurse position the laboring patient? a. On her back b. On her left side c. On her right side d. On her hands and knees ANS: B LOA is the desired fetal position for the birthing process. Positioning the patient on her left side will accomplish two objectives: (1) by the use of gravity, the fetus will most likely stay in the LOA position; and (2) increase perfusion of the placenta and increase oxygen to the fetus. Positioning the patient on her back decreases placental perfusion. Positioning on her right may facilitate internal rotation and move the fetus out of the LOA position. The hands and knees position is reserved to decrease cord compression, facilitate the fetus out of a posterior position, or increase oxygenation in the presence of hypoxia. Because none of these conditions are present, there is no need to implement this position. DIF: Cognitive Level: Analysis OBJ: Nursing Process Step: Implementation MSC: Patient Needs: Health Promotion and Maintenance 25. The primiparous patient at 39 weeks‘ gestation states to the nurse, ―I can breathe easier now.‖ What is the nurse‘s most appropriate response? a. ―Your labor will start any day now since the baby has dropped.‖ b. ―That process is called lightening. Do you have to urinate more frequently?‖ c. ―Contact your health care provider when your contractions are every 5 minutes for 1 hour.‖ d. ―You will likely not feel you baby‘s movements as much now, so do not be concerned.‖ ANS: B As the fetus descends toward the pelvic inlet (dropping), the woman notices that she breathes more easily because upward pressure on her diaphragm is reduced. However, increased pressure on her bladder causes her to urinate more frequently. Pressure of the fetal head in the pelvis also may cause leg cramps and edema. Lightening (descent of the fetus toward the pelvic inlet before labor) is most noticeable in primiparas and occurs about 2 to 3 weeks before the natural onset of labor. Instructions for labor, although correct, do not address the patient‘s statement of being able to breathe easier. Fetal movement continues throughout the final weeks of gestation. A decrease in fetal movement is a concerning sign and the health care provider must be notified. DIF: Cognitive Level: Application OBJ: Nursing Process Step: Implementation MSC: Patient Needs: Health Promotion and Maintenance 26. The nurse assesses a laboring patient‘s contraction pattern and notes the frequency at every 3 to 4 minutes, duration 50 to 60 seconds, and the intensity is moderate by palpation. What is the most accurate documentation for this contraction pattern? a.Stage 1, latent phase b. Stage 2, latent phase c. Stage 1, active phase d. Stage 2, active phase ANS: C In the active phase of stage 1, contractions are about 2 to 5 minutes apart, with a duration of about 40 to 60 seconds and an intensity that ranges from moderate to strong. During the latent phase of stage 1, the interval between contractions shortens until contractions are about 5 minutes apart. Duration increases to 30 to 40 seconds by the end of the latent phase. During stage 2, latent phase, the woman is resting and preparing to push; she likely has not experienced the pushing reflex (sometimes referred to as the Ferguson reflex). She is actively bearing down during the active phase of the second stage. DIF: Cognitive Level: Understanding OBJ: Nursing Process Step: Analysis MSC: Patient Needs: Health Promotion and Maintenance 27. A laboring patient states to the nurse, ―I have to push!‖ What is the next nursing action? a. Contact the health care provider. b. Examine the patient‘s cervix for dilation. c. Review with her how to bear down with contractions. d. Ask her partner to support her head with each push. ANS: B When the cervix is completely dilated, the head can descend through the pelvis and stimulate the pushing, reflex. Cervical dilation must first be confirmed because premature pushing efforts may result in cervical edema and corresponding delay in dilation. Once complete dilation has been confirmed, the nurse can notify the health care provider. Teaching positioning and pushing efforts is accomplished once complete dilation has been confirmed. DIF: Cognitive Level: Application OBJ: Nursing Process Step: Analysis MSC: Patient Needs: Health Promotion and Maintenance 28. After birth of the placenta the patient states, ―All of a sudden I feel very cold.‖ What is the most appropriate nursing action at this time? a. Place a warm blanket over the patient. b. Place the baby on the patient‘s abdomen. c. Tell the patient that chills are expected after birth. d. ―What do you mean by your words ‗very cold‘?‖ ANS: A Many women are chilled after birth. The cause of this reaction is unknown but probably relates to the sudden decrease in effort, loss of the heat produced by the fetus, decrease in intraabdominal pressure, and fetal blood cells entering the maternal circulation. The chill lasts for about 20 minutes and subsides spontaneously. A warm blanket, hot drink, or soup may help relieve the chill and make the woman more comfortable. Placing the baby on her abdomen may result in transfer of heat and make her feel even colder. Reassurance is appropriate after theblanket is provided. Validation of an expected physical response to the birthing process results in a delay of care and is unnecessary. DIF: Cognitive Level: Application OBJ: Nursing Process Step: Implementation MSC: Patient Needs: Health Promotion and Maintenance MULTIPLE RESPONSE 1. A 28-year-old gravida 1, para 0 patient who is at term calls the labor and birth unit stating that she thinks she is in labor. She states that she does have some vaginal discharge and feels wet; however, it is not bloody in nature. She relates a contraction pattern that is irregular, ranging from 5 to 7 minutes and lasting 30 seconds. Which questions should the nurse pose to the patient during this telephone triage? (Select all that apply.) a. Does she think that her membranes have ruptured? b. Is there any evidence of bloody show? c. Instruct the patient to keep monitoring her contraction pattern and call you back if they become more regular. d. When is her next scheduled visit with her health care provider? e. Tell her to come into the hospital for evaluation. ANS: A, E The cornerstone of obstetric triage is reassurance of maternal-fetal well-being. Thus in view of the assessment data that the patient provided, the nurse should ascertain membrane status and ask the patient to come in for evaluation. The patient has already indicated that the vaginal discharge was not bloody in nature. Having the patient continue to monitor at home would not provide assurance of maternal-fetal well-being. Asking the patient about the next scheduled physician visit does not address current health concerns of impending labor. DIF: Cognitive Level: Analysis OBJ: Nursing Process Step: Evaluation MSC: Patient Needs: Physiologic Integrity/Physiologic Adaptation 2. A patient asks the nurse how she can tell if labor is real. Which information should the nurse provide to this patient? (Select all that apply.) a. In true labor, the cervix begins to dilate. b. In true labor, the contractions are felt in the abdomen and groin. c. In true labor, contractions often resemble menstrual cramps during early labor. d. In true labor, contractions are inconsistent in frequency, duration, and intensity in the early stages. e. In true labor your contractions tend to increase in frequency, duration, and intensity with walking. ANS: A, C, E In true labor, the cervix begins to dilate, contractions often resemble menstrual cramps in the early stage, and labor contractions increase in frequency, duration, and intensity with walking. False labor contractions are felt in the abdomen and groin and the contractions are inconsistent in frequency, duration, and intensity. DIF: Cognitive Level: Application OBJ: Nursing Process Step: Implementation MSC: Patient Needs: Physiologic Integrity3. The nurse who elects to practice in the area of obstetrics often hears discussion regarding the four Ps. What are the four Ps that interact during childbirth? (Select all that apply.) a. Powers b. Passage c. Position d. Passenger e. Psyche ANS: A, B, D, E Powers: The two powers of labor are uterine contractions and pushing efforts. During the first stage of labor, through full cervical dilation, uterine contractions are the primary force moving the fetus through the maternal pelvis. At some point after full dilation, the woman adds her voluntary pushing efforts to propel the fetus through the pelvis. Passage: The passage for birth of the fetus consists of the maternal pelvis and its soft tissues. The bony pelvis is more important to the successful outcome of labor because bones and joints do not yield as readily to the forces of labor. Passenger: This is the fetus plus the membranes and placenta. Fetal lie, attitude, presentation, and position are all factors that affect the fetus as passenger. Psyche: The psyche is a crucial part of childbirth. Marked anxiety, fear, or fatigue decreases the woman‘s ability to cope. Position is not one of the four Ps. DIF: Cognitive Level: Analysis OBJ: Nursing Process Step: Assessment MSC: Patient Needs: Physiologic Integrity 4. The nurse is planning care for a patient during the fourth stage of labor. Which interventions should the nurse plan to implement? (Select all that apply.) a. Offer the patient a warm blanket. b. Place an ice pack on the perineum. c. Massage the uterus if it is boggy. d. Delay breastfeeding until the patient is rested. e. Explain to the patient that the lochia will be light pink in color. ANS: A, B, C The fourth stage of labor lasts from the birth of the placenta through the first 1 to 4 hours after birth. Many women are chilled after birth. A warm blanket, hot drink, or soup may help relieve the chill and make the woman more comfortable. Localized discomfort from birth trauma such as lacerations, episiotomy, edema, or hematoma is evident as the effects of local and regional anesthetics diminish. Ice packs on the perineum limit this edema and hematoma formation. A soft (boggy) uterus and increasing uterine size are associated with postpartum hemorrhage because large blood vessels at the placenta site are not compressed. The uterus should be massaged if it is not firm. The fourth stage is the best time to initiate breastfeeding if maternal and infant problems are absent. The vaginal drainage after childbirth is called lochia. The three stages are lochia rubra, lochia serosa, and lochia alba. Lochia rubra, consisting mostly of blood, is present in the fourth stage of labor. The color of the lochia will be bright red not pink.DIF: Cognitive Level: Application OBJ: Nursing Process Step: Planning MSC: Patient Needs: Physiologic Integrity 5. Which clinical finding should the nurse expect to assess in the third stage of labor that indicates the placenta has separated from the uterine wall? (Select all that apply.) a. A gush of blood appears. b. The uterus rises upward in the abdomen. c. The fundus descends below the umbilicus. d. The cord descends further from the vagina. e. The uterus becomes boggy and soft, with an elongated shape. ANS: A, B, D Four signs suggest placenta separation. The uterus has a spherical shape. The uterus rises upward in the abdomen as the placenta descends into the vagina and pushes the fundus upward. The cord descends further from the vagina. A gush of blood appears as blood trapped behind the placenta is released. The fundus rises upward above the umbilicus. A boggy uterus with an elongated shape would not be expected. DIF: Cognitive Level: Analysis OBJ: Nursing Process Step: Assessment MSC: Patient Needs: Physiologic Integrity 6. The clinical nurse educator is providing instruction to a group of new nurses during labor orientation. Which information regarding the factors that have a role in the initiation of labor should the educator include in this teaching session? (Select all that apply.) a. Progesterone levels become higher than estrogen levels. b. Natural oxytocin in conjunction with other substances plays a role. c. Stretching, pressure, and irritation of the uterus and cervix increase. d. The secretion of prostaglandins from the fetal membranes decreases. ANS: B, C Factors that appear to have a role in starting labor include the following: (1) natural oxytocin plays a part in labor‘s initiation in conjunction with other substances; and (2) stretching, pressure, and irritation of the uterus and cervix increase as the fetus reaches term size. The progesterone levels drop and estrogen levels increase. There is an increase in the secretion of prostaglandins from the fetal membranes. DIF: Cognitive Level: Application OBJ: Nursing Process Step: Planning MSC: Patient Needs: Physiologic Integrity Chapter 13: Pain Management During Childbirth Foundations of Maternal-Newborn & Women‘s Health Nursing, 7th Edition MULTIPLE CHOICE 1. Childbirth preparation can be considered successful if which of the following outcomes is achieved? a. Labor was pain-free. b. The birth experiences of friends and families were ignored.c. Only nonpharmacologic methods for pain control were used. d. The patient rehearsed labor and practiced skills to master pain. ANS: D Preparation allows the woman to rehearse for labor and to learn new skills to cope with the pain of labor and the expected behavioral changes. Childbirth preparation does not guarantee a pain-free labor. A woman should be prepared for pain and anesthesia–analgesia realistically. Friends and families can be an important source of support if they convey realistic information about labor pain. Women will not always achieve their desired level of pain control by using nonpharmacologic methods alone. DIF: Cognitive Level: Analysis OBJ: Nursing Process Step: Evaluation MSC: Patient Needs: Psychosocial Integrity 2. A woman with a known heroin habit is admitted in early labor. Which drug is contraindicated with opiate-dependent patients? a. Nalbuphine (Nubain) b. Hydroxyzine (Vistaril) c. Promethazine (Phenergan) d. Diphenhydramine (Benadryl) ANS: A Nalbuphine may precipitate withdrawal if given to an opiate-dependent woman. Hydroxyzine is an antihistamine with antiemetic effects. Promethazine usually relieves nausea and vomiting. Diphenhydramine is commonly used to relieve pruritus from epidural narcotics. DIF: Cognitive Level: Understanding OBJ: Nursing Process Step: Planning MSC: Patient Needs: Physiologic Integrity 3. A patient is admitted to the labor and birth room in active labor; contractions are 4 to 5 minutes apart and last for 30 seconds. The nurse needs to perform a detailed assessment. When is the best time to ask questions or perform procedures? a. After the contraction is over b. When it is all right with the coach c. During the increment of the next contraction d. After administration of analgesic-anesthetic ANS: A Reduce intrusions as much as possible. Longer assessments may span several contractions. The coach is the support person. The woman needs to feel confident in her ability to go through labor and birth, and she should be encouraged to express her own needs and concerns. The increment is the beginning of the next contraction. It is best to stop with questions and procedures during each contraction. An analgesic or anesthetic may cause adverse reactions in the woman, preventing her from answering questions correctly. DIF: Cognitive Level: Understanding OBJ: Nursing Process Step: Assessment MSC: Patient Needs: Health Promotion and Maintenance 4. Childbirth pain is different from other types of pain in that it is a.less intense. b. associated with a physiologic process. c. more responsive to pharmacologic management. d. designed to make one withdraw from the stimulus. ANS: B Childbirth pain is part of a normal process, whereas other types of pain usually signify an injury or illness. Childbirth pain is not less intense than other types of pain. Pain management during labor may affect the course and length of labor. The pain with childbirth is a normal process; however, it is not caused by the type of injury as when withdrawal from the stimulus occurs. DIF: Cognitive Level: Understanding OBJ: Nursing Process Step: Assessment MSC: Patient Needs: Physiologic Integrity 5. Excessive anxiety during labor heightens the patient‘s sensitivity to pain by increasing a. muscle tension. b. the pain threshold. c. blood flow to the uterus. d. rest time between contractions. ANS: A Anxiety and fear increase muscle tension, diverting oxygenated blood to the woman‘s brain and skeletal muscles. Prolonged tension results in general fatigue, increased pain perception, and reduced ability to use coping skills. Anxiety will decrease the pain threshold. Anxiety can decrease blood flow to the uterus. Anxiety will decrease the amount of rest the mother gets between contractions. DIF: Cognitive Level: Understanding OBJ: Nursing Process Step: Assessment MSC: Patient Needs: Psychosocial Integrity 6. Which fetal position may cause the laboring patient increased back discomfort? a. Left occiput anterior b. Left occiput posterior c. Right occiput anterior d. Right occiput transverse ANS: B In the left occiput posterior position, each contraction pushes the fetal head against the mother‘s sacrum, which results in intense back discomfort. Back labor is seen mostly when the fetus is in the posterior position. DIF: Cognitive Level: Understanding OBJ: Nursing Process Step: Assessment MSC: Patient Needs: Health Promotion and Maintenance 7. A major advantage of nonpharmacologic pain management is a. a more rapid labor is likely. b. more complete pain relief is possible.c. the woman remains fully alert at all times. d. there are no side effects or risks to the fetus. ANS: D Because nonpharmacologic pain management does not include analgesics, adjunct drugs, or anesthesia, it is harmless to the mother and the fetus. There is less pain relief with nonpharmacologic pain management during childbirth. Pain management may or may not alter the length of labor. At times, when pain is decreased, the mother relaxes and labor progresses at a quicker pace. The woman‘s alertness is not altered by medication, but the increase in pain will decrease alertness. DIF: Cognitive Level: Understanding OBJ: Nursing Process Step: Assessment MSC: Patient Needs: Physiologic Integrity 8. The best time to teach nonpharmacologic pain control methods to an unprepared laboring patient is during which stage? a. Latent phase b. Active phase c. Second stage d. Transition phase ANS: A The latent phase of labor is the best time for intrapartum teaching because the woman is usually anxious enough to be attentive yet comfortable enough to understand the teaching. During the active phase, the woman is focused internally and unable to concentrate on teaching. During the second stage, the woman is focused on pushing. She normally handles the pain better at this point because she is active in doing something to hasten the birth. During transition, the woman is focused on keeping control; she is unable to focus on anyone else or learn at this time. DIF: Cognitive Level: Understanding OBJ: Nursing Process Step: Planning MSC: Patient Needs: Health Promotion and Maintenance 9. The primary side effect of maternal narcotic analgesia in the newborn is a. tachypnea. b. bradycardia. c. acrocyanosis. d. respiratory depression. ANS: D An infant delivered within 1 to 4 hours of maternal analgesic administration is at risk for respiratory depression from the sedative effects of the narcotic. The infant who is having a side effect to maternal analgesics normally would have a decrease in respirations, not an increase. Bradycardia is not the anticipated side effect of maternal analgesics. Acrocyanosis is an expected finding in a newborn and is not related to maternal analgesics. DIF: Cognitive Level: Understanding OBJ: Nursing Process Step: Assessment MSC: Patient Needs: Physiologic Integrity10. The nerve block used in labor that provides anesthesia to the lower vagina and perineum is referred to as a(n) a. local. b. epidural. c. pudendal. d. spinal block. ANS: C A pudendal block anesthetizes the lower vagina and perineum to provide anesthesia for an episiotomy and use of low forceps, if needed. A local provides anesthesia for the perineum at the site of the episiotomy. An epidural provides anesthesia for the uterus, perineum, and legs. A spinal block provides anesthesia for the uterus, perineum, and down the legs. DIF: Cognitive Level: Understanding OBJ: Nursing Process Step: Assessment MSC: Patient Needs: Physiologic Integrity 11. The nurse is teaching a childbirth education class. Which information regarding excessive pain in labor should the nurse include in the session? a. It usually results in a more rapid labor. b. It has no effect on the outcome of labor. c. It is considered to be a normal occurrence. d. It may result in decreased placental perfusion. ANS: D When experiencing excessive pain, the woman may react with a stress response that diverts blood flow from the uterus and the fetus. Excessive pain may prolong the labor because of increased anxiety in the woman. It may affect the outcome of the labor, depending on the cause and the effect on the woman. Pain is considered normal for labor. However, excessive pain may be an indication of other problems and must be assessed. DIF: Cognitive Level: Application OBJ: Nursing Process Step: Implementation MSC: Patient Needs: Physiologic Integrity 12. Which patient will most likely have increased anxiety and tension during labor? a. Gravida 2 who refused any medication b. Gravida 2 who delivered a stillborn baby last year c. Gravida 1 who did not attend prepared childbirth classes d. Gravida 3 who has two children younger than 3 years ANS: B If a previous pregnancy had a poor outcome, the patient will probably be more anxious during labor and birth. The patient without childbirth education classes is not prepared for labor and will have increased anxiety during labor; however, the patient with a poor previous outcome is more likely to experience a greater degree of anxiety. A gravida 2 has previous experience and can anticipate what to expect. By refusing any medication, she is taking control over her situationand will have less anxiety. This gravida 3 has previous experience and is aware of what to expect. DIF: Cognitive Level: Analysis OBJ: Nursing Process Step: Assessment MSC: Patient Needs: Psychosocial Integrity 13. Which method of pain management would be safest for a gravida 3, para 2, admitted at 8 cm cervical dilation? a. Narcotics b. Spinal block c. Epidural anesthesia d. Breathing and relaxation techniques ANS: D Nonpharmacologic methods of pain management may be the best option for a woman in advanced labor. At 8 cm cervical dilation there is probably not enough time remaining to administer spinal anesthesia or epidural anesthesia. A narcotic given at this time may reach its peak at about the time of birth and result in respiratory depression in the newborn. DIF: Cognitive Level: Application OBJ: Nursing Process Step: Planning MSC: Patient Needs: Physiologic Integrity 14. A laboring patient who imagines her body opening to let the baby out is using a mental technique called a. imagery. b. effleurage. c. distraction. d. dissociation. ANS: A Imagery is a technique of visualizing images that will assist the woman in coping with labor. Effleurage is self-massage. Distraction can be used in the early latent phase by having the woman involved in another activity. Dissociation helps the woman learn to relax all muscles except those that are working. DIF: Cognitive Level: Understanding OBJ: Nursing Process Step: Assessment MSC: Patient Needs: Psychosocial Integrity 15. When administering a narcotic to a laboring patient, which statement explains why the nurse should inject the medication at the beginning of a contraction? a. The medication will be rapidly circulated. b. Less medication will be transferred to the fetus. c. The maternal vital signs will not be adversely affected. d. Full benefit of the medication is received during that contraction. ANS: B Injecting the medication at the beginning of a contraction, when blood flow to the placenta is normally reduced, limits transfer to the fetus. It will not increase the circulation of the medication.It will not alter the vital signs any more than giving it at another time. The full benefit of the medication will be received by the patient. DIF: Cognitive Level: Application OBJ: Nursing Process Step: Implementation MSC: Patient Needs: Physiologic Integrity 16. The method of anesthesia in labor that is considered the safest for the fetus is a. epidural block. b. pudendal block. c. local infiltration. d. spinal (subarachnoid) block. ANS: C Local infiltration of the perineum rarely has any adverse effects on the mother or the fetus. With an epidural, pudendal, or spinal block the fetus can be affected by maternal side effects and maternal hypotension. DIF: Cognitive Level: Understanding OBJ: Nursing Process Step: Assessment MSC: Patient Needs: Physiologic Integrity 17. To improve placental blood flow immediately after the injection of an epidural anesthetic, the nurse should a. give the woman oxygen. b. turn the woman to the right side. c. decrease the intravenous infusion rate. d. place a wedge under the woman‘s right hip. ANS: D Tilting the woman‘s pelvis to the left side relieves compression of the vena cava and compensates for a lower blood pressure without interfering with dispersal of the epidural medication. Oxygen administration will not improve placental blood flow. The woman needs to maintain the supine position for proper dispersal of the medication. Placing a wedge under the hip will relieve compression of the vena cava. The intravenous infusion rate needs to be increased to prevent hypotension. DIF: Cognitive Level: Application OBJ: Nursing Process Step: Implementation MSC: Patient Needs: Physiologic Integrity 18. Which physiologic effect may occur in the presence of increased maternal pain perception during labor? a. Increase in uterine contractions in response to catecholamine secretion b. Decrease in blood pressure in response to alpha receptors c. Decreased perfusion to the placenta in response to catecholamine secretion d. Increased uterine blood flow, causing increase in maternal blood pressure ANS: C Decreased perfusion to and from the placenta occurs as result of catecholamine secretion. A decrease in uterine contractions is seen in response to catecholamine secretion. Maternal bloodpressure is increased in response to alpha receptors. Decreased uterine blood flow causes an increase in maternal blood pressure. DIF: Cognitive Level: Analysis OBJ: Nursing Process Step: Assessment MSC: Patient Needs: Physiologic Integrity/Physiologic Adaptation 19. Which of the following factors would affect pain perception or tolerance for the laboring patient? a. Right occiput posterior fetal position during labor b. Bishop score of 10 prior to the induction of labor c. Gynecoid pelvis d. Absence of Ferguson‘s reflex ANS: A A fetus in the posterior position during labor can cause increased back pain to the mother because it is spine against spine. A Bishop score of 10 indicates that conditions are favorable for induction; the cervix is soft, anterior, effaced, and dilated and the presenting part is engaged. A gynecoid pelvic structure is considered to be an adequate passage for vaginal birth. Ferguson‘s reflex occurs when a contraction is stimulated as a result of vaginal stimulation. DIF: Cognitive Level: Analysis OBJ: Nursing Process Step: Evaluation MSC: Patient Needs: Physiologic Integrity/Physiologic Adaptation 20. A patient in labor is approaching the transition stage and already has an epidural in place. An additional dose of medication has been prescribed and administered to the patient. Which priority intervention should be performed in order to evaluate the clinical response to treatment? a. Obtain a pain scale response from the patient based on a 0 to 10 scale. b. Document maternal blood pressure and fetal heart rates following medication administration and observe for any variations. c. Document intake and output on the electronic health record (EHR). d. Increase the flow rate of prescribed parenteral fluid to maintain hydration. ANS: B Association of Women‘s Health, Obstetric and Neonatal Nurses (AWHONN) evidence-based practice guidelines note that maternal blood pressure and fetal heart tones should be assessed following any bolus of additional medication via the epidural route. Obtaining a pain scale response is not typically used for the laboring patient but used for postoperative and/or chronic pain patients. Intake and output should be documented as part of the clinical record but is not the priority intervention based on this patient‘s situation. Increasing the flow rate of parenteral fluids requires a physician‘s order, and there is no clinical evidence that this is needed. Giving parenteral fluids in excess can lead to fluid retention and fluid volume excess. DIF: Cognitive Level: Analysis OBJ: Nursing Process Step: Implementation MSC: Patient Needs: Safe and Effective Care Environment/Establishing Priorities 21. The process of labor places significant metabolic demands on the obstetric patient. Which physiologic findings would be expected? a. Decreased maternal blood pressure as a result of stimulation of alpha receptors b. Uterine vasoconstriction as a result of stimulation of beta receptorsc. Increased maternal demand for oxygen d. Increased blood flow to placenta because of catecholamine release ANS: C With regard to labor, one would expect to see an increase in maternal blood pressure because of stimulation of alpha receptors. Uterine vasoconstriction would occur in response to stimulation of alpha receptors. One would expect to see a decrease in blood flow to the placenta. The maternal metabolic rate is increased during labor, along with an increase in maternal demand for oxygen. DIF: Cognitive Level: Analysis OBJ: Nursing Process Step: Assessment MSC: Patient Needs: Physiologic Integrity/Physiologic Adaptation 22. A labor patient, gravida 2, para 1, at term has received meperidine (Demerol) for pain control during labor. Her most recent dose was 15 minutes ago and birth is now imminent. Maternal vital signs have been stable and the EFM tracing has not shown any baseline changes. Which medication does the nurse anticipate would be required in the birth room for administration? a. Oxytocin (Pitocin) b. Naloxone (Narcan) c. Bromocriptine (Parlodel) d. Oxygen ANS: B Because birth is imminent, and considering that the patient has had a recent dose of narcotics, the nurse anticipates that naloxone (Narcan) will be administered to the newborn to combat the effects of the opioid. Although Pitocin will be given following birth of the placenta, the newborn will be delivered prior to that and will receive priority intervention. Parlodel is not typically given in the labor and birth area any more. It was previously used to suppress lactation. At present, there is no need for the administration of oxygen because there is no evidence that the mother is showing any signs of respiratory depression. DIF: Cognitive Level: Analysis OBJ: Nursing Process Step: Planning MSC: Patient Needs: Physiologic Integrity/Physiologic Adaptation 23. Which statement is true with regard to the type of pain associated with childbirth experience? a. Pain is constant throughout the labor experience. b. Labor pain during childbirth is considered to be an abnormal response. c. Pain associated with childbirth is self-limiting. d. Pain associated with childbirth does not allow for adequate preparation. ANS: C The pain associated with childbirth is self-limiting in that it typically stops once the child is delivered. Pain is intermittent during the labor experience. Labor pain is considered to be a normal response during childbirth. Pregnant woman can prepare for the expected pain of childbirth by taking prepared childbirth classes and using relaxation techniques during the course of labor.DIF: Cognitive Level: Analysis OBJ: Nursing Process Step: Planning MSC: Patient Needs: Physiologic Integrity/Physiologic Adaptation 24. A patient in labor reports a feeling of burning pain during the second stage of labor. This type of pain is associated with a. visceral pain. b. tissue ischemia. c. somatic pain. d. cervical dilation. ANS: C This is an example of somatic pain experienced as a result of distention of the vagina and perineum during the second stage of labor. Visceral pain occurs in response to pressure on pelvic structures. Pain associated with ischemic tissue is a result of decreased blow flow to the uterus. The pain of cervical dilation is a major pain source during labor but, during the second stage of labor, the patient is already fully dilated so this would not be a factor. DIF: Cognitive Level: Analysis OBJ: Nursing Process Step: Assessment MSC: Patient Needs: Physiologic Integrity/Physiologic Adaptation 25. A patient presents to the labor and birth area for emergent birth. Vaginal exam reveals that the patient is fully dilated, vertex, +2 station, with ruptured membranes. The patient is extremely apprehensive because this is her first childbirth experience and asks for an epidural to be administered now. What is the priority nursing response based on this patient assessment? a. Use contact anesthesia for an epidural and prepare the patient per protocol. b. Tell the patient that she will not need any pain medication because the birth will be over in a matter of minutes and the pain will stop. c. Assist the patient with nonpharmacologic methods of pain distraction during this time as you prepare for vaginal birth. d. Call the physician for admitting orders. ANS: C By assisting the patient with nonpharmacologic methods of pain distraction, the nurse is focusing on the patient‘s needs while still preparing for vaginal birth. The patient presents in an emergent situation with birth being imminent. Thus there is not enough time to administer an epidural. Telling the patient that she will not need any pain medication because the birth will be over soon does not address the patient‘s concerns of apprehension and therefore is not therapeutic. Because this is an emergency birth situation, the nurse should be attending to the patient. If needed, another nurse and/or supervisor can contact the physician. DIF: Cognitive Level: Analysis OBJ: Nursing Process Step: Planning MSC: Patient Needs: Safe and Effective Care Environment 26. A labor patient has brought in a photograph of her two children and asks the nurse to place it on the wall so that she can look at it during labor contractions. This is an example of a. focal point. b. distraction. c.effleurage. d. relaxation. ANS: A The use of a focal point (image and/or point reference in the labor room) is an example of nonpharmacologic pain control during labor. The image of the patient‘s children is not serving as a method of distraction. Effleurage is the use of massage techniques to minimize pain perception. The image of the patient‘s children is not serving as a method of relaxation. DIF: Cognitive Level: Application OBJ: Nursing Process Step: Planning MSC: Patient Needs: Health Promotion and Maintenance 27. A pregnant woman in labor is quite anxious and has been breathing rapidly during contractions. She now complains of a tingling sensation in her fingers. What is the priority nursing intervention at this time? a. Perform a vaginal exam to denote progress. b. Reposition the patient to a side lying position. c. Instruct the patient to breathe into her cupped hands. d. Notify the physician about current findings. ANS: C This patient is exhibiting signs of hyperventilation associated with a rapid breathing pattern, which can occur during the labor process. The nurse should instruct the patient to breathe into her cupped hands to retain carbon dioxide that is being lost from the hyperventilation process. A vaginal exam is not indicated because there is no evidence of fetal distress and/or change in labor progress. Repositioning the patient may be an option but is not the priority intervention at this time. Notifying the physician is not appropriate at this time because the RN should attend to actions that are readily available to her based on her scope of practice and standard of care. The physician may have to be notified once the intervention has been performed. DIF: Cognitive Level: Analysis OBJ: Nursing Process Step: Implementation MSC: Patient Needs: Safe and Effective Care Environment 28. A laboring patient has asked the nurse to assist her in utilizing a cutaneous stimulation strategy for pain management. The nurse would a. assist her into the shower. b. apply a heat pack to lower back. c. help her to create a relaxing mental scene. d. encourage cleansing breaths and slow-paced breathing. ANS: B Cutaneous stimulation includes self-massage, massage by others, counterpressure, touch, thermal stimulation, and acupressure. A shower, tub, and whirlpool are forms of hydrotherapy; creating a relaxed mental scene is mental stimulation. The use of cleansing breaths and patterned breathing is part of breathing techniques for labor. DIF: Cognitive Level: Application OBJ: Nursing Process Step: Implementation MSC: Patient Needs: Health Promotion and Maintenance 29. To relieve a mild postdural puncture headache, the nurse should encourage the intake of a.milk. b. orange juice. c. tea or coffee. d. beef or chicken bouillon. ANS: C Caffeine is an oral therapy that is beneficial in relieving postdural puncture headache. Milk, juices, and bouillon will add oral hydration but lack the added benefit of the caffeine. Some patients prefer a cold caffeinated soft drink over coffee or tea. DIF: Cognitive Level: Application OBJ: Nursing Process Step: Implementation MSC: Patient Needs: Physiologic Integrity 30. Which patient will be most receptive to teaching about nonpharmacologic pain control methods? a. Gravida 1, para 0, in transition b. Gravida 2, para 1, admitted at 8 cm c. Gravida 1, para 0, dilated 2 cm, 80% effaced d. Gravida 3, para 2, complaining of intense perineal pressure ANS: C The latent phase of labor is the best time for intrapartum teaching; the latent phase of labor is the first centimeter of cervical dilation. Patients in the transition phase (8 to 10 cm) are experiencing intense pain and are not receptive to teaching. A multigravida complaining of intense perineal pressure indicates a patient whose birth is imminent. DIF: Cognitive Level: Analysis OBJ: Nursing Process Step: Analysis MSC: Patient Needs: Health Promotion and Maintenance 31. The nurse is providing care to a patient in the active phase of the first stage of labor. The patient is crying out loudly with each contraction. What is the nurse‘s most respectful approach for this patient? a. Ask the patient‘s labor coach if this is a usual expression of pain for her. b. Refer to the patient‘s chart to determine any orders for pain medication. c. Tell the patient that she is disturbing the other laboring patients on the unit. d. Encourage the patient to try to suppress her noisiness during contractions. ANS: A Women should be encouraged to express themselves in any way they find comforting, and the diversity of their expressions must be respected. Loud and vigorous expression may be a woman‘s personal pain coping mechanism, whereas a quiet woman may need medication relief but feels the need to remain stoic. Accepting a woman‘s individual response to labor and pain promotes a therapeutic relationship. Restraint is difficult because noisy women are challenging to work with and may disturb others. DIF: Cognitive Level: Application OBJ: Nursing Process Step: Assessment MSC: Patient Needs: Health Promotion and Maintenance32. A multipara‘s labor plan includes the use of jet hydrotherapy during the active phase of labor. What is the priority patient assessment prior to assisting the patient with this request? a. Maternal pulse b. Maternal temperature c. Maternal blood pressure d. Maternal blood glucose level ANS: B A shower, tub bath, or whirlpool bath is relaxing and provides thermal stimulation. Several studies have shown benefits of water therapy during labor, including immersion in a tub or whirlpool (jet hydrotherapy, or Jacuzzi). The major concern regarding immersion therapy has been newborn and postpartum maternal infections caused by microorganisms in the water. Infections can be caused by the woman‘s own ascending vaginal bacteria or by preexisting organisms in an improperly cleaned tub. Several studies have not found a significant association between newborn or postpartum maternal infections and the use of immersion hydrotherapy with proper cleaning. DIF: Cognitive Level: Application OBJ: Nursing Process Step: Assessment MSC: Patient Needs: Health Promotion and Maintenance 33. A patient in active labor requests an epidural for pain management. What is the nurse‘s most appropriate intervention at this juncture? a. Assess the fetal heart rate pattern over the next 30 minutes. b. Take the patient‘s blood pressure every 5 minutes for 15 minutes. c. Determine the patient‘s contraction pattern for the next 30 minutes. d. Initiate an IV infusion of lactated Ringer‘s solution at 2000 mL/hour over 30 minutes. ANS: D Rapid infusion of a nondextrose IV solution, often warmed, such as lactated Ringer‘s or normal saline, before initiation of the block fills the vascular system to offset vasodilation. Preload IV quantities are at least 500 to 1000 mL infused rapidly. Vasodilation with corresponding hypotension can reduce placental perfusion and is most likely to occur within the first 15 minutes after the initiation of the epidural. Determining the fetal heart rate every 30 minutes is the standard of care. The patient is in active labor, which indicates a contraction pattern resulting in cervical dilation. DIF: Cognitive Level: Analysis OBJ: Nursing Process Step: Analysis MSC: Patient Needs: Health Promotion and Maintenance MULTIPLE RESPONSE 1. You are preparing a patient for epidural placement by a nurse anesthetist in the LDR. Which interventions should be included in the plan of care? (Select all that apply.) a. Administer a bolus of 500 to 1000 mL of D5 normal saline prior to catheter placement. b. Have ephedrine available at bedside during catheter placement. c. Monitor blood pressure of patient frequently during catheter insertion and for the first 15 minutes of epidural administration.d. Insert a Foley catheter prior to epidural catheter placement. e. Monitor the patient for hypertension in response to epidural insertion. ANS: B, C A bolus of nondextrose fluid is recommended prior to epidural administration to prevent maternal hypotension. Ephedrine should be available at the bedside in case maternal hypotension is exhibited. Blood pressure should be monitored frequently during insertion and for the first 15 minutes of therapy. It is not necessary to insert a Foley catheter prior to epidural catheter placement. Hypertension is not a common clinical response to this treatment but hypotension is. DIF: Cognitive Level: Application OBJ: Nursing Process Step: Implementation MSC: Patient Needs: Physiologic Integrity/Pharmacologic and Parenteral Therapies 2. While developing an intrapartum care plan for the patient in early labor, it is important that the nurse recognize that psychosocial factors may influence a woman‘s experience of pain. These include which of the following? (Select all that apply.) a. Culture b. Anxiety and fear c. Support systems d. Preparation for childbirth e. Previous experiences with pain ANS: A, B, C, D, E Culture: A woman‘s sociocultural roots influence how she perceives, interprets, and responds to pain during childbirth. Some cultures encourage loud and vigorous expressions of pain, whereas others value self-control. The nurse should avoid praising some behaviors (stoicism) while belittling others (noisy expression). Anxiety and fear: Extreme anxiety and fear magnify sensitivity to pain and impair a woman‘s ability to tolerate it. Anxiety and fear increase muscle tension in the pelvic area, which counters the expulsive forces of uterine contractions and pushing efforts. Support systems: An anxious partner is less able to provide help and support to a woman during labor. A woman‘s family and friends can be an important source of support if they convey realistic and positive information about labor and birth. Preparation for childbirth: This does not ensure a pain-free labor. Preparation does reduce anxiety and fear. It also allows a woman to rehearse for labor. Previous experiences with pain: Fear and withdrawal are natural responses to pain during labor. Learning about these normal sensations ahead of time helps a woman suppress her natural reactions of fear regarding the impending birth. If a woman previously had a long and difficult labor, she is likely to be anxious. She may also have learned ways to cope and may use these skills to adapt to the present labor experience. DIF: Cognitive Level: Application OBJ: Nursing Process Step: Planning MSC: Patient Needs: Psychosocial Integrity 3. The nurse detects hypotension in a laboring patient after an epidural. Which actions should the nurse plan to implement? (Select all that apply.) a. Encourage the patient to drink fluids.b. Place the patient in a Trendelenburg position. c. Administer a normal saline bolus as prescribed. d. Administer oxygen at 8 to 10 L/minute per face mask. e. Administer IV ephedrine in 5- to 10-mg increments as prescribed. ANS: C, D, E If hypotension occurs after an epidural has been placed, techniques such as a rapid nondextrose IV fluid bolus, maternal repositioning, and oxygen administration are implemented. If those interventions are ineffective, IV ephedrine in 5- to 10-mg increments can be prescribed to promote vasoconstriction to raise the blood pressure. The patient in active labor should not be encouraged to drink fluids. In a Trendelenburg position, the body is flat, with the feet elevated. This would not be a position to use for a pregnant patient. DIF: Cognitive Level: Application OBJ: Nursing Process Step: Planning MSC: Patient Needs: Physiologic Integrity 4. The nurse is preparing a patient for a cesarean birth scheduled to be performed under general anesthesia. Which should the nurse plan to administer, if ordered by the health care provider, to prevent aspiration of gastric contents? (Select all that apply.) a. Citric acid (Bicitra) b. Ranitidine (Zantac) c. Hydroxyzine (Vistaril) d. Glycopyrrolate (Robinul) e. Promethazine (Phenergan) ANS: A, B, D To prevent aspiration of gastric contents during general anesthesia administration of medications to raise the gastric pH and make secretions less acidic, such as citric acid (Bicitra) and ranitidine (Zantac) may be prescribed. In addition, medications to reduce secretions, such as glycopyrrolate (Robinul) may be prescribed. Hydroxyzine (Vistaril) and promethazine (Phenergan) are used to prevent and relieve nausea often associated with opioids. DIF: Cognitive Level: Analysis OBJ: Nursing Process Step: Planning MSC: Patient Needs: Physiologic Integrity COMPLETION 1. The health care provider‘s prescription reads diphenhydramine (Benadryl), 25 mg IV stat. The medication vial reads diphenhydramine (Benadryl), 50 mg/mL. The nurse should prepare how many milliliters to administer the correct dose? Record your answer to one decimal point. mL ANS: 0.5 Desired/available volume = milliliters per dose 25 mg/50 mg 1 mL = 0.5 mL/dose DIF: Cognitive Level: Application OBJ: Nursing Process Step: Implementation MSC: Patient Needs: Safe and Effective Care Environment 2. The nurse is administering fentanyl (Sublimaze) to a patient in labor. The health care provider‘s prescription reads fentanyl (Sublimaze), 100 mcg IV stat. The medication vial readsfentanyl (Sublimaze), 50 mcg/mL. The nurse should prepare how many milliliters to administer the correct dose? Record your answer as a whole number. mL ANS: 2 Desired/available volume = milliliters per dose 100 mcg/50 mcg 1 mL = 2 mL/dose DIF: Cognitive Level: Application OBJ: Nursing Process Step: Implementation MSC: Patient Needs: Safe and Effective Care Environment Chapter 15: Nursing Care During Labor and Birth Foundations of Maternal-Newborn & Women‘s Health Nursing, 7th Edition MULTIPLE CHOICE 1. The nurse is preparing to perform Leopold‘s maneuvers. Please select the rationale for the consistent use of these maneuvers by obstetric providers? a. To determine the status of the membranes b. To determine cervical dilation and effacement c. To determine the best location to assess the fetal heart rate d. To determine whether the fetus is in the posterior position ANS: C Leopold‘s maneuvers are often performed before assessing the fetal heart rate (FHR). These maneuvers help identify the best location to obtain the FHR. A pH test or fern test can be performed to determine the status of the fetal membranes. Dilation and effacement are best determined by vaginal examination. Assessment of fetal position is more accurate with vaginal examination. DIF: Cognitive Level: Application OBJ: Nursing Process Step: Planning MSC: Patient Needs: Health Promotion and Maintenance 2. Which comfort measure should the nurse utilize in order to enable a laboring woman to relax? a. Recommend frequent position changes. b. Palpate her filling bladder every 15 minutes. c. Offer warm wet cloths to use on the patient‘s face and neck. d. Keep the room lights lit so the patient and her coach can see everything. ANS: A Frequent maternal position changes reduce the discomfort from constant pressure and promote fetal descent. A full bladder intensifies labor pain. The bladder should be emptied every 2 hours. Women in labor become very hot and perspire. Cool cloths will provide greater relief. Soft indirect lighting is more soothing than irritating bright lights. DIF: Cognitive Level: Application OBJ: Nursing Process Step: Implementation MSC: Patient Needs: Physiologic Integrity 3. Which assessment finding is an indication of hemorrhage in the recently delivered postpartum patient? a. Elevated pulse rateb. Elevated blood pressure c. Firm fundus at the midline d. Saturation of two perineal pads in 4 hours ANS: A An increasing pulse rate is an early sign of excessive blood loss. If the blood volume were diminishing, the blood pressure would decrease. A firm fundus indicates that the uterus is contracting and compressing the open blood vessels at the placental site. Saturation of one pad within the first hour is the maximum normal amount of lochial flow. Two pads within 4 hours is within normal limits. DIF: Cognitive Level: Analysis OBJ: Nursing Process Step: Assessment MSC: Patient Needs: Physiologic Integrity 4. Which intervention is an essential part of nursing care for a laboring patient? a. Helping the woman manage the pain b. Eliminating the pain associated with labor c. Feeling comfortable with the predictable nature of intrapartal care d. Sharing personal experiences regarding labor and birth to decrease her anxiety ANS: A Helping a patient manage the pain is an essential part of nursing care because pain is an expected part of normal labor and cannot be fully relieved. Labor pain cannot be fully relieved. The labor nurse should always be assessing for unpredictable occurrences. Decreasing anxiety is important; however, managing pain is a top priority. DIF: Cognitive Level: Application OBJ: Nursing Process Step: Planning MSC: Patient Needs: Health Promotion and Maintenance 5. A patient at 40 weeks‘ gestation should be instructed to go to a hospital or birth center for evaluation when she experiences a. increased fetal movement. b. irregular contractions for 1 hour. c. a trickle of fluid from the vagina. d. thick pink or dark red vaginal mucus. ANS: C A trickle of fluid from the vagina may indicate rupture of the membranes, requiring evaluation for infection or cord compression. Decreased or the lack of fetal movement requires further assessment. Irregular contractions are a sign of false labor and do not require further assessment. Bloody show may occur before the onset of true labor. It does not require professional assessment unless the bleeding is pronounced. DIF: Cognitive Level: Application OBJ: Nursing Process Step: Implementation MSC: Patient Needs: Physiologic Integrity 6. Which patient at term should proceed to the hospital or birth center the immediately after labor begins? a.Gravida 2, para 1, who lives 10 minutes away b. Gravida 1, para 0, who lives 40 minutes away c. Gravida 2, para 1, whose first labor lasted 16 hours d. Gravida 3, para 2, whose longest previous labor was 4 hours ANS: D Multiparous women usually have shorter labors than do nulliparous women. The woman described in option D is multiparous with a history of rapid labors, increasing the likelihood that her infant might be born in uncontrolled circumstances. A gravida 2 would be expected to have a longer labor than the gravida in option C. The fact that she lives close to the hospital allows her to stay home for a longer period of time. A gravida 1 will be expected to have the longest labor. The gravida 2 would be expected to have a longer labor than the gravida 3, especially because her first labor was 16 hours. DIF: Cognitive Level: Analysis OBJ: Nursing Process Step: Evaluation MSC: Patient Needs: Safe and Effective Care Environment 7. A woman who is gravida 3, para 2 enters the intrapartum unit. The most important nursing assessments include a. contraction pattern, amount of discomfort, and pregnancy history. b. fetal heart rate, maternal vital signs, and the woman‘s nearness to birth. c. last food intake, when labor began, and cultural practices the couple desires. d. identification of ruptured membranes, the woman‘s gravida and para, and access to a support person. ANS: B All options describe relevant intrapartum nursing assessments, but the focus assessment has priority. If the maternal and fetal conditions are normal and birth is not imminent, other assessments can be performed in an unhurried manner. Contraction pattern, amount of discomfort, and pregnancy history are important nursing assessments but do not take priority if the birth is imminent. Last food intake, when labor began, and cultural practices the couple desires is an assessment that can occur later in the admission process, if time permits. Identification of ruptured membranes, the woman‘s gravida and para, and her support person are assessments that can occur later in the admission process if time permits. DIF: Cognitive Level: Application OBJ: Nursing Process Step: Assessment MSC: Patient Needs: Physiologic Integrity 8. A primigravida at 39 weeks of gestation is observed for 2 hours in the intrapartum unit. The fetal heart rate has been normal. Contractions are 5 to 9 minutes apart, 20 to 30 seconds in duration, and of mild intensity. Cervical dilation is 1 to 2 cm and uneffaced (unchanged from admission). Membranes are intact. The nurse should expect the patient to be a. discharged home with a sedative. b. admitted for extended observation. c. admitted and prepared for a cesarean birth. d. discharged home to await the onset of true labor.ANS: D The situation describes a patient with normal assessments who is probably in false labor and will probably not deliver rapidly once true labor begins. The patient will probably be discharged, and there is no indication that a sedative is needed. These are all indications of false labor; there is no indication that further assessment or observations are indicated. These are all indications of false labor without fetal distress. There is no indication that a cesarean birth is indicated. DIF: Cognitive Level: Application OBJ: Nursing Process Step: Assessment MSC: Patient Needs: Health Promotion and Maintenance 9. The nurse auscultates the fetal heart rate and determines a rate of 152 bpm. Which nursing intervention is most appropriate at this time? a. Inform the mother that the fetal heart rate is normal. b. Reassess the fetal heart rate in 5 minutes because the rate is too high. c. Report the fetal heart rate to the physician or nurse-midwife immediately. d. Suggest to the mother that she is going to have a boy because the heart rate is fast. ANS: A The FHR is within the normal range, so no other action is indicated at this time. The FHR is within the expected range; reassessment should occur, but not in 5 minutes. The FHR is within the expected range; no further action is necessary at this point. The gender of the baby cannot be determined by the FHR. DIF: Cognitive Level: Comprehension OBJ: Nursing Process Step: Implementation MSC: Patient Needs: Health Promotion and Maintenance 10. Which clinical finding would be an indication to the nurse that the fetus may be compromised? a. Active fetal movements b. Fetal heart rate in the 140s c. Contractions lasting 90 seconds d. Meconium-stained amniotic fluid ANS: D When fetal oxygen is compromised, relaxation of the rectal sphincter allows passage of meconium into the amniotic fluid. Active fetal movement is an expected occurrence. The expected FHR range is 120 to 160 bpm. The fetus should be able to tolerate contractions lasting 90 seconds if the resting phase is sufficient to allow for a return of adequate blood flow. DIF: Cognitive Level: Application OBJ: Nursing Process Step: Assessment MSC: Patient Needs: Physiologic Integrity 11. The nurse is caring for a low-risk patient in the active phase of labor. At which interval should the nurse assess the fetal heart rate? a. Every 15 minutes b. Every 30 minutes c. Every 45 minutesd. Every 1 hour ANS: B For the fetus at low risk for complications, guidelines for frequency of assessments are at least every 30 minutes during the active phase of labor. 15-minute assessments would be appropriate for a fetus at high risk. 45-minute assessments during the active phase of labor are not frequent enough to monitor for complications. 1-hour assessments during the active phase of labor are not frequent enough to monitor for complications. DIF: Cognitive Level: Application OBJ: Nursing Process Step: Implementation MSC: Patient Needs: Physiologic Integrity 12. Which nursing assessment indicates that a patient who is in the second stage of labor is almost ready to give birth? a. Bloody mucous discharge increases. b. The vulva bulges and encircles the fetal head. c. The membranes rupture during a contraction. d. The fetal head is felt at 0 station during the vaginal examination. ANS: B A bulging vulva that encircles the fetal head describes crowning, which occurs shortly before birth. Bloody show occurs throughout the labor process and is not an indication of an imminent birth. Rupture of membranes can occur at any time during the labor process and does not indicate an imminent birth. Birth of the head occurs when the station is +4. A zero station indicates engagement. DIF: Cognitive Level: Analysis OBJ: Nursing Process Step: Assessment MSC: Patient Needs: Physiologic Integrity 13. During labor a vaginal examination should be performed only when necessary because of the risk of a. infection. b. fetal injury. c. discomfort. d. perineal trauma. ANS: A Vaginal examinations increase the risk of infection by carrying vaginal microorganisms upward toward the uterus. Properly performed vaginal examinations should not cause fetal injury. Vaginal examinations may be uncomfortable for some women in labor, but that is not the main reason for limiting them. A properly performed vaginal examination should not cause perineal trauma. DIF: Cognitive Level: Understanding OBJ: Nursing Process Step: Assessment MSC: Patient Needs: Physiologic Integrity 14. A 25-year-old primigravida patient is in the first stage of labor. She and her husband have been holding hands and breathing together through each contraction. Suddenly, the patient pushes her husband‘s hand away and shouts, ―Don‘t touch me!‖ This behavior is most likely a. a sign of abnormal labor progress.b. an indication that she needs analgesia. c. normal and related to hyperventilation. d. common during the transition phase of labor. ANS: D The transition phase of labor is often associated with an abrupt change in behavior, including increased anxiety and irritability. This change of behavior is an expected occurrence during the transition phase. If she is in the transitional phase of labor, analgesia may not be appropriate if the birth is near. Hyperventilation will produce signs of respiratory alkalosis. DIF: Cognitive Level: Application OBJ: Nursing Process Step: Assessment MSC: Patient Needs: Psychosocial Integrity 15. At 1 minute after birth, the nurse assesses the newborn to assign an Apgar score. The apical heart rate is 110 bpm, and the infant is crying vigorously with the limbs flexed. The infant‘s trunk is pink and the hands and feet are blue. The Apgar score for this infant is a. 7. b. 8. c. 9. d. 10. ANS: C The Apgar score is 9 because 1 point is deducted from the total score of 10 for the infant‘s blue hands and feet. The baby received 2 points for each of the categories except color. Because the infant‘s hands and feet were blue, this category is given a grade of 1. The baby received 2 points for each of the categories except color. Because the infant‘s hands and feet were blue, this category is given a grade of 1. The infant had 1 point deducted because of the blue color of the hands and feet. DIF: Cognitive Level: Application OBJ: Nursing Process Step: Evaluation MSC: Patient Needs: Physiologic Integrity 16. If a woman‘s fundus is soft 30 minutes after birth, the nurse‘s first action should be to a. massage the fundus. b. take the blood pressure. c. notify the physician or nurse-midwife. d. place the woman in Trendelenburg position. ANS: A The nurse‘s first response should be to massage the fundus to stimulate contraction of the uterus to compress open blood vessels at the placental site, limiting blood loss. The blood pressure is an important assessment to determine the extent of blood loss but is not the top priority. Notification should occur after all nursing measures have been attempted with no favorable results. The Trendelenburg position is contraindicated for this woman at this point. This position would not allow for appropriate vaginal drainage of lochia. The lochia remaining in the uterus would clot and produce further bleeding. DIF: Cognitive Level: Application OBJ: Nursing Process Step: ImplementationMSC: Patient Needs: Physiologic Integrity 17. The nurse thoroughly dries the infant immediately after birth primarily to a. reduce heat loss from evaporation. b. stimulate crying and lung expansion. c. increase blood supply to the hands and feet. d. remove maternal blood from the skin surface. ANS: A Infants are wet with amniotic fluid and blood at birth, which accelerates evaporative heat loss. Rubbing the infant does stimulate crying but is not the main reason for drying the infant. The main purpose of drying the infant is to prevent heat loss. Drying the infant after birth does not remove all of the maternal blood. DIF: Cognitive Level: Understanding OBJ: Nursing Process Step: Implementation MSC: Patient Needs: Physiologic Integrity 18. The nurse notes that a patient who has given birth 1 hour ago is touching her infant with her fingertips and talking to him softly in high-pitched tones. Based on this observation, which action should the nurse take? a. Request a social service consult for psychosocial support. b. Observe for other signs that the mother may not be accepting of the infant. c. Document this evidence of normal early maternal-infant attachment behavior. d. Determine whether the mother is too fatigued to interact normally with her infant. ANS: C Normal early maternal-infant behaviors are tentative and include fingertip touch, eye contact, and using a high-pitched voice when talking to the infant. There is no indication at this point that a social service consult is necessary. The signs are of normal attachment behavior. These are signs of normal attachment behavior; no other assessment is necessary at this point. The mother may be fatigued but is interacting with the infant in an expected manner. DIF: Cognitive Level: Analysis OBJ: Nursing Process Step: Evaluation MSC: Patient Needs: Psychosocial Integrity 19. Which nursing diagnosis would take priority in the care of a primipara patient with no visible support person in attendance? The patient has entered the second stage of labor after a first stage of labor lasting 4 hours. a. Fluid volume deficit (FVD) related to fluid loss during labor and birth process b. Fatigue related to length of labor requiring increased energy expenditure c. Acute pain related to increased intensity of contractions d. Anxiety related to imminent birth process ANS: D A primipara is experiencing the birthing event for the first time and may experience anxiety due to fear of the unknown. It would be important to recognize this because the patient is alone in the labor-birth room and will need additional support and reassurance. Although FVD may occuras a result of fluid loss, prospective management of labor patients includes the use of parenteral fluid therapy; the patient should be monitored for FVD and, if symptoms warrant, receive intervention. Because the patient has been in labor for 4 hours, this is not considered to be a prolonged labor pattern for a primipara patient. Although the patient may be tired, this nursing diagnosis would not be a priority unless there were other symptoms manifested. The patient is entering the second stage of labor; therefore she will be allowed to push with contractions. In terms of pain management, medication will not be administered at this time because of imminent birth. DIF: Cognitive Level: Analysis OBJ: Nursing Process Step: Nursing Diagnosis MSC: Patient Needs: Psychosocial Integrity 20. Which of the following behaviors would be applicable to a nursing diagnosis of ―risk for injury‖ in a patient who is in labor? a. Length of second-stage labor is 2 hours. b. Patient has received an epidural for pain control during the labor process. c. Patient is using breathing techniques during contractions to maximize pain relief. d. Patient is receiving parenteral fluids during the course of labor to maintain hydration. ANS: B A patient who has received medication during labor is at risk for injury as a result of altered sensorium, so this presentation is applicable to the diagnosis. A length of 2 hours for the second stage of labor is within the range of normal. Breathing techniques help maintain control over the labor process. Fluids administered during the labor process are used to prevent potential fluid volume deficit. DIF: Cognitive Level: Application OBJ: Nursing Process Step: Diagnosis MSC: Patient Needs: Safe and Effective Care Environment/Management of Care 21. A nursing priority during admission of a laboring patient who has not had prenatal care is a. obtaining admission labs. b. identifying labor risk factors. c. discussing her birth plan choices. d. explaining importance of prenatal care. ANS: B When a patient has not had prenatal care, the nurse must determine through interviewing and examination the presence of any pregnancy or labor risk factors, obtain admission labs, and discuss birth plan choices. Explaining the importance of prenatal care can be accomplished after the patient‘s history has been completed. DIF: Cognitive Level: Analysis OBJ: Nursing Process Step: Analysis MSC: Patient Needs: Physiologic Integrity 22. The patient in labor experiences a spontaneous rupture of membranes. Which information related to this event must the nurse include in the patient‘s record? a. Fetal heart rate b. Pain level c.Test results ensuring that the fluid is not urine d. The patient‘s understanding of the event ANS: A Charting related to membrane rupture includes the time, FHR, and character and amount of the fluid. Pain is not associated with this event. When it is obvious that the fluid is amniotic fluid, which is anticipated during labor, it is not necessary to verify this by testing. The patient‘s understanding of the event would only need to be documented if it presents a problem. DIF: Cognitive Level: Understanding OBJ: Nursing Process Step: Assessment MSC: Patient Needs: Physiologic Integrity: Reduction of Risk Potential 23. The nurse assesses the amniotic fluid. Which characteristic presents the lowest risk of fetal complications? a. Bloody b. Clear with bits of vernix caseosa c. Green and thick d. Yellow and cloudy with foul odor ANS: B Amniotic fluid should be clear and may include bits of vernix caseosa, the creamy white fetal skin lubricant. Green fluid indicates that the fetus passed meconium before birth. The newborn may need extra respiratory suctioning at birth if the fluid is heavily stained with meconium. Cloudy, yellowish, strong-smelling, or foul-smelling fluid suggests infection. Bloody fluid may indicate partial placental separation. DIF: Cognitive Level: Application OBJ: Nursing Process Step: Assessment MSC: Patient Needs: Physiologic Integrity: Reduction of Risk Potential 24. The nurse is preparing to initiate intravenous (IV) access on a patient in the active phase of labor. Which size IV cannula is best for this patient? a. 18-gauge b. 20-gauge c. 22-gauge d. 24-gauge ANS: A The nurse should select the largest bore cannula possible. An 18-gauge cannula is the largest size available. A 24-gauge cannula would be the smallest. IV access is initiated for hydration prior to epidural placement and for use in an emergency. Both require the rapid administration of fluid, which is most easily accomplished with a large bore cannula. DIF: Cognitive Level: Understanding OBJ: Nursing Process Step: Planning MSC: Patient Needs: Health Promotion and Maintenance 25. The nurse is reviewing the cardinal maneuvers of labor and birth with a group of nursing students. Which maneuver will immediately follow the birth of the baby‘s head? a. Expulsion b. Restitutionc. Internal rotation d. External rotation ANS: B After the head emerges, it realigns with the shoulders (restitution). External rotation occurs as the fetal shoulders rotate internally, aligning their transverse diameter with the anteroposterior diameter of the pelvic outlet. Expulsion occurs when the baby is completely delivered. Internal rotation occurs prior to birth of the head. DIF: Cognitive Level: Understanding OBJ: Nursing Process Step: Planning MSC: Patient Needs: Health Promotion and Maintenance 26. The nurse is performing Leopold‘s maneuvers on her patient. Which figure depicts the Leopold‘s maneuver that determines whether the fetal presenting part is engaged in the maternal pelvis? Refer to Figures A to D. a. b. c. d. ANS: C The maneuver that determines whether the presenting part is engaged (widest diameter at or below a zero station) in the maternal pelvis is performed by palpating the suprapubic area. Next, an attempt is made to grasp the presenting part gently between the thumb and fingers. If the presenting part is not engaged, the grasping movement of the fingers moves it upward in the uterus. If the presenting part is engaged, the fetus will not move upward in the uterus. Palpating the uterine fundus distinguishes between a cephalic and breech presentation. Holding the left hand steady on one side of the uterus while palpating the opposite side of the uterus determines on which side of the uterus is the fetal back and on which side are the fetal arms and legs. Placing your hands on each side of the uterus with fingers pointed toward the inlet determines whether the head is flexed (vertex) or extended (face). DIF: Cognitive Level: Analysis OBJ: Nursing Process Step: Implementation MSC: Patient Needs: Physiologic Integrity 27. After a forceps-assisted birth, the patient is observed to have continuous bright red lochia and a firm fundus. Which other data would indicate the presence of a potential vaginal wall hematoma? a. Lack of an episiotomy b. Mild, intermittent perineal pain c. Lack of pain in the perineal area d. Edema and discoloration of the labia and perineum ANS: D The nurse should monitor for edema and discoloration. Using a cold application to the labia and perineum reduces pain by numbing the area and limiting bruising and edema for the first 12 hours. An episiotomy is performed as the fetal head distends the perineum. The pain with vaginal hematoma is severe and constant. The pain associated with vaginal hematoma is severe. DIF: Cognitive Level: Understanding OBJ: Nursing Process Step: Assessment MSC: Patient Needs: Physiologic Integrity 28. Which patient presentation is an acceptable indication for serial oxytocin induction of labor?a. Multiple fetuses b. Polyhydramnios c. History of long labors d. Past 42 weeks of gestation ANS: D Continuing a pregnancy past the normal gestational period is likely to be detrimental to fetal health. Multiple fetuses overdistend the uterus, making induction of labor high risk. Polyhydramnios also overdistends the uterus, creating a high risk for induction. A history of rapid labors is a reason for induction of labor because of the possibility that the baby would otherwise be born in uncontrolled circumstances. DIF: Cognitive Level: Understanding OBJ: Nursing Process Step: Assessment MSC: Patient Needs: Physiologic Integrity 29. The nurse is explaining the technique of internal version to a nursing orientee. Which statement best describes the technique of internal version? a. Manipulation of the fetus from a breech to a cephalic presentation before labor begins b. Manipulation of the fetus from a transverse lie to a longitudinal lie before cesarean birth c. Manipulation of the second twin from an oblique lie to a transverse lie before labor begins d. Manipulation of the second twin from a transverse lie to a breech presentation during vaginal birth ANS: D Internal version is used only during vaginal birth to manipulate the second twin into a presentation that allows it to be born vaginally. For internal version to occur, the cervix needs to be completely dilated. For internal version to occur, the cervix needs to be dilated. Internal version is done to turn the second twin after the first twin is born. DIF: Cognitive Level: Application OBJ: Nursing Process Step: Assessment MSC: Patient Needs: Physiologic Integrity 30. A maternal indication for the use of vacuum extraction is a. a wide pelvic outlet. b. maternal exhaustion. c. a history of rapid deliveries. d. failure to progress past 0 station. ANS: B The patient who is exhausted will be unable to assist with the expulsion of the fetus. With a wide pelvic outlet, vacuum extraction would not be necessary. With a rapid birth, vacuum extraction would not be necessary. A station of 0 is too high for a vacuum extraction. DIF: Cognitive Level: Understanding OBJ: Nursing Process Step: Assessment MSC: Patient Needs: Physiologic Integrity 31. For which patient should the oxytocin (Pitocin) infusion be discontinued immediately?a. A patient in transition with contractions every 2 minutes lasting 90 seconds each b. A patient in early labor with contractions every 5 minutes lasting 40 seconds each c. A patient in active labor with contractions every 3 minutes lasting 60 seconds each d. A patient in active labor with contractions every 2 to 3 minutes lasting 70 to 80 seconds each ANS: A This patient‘s contraction pattern represents hyperstimulation, and inadequate resting time occurs between contractions to allow placental perfusion. Oxytocin may assist this patient‘s contractions to become closer and more efficient when the contractions are 5 minutes apart. There is an appropriate resting period between this patient‘s contractions. There is an appropriate resting period between this patient‘s contractions for her stage of labor. DIF: Cognitive Level: Analysis OBJ: Nursing Process Step: Evaluation MSC: Patient Needs: Physiologic Integrity 32. Immediately following the forceps-assisted birth of an infant, which action should the nurse implement? a. Assess the infant for signs of trauma. b. Apply a cold pack to the infant‘s scalp. c. Give the infant prophylactic antibiotics. d. Measure the circumference of the infant‘s head. ANS: A Forceps birth can result in local irritation, bruising, or lacerations of the fetal scalp. This would put the infant at risk for cold stress and would be contraindicated. Prophylactic antibiotics are not necessary with a forceps birth. Measuring the circumference of the head is part of the initial nursing assessment. DIF: Cognitive Level: Application OBJ: Nursing Process Step: Implementation MSC: Patient Needs: Physiologic Integrity 33. While assisting with a vacuum extraction birth, which alteration should the nurse immediately report to the obstetric provider? a. Maternal pulse rate of 100 bpm b. Maternal blood pressure of 120/70 mm Hg c. Persistent fetal bradycardia below 100 bpm d. Decreased intensity of uterine contractions ANS: C Fetal bradycardia may indicate fetal distress and may require immediate intervention. Maternal pulse rate may increase due to the pushing process. Blood pressure of 120/70 mm Hg is within expected norms for this stage of labor. Decreased intensity of uterine contractions indicates the birth is imminent at this point. DIF: Cognitive Level: Application OBJ: Nursing Process Step: Assessment MSC: Patient Needs: Physiologic Integrity34. To monitor for potential hemorrhage in the patient who has just had a cesarean birth, which action should the recovery room nurse implement? a. Monitor her urinary output. b. Maintain an intravenous infusion at 1 mL/hour. c. Assess the abdominal dressings for drainage. d. Assess the uterus for firmness every 15 minutes. ANS: D Maintaining contraction of the uterus is important for controlling bleeding from the placental site. Maintaining proper fluid balance will not control hemorrhage. Monitoring urine output is an important assessment, but hemorrhage will first be noted vaginally. Assessing the abdominal dressing is an important assessment to prevent future hemorrhaging from occurring but is not the first priority assessment in the recovery room. DIF: Cognitive Level: Application OBJ: Nursing Process Step: Implementation MSC: Patient Needs: Health Promotion and Maintenance 35. The nurse is preparing to administer a vaginal prostaglandin preparation to ripen the cervix of her patient. With which patient should the nurse question the use of vaginal prostaglandin as a cervical ripening agent? a. The patient who has a Bishop‘s score of 5 b. The patient who is at 42 weeks of gestation c. The patient who had a previous low transverse cesarean birth d. The patient who had previous surgery in the upper uterus ANS: D Prostaglandins are contraindicated in patients who have had a previous surgery in the upper uterus, such as a previous classic cesarean incision or extensive surgery for uterine fibroids. A side effect of prostaglandin administration is hyperstimulation of the uterus. A Bishop‘s score of 5, 42 weeks of gestation, or a previous low transverse cesarean birth are not contraindications for cervical ripening. DIF: Cognitive Level: Analysis OBJ: Nursing Process Step: Evaluation MSC: Patient Needs: Physiologic Integrity 36. A patient who is receiving oxytocin (Pitocin) infusion for the augmentation of labor is experiencing a contraction pattern of more than eight contractions in a 10-minute period. Which intervention would be a priority? a. Increase rate of Pitocin infusion to help spread out contraction pattern. b. Place oxygen on patient at 8 to 10 L/minute via face mask and turn patient to left side. c. Stop Pitocin infusion. d. Call physician to obtain an order for initiation of magnesium sulfate. ANS: C The patient is exhibiting uterine tachysystole (uterine tetany). Priority intervention is to stop the infusion. The next course of action is to place oxygen on the patient and reposition and increasethe flow rate of the primary infusion. If the condition does not improve, the physician may be called for additional orders. DIF: Cognitive Level: Analysis OBJ: Nursing Process Step: Implementation MSC: Patient Needs: Safe and Effective Care Environment 37. On vaginal exam, the patient‘s cervix is anterior, soft, 70% effaced, dilated 2 cm, and the presenting part is at 0 station. The Bishop‘s score for this patient is a. 6. b. 9. c. 10. d. 12. ANS: B On the Bishop‘s scoring system, an anterior cervix = 2 points, soft cervix = 2 points, 70% effaced = 2 points, 2 cm dilated = 1 point, and 0 station = 2 points, for a total score of 9. DIF: Cognitive Level: Application OBJ: Nursing Process Step: Analysis MSC: Patient Needs: Health Promotion and Maintenance 38. Which assessment would be important for a 6-hour-old infant who has bruising over the cheeks from a forceps birth? a. Presence of newborn reflexes b. Symmetry of facial movements c. Caput and molding of the head d. Anterior and posterior fontanels ANS: B Following a forceps birth, the infant may have ecchymoses and facial nerve injury. Facial asymmetry suggests facial nerve damage. Changes in newborn reflexes, presence of caput and molding, and changes in the anterior and posterior fontanels are not risks associated with trauma to the infant‘s face. DIF: Cognitive Level: Analysis OBJ: Nursing Process Step: Analysis MSC: Patient Needs: Health Promotion and Maintenance 39. Which aspect of newborn assessment may be limited by the application of a vacuum extractor at birth? a. Anterior fontanel b. Coronal suture lines c. Posterior fontanel d. Biparietal diameter ANS: C The vacuum extractor is applied on the occipital bone and may create scalp edema at the application site. The posterior fontanel connects the occipital bone to the parietal bones. The anterior fontanel, coronal suture lines, and parietal bones are not part of the application area for a vacuum extractor.DIF: Cognitive Level: Analysis OBJ: Nursing Process Step: Analysis MSC: Patient Needs: Health Promotion and Maintenance 40. Which breech presentation should the nurse recognize as being favorable for an external cephalic version? a. 36-week gestation with low-lying placenta b. 38-week gestation with one previous cesarean c. 37-week gestation with fetal weight of 7 lb d. 40-week gestation with several uterine fibroids ANS: C An external cephalic version (changing the fetal presentation from breech to cephalic) is more successful when the pregnancy is at least 37 weeks and there is still adequate room and fluid to manipulate the fetus but prior to term or onset of labor. A low-lying placenta, previous cesarean birth, and uterine fibroids are contraindications for version. DIF: Cognitive Level: Analysis OBJ: Nursing Process Step: Analysis MSC: Patient Needs: Physiologic Integrity/Physiologic Adaptation 41. Following an external cephalic version, which assessment finding indicates a complication? a. Onset of irregular contractions b. Maternal blood pressure of 110/70 mm Hg c. Deceleration of FHR to 88 bpm d. Maternal pulse rate of 100 bpm ANS: C A serious risk of external cephalic version is that the fetus may become entangled in the umbilical cord, compressing its vessels and resulting in hypoxia. The onset of irregular contractions, maternal blood pressure of 110/70 mm Hg, and maternal pulse rate of 100 bpm are normal findings. DIF: Cognitive Level: Analysis OBJ: Nursing Process Step: Assessment MSC: Patient Needs: Physiologic Integrity/Physiologic Adaptation 42. The pregnant patient expresses a desire to schedule birth during the baby‘s father‘s furlough from military service. The nurse explains that prior to induction of labor, it is essential to determine which clinical finding? a. Dilated cervix b. Fetal lung maturity c. Rupture of membranes d. Uterine hypertonia ANS: B Reassurance of fetal lung maturity is essential before elective procedures such as induction or cesarean. The cervix must be favorable for dilation but need not be dilated prior to induction. Prior rupture of membranes is not necessary for induction. Uterine hypertonia is a risk factor associated with induction of labor.DIF: Cognitive Level: Knowledge OBJ: Nursing Process Step: Assessment MSC: Patient Needs: Health Promotion and Maintenance 43. The labor nurse is developing a plan of care for a patient admitted in active labor with spontaneous rupture of the membranes 6 hours prior to admission with clear fluid. On admission, vital signs were as follows: maternal heart rate (HR) 92 bpm; fetal rate (FHR) baseline, 150 to 160 bpm; blood pressure, 124/76 mm Hg; temperature 37.2 C (99 F). What is the priority nursing action for this patient? a. Fetal acoustic stimulation b. Assess temperature every 2 hours c. Change absorption pads under her hips every 2 hours d. Review white blood cell count (WBC) drawn at admission ANS: B The woman‘s temperature should be assessed at least every 2 to 4 hours after the membranes rupture. Elevations above 38C (100.4F) should be reported. A rising FHR and fetal tachycardia (above 160 bpm) may precede maternal fever. The fetal heart rate is at the high end of the acceptable range and the maternal temperature is slightly above normal. These parameters warrant watching closely with more frequent vital signs. The WBC is often falsely elevated in labor, largely related to the stress of labor. The FHR with a baseline of 150 to 160 bpm demonstrates moderate variability, and fetal acoustic stimulation is not warranted. Amniotic fluid is emitted from the vagina at variable rates and the underpad needs to be changed as needed. DIF: Cognitive Level: Analysis OBJ: Nursing Process Step: Assessment MSC: Patient Needs: Health Promotion and Maintenance MULTIPLE RESPONSE 1. A laboring patient is 10 cm dilated; however, she does not feel the urge to push. The nurse understands that according to laboring down, the advantages of waiting until an urge to push are which of the following? (Select all that apply.) a. Less maternal fatigue b. Less birth canal injuries c. Decreased pushing time d. Faster descent of the fetus e. An increase in frequency of contractions ANS: A, B, C Delayed pushing has been shown to result in less maternal fatigue and decreased pushing time. Pushing vigorously sooner than the onset of the reflexive urge may contribute to birth canal injury because her vaginal tissues are stretched more forcefully and rapidly than if she pushed spontaneously and in response to her body‘s signals. A brief slowing of contractions often occurs at the beginning of the second stage. DIF: Cognitive Level: Analysis OBJ: Nursing Process Step: Evaluation MSC: Patient Needs: Physiologic Integrity 2. Which interventions should be performed in the birth room to facilitate thermoregulation of the newborn? (Select all that apply.) a.Place the infant covered with blankets in the radiant warmer. b. Dry the infant off with sterile towels. c. Place stockinette cap on infant‘s head. d. Bathe the newborn within 30 minutes of birth. e. Remove wet linen as needed. ANS: B, C, E Following birth, the newborn is at risk for hypothermia. Therefore nursing interventions are aimed at maintaining warmth. Drying the infant off, in addition to maintaining warmth, helps stimulate crying and lung expansion, which helps in the transition period following birth. Placing a cap on the infant‘s head helps prevent heat loss. Removal of wet linens helps minimize further heat loss caused by exposure. Newborns should not be covered while in a radiant warmer with blankets because this will impede birth of heat transfer. Bathing a newborn should be delayed for at least a few hours so that the newborn temperature can stabilize during the transition period. DIF: Cognitive Level: Analysis OBJ: Nursing Process Step: Implementation MSC: Patient Needs: Health Promotion and Maintenance: Ante/Intra/Postpartum and Newborn Care 3. When caring for a patient in labor who is considered to be at low risk, which assessments should be included in the plan of care? (Select all that apply.) a. Check the DTR each shift. b. Monitor and record vital signs frequently during the course of labor. c. Document the FHR pattern, noting baseline and response to contraction patterns. d. Indicate on the EFM tracing when maternal position changes are done. e. Provide food, as tolerated, during the course of labor. ANS: B, C, D Nursing care of the normal laboring patient would include monitoring and documentation of vital signs as part of the labor assessment, documentation the FHR, checking patterns to look for assurance of fetal well-being by evaluating baseline and the fetal response to contraction patterns, and noting any position changes on the monitor tracing to evaluate the fetal response. Providing dietary offerings during the course of labor is not part of the nursing care plan because the introduction of food may lead to nausea and vomiting in response to the labor process and might affect the mode of birth. DIF: Cognitive Level: Analysis OBJ: Nursing Process Step: Implementation MSC: Patient Needs: Health Promotion and Maintenance: Ante/Intra/Postpartum and Newborn Care 4. The nurse is monitoring a patient in the active stage of labor. Which conditions associated with fetal compromise should the nurse monitor? (Select all that apply.) a. Maternal hypotension b. Fetal heart rate of 140 to 150 bpm c.Meconium-stained amniotic fluid d. Maternal fever—38C (100.4F) or higher e. Complete uterine relaxation of more than 30 seconds between contractions ANS: A, C, D Conditions associated with fetal compromise include maternal hypotension (may divert blood flow away from the placenta to ensure adequate perfusion of the maternal brain and heart), meconium-stained (greenish) amniotic fluid, and maternal fever (38C [100.4F] or higher). Fetal heart rate of 110 to 160 bpm for a term fetus is normal. Complete uterine relaxation is a normal finding. DIF: Cognitive Level: Analysis OBJ: Nursing Process Step: Evaluation MSC: Patient Needs: Physiologic Integrity 5. The nurse is caring for a patient in the fourth stage of labor. Which assessment findings should the nurse identify as a potential complication? (Select all that apply.) a. Soft boggy uterus b. Maternal temperature of 37.2C (99F) c. High uterine fundus displaced to the right d. Intense vaginal pain unrelieved by analgesics e. Half of a lochia pad saturated in the first hour after birth ANS: A, C, D Assessment findings that may indicate a potential complication in the fourth stage include a soft boggy uterus, high uterine fundus displaced to the right, and intense vaginal pain unrelieved by analgesics. The maternal temperature may be slightly elevated after birth because of the inflammation to tissues, and half of a lochia pad saturated in the first hour after birth is within expected amounts. DIF: Cognitive Level: Analysis OBJ: Nursing Process Step: Assessment MSC: Patient Needs: Physiologic Integrity 6. Induction of labor is considered an acceptable obstetric procedure if it is a safe time to deliver the fetus. The charge nurse on the labor and birth unit is often asked to schedule patients for this procedure and therefore must be cognizant of the specific conditions appropriate for labor induction, including which of the following? (Select all that apply.) a. Fetal death b. Postterm pregnancy c. Rupture of membranes at or near term d. Convenience of the patient or her health care provider e. Chorioamnionitis (inflammation of the amniotic sac) ANS: A, B, C, E Fetal death, postterm pregnancy, rupture of members, and chorioamnionitis are all acceptable indications for induction. Other conditions include intrauterine growth retardation (IUGR), maternal-fetal blood incompatibility, hypertension, and placental abruption. Elective inductionsfor convenience of the patient or her provider are not recommended; however, they have become common. Factors such as rapid labors and living a long distance from a health care facility may be a valid reason in such a circumstance. DIF: Cognitive Level: Application OBJ: Nursing Process Step: Planning MSC: Patient Needs: Physiologic Integrity COMPLETION 1. The nurse in the birth room receives an order to give a newborn 0.3 mg of naloxone (Narcan) intramuscularly. The medication vial reads naloxone (Narcan), 0.4 mg/mL. The nurse should prepare how many milliliters to administer the correct dose? Fill in the blank and record your answer using two decimal places. mL ANS: 0.75 Use the medication calculation formula to calculate the correct dose: Desired/available volume = milliliters per dose (0.3 mg/0.4 mg) 1 mL = 0.75 mL/dose DIF: Cognitive Level: Application OBJ: Nursing Process Step: Implementation MSC: Patient Needs: Safe and Effective Care Environment Chapter 16: Intrapartum Complications Foundations of Maternal-Newborn & Women‘s Health Nursing, 7th Edition MULTIPLE CHOICE 1. Which pelvic shape is most conducive to vaginal labor and birth? a. Android b. Gynecoid c. Platypelloid d. Anthropoid ANS: B The gynecoid pelvis is round and cylinder-shaped, with a wide pubic arch and is considered the most suitable for a vaginal birth. An android pelvis has been described as heart shaped, with more prominent ischial spines and a narrow pubic arch. A vaginal birth will be more difficult, with the need for harder pushing and often some form of instrumentation. The anthropoid pelvis is a long narrow oval, with a narrow pubic arch. It is more favorable than the android or platypelloid pelvic shape. The platypelloid pelvis is flat, wide, short, and oval and has a very poor prognosis for vaginal birth. Most women have characteristics from two or more types of pelvic shapes. DIF: Cognitive Level: Understanding OBJ: Nursing Process Step: Assessment MSC: Patient Needs: Health Promotion and Maintenance 2. Which action by the nurse prevents infection in the labor and birth area? a. Using clean techniques for all procedures b. Keeping underpads and linens as dry as possible c. Cleaning secretions from the vaginal area by using a back to front motion d. Performing vaginal examinations every hour while the patient is in active labor ANS: BBacterial growth prefers a moist, warm environment. Use an aseptic technique if membranes are not ruptured; use a sterile technique if membranes are ruptured. Vaginal drainage should be removed with a front to back motion to decrease fecal contamination. Vaginal examinations should be limited to decrease transmission of vaginal organisms into the uterine cavity. DIF: Cognitive Level: Application OBJ: Nursing Process Step: Implementation MSC: Patient Needs: Safe and Effective Care Environment 3. A pregnant patient with premature rupture of membranes is at higher risk for postpartum infection. Which assessment data indicates a potential infection? a. Fetal heart rate, 150 beats/minute b. Maternal temperature, 37.2C (99F) c. Cloudy amniotic fluid, with strong odor d. Lowered maternal pulse and decreased respiratory rates ANS: C Amniotic fluid should be clear and have a mild odor, if any. Fetal tachycardia of greater than 160 beats/minute is often the first sign of intrauterine infection. A temperature of 38C (100.4F) or higher is a classic symptom of infection. Vital signs should be assessed hourly to identify tachycardia or tachypnea, which often accompany temperature elevation. DIF: Cognitive Level: Analysis OBJ: Nursing Process Step: Evaluation MSC: Patient Needs: Physiologic Integrity 4. A patient with polyhydramnios is admitted to a labor-birth-recovery-postpartum (LDRP) suite. Her membranes rupture and the fluid is clear and odorless; however, the fetal heart monitor indicates bradycardia and variable decelerations. Which action should be taken next? a. Perform Leopold maneuvers. b. Perform a vaginal examination. c. Apply warm saline soaks to the vagina. d. Place the patient in a high Fowler position. ANS: B A prolapsed cord may not be visible but may be palpated on vaginal examination. The priority is to relieve pressure on the umbilical cord. Leopold maneuvers are not an appropriate action at this time. Moist towels retard cooling and drying of the prolapsed cord, but it is hoped the fetus will be delivered before this occurs. The high Fowler position will increase cord compression and decrease fetal oxygenation. DIF: Cognitive Level: Application OBJ: Nursing Process Step: Implementation MSC: Patient Needs: Physiologic Integrity 5. Which technique is least effective for the patient with persistent occiput posterior position? a. Squatting b. Lying supine and relaxing c. Sitting or kneeling, leaning forward with support d. Rocking the pelvis back and forth while on hands and kneesANS: B Lying supine increases the discomfort of back labor. Squatting aids rotation and fetal descent. A sitting or kneeling position may help the fetal head to rotate to occiput anterior. Rocking the pelvis encourages rotation from occiput posterior to occiput anterior. DIF: Cognitive Level: Application OBJ: Nursing Process Step: Implementation MSC: Patient Needs: Health Promotion and Maintenance 6. Birth for the nulliparous patient with a fetus in a breech presentation is usually a. cesarean birth. b. vaginal birth. c. vacuumed extraction. d. forceps-assisted birth. ANS: A Birth for the nulliparous patient with a fetus in breech presentation is almost always cesarean birth. The greatest fetal risk in the vaginal birth of breech presentation is that the head (largest part of the fetus) is the last to be delivered. The birth of the rest of the baby must be quick so the infant can breathe. Serious trauma to maternal or fetal tissues is likely if the vacuum extractor birth is difficult. Most breech births are difficult. The health care provider may assist rotation of the head with forceps. A cesarean birth may be required. DIF: Cognitive Level: Understanding OBJ: Nursing Process Step: Assessment MSC: Patient Needs: Physiologic Integrity 7. Which patient situation presents the greatest risk for the occurrence of hypotonic dysfunction during labor? a. A primigravida who is 17 years old b. A 22-year-old multiparous patient with ruptured membranes c. A primigravida who has requested no analgesia during her labor d. A multiparous patient at 39 weeks of gestation who is expecting twins ANS: D Overdistention of the uterus in a multiple pregnancy is associated with hypotonic dysfunction because the stretched uterine muscle contracts poorly. A young primigravida usually will have good muscle tone in the uterus. This prevents hypotonic dysfunction. There is no indication that this patient‘s uterus is overdistended, which is the main cause of hypotonic dysfunction. A primigravida usually will have good uterine muscle tone, and there is no indication of an overdistended uterus. DIF: Cognitive Level: Analysis OBJ: Nursing Process Step: Assessment MSC: Patient Needs: Physiologic Integrity 8. Which factor is most likely to result in fetal hypoxia during a dysfunctional labor? a. Incomplete uterine relaxation b. Maternal fatigue and exhaustion c. Maternal sedation with narcotics d.Administration of tocolytic drugs ANS: A A high uterine resting tone, with inadequate relaxation between contractions, reduces maternal blood flow to the placenta and decreases the fetal oxygen supply. Maternal fatigue usually does not decrease uterine blood flow. Maternal sedation will sedate the fetus but should not decrease blood flow. Tocolytic drugs decrease contractions. This will increase uterine blood flow. DIF: Cognitive Level: Understanding OBJ: Nursing Process Step: Assessment MSC: Patient Needs: Physiologic Integrity 9. After a birth complicated by a shoulder dystocia, the infant‘s Apgar scores were 7 at 1 minute and 9 at 5 minutes. The infant is now crying vigorously. The nurse in the birthing room should a. palpate the infant‘s clavicles. b. encourage the parents to hold the infant. c. perform a complete newborn assessment. d. give supplemental oxygen with a small face mask. ANS: A Because of the shoulder dystocia, the infant‘s clavicles may have been fractured. Palpation is a simple assessment to identify crepitus or deformity that requires follow-up. The infant needs to be assessed for clavicle fractures before excessive movement. A complete newborn assessment is necessary for all newborns, but assessment of the clavicle is top priority for this infant. The Apgar indicates that no respiratory interventions are needed. DIF: Cognitive Level: Understanding OBJ: Nursing Process Step: Assessment MSC: Patient Needs: Physiologic Integrity 10. A laboring patient in the latent phase is experiencing uncoordinated irregular contractions of low intensity. How should the nurse respond to complaints of constant cramping pain? a. ―You are only 2 cm dilated, so you should rest and save your energy for when the contractions get stronger.‖ b. ―Let me take off the monitor belts and help you get into a more comfortable position.‖ c. ―You must breathe more slowly and deeply so there is greater oxygen supply for your uterus. That will decrease the pain.‖ d. ―I have notified the doctor that you are having a lot of discomfort. Let me rub your back and see if that helps.‖ ANS: D Intervention is needed to manage the dysfunctional pattern. Offering support and comfort is important to help the patient cope with the situation, no matter at what stage. It is important to get her into a more comfortable position and fetal monitoring should continue. An alteration in breathing pattern will not decrease the pain in this situation. DIF: Cognitive Level: Application OBJ: Nursing Process Step: Implementation MSC: Patient Needs: Health Promotion and Maintenance 11. Which nursing action should be initiated first when there is evidence of prolapsed cord? a. Notify the health care provider. b. Apply a scalp electrode.c. Prepare the mother for an emergency cesarean birth. d. Reposition the mother with her hips higher than her head. ANS: D The priority is to relieve pressure on the cord. Changing the maternal position will shift the position of the fetus so that the cord is not compressed. Notifying the health care provider is a priority but not the first action. It would not be appropriate to apply a scalp electrode at this time. Preparing the mother for a cesarean birth would not be the first priority. The nurse may need to hold the presenting part away from the cord until delivery is complete. DIF: Cognitive Level: Application OBJ: Nursing Process Step: Implementation MSC: Patient Needs: Physiologic Integrity 12. A patient who has had two previous cesarean births is in active labor when she suddenly complains of pain between her scapulae. Which should be the nurse‘s priority action? a. Notify the health care provider promptly. b. Observe for abnormally high uterine resting tone. c. Decrease the rate of nonadditive intravenous fluid. d. Reposition the patient with her hips slightly elevated. ANS: A Pain between the scapulae may occur when the uterus ruptures because blood accumulates under the diaphragm. This is an emergency that requires medical intervention. Observing for high uterine resting tones should have been done before the sudden pain. High uterine resting tones put the patient at high risk for uterine rupture. The patient is now at high risk for shock. Nonadditive intravenous fluids should be increased. Repositioning the patient with her hips slightly elevated is the treatment for a prolapsed cord. That position in this scenario would cause respiratory difficulties. DIF: Cognitive Level: Application OBJ: Nursing Process Step: Implementation MSC: Patient Needs: Physiologic Integrity 13. Which factor should alert the nurse to the potential for a prolapsed umbilical cord? a. Oligohydramnios b. Pregnancy at 38 weeks of gestation c. Presenting part at a station of –3 d. Meconium-stained amniotic fluid ANS: C Because the fetal presenting part is positioned high in the pelvis and is not well applied to the cervix, a prolapsed cord could occur if the membranes rupture. Hydramnios puts the patient at high risk for a prolapsed umbilical cord. A very small fetus, normally preterm, puts the patient at risk for a prolapsed umbilical cord. Meconium-stained amniotic fluid shows that the fetus already has been compromised but does not increase the chance of a prolapsed cord. DIF: Cognitive Level: Understanding OBJ: Nursing Process Step: Assessment MSC: Patient Needs: Physiologic Integrity 14. The fetus in a breech presentation is often born by cesarean birth because a.the buttocks are much larger than the head. b. compression of the umbilical cord is more likely. c. internal rotation cannot occur if the fetus is breech. d. postpartum hemorrhage is more likely if the patient delivers vaginally. ANS: B After the fetal legs and trunk emerge from the patient‘s vagina, the umbilical cord can be compressed between the maternal pelvis and the fetal head if a delay occurs in the birth of the head. The head is the largest part of a fetus. Internal rotation can occur with a breech. There is no relationship between breech presentation and postpartum hemorrhage. DIF: Cognitive Level: Understanding OBJ: Nursing Process Step: Assessment MSC: Patient Needs: Physiologic Integrity 15. A patient who is 32 weeks pregnant telephones the nurse at her obstetrician‘s office and complains of constant backache. She asks what pain reliever is safe for her to take. The best nursing response is a. ―You should come into the office and let the doctor check you.‖ b. ―Acetaminophen is acceptable during pregnancy. You should not take aspirin, however.‖ c. ―Back pain is common at this time during pregnancy because you tend to stand with a sway back.‖ d. ―Avoid medication because you are pregnant. Try soaking in a warm bath or using a heating pad on low before taking any medication.‖ ANS: A A prolonged backache is one of the subtle symptoms of preterm labor. Early intervention may prevent preterm birth. The patient needs to be assessed for preterm labor before providing pain relief. DIF: Cognitive Level: Application OBJ: Nursing Process Step: Implementation MSC: Patient Needs: Physiologic Integrity 16. Which is the priority nursing assessment for the patient undergoing tocolytic therapy with terbutaline (Brethine)? a. Intake and output b. Maternal blood glucose level c. Internal temperature and odor of amniotic fluid d. Fetal heart rate, maternal pulse, and blood pressure ANS: D All assessments are important; however, those most relevant to tocolytic therapy include the fetal heart rate and maternal pulse, which tend to increase, and the maternal blood pressure, which tends to exhibit a wide pulse pressure. Intake and output and glucose are not important assessments to monitor for side effects of terbutaline. Internal temperature and odor of amniotic fluid are important if the membranes have ruptured; however, these are not relevant to the medication. DIF: Cognitive Level: Application OBJ: Nursing Process Step: PlanningMSC: Patient Needs: Health Promotion and Maintenance 17. Which clinical finding during assessment indicates uterine rupture? a. Fetal tachycardia occurs. b. The patient becomes dyspneic. c. Labor progresses unusually quickly. d. Contractions abruptly stop during labor. ANS: D A large rupture of the uterus will disrupt its ability to contract. Fetal tachycardia is a sign of hypoxia. With a large rupture, the nurse should be alert for the earlier signs. Dyspnea is not an early sign of a rupture. Contractions will stop with a rupture. DIF: Cognitive Level: Understanding OBJ: Nursing Process Step: Assessment MSC: Patient Needs: Physiologic Integrity 18. Which intervention should be incorporated in the plan of care for a labor patient who is experiencing hypertonic labor? Vaginal exam is unchanged from prior exam—3 cm, 80% effaced, and 0 station presenting part vertex. a. Augmentation of labor with oxytocin (Pitocin) b. AROM c. Performing a vaginal exam to denote progress d. Preparing the patient for epidural administration as ordered by the physician ANS: D The administration of an epidural may help relieve increased uterine resting tone by decreasing maternal pain sensation. Hypertonic labor pattern indicates increased uterine resting tone; therefore augmentation would not be advised at this time because it would cause further uterine irritation in the form of contractions. Rupture of membranes would not be warranted at this time because the critical issue is to resolve the increased uterine resting tone. There is no indication that a vaginal exam is required at this time based on the information provided. DIF: Cognitive Level: Analysis OBJ: Nursing Process Step: Implementation MSC: Patient Needs: Physiologic Integrity/Physiologic Adaptation 19. During the course of the birth process, the physician suspects that a shoulder dystocia is occurring and asks the nurse for assistance. Which priority action should be taken in response to this request? a. Put pressure on the fundus. b. Ask the physician if he or she would like you to prepare for a surgical method of birth. c. Tell the patient not to push until you prepare the vacuum extraction device for physician. d. Reposition the patient to facilitate birth. ANS: B In the presence of a suspected shoulder dystocia, a surgical birth method is typically indicated to avoid complications from this type of abnormal presentation. Fundal pressure is no longer recommended as a treatment strategy because it may cause additional problems. Vacuumextraction will not help to resolve the birth issue and may lead to further complications. Repositioning of the patient may not be effective to relieve this condition and facilitate birth. DIF: Cognitive Level: Analysis OBJ: Nursing Process Step: Planning MSC: Patient Needs: Safe and Effective Care Environment/Establishing Priorities 20. A pregnant patient who has had a prior obstetric history of preterm labors is pregnant with her third child. The physician has ordered a fetal fibronectin test. Which instructions should be given to the patient regarding this clinical test? a. Patient must be NPO prior to testing. b. Blood work will be drawn every week to help confirm the start of preterm labor. c. Patient should refrain from sexual activity prior to testing. d. A urine specimen will be collected for testing. ANS: C Fetal fibronectin testing has a predictive value relative to the onset of preterm labor. A specimen is collected from the vaginal area. False-positive results can occur in response to excessive cervical manipulation, in the presence of bleeding, and as a result of sexual activity. DIF: Cognitive Level: Application OBJ: Nursing Process Step: Planning MSC: Patient Needs: Physiologic Integrity/Physiologic Adaptation 21. An obstetric patient has been identified as being high risk. The patient has had activities restricted (placed on bed rest) until the end of the pregnancy. Currently, she is at 32 weeks‘ gestation and has two other children at home, ages 3 and 6. The patient‘s husband works at home. A nursing diagnosis of Impaired Home Maintenance is noted. Which statement potentially identifies a long-term goal? a. The patient and husband will be able to adapt their schedules accordingly to meet activities of daily living until the patient‘s next scheduled antepartum visit the following week. b. The patient and husband will hire a nanny to act as an additional caregiver for the next month. c. The patient will continue to take care of her children at home, taking frequent rest periods. d. The patient and husband will make arrangements for child care routine activity assistance for the rest of the pregnancy. ANS: D A long-term goal is based on acknowledgment of prescribed clinical treatment conditions for the specified timeframe. Planning for caregiving for the next week or month provide evidence of short-term goals. It is not realistic for the patient to take care of her children at home with rest period because the patient will not be maintaining the prescribed therapy regimen and thus may be at risk to further develop complications. DIF: Cognitive Level: Analysis OBJ: Nursing Process Step: Evaluation MSC: Patient Needs: Health Promotion and Maintenance 22. A labor patient has been diagnosed with cephalopelvic disproportion (CPD) following attempts at pushing for 2 hours with no progress. Based on this information, which birth method is most appropriate? a. Vaginal birth with vacuum extraction b.Augmentation of labor with oxytocin (Pitocin) to improve contraction pattern and strengthen contractions c. Cesarean section d. Insertion of Foley catheter into empty bladder to provide more room for fetal descent ANS: C The presence of CPD is a contraindication for vaginal birth. To prevent further complications, the patient should be prepped for a cesarean section. DIF: Cognitive Level: Analysis OBJ: Nursing Process Step: Evaluation MSC: Patient Needs: Pathophysiologic Integrity/Medical Emergency 23. A patient is diagnosed with anaphylactoid syndrome of pregnancy. Which therapeutic intervention does the nurse expect will be included in the plan of care? a. Administration of antihypertensive medication b. Initiation of CPR and other life support measures c. Respiratory treatments with nebulizers d. Internal fetal monitoring ANS: B Anaphylactoid syndrome was previously known as amniotic fluid embolism. This is a rare complication that results in a medical emergency in which CPR measures are initiated and mechanical ventilation, correction of shock and hypotension, and blood component therapy are also begun. Meconium-stained fluid is associated with particulate matter that may be found in the maternal circulation. Internal fetal monitoring may provide a potential source of entry because it is an invasive procedure. The use of nebulizers is not indicated. The patient with this condition will be hypotensive, not hypertensive. DIF: Cognitive Level: Analysis OBJ: Nursing Process Step: Implementation MSC: Patient Needs: Pathophysiologic Integrity/Medical Emergency 24. A 20-year-old gravida 1, para 0 woman, is evaluated to be at 42 weeks‘ gestation on admission to the labor and birth unit. The patient is not in labor at the current time; however, she has been sent over by the physician to be admitted for the induction of labor. The patient indicates to you that she would rather go home and wait for natural labor to start. How should the nurse respond to the patient‘s request? a. There is no way to tell if any complications would arise. Because the patient is not presenting with any problems, the nurse should call the health care provider and inform her or him of the patient‘s decision to go home and wait. b. Inform the patient that there are a number of serious concerns related to a postdate pregnancy and that she would be better off to be monitored in a clinical setting. c. Tell the patient that an assessment will be done and if there are no findings indicating that an induction of labor would be favorable, the patient will be sent home. d. Tell the patient that confirmation of a due date can be off by 2 weeks and possibly be even later than 42 weeks, so it is better to follow the physician‘s directions. ANS: BThe most serious concern related to a postdate pregnancy is that of fetal compromise based on the fact that the placenta function deteriorates. Although one can appreciate that the patient wants to have a natural labor experience, some women do not go into labor for various physiologic reasons. Therefore it is best for the patient to remain in a supervised clinical setting. Indicating that the patient could possibly go home would place the patient at risk and the nurse at risk for practicing outside of his or her scope of practice. Even though there can be a difference in the calculated due date, it is highly unlikely that the pregnancy has gone longer than 42 weeks. DIF: Cognitive Level: Analysis OBJ: Nursing Process Step: Implementation MSC: Patient Needs: Pathophysiologic Integrity/Medical Emergency 25. Which presentation is least likely to occur with a hypotonic labor pattern? a. Prolonged labor duration b. Fetal distress c. Maternal comfort during labor d. Irregular labor contraction pattern ANS: B A hypotonic labor pattern indicates that uterine contractions are variable in nature and weak and thus do not affect cervical change in a timely manner. Labor patterns are prolonged in duration and patients are typically comfortable but can become easily tired and frustrated because of the inability of their labor to progress to conclusion. The least likely occurrence is that of fetal distress, because the uterine contraction pattern is not coordinated and/or strong enough to exert pressure. DIF: Cognitive Level: Application OBJ: Nursing Process Step: Assessment MSC: Patient Needs: Pathophysiologic Integrity 26. Which finding on vaginal examination would be a concern if a spontaneous rupture of the membranes has occurred? a. Cephalic presentation b. Left occiput position c. Dilation 2 cm d. Presenting part at station ANS: D If membranes rupture while the presenting part is at a high station, prolapse of the umbilical cord is more likely; a cephalic presentation, left occiput position, and dilation of 2 cm are normal findings. DIF: Cognitive Level: Analysis OBJ: Nursing Process Step: Analysis MSC: Patient Needs: Safe and Effective Care Environment/Management of Care 27. Which intervention would be most effective if the fetal heart rate drops following a spontaneous rupture of the membranes? a. Apply oxygen at 8 to 10 L/minute. b. Stop the Pitocin infusion. c.Position the patient in the knee-chest position. d. Increase the main line infusion to 150 mL/hour. ANS: C A drop in the fetal heart rate following rupture of the membranes indicates a compressed or prolapsed umbilical cord. Immediate action is necessary to relieve pressure on the cord. The knee-chest position uses gravity to shift the fetus out of the pelvis and relieves pressure on the umbilical cord, applying oxygen will not be effective until compression is relieved, and stopping the Pitocin infusion and increasing the main line fluid do not directly affect cord compression. DIF: Cognitive Level: Application OBJ: Nursing Process Step: Analysis MSC: Patient Needs: Safe and Effective Care Environment/Management of Care 28. Which finding would be indicative of an adverse response to terbutaline (Brethine)? a. Fetal heart rate (FHR) of 134 bpm b. Heart rate of 122 bpm c. Two episodes of diarrhea d. Fasting blood glucose level of 100 mg/dL ANS: B Terbutaline (Brethine) stimulates beta-adrenergic receptors of the sympathetic system. This action results primarily in bronchodilation, inhibition of uterine muscle activity, increased pulse rate, and widening of pulse pressure. An FHR of 134 bpm and fasting blood glucose level of 100 mg/dL are normal findings, and diarrhea is not a side effect associated with this medication. DIF: Cognitive Level: Analysis OBJ: Nursing Process Step: Assessment MSC: Patient Needs: Physiologic Integrity/Pharmacologic and Parenteral Therapies 29. A dose of dexamethasone 12 mg was administered to a patient in preterm labor at 0830 hours on March 12. The nurse knows that the next dose must be scheduled for a. 1430 hours on March 12th. b. 2030 hours on March 12th. c. 0830 hours on March 13th. d. 1430 hours on March 13th. ANS: C The current recommendation for betamethasone for threatened preterm birth is two doses of 12 mg 24 hours apart; 1430 hours on March 12th, 2030 hours on March 12th, and 1430 hours on March 13th do not fall within this recommendation. The next dose should be scheduled for 0830 hours on March 13th. DIF: Cognitive Level: Application OBJ: Nursing Process Step: Implementation MSC: Patient Needs: Physiologic Integrity/Pharmacologic and Parenteral Therapies 30. When reviewing the prenatal record of a patient at 42 weeks‘ gestation, the nurse recognizes that induction of labor is based upon which indication a. reduced amniotic fluid volume. b. cervix 2 cm at last prenatal visit. c.fundal height measured at the xyphoid process. d. 1-lb weight gain at each of the last two weekly visits. ANS: A Reduced amniotic fluid volume (oligohydramnios) often accompanies placental insufficiency and can result in fetal hypoxia. Lack of adequate amniotic fluid can result in umbilical cord compression; cervix 2 cm at last prenatal visit, fundal height measured at the xyphoid process, and 1-lb weight gain at each of the last two weekly visits are normal prenatal findings for a 42- week gestation. DIF: Cognitive Level: Analysis OBJ: Nursing Process Step: Analysis MSC: Patient Needs: Health Promotion and Maintenance 31. Which assessment finding in the postpartum patient following a uterine inversion indicates normovolemia? a. Blood pressure of 100/60 mm Hg b. Urine output >30 mL/hour c. Rebound skin turgor <5 seconds d. Pulse rate <120 beats/minute ANS: B In the presence of normal volume, urinary output will be equal to or greater than 30 mL/hour; blood pressure of 100/60 mm Hg, rebound skin turgor <5 seconds, and pulse rate <120 beats/minute may be indications of hypovolemia. DIF: Cognitive Level: Analysis OBJ: Nursing Process Step: Assessment MSC: Patient Needs: Physiologic Integrity/Physiologic Adaptation 32. Which assessment finding indicates a complication in the patient attempting a vaginal birth after cesarean (VBAC)? a. Complaint of pain between the scapulae b. Change in fetal baseline from 128 to 132 bpm c. Contractions every 3 minutes lasting 70 seconds d. Pain level of 6 on scale of 0 to 10 during acme of contraction ANS: A A patient attempting a VBAC is at greater risk for uterine rupture. As blood leaks into the abdomen, pain occurs between the scapulae or in the chest because of irritation from blood below the diaphragm; a change in the fetal baseline from 128 to 132 bpm, contractions every 3 minutes lasting 70 seconds, and a pain level of 6 on a scale of 0 to 10 during the acme of contraction would be normal findings during labor. DIF: Cognitive Level: Analysis OBJ: Nursing Process Step: Assessment MSC: Patient Needs: Safe and Effective Care Environment/Management of Care 33. The labor nurse is providing care to a multigravida with moderate to strong contractions every 2 to 3 minutes, duration 45 to 60 seconds. On admission, her cervical assessment was 5 cm, 80%, and 2. An epidural was administered shortly thereafter. Two hours after admission, her contraction pattern remains the same and her cervical assessment is 5 cm, 90%, and 2. What is the nurse‘s next action? a.Palpate the patient‘s bladder for fullness. b. Contact the health care provider for a prescription to augment the labor. c. Obtain an order for an internal pressure catheter. d. Reassure the patient that she is making adequate progress. ANS: A The fetal presenting part is expected to descend at a minimal rate of 1 cm/hour in the nullipara and 2 cm/hour in the parous woman. Despite an active labor pattern, cervical dilation and descent have not occurred for 2 hours. The nurse must consider the possibility of an obstruction. During labor, a full bladder is a common soft tissue obstruction. Bladder distention reduces available space in the pelvis and intensifies maternal discomfort. The woman should be assessed for bladder distention regularly and encouraged to void every 1 to 2 hours. Catheterization may be needed if she cannot urinate or if epidural analgesia depresses her urge to void. Even with a catheter, the nurse must assess for flow of urine and a distended bladder. DIF: Cognitive Level: Synthesis OBJ: Nursing Process Step: Analysis MSC: Patient Needs: Health Promotion and Maintenance 34. Which patient is most at risk for a uterine rupture? a. A gravida 4 who had a classic cesarean incision b. A gravida 5 who had two vaginal births and one cesarean birth c. A gravida 3 who has had two low-segment transverse cesarean births d. A gravida 2 who had a low-segment vertical incision for birth of a 10-lb infant ANS: A The classic cesarean incision is made into the upper uterine segment. This part of the uterus contracts forcefully during labor, and an incision in this area may rupture in subsequent pregnancies. The patient who had two vaginal deliveries and one cesarean is not a high-risk candidate. Low-segment transverse cesarean scars do not predispose her to uterine rupture. Low-segment incisions do not raise the risk of uterine ruptures. DIF: Cognitive Level: Understanding OBJ: Nursing Process Step: Assessment MSC: Patient Needs: Physiologic Integrity 35. A pregnant woman develops hypertension. The nurse monitors the patient‘s blood pressure closely at subsequent visits because the nurse is aware that hypertension is associated with which complication? a. Abruptio placentae b. Cardiac abnormalities in the neonate c. Neonatal jaundice d. Reduced placental blood flow ANS: D Hypertension associated with pregnancy is associated with reduced placental blood flow. Abruptio placentae, cardiac abnormalities in the neonate, and neonatal jaundice are not directly related to maternal hypertension. DIF: Cognitive Level: Understanding OBJ: Nursing Process Step: AssessmentMSC: Patient Needs: Physiologic Integrity/Reduction of Risk Potential 36. After birth, the nurse monitors the mother for postpartum hemorrhage secondary to uterine atony. Which clinical finding would increase the nurse‘s concern regarding this risk? a. Hypovolemia b. Iron-deficiency anemia c. Prolonged use of oxytocin d. Uteroplacental insufficiency ANS: C Postpartum uterine atony is more likely if she has received oxytocin for a long time because the uterine muscle becomes fatigued and does not contract effectively to compress vessels at the placental site. DIF: Cognitive Level: Understanding OBJ: Nursing Process Step: Assessment MSC: Patient Needs: Physiologic Integrity/Reduction of Risk Potential MULTIPLE RESPONSE 1. Emergency measures used in the treatment of a prolapsed cord include which of the following? (Select all that apply.) a. Administration of oxygen via face mask at 8 to 10 L/minute b. Maternal change of position to knee-chest c. Administration of tocolytic agent d. Administration of oxytocin (Pitocin) e. Vaginal elevation f. Insertion of cord back into vaginal area ANS: A, B, C, E Prolapsed cord is a medical emergency. Oxygen should be administered to the mother to increase perfusion from mother to fetus. The maternal position change to knee-chest or Trendelenburg to offset pressure on the presenting cord should be done. A tocolytic drug such as terbutaline inhibits contractions, increasing placental blood flow and reducing intermittent pressure of the fetus against the pelvis and cord. Vaginal elevation should be done to offset pressure on the presenting cord. Pitocin and manipulation of the cord by reinsertion are contraindicated. DIF: Cognitive Level: Analysis OBJ: Nursing Process Step: Implementation MSC: Patient Needs: Pathophysiologic Integrity/Medical Emergency 2. Which presentation is most likely to occur with a hypertonic labor pattern? (Select all that apply.) a. Increased risk for placenta previa b. Painful uterine contractions c. Increased resting tone d.Uterine vasodilation e. Increased uterine pressure f. Effective uterine contraction ANS: B, C, E Hypertonic labor patterns indicate increased uterine pressure and resting tone. Uterine ischemia occurs, leading to vasoconstriction and constant cramplike abdominal pain. Thus there is an increased risk for placental abruption as compared with placenta previa, which is based upon malpresentation of the placental attachment. The contractions are painful but not effective for progression of labor. DIF: Cognitive Level: Application OBJ: Nursing Process Step: Assessment MSC: Patient Needs: Pathophysiologic Integrity Chapter 17: Postpartum Adaptations and Nursing Care Foundations of Maternal-Newborn & Women‘s Health Nursing, 7th Edition MULTIPLE CHOICE 1. Which patient would be most likely to have severe afterbirth pains and request a narcotic analgesic? a. Gravida 5, para 5 b. Primipara who delivered a 7-lb boy c. Patient who is bottle feeding her first child d. Patient who is breastfeeding her second child ANS: A The discomfort of afterpains is more acute for multiparas because repeated stretching of muscle fibers leads to loss of uterine muscle tone. The uterus of a primipara tends to remain contracted. Afterpains are particularly severe during breastfeeding, not bottle feeding. The non-nursing mother may have engorgement problems that will cause her discomfort. The patient who is nursing her second child will have more afterpains than her first pregnancy; however, they will not be as severe as the grand multiparous patient. DIF: Cognitive Level: Understanding OBJ: Nursing Process Step: Assessment MSC: Patient Needs: Physiologic Integrity 2. Which maternal event is abnormal in the early postpartal period? a. Diuresis and diaphoresis b. Flatulence and constipation c. Extreme hunger and thirst d. Lochial color changes from rubra to alba ANS: D For the first 3 days after childbirth, lochia is termed rubra. Lochia serosa follows, and then at about 11 days, the discharge becomes clear, colorless, or white. The body rids itself of increased plasma volume. Urine output of 3000 mL/day is common for the first few days after birth and is facilitated by hormonal changes in the mother. Bowel tone remains sluggish for days. Many women anticipate pain during defecation and are unwilling to exert pressure on theperineum. The new mother is hungry because of energy used in labor and thirsty because of fluid restrictions during labor. DIF: Cognitive Level: Analysis OBJ: Nursing Process Step: Assessment MSC: Patient Needs: Health Promotion and Maintenance 3. Which fundal assessment finding at 12 hours after birth requires further assessment? a. The fundus is palpable at the level of the umbilicus. b. The fundus is palpable two fingerbreadths above the umbilicus. c. The fundus is palpable one fingerbreadth below the umbilicus. d. The fundus is palpable two fingerbreadths below the umbilicus. ANS: B The fundus rises to the umbilicus after birth and remains there for about 24 hours. A fundus that is above the umbilicus may indicate uterine atony or urinary retention. The fundus palpable at the umbilicus is an appropriate assessment finding for 12 hours postpartum. The fundus palpable one fingerbreadth below the umbilicus is an appropriate assessment finding for 12 hours postpartum. The fundus palpable two fingerbreadths below the umbilicus is an unusual finding for 12 hours postpartum; however, it is still appropriate. DIF: Cognitive Level: Application OBJ: Nursing Process Step: Assessment MSC: Patient Needs: Physiologic Integrity 4. If the patient‘s white blood cell (WBC) count is 25,000/mm3 on her second postpartum day, which action should the nurse take? a. Document the finding. b. Inform the health care provider. c. Begin antibiotic therapy immediately. d. Have the laboratory draw blood for reanalysis. ANS: A An increase in WBC count to 25,000/mm3 during the postpartum period is considered normal and not a sign of infection. The nurse should document the finding. There is no reason to alert the health care provider. Antibiotics are not needed because the elevated WBCs are caused by the stress of labor and not an infectious process. There is no need for reassessment as it is expected for the WBCs to be elevated. DIF: Cognitive Level: Application OBJ: Nursing Process Step: Implementation MSC: Patient Needs: Health Promotion and Maintenance 5. Postpartal overdistention of the bladder and urinary retention can lead to which complication? a. Fever and increased blood pressure b. Postpartum hemorrhage and eclampsia c. Urinary tract infection and uterine rupture d. Postpartum hemorrhage and urinary tract infection ANS: DIncomplete emptying and overdistention of the bladder can lead to urinary tract infection. Overdistention of the bladder displaces the uterus and prevents contraction of the uterine muscle. There is no correlation between bladder distention and blood pressure or fever. There is no correlation between bladder distention and eclampsia. The risk of uterine rupture decreases after the birth. DIF: Cognitive Level: Understanding OBJ: Nursing Process Step: Assessment MSC: Patient Needs: Physiologic Integrity 6. A postpartum patient asks, ―Will these stretch marks ever go away?‖ Which is the nurse‘s best response? a. ―No, never.‖ b. ―Yes, eventually.‖ c. ―They will fade to silvery lines but won‘t disappear completely.‖ d. ―They will continue to fade and should be gone by your 6-week checkup.‖ ANS: C Stretch marks never disappear altogether, but they do gradually fade to silvery lines. Stating never is true, but more information can be added, such as the changes that will occur with the stretch marks. Stretch marks do not disappear. DIF: Cognitive Level: Application OBJ: Nursing Process Step: Implementation MSC: Patient Needs: Health Promotion and Maintenance 7. A pregnant patient asks when the dark line on her abdomen (linea nigra) will go away. The nurse knows the pigmentation will fade after birth due to a. increased estrogen. b. increased progesterone. c. decreased human placental lactogen. d. decreased melanocyte-stimulating hormone. ANS: D Melanocyte-stimulating hormone increases during pregnancy and is responsible for changes in skin pigmentation; the amount decreases after birth. Estrogen levels decrease after birth. Progesterone levels decrease after birth. Human placental lactogen production continues to aid in lactation. However, it does not affect pigmentation. DIF: Cognitive Level: Understanding OBJ: Nursing Process Step: Assessment MSC: Patient Needs: Health Promotion and Maintenance 8. Which clinical finding should the nurse suspect if the fundus is palpated on the right side of the abdomen above the expected level? a. Distended bladder b. Normal involution c. Been lying on her right side too long d. Stretched ligaments that are unable to support the uterus ANS: AThe presence of a full bladder will displace the uterus. A palpated fundus on the right side of the abdomen above the expected level is not an expected finding. Position of the patient should not alter uterine position. The problem is a full bladder displacing the uterus. DIF: Cognitive Level: Understanding OBJ: Nursing Process Step: Assessment MSC: Patient Needs: Health Promotion and Maintenance 9. Which situation would require the administration of Rho(D) immune globulin? a. Mother Rh-negative, baby Rh-positive b. Mother Rh-negative, baby Rh-negative c. Mother Rh-positive, baby Rh-positive d. Mother Rh-positive, baby Rh-negative ANS: A An Rh-negative mother delivering an Rh-positive baby may develop antibodies to fetal cells that entered her bloodstream when the placenta separated. The Rho(D) immune globulin works to destroy the fetal cells in the maternal circulation before sensitization occurs. When the blood types are alike as with mother Rh-negative, baby Rh-negative, no antibody formation would be anticipated. If the Rh-positive blood of the mother comes in contact with the Rh-negative blood of the infant, no antibodies would develop because the antigens are in the mother‘s blood, not the infant‘s. DIF: Cognitive Level: Analysis OBJ: Nursing Process Step: Evaluation MSC: Patient Needs: Physiologic Integrity 10. If the rubella vaccine is indicated for a postpartum patient, which instructions should be provided? a. No specific instructions b. Drinking plenty of fluids to prevent fever c. Recommendation to stop breastfeeding for 24 hours after the injection d. Explanation of the risks of becoming pregnant within 28 days following injection ANS: D Potential risks to the fetus can occur if pregnancy results within 3 months after rubella vaccine administration. The mother does need to understand potential side effects and that pregnancy is discouraged for 3 months. The mother should be afebrile before the vaccine. Small amounts of the vaccine do cross the breast milk, but it is believed that there is no need to discontinue breastfeeding. DIF: Cognitive Level: Application OBJ: Nursing Process Step: Implementation MSC: Patient Needs: Health Promotion and Maintenance 11. Which measure is optimal in order to prevent abdominal distention following a cesarean birth? a. Rectal suppositories b. Carbonated beverages c. Early and frequent ambulation d.Tightening and relaxing abdominal muscles ANS: C Activity can aid the movement of accumulated gas in the gastrointestinal tract. Rectal suppositories can be helpful after distention occurs; however, do not prevent it. Carbonated beverages may increase distention. Ambulation is the best prevention. Abdominal strengthening will not prevent distention. DIF: Cognitive Level: Application OBJ: Nursing Process Step: Planning MSC: Patient Needs: Physiologic Integrity 12. To assess fundal contraction 6 hours after cesarean birth, which technique should the nurse utilize? a. Assess lochial flow rather than palpating the fundus. b. Palpate forcefully through the abdominal dressing. c. Place hands on both sides of the abdomen and press downward. d. Gently palpate, applying the same technique used for vaginal deliveries. ANS: D Assessment of the fundus is the same for vaginal and cesarean deliveries. Forceful palpation should never be used. The top of the fundus, not the sides, should be palpated and massaged. Assessing lochial flow is not adequate; the fundus also needs to be checked. DIF: Cognitive Level: Application OBJ: Nursing Process Step: Implementation MSC: Patient Needs: Health Promotion and Maintenance 13. The nurse has completed a postpartum assessment on a patient who delivered 1 hour ago. Which amount of lochia consists of a moderate amount? a. Saturated peripad b. 10 to 15 cm (4- to 6-inch) stain on the peripad c. 2.5 to 10 cm (1- to 4-inch) stain on the peripad d. Less than a 1-inch stain on the peripad ANS: B Because estimating the amount of lochia is difficult, nurses frequently record flow by estimating the amount of lochia in 1 hour using the following labels: Scant—less than 2.5 cm (1-inch) stain on the peripad Light—less than a 10 cm (4 inch) stain Moderate— less than a 15 cm (6 inch) stain Heavy—saturated peripad Excessive—saturated peripad in 15 minutes Determining the time interval that the peripad is in place is also important. Lochia is less for women who have had a cesarean birth because some of the endometrial lining is removed during surgery. DIF: Cognitive Level: Analysis OBJ: Nursing Process Step: Assessment MSC: Patient Needs: Health Promotion and Maintenance 14. The postpartum nurse has completed discharge teaching for a patient being discharged after an uncomplicated vaginal birth. Which statement by the patient indicates that further teaching is necessary? a.―I may not have a bowel movement until the 2nd postpartum day.‖ b. ―If I breastfeed and supplement with formula, I won‘t need any birth control.‖ c. ―I know my normal pattern of bowel elimination won‘t return until about 8 to 10 days.‖ d. ―If I am not breastfeeding, I should use birth control when I resume sexual relations with my husband.‖ ANS: B For some women, ovulation resumes as early as 3 weeks postpartum. Therefore contraceptive measures are important considerations when sexual relations are resumed for lactating and nonlactating women. Further teaching would be needed if the patient does not feel any need for birth control with breastfeeding and supplementing with formula. The first stool usually occurs within 2 to 3 days postpartum. Normal patterns of bowel elimination generally resume by 8 to 14 days after birth. DIF: Cognitive Level: Analysis OBJ: Nursing Process Step: Evaluation MSC: Patient Needs: Health Promotion and Maintenance 15. The nurse is caring for a postpartum patient who delivered by the vaginal route 12 hours ago. Which assessment finding should the nurse report to the health care provider? a. Pulse rate of 50 b. Temperature of 38C (100.4F) c. Firm fundus, but excessive lochia d. Lightheaded when moving from a lying to standing position ANS: C Excessive lochia in the presence of a contracted uterus suggests lacerations of the birth canal. The health care provider must be notified so that lacerations can be located and repaired. Bradycardia, defined as a pulse rate of 40 to 50 beats per minute (bpm), may occur as the large amount of blood that returns to the central circulation after birth of the placenta. A temperature of up to 38C (100.4F) is common during the first 24 hours after childbirth and may be caused by dehydration or normal postpartum leukocytosis. The resulting engorgement of abdominal blood vessels contributes to a rapid fall in BP of 15 to 20 mm Hg systolic when the woman moves from a recumbent to a sitting position. This change causes mothers to feel dizzy or lightheaded or to faint when they stand. DIF: Cognitive Level: Application OBJ: Nursing Process Step: Implementation MSC: Patient Needs: Physiologic Integrity 16. To facilitate adequate urinary elimination during the postpartum period, the nurse should incorporate which intervention into the plan of care? a. Have the patient drink carbonated beverages to promote urinary excretion. b. Tell the patient that because of postpartum diuresis there is less risk to develop dehydration. c. Limit fluid intake to prevent polyuria. d. Teach the patient to perform pelvic floor exercises to combat potential stress incontinence. ANS: DEducating the patient to use pelvic floor exercises (Kegel exercises) will help strengthen pelvic floor muscles. Carbonated beverages will lead to increased gas and potential gastrointestinal discomfort. During the postpartum period, the patient is at greater risk for dehydration and thus should increase fluids. Limitation of fluids is not warranted during the postpartum period. DIF: Cognitive Level: Application OBJ: Nursing Process Step: Implementation MSC: Patient Needs: Health Promotion and Maintenance 17. When assessing the A of the acronym REEDA, the nurse should evaluate the a. skin color. b. degree of edema. c. edges of the episiotomy. d. episiotomy for discharge. ANS: C In the acronym REEDA, the A refers to approximation of the edges of the episiotomy; the other letters of the acronym refer to other components of wound assessment: R = redness, E = edema, E = ecchymosis, and D = drainage. DIF: Cognitive Level: Application OBJ: Nursing Process Step: Assessment MSC: Patient Needs: Health Promotion and Maintenance 18. Which assessment finding 24 hours after vaginal birth would indicate a need for further intervention? a. Pain level 5 on scale of 0 to 10 b. Saturated pad over a 2-hour period c. Urinary output of 500 mL in one voiding d. Uterine fundus 2 cm above the umbilicus ANS: D By the second postpartum day, the fundus descends by approximately 1 cm/day and should be 1 cm below the umbilicus; pain level of 5, saturated pad over a 2-hour time period, and urinary output of 500 mL in one voiding are normal findings in the postpartum patient. DIF: Cognitive Level: Analysis OBJ: Nursing Process Step: Analysis MSC: Patient Needs: Physiologic Integrity 19. The nurse is providing care to a patient who delivered a 3525-g infant 14 hours ago. The nurse palpates the fundus of the uterus as firm and at the umbilicus. What is the nurse‘s priority action related to this finding? a. Inform the health care provider. b. Encourage the patient to urinate. c. Massage the uterus to expel clots. d. Document the finding in the patient‘s chart. ANS: D The location of the uterine fundus helps determine whether involution is progressing normally. Immediately after birth, the uterus is about the size of a large grapefruit or softball and weighsapproximately 1000 g (2.2 lb). The fundus can be palpated midway between the symphysis pubis and umbilicus in the midline of the abdomen. Within 12 hours, the fundus rises to approximately the level of the umbilicus. This finding is expected and can be followed with documentation. No further action is needed. DIF: Cognitive Level: Application OBJ: Nursing Process Step: Assessment MSC: Patient Needs: Health Promotion and Maintenance 20. The nurse is providing care to a patient 2 hours after a cesarean birth. In the hand-off report, the preceding nurse indicated that the patient‘s lochia was scant rubra. On initial assessment, the oncoming nurse notes the patient‘s peripad is saturated with lochia rubra immediately after breastfeeding her infant. What is the nurse‘s priority action with this finding? a. Weigh the peripad. b. Replace the peripad. c. Contact the health care provider. d. Document the finding in the patient‘s chart. ANS: C The lochia of the cesarean birth mother will go through the same phases as that of the woman who had a vaginal birth; however, the amount will be reduced. The finding of a saturated pad is abnormal, even after breastfeeding, and an indication of hemorrhage. The health care provider needs to be notified immediately. Weighing the peripad will give an estimation of the blood loss; but, this assessment can result in a delay of care. Replacing the peripad and documentation of the findings are appropriate when the data are within normal limits. DIF: Cognitive Level: Analysis OBJ: Nursing Process Step: Evaluation MSC: Patient Needs: Health Promotion and Maintenance 21. The nurse includes the addition of ice sitz baths for the postpartum patient. Which assessment finding indicates the treatment has been effective? a. No swelling or edema to the perineal area b. Patient complains that the sitz bath is too cold. c. Patient reports she took two sitz baths in 12 hours. d. Edges of the perineal laceration are well approximated. ANS: A Sitz baths may be offered two to four times a day to women with episiotomies, painful hemorrhoids, or perineal edema. Sitz baths provide continuous circulation of water and cleanse and comfort the traumatized perineum. Cool water reduces pain caused by edema and may be most effective within the first 24 hours. Ice can be added to cool the water to a comfortable level as the woman sits in it. Approximation of the edges of a wound facilitates wound healing. The purpose of the cold sitz bath is to decrease the edema secondary to tissue trauma. DIF: Cognitive Level: Analysis OBJ: Nursing Process Step: Evaluation MSC: Patient Needs: Health Promotion and Maintenance 22. Which description best explains the term reciprocal attachment behavior? a. Behavior during the sensitive period when the infant is in the quiet alert stage b. Positive feedback that the infant exhibits toward parents during the attachment processc. Unidirectional behavior exhibited by the infant, initiated and enhanced by eye contact d. Behavior by the infant during the sensitive period to elicit feelings of ―falling in love‖ from the parents ANS: B In this definition, reciprocal refers to the feedback from the infant during the attachment process. The quiet alert state is a good time for bonding; however, does not define reciprocal attachment. Reciprocal attachment deals with feedback behavior and is not unidirectional. DIF: Cognitive Level: Understanding OBJ: Nursing Process Step: Assessment MSC: Patient Needs: Health Promotion and Maintenance 23. The postpartum patient who continually repeats the story of her labor, birth, and recovery experiences is performing which of the following tasks? a. Making the birth experience ―real‖ b. Accepting her response to labor and birth c. Providing others with her knowledge of events d. Taking hold of the events leading to her labor and birth ANS: A Reliving the birth experience makes the event real and helps the mother realize that the pregnancy is over and that the infant is born and is now a separate individual. She is in the taking-in phase, trying to make the birth experience seem real. This is to satisfy her needs, not the needs of others. DIF: Cognitive Level: Understanding OBJ: Nursing Process Step: Assessment MSC: Patient Needs: Psychosocial Integrity 24. During which stage of role attainment do the parents become acquainted with their baby and combine parenting activities with cues from the infant? a. Formal b. Informal c. Personal d. Anticipatory ANS: A A major task of the formal stage of role attainment is getting acquainted with the infant. The informal stage begins once the parents have learned appropriate responses to their infant‘s cues. The personal stage is attained when parents feel a sense of harmony in their role. The anticipatory stage begins during the pregnancy when the parents choose a physician and attend childbirth classes. DIF: Cognitive Level: Understanding OBJ: Nursing Process Step: Assessment MSC: Patient Needs: Psychosocial Integrity 25. The nurse observes a patient on her first postpartum day sitting in bed while her newborn lies awake in the bassinet. Which action is most appropriate for the nurse to take at this time? a. Hand the baby to the woman. b.Explain ―taking-in‖ to the woman. c. Offer to hand the baby to the woman. d. No action, because this situation is perfectly acceptable. ANS: A During the taking-in phase of maternal adaptation, in which the mother may be passive and dependent, the nurse should encourage bonding when the infant is in the quiet alert stage. This is done best by simply giving the baby to the mother. She learns best during the taking-hold phase. The woman is dependent and passive at this stage and may have difficulty making a decision. This is expected behavior during the taking-in phase; however, interventions that facilitate infant bonding can be taken. DIF: Cognitive Level: Application OBJ: Nursing Process Step: Implementation MSC: Patient Needs: Psychosocial Integrity 26. The postpartum nurse is observing a patient holding the baby she delivered less than 24 hours ago. The partner is watching his wife and asking questions about newborn care. The 4- year-old big brother is punching his mother on the back. What should the nurse do next? a. Report the incident to the social services department. b. Advise the parents that the older son needs to be reprimanded. c. No action; this is a normal family adjusting to family change. d. Report to oncoming staff that the mother is probably not a good disciplinarian. ANS: C The observed behaviors are normal variations of families adjusting to change. There is no need to report this one incident. Giving advice at this point would make the parents feel inadequate. This is normal for an adjusting family. DIF: Cognitive Level: Analysis OBJ: Nursing Process Step: Assessment MSC: Patient Needs: Psychosocial Integrity 27. During which phase of maternal adjustment will the mother relinquish the baby of her fantasies and accept the real baby? a. Letting-go b. Taking-in c. Taking-on d. Taking-hold ANS: A Accepting the real infant and relinquishing the fantasy infant occurs during the letting-go phase of maternal adjustment. In the taking-in phase, the mother is primarily focused on her own needs. There is no taking-on phase of maternal adjustment. During the taking-hold phase, the mother assumes responsibility for her own care and shifts her attention to the infant. DIF: Cognitive Level: Understanding OBJ: Nursing Process Step: Assessment MSC: Patient Needs: Psychosocial Integrity 28. A new father calls the nurse‘s station stating that his wife, who delivered last week, is happy one minute and crying the next. He states, ―She was never like this before the baby was born.‖ How should the nurse best respond?a. Reassure him that this behavior is normal. b. Advise him to get immediate psychological help for her. c. Tell him to ignore the mood swings because they will go away. d. Instruct him in the signs, symptoms, and duration of postpartum blues. ANS: A Before providing further instructions, inform family members of the fact that postpartum blues are a normal process to allay anxieties and increase receptiveness to learning. Postpartum blues are a normal process that is short-lived; no medical intervention is needed. Telling him to ignore the moods blocks communication and may belittle the husband‘s concerns. Patient teaching is important; however, his anxieties need to be allayed before he will be receptive to teaching. DIF: Cognitive Level: Application OBJ: Nursing Process Step: Implementation MSC: Patient Needs: Psychosocial Integrity 29. To promote bonding and attachment immediately after birth, which action should the nurse take? a. Assist the mother in feeding her baby. b. Allow the mother quiet time with her infant. c. Teach the mother about the concepts of bonding and attachment. d. Assist the mother in assuming an en face position with her newborn. ANS: D Assisting the mother in assuming an en face position with her newborn will support the bonding process. After birth is a good time to initiate breastfeeding, but first the mother needs time to explore the new infant and begin the bonding process. The mother should be given as much privacy as possible; however, nursing assessments must still be continued during this critical time. The mother has just delivered and is more focused on the infant; she will not be receptive to teaching at this time. DIF: Cognitive Level: Application OBJ: Nursing Process Step: Implementation MSC: Patient Needs: Health Promotion and Maintenance 30. Which patient is more likely to have less stress adjusting to her role as a mother? a. A 26-year-old woman who is returning to work in 10 weeks b. A 35-year-old anxious mother who has had no contact with babies or children c. A 16-year-old teenager who lives with her parents and has a strained relationship with her mother d. A 25-year-old woman who knew at 16 weeks of gestation that she was pregnant with twins, who were delivered by cesarean birth ANS: A The woman who has the least amount of stress in her life will adjust more quickly to her role as a mother. The anxious mother with no real experience with babies may have a difficult time adjusting to motherhood. The teenager has a significant amount of stress in her life, which couldmake adjusting to her role as a mother more difficult. The 25-year-old mother has the added stress of twins, which may make motherhood adjustment more difficult. DIF: Cognitive Level: Understanding OBJ: Nursing Process Step: Assessment MSC: Patient Needs: Psychosocial Integrity 31. Which anticipatory guidance action by the nurse makes role transition to parenthood easier? a. Helps the new parents identify resources. b. Recommends employing babysitters frequently. c. Tells the parents about the realities of parenthood. d. Offers a home phone number and tells parents to call if they have a question. ANS: A Available resources within the community can assist the parents in role transition. Some parents may not be able to afford babysitters. Also, this removes them from the parenthood role. Each adult sees parenthood in a different light. They cannot be compared. Searching out resources for the parents is an important task; however, the nurse should not give her personal number to patients. DIF: Cognitive Level: Application OBJ: Nursing Process Step: Implementation MSC: Patient Needs: Psychosocial Integrity 32. Which action should the nurse take in order to provide support and encouragement to the new postpartum patient? a. Recount how she solved her own problems. b. Correct the new mother at every opportunity. c. Praise the mother‘s early attempts at infant care. d. Explain to the new mother that everything will be fine. ANS: C Positive reinforcement of the mother‘s attempt to provide care to the newborn will promote a healthy self-concept. The mother needs to learn how to solve problems on her own. Each person may use different techniques that work for that person. Correcting her actions would be discouraging to a new mother. She needs encouragement. Saying everything will be fine is blocking communication and further teaching. DIF: Cognitive Level: Application OBJ: Nursing Process Step: Implementation MSC: Patient Needs: Health Promotion and Maintenance 33. Which should the nurse do to provide support to a patient who must return to full-time employment 6 weeks after a vaginal birth? a. Discuss child care arrangements with her. b. Allow her to solve the problem on her own. c. Reassure her that she‘ll get used to leaving her baby. d. Allow her to express her positive and negative feelings freely. ANS: DAllowing the patient to express feelings will provide positive support in her process of maternal adjustment. Discussing child care arrangements is an important step in anticipatory guidance, although this is not the best way to offer support. The new mother should be instrumental in solving the problem; however, allowing her time to express her feelings and talk the problem over will assist her in making this decision. Reassuring her that she will get used to leaving the baby blocks communication and belittles the patient‘s feelings. DIF: Cognitive Level: Application OBJ: Nursing Process Step: Implementation MSC: Patient Needs: Psychosocial Integrity 34. The postpartum nurse is reviewing dietary practices for an Asian patient. Which of the following should the nurse expect to observe as a dietary practice for this culture? a. Special foods brought from home. b. Preference for fresh fruits. c. Preference for ―cold‖ foods. d. Request for ice water instead of hot water. ANS: A Specific foods brought from home are a welcome sign of caring in many cultures. Some Asians believe that after childbirth the woman should eat only ―hot‖ foods such as chicken, meat, and fish. Fresh fruit would be considered a ―cold‖ food. Although ice water is commonly given to hospital patients, it is not acceptable to many Asians. For example, Southeast Asian women may refuse cold or ice water and prefer hot water or other warm beverages to keep warm. DIF: Cognitive Level: Application OBJ: Nursing Process Step: Assessment MSC: Patient Needs: Health Promotion and Maintenance 35. An example of binding in during the postpartum period is a a. new mother telling her friends all about her labor and birth experience. b. father looking at his newborn and stating that he ―looks like I did when I was a baby.‖ c. mother reporting increasing anxiety during the postpartum period because she feels like she is without support. d. mother wanting some time alone so that she can catch up on needed sleep. ANS: B A new mother telling her friends all about her labor and birth experience is an example of binding in or claiming. A new mother telling her friends all about her labor and birth experience is an example of the taking-in phase of maternal adaptation. A mother who reports increasing anxiety during the postpartum period because she feels like she is all alone may be problematic and indicates that the patient is experiencing significant stressors during the postpartum period. A mother wanting some time alone so that she can catch up on needed sleep is a normal reaction to the demands of the newborn and reflects that the patient may need additional support during this time. DIF: Cognitive Level: Analysis OBJ: Nursing Process Step: Evaluation MSC: Patient Needs: Psychosocial Integrity 36. Which of the following behaviors would be applicable to a nursing diagnosis of Risk for Impaired Parenting? a. En face behavior is observed between father and infant.b. Mother relates that she feels exhilarated postbirth. c. Mother states that she feels excessive fatigue as a result of the childbirth experience. d. Father displays finger tipping behavior toward infant. ANS: C Fatigue can contribute to altered parenting, because it may affect the level of interaction between parent and child. En face behavior acknowledges maternal-paternal attachment. A feeling of exhilaration is normal following a changing life cycle event such as childbirth. Finger tipping behavior conveys a sense of identification or claiming behavior. DIF: Cognitive Level: Application OBJ: Nursing Process Step: Diagnosis MSC: Patient Needs: Health Promotion and Maintenance 37. A family is concerned about how their 2-year-old son is going to react to the new baby. Which intervention would help facilitate sibling attachment? a. Have the mother and father spend individual time with their son to allay potential anxiety over the new baby coming in and displacing his position in the family as the only child. b. Make sure that their son is supervised at all times when the baby is brought home from the hospital and is in his presence. c. Include the son in helping to take care of the baby and reinforce the label of ―big brother‖ as a special role. d. Observe the son‘s reaction to the baby and let him decide when he wants to be introduced to his new sibling. ANS: C Providing the older son with a special role designation and involving him in the care of the baby will facilitate sibling attachment. Spending individual time with the older child is recommended but will not facilitate sibling attachment. Although the older child should be supervised because of his age in terms of infant safety, this level of overprotection may inhibit sibling attachment. Observation of his behavior may be warranted; however, the age of the child (2 years) does not warrant this type of control. DIF: Cognitive Level: Analysis OBJ: Nursing Process Step: Implementation MSC: Patient Needs: Psychosocial Integrity 38. The nurse is developing a plan of care for the patient‘s fourth stage of labor. One nursing intervention is to promote bonding. Specifically, which nursing action will facilitate the bonding process? a. Encourage the patient to call the baby by his or her first name. b. Stimulate the grasp reflex by placing the patient‘s finger in the infant‘s palm. c. Ask the patient if she wants her baby placed on her chest immediately after birth. d. Assess for familial characteristics and remark on the resemblance to the patient or the father. ANS: C Bonding refers to the rapid initial attraction felt by parents for their infant. It is unidirectional, from parent to child, and is enhanced when parents and infants are permitted to touch and interact during the first 30 to 60 minutes after birth. During this time, the infant is in a quiet, alert stateand seems to gaze directly at the parents. Infants are often placed skin to skin on the mother‘s chest or abdomen for bonding time immediately after birth. Nurses frequently delay procedures such as measurements and medication administration that would interfere with this time, so that parents can focus on their newborn baby. Attachment follows a progressive or developmental course that changes over time. It is rarely instantaneous. Unlike bonding, attachment is reciprocal—it occurs in both directions between parent and infant. DIF: Cognitive Level: Application OBJ: Nursing Process Step: Implementation MSC: Patient Needs: Health Promotion and Maintenance 39. A postpartum patient calls the clinic and reports to the nurse feelings of fatigue, tearfulness, and anxiety. What is the nurse‘s most appropriate response at this time? a. ―When did these symptoms begin?‖ b. ―Sounds like normal postpartum depression.‖ c. ―Are you having trouble getting enough sleep?‖ d. ―Are you able to get out of bed and provide care for your baby?‖ ANS: D Postpartum blues must be distinguished from postpartum depression and postpartum psychosis, which are disabling conditions and require therapeutic management for full recovery. Nurses need to assess the depression to ascertain if she is unable to cope with daily life. Postpartum blues are self-limiting and frequently occur by the fifth postpartum day and resolve in 2 weeks. The response ―Sounds like postpartum depression‖ does not offer the patient any help or encouragement through this challenging time. Asking if she is getting enough sleep does not add to the assessments already identified in the stem. Enough information exists to determine that she has the signs and symptoms of postpartum blues. The nurse must differentiate between postpartum blues and depression. DIF: Cognitive Level: Analysis OBJ: Nursing Process Step: Assessment MSC: Patient Needs: Health Promotion and Maintenance MULTIPLE RESPONSE 1. Which vaccinations are indicated for the postpartum patient if she does not have immunity? (Select all that apply.) a. Pertussis b. Rubella c. Diphtheria, tetanus (Tdap) d. RhoGAM e. Varicella ANS: A, B, C, E If a patient who has delivered does not have evidence of immunity, CDC recommendations advise that pertussis, rubella, Tdap, and varicella should be administered. RhoGAM is required if there is evidence of sensitization in response to Rh factor identification based on maternal and fetal blood results. DIF: Cognitive Level: Application OBJ: Nursing Process Step: Planning MSC: Patient Needs: Health Promotion and Maintenance2. The nurse is planning comfort measures to implement for a patient after a vaginal birth. Which measures should the nurse plan to include in the patient‘s care plan? (Select all that apply.) a. Sitz baths four times a day b. Use of only warm water with the sitz baths c. Topical anesthetic spray after perineal care d. Ice pack to the perineum for the first 24 hours e. Relax the perineal and buttock areas when sitting ANS: A, C, D Sitz baths provide continuous circulation of water, cleansing and comforting the traumatized perineum. Ice causes vasoconstriction and is most effective if applied soon after the birth to prevent edema and to numb the area. Anesthetic sprays decrease surface discomfort and allow more comfortable ambulation. Cool water in the sitz bath reduces pain caused by edema and may be most effective within the first 24 hours. The mother should be advised to squeeze her buttocks together, not relax them, before sitting, and to lower her weight slowly onto her buttocks. DIF: Cognitive Level: Application OBJ: Nursing Process Step: Implementation MSC: Patient Needs: Physiologic Integrity 3. The nurse is teaching a non–breastfeeding patient measure to suppress lactation. Which information should the nurse include in the teaching session? (Select all that apply.) a. Avoid massaging the breasts. b. Allow warm shower water to run over the breasts. c. If the breasts become engorged, pumping is recommended. d. Ice packs or cabbage leaves can be applied to the breasts to relieve discomfort. e. Wear a sports bra 24 hours a day until the breasts become soft. ANS: A, D, E The patient should be advised to avoid massaging the breasts because this will stimulate milk production. Instruct the patient to wear a sports bra or other well-fitting bra 24 hours a day until the breasts become soft. Manage breast discomfort by application of ice, or cold cabbage leaves, which reduce vasocongestion. Advise the patient to refrain from allowing warm water to fall directly on the breasts during showers and pumping because these actions will stimulate milk production. DIF: Cognitive Level: Application OBJ: Nursing Process Step: Implementation MSC: Patient Needs: Physiologic Integrity 4. The nurse is conducting discharge teaching for a patient going home after a cesarean birth. Which signs and symptoms should the patient be taught to report? (Select all that apply.) a. Mild incisional pain b. Feeling of pelvic fullness c.Lochia changing from red to pink in color d. Frequency, urgency, or burning on urination e. Redness or edema of the abdominal incision ANS: B, D, E The signs and symptoms to watch for after a cesarean birth are feelings of pelvic fullness, frequency, urgency or burning on urination, and redness or edema of the abdominal incision. Mild incisional pain is expected and the lochia should change from a bright red (rubra) to a pinkish color (serosa). DIF: Cognitive Level: Application OBJ: Nursing Process Step: Implementation MSC: Patient Needs: Physiologic Integrity 5. Which of the following are nursing measures that can promote parent-infant bonding and attachment? (Select all that apply.) a. Provide comfort and ample time for rest. b. Keep the baby wrapped to avoid cold stress. c. Position the infant face to face with the mother. d. Point out the characteristics of the infant in a positive way. e. Limit the amount of modeling so the mother doesn‘t feel insecure. ANS: A, C, D Provide comfort and ample time for rest, because the mother must replenish her energy and be relatively free of discomfort before she can progress to initiating care of the infant. Position the infant in an en face position and discuss the infant‘s ability to see the parent‘s face. Face to face and eye to eye contact is a first step in establishing mutual interaction between the infant and parent. Point out the characteristics of the infant in a positive way: ―She has such pretty little hands and beautiful eyes.‖ The baby should be kept warm, but parents should be assisted to unwrap the baby (keeping or rewrapping the body part not being inspected) to inspect the toes, fingers, and body. The nurse should model behaviors by holding the infant close, making eye contact with the infant, and speaking in high-pitched, soothing tones. DIF: Cognitive Level: Application OBJ: Nursing Process Step: Implementation MSC: Patient Needs: Health Promotion and Maintenance COMPLETION 1. The postpartum nurse is administering ibuprofen (Advil) to a patient with episiotomy discomfort. The prescribed order is 400 mg of Advil by mouth every 6 to 8 hours PRN for discomfort. The Advil sent by the pharmacy is 200 mg/tablet. How many tablet(s) should the nurse administer to the patient? Record your answer as a whole number. tab(s) ANS: 2 Use the medication calculation formula to calculate the correct dose. Formula: Desired/available volume = mg/dose 400 mg/200 mg 1 tab = 2 tabs DIF: Cognitive Level: Application OBJ: Nursing Process Step: Implementation MSC: Patient Needs: Safe and Effective Care EnvironmentChapter 18: Postpartum Maternal Complications Foundations of Maternal-Newborn & Women‘s Health Nursing, 7th Edition MULTIPLE CHOICE 1. Which statement by a postpartum patient indicates that further teaching regarding thrombus formation is unnecessary? a. ―I‘ll keep my legs elevated with pillows.‖ b. ―I‘ll sit in my rocking chair most of the time.‖ c. ―I‘ll stay in bed for the first 3 days after my baby is born.‖ d. ―I‘ll put my support stockings on every morning before rising.‖ ANS: D Venous congestion begins as soon as the patient stands up. The stockings should be applied before she rises from the bed in the morning. The patient should avoid knee pillows because they increase pressure on the popliteal space. Sitting in a chair with legs in a dependent position causes pooling of blood in the lower extremities. As soon as possible, the patient should ambulate frequently. DIF: Cognitive Level: Application OBJ: Nursing Process Step: Evaluation MSC: Patient Needs: Health Promotion and Maintenance 2. The nurse understands that late postpartum hemorrhage may be prevented by a. manually removing the placenta. b. inspecting the placenta after birth. c. administering broad-spectrum antibiotics. d. pulling on the umbilical cord to hasten the birth of the placenta. ANS: B If a portion of the placenta is missing, the clinician can explore the uterus, locate the missing fragments, and remove the potential cause of late postpartum hemorrhage. Manual removal of the placenta increases the risk of postpartum hemorrhage. Broad-spectrum antibiotics will be given if postpartum infection is suspected. The placenta is usually delivered 5 to 30 minutes after birth of the baby without pulling on the cord. That can cause uterine inversion. DIF: Cognitive Level: Application OBJ: Nursing Process Step: Implementation MSC: Patient Needs: Physiologic Integrity 3. A multiparous patient is admitted to the postpartum unit after a rapid labor and birth of a 4000-g infant. Her fundus is boggy, lochia is heavy, and vital signs are unchanged. The nurse has the patient void and massages her fundus; however, the fundus remains difficult to find and the rubra lochia remains heavy. Which action should the nurse take next? a. Recheck vital signs. b. Insert a Foley catheter. c. Notify the health care provider. d. Continue to massage the fundus. ANS: CTreatment of excessive bleeding requires the collaboration of the health care provider and the nurses. Do not leave the patient alone. The nurse should call the clinician while a second nurse rechecks the vital signs. The patient has voided successfully, therefore a Foley catheter is not needed at this time. The uterine muscle can be overstimulated by massage, leading to uterine atony and rebound hemorrhage. DIF: Cognitive Level: Application OBJ: Nursing Process Step: Implementation MSC: Patient Needs: Physiologic Integrity 4. Early postpartum hemorrhage is defined as a blood loss greater than a. 500 mL within 24 hours after a vaginal birth. b. 750 mL within 24 hours after a vaginal birth. c. 1000 mL within 48 hours after a cesarean birth. d. 1500 mL within 48 hours after a cesarean birth. ANS: B The average amount of bleeding after a vaginal birth is 500 mL. Early postpartum hemorrhage occurs in the first 24 hours, not 48 hours. Blood loss after a cesarean birth averages 1000 mL. Late postpartum hemorrhage is 48 hours and later. DIF: Cognitive Level: Understanding OBJ: Nursing Process Step: Assessment MSC: Patient Needs: Physiologic Integrity 5. A steady trickle of bright red blood from the vagina in the presence of a firm fundus suggests a. uterine atony. b. perineal hematoma. c. infection of the uterus. d. lacerations of the genital tract. ANS: D Undetected lacerations will bleed slowly and continuously. Bleeding from lacerations will not be affected by uterine contraction. The fundus would be boggy with a clinical finding of uterine atony. A hematoma would occur internally with swelling and discoloration. With an infection of the uterus, there would be an odor to the lochia and systemic symptoms such as fever and malaise. DIF: Cognitive Level: Understanding OBJ: Nursing Process Step: Assessment MSC: Patient Needs: Physiologic Integrity 6. A postpartum patient would be at increased risk for postpartum hemorrhage if she delivered a(n) a. 5-lb, 2-oz infant with outlet forceps. b. 6.5-lb infant after a 2-hour labor. c. 7-lb infant after an 8-hour labor. d. 8-lb infant after a 12-hour labor. ANS: BA rapid labor and birth may cause exhaustion of the uterine muscle and prevent contraction. Delivering a 5-lb, 2-oz infant with outlet forceps would put this patient at risk for lacerations due to the use of forceps. A 7-lb infant after an 8-hour labor is a normal labor progression. Less than 3 hours is considered a rapid labor and can produce uterine muscle exhaustion. An 8-lb infant after a 12-hour labor is a normal labor progression. Less than 3 hours is a rapid birth and may cause the uterine muscles failure to contract. DIF: Cognitive Level: Understanding OBJ: Nursing Process Step: Assessment MSC: Patient Needs: Physiologic Integrity 7. The nurse should expect medical intervention for subinvolution to include a. oral fluids to 3000 mL/day. b. intravenous fluid and blood replacement. c. oxytocin intravenous infusion for 8 hours. d. oral methylergonovine maleate (Methergine) for 48 hours. ANS: D Methergine provides sustained contraction of the uterus. There is no correlation between dehydration and subinvolution. There is no indication that excessive blood loss has occurred. Oxytocin provides intermittent contractions. DIF: Cognitive Level: Understanding OBJ: Nursing Process Step: Planning MSC: Patient Needs: Physiologic Integrity 8. If nonsurgical treatment for subinvolution is ineffective, which surgical procedure is appropriate to correct the cause of this condition? a. Hysterectomy b. Laparoscopy c. Laparotomy d. Dilation and curettage (D&C) ANS: D D&C allows examination of the uterine contents and removal of any retained placenta or membranes. A hysterectomy is the removal of the uterus and is not indicated in this situation. A laparoscopy is the insertion of an endoscope through the abdominal wall to examine the peritoneal cavity and would also not be necessary at this juncture. A laparotomy is a surgical incision into the peritoneal cavity to explore the peritoneal cavity. This patient requires a D&C rather than a laparotomy. DIF: Cognitive Level: Understanding OBJ: Nursing Process Step: Assessment MSC: Patient Needs: Physiologic Integrity 9. A positive sign of thrombophlebitis includes a. visible varicose veins. b. positive Homans sign. c. pedal edema in the affected leg. d. local tenderness, heat, and swelling.ANS: D Tenderness, heat, and swelling are classic signs of thrombophlebitis that appear at the site of the inflammation. Varicose veins may predispose the patient to thrombophlebitis; however, are not an indication of thrombophlebitis. A positive Homans sign is indicative of deep vein thrombosis (DVT). DIF: Cognitive Level: Understanding OBJ: Nursing Process Step: Assessment MSC: Patient Needs: Physiologic Integrity 10. Which nursing measure would be most appropriate to prevent thrombophlebitis in the recovery period following a cesarean birth? a. Limit the patient‘s oral intake of fluids for the first 24 hours. b. Assist the patient in performing leg exercises every 2 hours. c. Ambulate the patient as soon as her vital signs are stable. d. Roll a bath blanket and place it firmly behind the patient‘s knees. ANS: B Leg exercises promote venous blood flow and prevent venous stasis while the patient is still on bed rest. Limiting oral intake will produce hemoconcentration, which may lead to thrombophlebitis. The patient may not have full return of leg movements, and ambulating at this time is contraindicated. The blanket behind the knees will cause pressure and decrease venous blood flow. DIF: Cognitive Level: Application OBJ: Nursing Process Step: Implementation MSC: Patient Needs: Health Promotion and Maintenance 11. Which temperature indicates the presence of postpartum infection? a. 37.5°C (99.6°F) in the first 48 hours b. 37.7°C (100°F) for 2 days postpartum c. 38°C (100.4°F) in the first 24 hours d. 38.2°C (100.8°F) on the second and third postpartum days ANS: D A temperature elevation of greater than 38°C (100.4°F) on two postpartum days, not including the first 24 hours, signifies infection. 37.5°C (99.6°F) in the first 48 hours is an expected finding due to dehydration. To be classified as an infection, the temperature needs to be greater than 38°C (100.4°F). It is anticipated that women have an elevated temperature the first 24 hours after delivery. DIF: Cognitive Level: Understanding OBJ: Nursing Process Step: Assessment MSC: Patient Needs: Health Promotion and Maintenance 12. A white blood cell (WBC) count of 35,000 cells/mm3 on the morning of the first postpartum day indicates a. possible infection. b. normal WBC limit. c. serious infection. d.suspicion of a sexually transmitted disease. ANS: A A WBC count in the upper ranges of normal (20,000 to 30,000 cells/mm3) may indicate an infection. An elevated WBC count is anticipated but becomes a concern as it hits the upper range. An elevated WBC count may be an indication of different types of infection. DIF: Cognitive Level: Understanding OBJ: Nursing Process Step: Assessment MSC: Patient Needs: Physiologic Integrity 13. The patient who is being treated for endometritis is placed in the Fowler position because this position a. promotes comfort and rest. b. facilitates drainage of lochia. c. prevents spread of infection to the urinary tract. d. decreases tension on the reproductive organs. ANS: B Lochia and infectious material are eliminated by gravity drainage. The Fowler position may not be the position of comfort, but it does allow for drainage. Good hygiene practice aids in preventing the spread of infection to the urinary tract. This position aids in the drainage of lochia and infectious material. DIF: Cognitive Level: Comprehension OBJ: Nursing Process Step: Implementation MSC: Patient Needs: Physiologic Integrity 14. Nursing measures that help prevent postpartum urinary tract infection include a. forcing fluids to at least 3000 mL/day. b. promoting bed rest for 12 hours after birth. c. encouraging the intake of grapefruit juice and carbonated beverages. d. discouraging voiding until the sensation of a full bladder is present. ANS: A Adequate fluid intake prevents urinary stasis, dilutes urine, and flushes out waste products. The patient should be encouraged to ambulate early. Drinks that acidify urine also inhibit bacterial growth. These include apricot, plum, prune, and cranberry juice. Grapefruit juice and soda should be avoided as they increase urine alkalinity. With pain medications, trauma to the area, and anesthesia, the sensation of a full bladder may be decreased. The patient needs to be encouraged to void frequently. DIF: Cognitive Level: Application OBJ: Nursing Process Step: Implementation MSC: Patient Needs: Physiologic Integrity 15. Which measure may prevent mastitis in a breastfeeding patient? a. Wearing a tight-fitting bra. b. Applying ice packs prior to feeding. c. Initiating early and frequent feedings. d. Nursing the infant for 5 minutes on each breast.ANS: C Early and frequent feedings prevent stasis of milk, which contributes to engorgement and mastitis. Five minutes does not empty the breast adequately. This will produce stasis of the milk. A firm-fitting bra will support the breast, but not prevent mastitis. The breast should not be bound. Warm packs before feeding will increase the flow of milk. DIF: Cognitive Level: Application OBJ: Nursing Process Step: Implementation MSC: Patient Needs: Physiologic Integrity 16. A patient with mastitis is concerned about breastfeeding while she has an active infection. Which is an appropriate response by the nurse? a. Organisms will be inactivated by gastric acid. b. Organisms that cause mastitis are not passed through the milk. c. The infant is not susceptible to the organisms that cause mastitis. d. The infant is protected from infection by immunoglobulins in the breast milk. ANS: B The organisms are localized in the breast tissue and are not excreted in the breast milk. The organism will not get into the infant‘s gastrointestinal system. Because of an immature immune system, infants are susceptible to many infections; however, this infection is in the breast tissue and is not excreted in the breast milk. The patient is just producing the immunoglobulin from this infection, so it is not available for the infant. DIF: Cognitive Level: Application OBJ: Nursing Process Step: Implementation MSC: Patient Needs: Physiologic Integrity 17. The nurse suspecting a uterine infection in a postpartum patient should assess the a. episiotomy site. b. odor of the lochia. c. abdomen for distention. d. pulse and blood pressure. ANS: B An abnormal odor of the lochia indicates infection in the uterus. The infection may move to the episiotomy site if proper hygiene is not followed. The abdomen becomes distended usually because of a decrease of peristalsis, such as after cesarean section. The pulse may be altered with an infection, but the odor of the lochia will be an earlier sign and will be more specific. DIF: Cognitive Level: Application OBJ: Nursing Process Step: Assessment MSC: Patient Needs: Physiologic Integrity 18. Following a difficult vaginal birth of a singleton pregnancy, the patient starts bleeding heavily. Clots are expressed and a Foley catheter is inserted to empty the bladder because the uterine fundus is soft and displaced laterally from midline. Vital signs are 37.6°C (99.8°F), pulse 90 beats/minute, respirations 20 breaths per minute, and BP 130/90 mm Hg. Which pharmacologic intervention is indicated? a. Oxytocin (Pitocin) to be administered in a piggyback solution b. Administration of methylergonovine (Methergine) c.Administration of prostaglandin analog d. Increase in parenteral fluids ANS: C Prostaglandin analogs can be administered intramuscularly to stop uterine bleeding. Although Pitocin may be indicated in an attempt to stop uterine bleeding, it is not administered in a piggyback solution. Methergine is contraindicated in the presence of hypertension. Increasing fluids will not stop uterine bleeding. DIF: Cognitive Level: Analysis OBJ: Nursing Process Step: Implementation MSC: Patient Needs: Physiologic Integrity/Pharmacologic Parental Therapies 19. Following a vaginal birth, a patient has lost a significant amount of blood and is starting to experience signs of hypovolemic shock. Which clinical signs would be consistent with this diagnosis? a. Decrease in blood pressure, with an increase in pulse pressure b. Compensatory response of tachycardia and decreased pulse pressure c. Decrease in heart rate and an increase in respiratory effort d. Flushed skin ANS: B Clinical signs consistent with the early stages of hypovolemic shock include normal blood pressure, decreased pulse pressure, compensatory tachycardia, and pale, cool skin color. DIF: Cognitive Level: Application OBJ: Nursing Process Step: Assessment MSC: Patient Needs: Physiologic Integrity/Physiologic Adaptation 20. A patient has been treated with oxytocin (Pitocin) for postpartum hemorrhage. Bleeding has stabilized and slowed down considerably. The peripad in place reveals a moderate amount of bright red blood, with no clots expelled when massaging the fundus. The patient now complains of having difficulty breathing. Auscultation of breath sounds reveals adventitious sounds. Based on this clinical presentation, the priority nursing action is to a. evaluate intake and output of the past 12 hours following birth. b. initiate a rapid response intervention. c. obtain an order from the physician for type and crossmatch of 2 units packed red blood cells (PRBCs). d. reposition the patient and reassess in 15 minutes. Initiate frequent vital sign assessments. ANS: B Oxytocin (Pitocin) can have antidiuretic effects when used in large amounts. Given the recent patient history, she has received an additional Pitocin infusion relative to the direct observation of postpartum hemorrhage. Adventitious breath sounds and the patient‘s complaints of difficulty breathing suggest that the patient is progressing to pulmonary edema. An appropriate intervention is to initiate a rapid response intervention so that the patient can be stabilized. Calling the physician for a type and crossmatch order is not indicated. Repositioning the patient, even with the initiation of frequent vital signs, will not treat the emerging clinical condition. Evaluation of intake and output, although necessary, is not the priority nursing action at this time. DIF: Cognitive Level: Analysis OBJ: Nursing Process Step: ImplementationMSC: Patient Needs: Safe and Effective Care Environment/Establishing Priorities 21. A postpartum patient has developed deep vein thrombosis (DVT) and treatment with warfarin (Coumadin) has been initiated. Which dietary selection should be modified in view of this treatment regimen? a. Fresh fruits b. Milk c. Lentils d. Soda ANS: C Foods that are high in vitamin K should be restricted and/or limited in consumption while on Coumadin therapy. Vitamin K is the antidote to Coumadin activity. DIF: Cognitive Level: Application OBJ: Nursing Process Step: Planning MSC: Patient Needs: Physiologic Integrity/Pharmacologic and Parenteral Therapies 22. To determine an adverse response to carboprost tromethamine (Hemabate), the nurse should frequently assess a. temperature. b. lochial flow. c. fundal height. d. breath sounds. ANS: D Pulmonary edema is a potential adverse effect of carboprost tromethamine (Hemabate). Auscultation of breath sounds will identify pulmonary edema; temperature, lochial flow, and fundal height are not affected by this medication. DIF: Cognitive Level: Application OBJ: Nursing Process Step: Assessment MSC: Patient Needs: Physiologic Integrity 23. If the nurse suspects a complication of a low forceps birth labor, she should immediately a. administer a strong oral analgesic. b. assess the perineal and vaginal areas. c. assess the position of the uterine fundus. d. review the labor record for duration of second stage. ANS: B A low forceps birth may result in significant vaginal trauma. Assessment will provide information on the extent of trauma of the perineum and vagina. Administering an analgesic may interfere with obtaining an accurate assessment of the problem, assessing the position of the uterine fundus will not provide any information on vaginal or perineal trauma, and reviewing the labor record may support the suspicion that trauma has occurred but will not identify extent of trauma. DIF: Cognitive Level: Application OBJ: Nursing Process Step: Implementation MSC: Patient Needs: Health Promotion and Maintenance24. Prior to ambulating the patient whose admission hemoglobin level was 10.2 g/dL to the bathroom, the nurse should a. request repeat hemoglobin and hematocrit. b. assess the resting pulse rate. c. dangle her on the side of the bed. d. administer the ordered oral analgesic. ANS: C Patients with a low hemoglobin level prior to birth will most likely have a drop in the hemoglobin level following birth. A low hemoglobin level will result in dizziness and place the patient at risk for fainting when first ambulating. Having the patient sit on the side of the bed and dangle her legs prior to standing will allow for the blood pressure to stabilize and prevent fainting. Requesting additional labs will delay ambulation at a time when the patient needs to empty her bladder, assessing the resting pulse rate will not provide any information about the effect of ambulation on her cardiovascular system, and administering an ordered oral analgesic may contribute to feelings of faintness. DIF: Cognitive Level: Application OBJ: Nursing Process Step: Implementation MSC: Patient Needs: Physiologic Integrity 25. If a late postpartum hemorrhage is documented on a patient who delivered 3 days ago, the nurse recognizes that this hemorrhage occurred a. on the first postpartum day. b. during recovery phase of labor. c. during the third stage of labor. d. on the second postpartum day. ANS: D A late postpartum hemorrhage occurs after the first 24 hours and up to 12 weeks after birth. The first postpartum day, during the recovery phase, and during the third stage are all within the first 24 hours after birth and would be classified as early postpartum hemorrhage. DIF: Cognitive Level: Analysis OBJ: Nursing Process Step: Analysis MSC: Patient Needs: Health Promotion and Maintenance 26. Which patient data received during report should the nurse recognize as being at risk for postpartum complications? a. Gravida 5, para 5 b. Labor duration of 4 hours c. Infant weight greater than 3800 g d. Epidural anesthesia for labor and birth ANS: A Multiparity (five or more deliveries) is a risk factor for postpartum uterine atony and hemorrhage. A labor duration of 4 hours is not a risk factor because it is not a precipitate labor and birth (less than 3 hours), infant weight of 3800 g is not a risk factor because the infant is not macrosomic,and epidural anesthesia is not a risk factor because epidural anesthesia does not affect uterine contractions. DIF: Cognitive Level: Application OBJ: Nursing Process Step: Analysis MSC: Patient Needs: Health Promotion and Maintenance 27. Before administering methylergonovine (Methergine), the nurse checks the a. color of the lochia. b. blood pressure. c. location of the fundus. d. last administration of analgesics. ANS: B Methylergonovine (Methergine) elevates the blood pressure and should not be given to a woman who is hypertensive. The color of the lochia, location of the fundus, and analgesics are not related to the administration of or contraindicated to this medication. DIF: Cognitive Level: Application OBJ: Nursing Process Step: Analysis MSC: Patient Needs: Physiologic Integrity/Pharmacologic and Parenteral Therapies 28. To evaluate the desired response of methylergonovine (Methergine), the nurse would assess the patient‘s a. uterine tone. b. pain level. c. blood pressure. d. last voiding. ANS: A Methylergonovine (Methergine) simulates sustained contraction of the uterus as evidenced by the tone of the uterus. The pain level, blood pressure, and voiding patterns are not related to the effectiveness of the medication. DIF: Cognitive Level: Application OBJ: Nursing Process Step: Evaluation MSC: Patient Needs: Physiologic Integrity/Pharmacologic and Parenteral Therapies 29. As you receive a report, which assessment finding should you recognize as an indication of a vaginal laceration? a. Fundus firm at the umbilicus b. Pulse of 90 bpm, blood pressure of 110/78 mm Hg c. Bright red continuous trickle of blood from vagina d. Patient requested pain medication twice during last shift ANS: C Lacerations of the birth canal should always be suspected if excessive bleeding continues when the fundus is firm. Bleeding from the genital tract often is bright red, in contrast to the darker red color of lochia; a firm fundus, pulse of 90 bpm, blood pressure of 110/78 mm Hg, and being medicated twice in one shift are common findings in the postpartum patient. DIF: Cognitive Level: Analysis OBJ: Nursing Process Step: AnalysisMSC: Patient Needs: Physiologic Integrity/Physiologic Adaptation 30. The nurse observes the patient as she ambulates to the bathroom. Which clinical finding might indicate development of a DVT (deep vein thrombosis)? a. Slow gait b. Shuffling gait c. Stiffness of right leg d. Leans on husband for support ANS: C Deep vein thrombosis may cause pain on ambulation and stiffness of the affected leg. A slow gait, shuffling gait, and needing ambulatory support are common observations of the postpartum patient because of weakness and discomfort of the perineum. DIF: Cognitive Level: Analysis OBJ: Nursing Process Step: Analysis MSC: Patient Needs: Physiologic Integrity/Physiologic Adaptation 31. If a DVT (deep vein thrombosis) is suspected, the nurse should a. perform a Homans sign on the affected leg. b. dorsiflex the foot of the affected leg. c. palpate the affected leg for edema and pain. d. place the patient on bed rest, with the affected leg elevated. ANS: D Initial treatment of DVT is bed rest with the leg elevated to decrease swelling and promote venous return. Performing a Homans sign, dorsiflexing the foot, and palpating the leg are contraindicated actions that may dislodge a DVT and result in a pulmonary embolism. DIF: Cognitive Level: Application OBJ: Nursing Process Step: Implementation MSC: Patient Needs: Safe and Effective Care Environment/Management of Care 32. If the nurse suspects a pulmonary embolism in the patient who suddenly complains of chest pain, she or he should immediately a. assess for abnormal breath sounds. b. apply O2 via tight face mask at 8 to 10 L/minute. c. position the patient in a supine position with the head of the bed flat. d. monitor pulse oximetry for decreased oxygen saturation. ANS: B Administration of oxygen will increase oxygen saturation and decrease hypoxia; assessing breath sounds and monitoring pulse oximetry provide assessment data but do not correct the problem. A supine position with the head of the bed flat is incorrect because the head of the bed should be elevated to facilitate respiratory function. DIF: Cognitive Level: Application OBJ: Nursing Process Step: Implementation MSC: Patient Needs: Safe and Effective Care Environment/Management of Care 33. To prevent infection of the reproductive tract, the nurse should instruct the patient to a.change the peripad once per shift. b. cleanse the perineum from front to back. c. perform pericare at least twice during the shift. d. increase fluid intake to 2500 to 3000 mL/day. ANS: B Lack of knowledge of hygiene measures increases the risk of postpartum infection. Wiping the perineum from front to back prevents introduction of infection into the reproductive tract from the anal area. Changing the peripad once per shift and performing pericare twice in a shift are incorrect because these interventions should be done at every voiding or bowel elimination, and increasing fluid intake does not prevent infection of the reproductive tract. DIF: Cognitive Level: Application OBJ: Nursing Process Step: Implementation MSC: Patient Needs: Health Promotion and Maintenance 34. The nurse notes that the fundus of a postpartum patient is boggy, shifted to the left of the midline, and 2 cm above the umbilicus. What is the nurse‘s priority action? a. Massage the fundus of the uterus. b. Assist the patient out of bed to void. c. Increase the infusion of oxytocin (Pitocin). d. Ask another nurse to bring in a straight catheter tray. ANS: A If the uterus is not firmly contracted, the first intervention is to massage the fundus until it is firm and to express clots that may have accumulated in the uterus. One hand is placed just above the symphysis pubis to support the lower uterine segment, while the other hand gently but firmly massages the fundus in a circular motion. Clots that may have accumulated in the uterine cavity interfere with the ability of the uterus to contract effectively. They are expressed by applying firm but gentle pressure on the fundus in the direction of the vagina. If the uterus does not remain contracted as a result of uterine massage or if the fundus is displaced, the bladder may be distended. A full bladder lifts the uterus, moving it up and to the side, preventing effective contraction of the uterine muscles. Assist the mother to urinate or catheterize her to correct uterine atony caused by bladder distention. Note the urine output. When the fundus is boggy, begin uterine massage. Check the woman‘s bladder for distention and have her empty it if necessary. If she is not able to void and the bladder is distended, catheterize the patient. Weigh blood-soaked pads. DIF: Cognitive Level: Application OBJ: Nursing Process Step: Implementation MSC: Patient Needs: Health Promotion and Maintenance MULTIPLE RESPONSE 1. Which information should the nurse recognize as contributing to mastitis in the breastfeeding mother? (Select all that apply.) a. Insufficient emptying b. Feeding every 2 hours c. Supplementing feedings d.Blisters on both nipples e. Alternating breastfeeding positions ANS: A, C, D Mastitis may develop because of stasis of milk, inadequate emptying of the breast, skipped feedings, and introduction of bacteria through injured areas of the nipple. Feeding every 2 hours and alternating breastfeeding positions are both interventions that promote emptying of the breasts and support successful breastfeeding. DIF: Cognitive Level: Analysis OBJ: Nursing Process Step: Analysis MSC: Patient Needs: Safe Effective Care Environment/Management of Care 2. The visiting nurse must be aware that women who have had a postpartum hemorrhage are subject to a variety of complications after discharge from the hospital. These include which of the following? (Select all that apply.) a. Anemia b. Dehydration c. Exhaustion d. Postpartum infection e. Failure to attach to her infant ANS: A, C, D, E Postpartum hemorrhage often results in anemia, and iron therapy may need to be initiated. Exhaustion is common after hemorrhage. It may take the new patient weeks to feel like herself again. Fatigue may interfere with normal parent-infant bonding and the attachment processes. The patient is likely to require assistance with housework and infant care. Excessive blood loss increases the risk for infection. The excessive blood loss that this patient has experienced is likely to lead to risk for infection rather than dehydration. It is important that all mothers be educated about adequate fluid intake after birth. DIF: Cognitive Level: Application OBJ: Nursing Process Step: Planning MSC: Patient Needs: Physiologic Integrity Chapter 19: Normal Newborn: Processes of Adaptation Foundations of Maternal-Newborn & Women‘s Health Nursing, 7th Edition MULTIPLE CHOICE 1. A nursing student is helping the mother-baby nurse with morning vital signs. A baby born 10 hours ago via cesarean birth is found to have moist lung sounds. Which is the best interpretation of this information? a. This is an emergency situation. b. The neonate must have aspirated surfactant. c. If this baby was born vaginally, it could indicate a pneumothorax. d. The lungs of a baby delivered by cesarean birth may sound moist for 24 hours after birth. ANS: D The condition will resolve itself within a few hours. For this common condition of newborns, surfactant acts to keep the expanded alveoli partially open between respirations. During vaginalbirth, the absorption of remaining fluid in the lungs is accelerated by the process of labor and birth. Remaining lung fluid will move into interstitial spaces and be absorbed by the circulatory and lymphatic systems. This is a common condition for infants delivered by cesarean birth. Surfactant is produced by the lungs; therefore aspiration is not a concern. It is common to have some fluid left in the lungs; this will be absorbed within a few hours. DIF: Cognitive Level: Analysis OBJ: Nursing Process Step: Assessment MSC: Patient Needs: Health Promotion and Maintenance 2. Which of the following organs are nonfunctional during fetal life? a. Eyes and ears b. Lungs and liver c. Kidneys and adrenals d. Gastrointestinal system ANS: B Most of the fetal blood flow bypasses the nonfunctional lungs and liver. Near term, the eyes are open and the fetus can hear. Kidneys and adrenals function during fetal life. The fetus continuously swallows amniotic fluid, which is filtered through the kidneys. The gastrointestinal system functions during fetal life. DIF: Cognitive Level: Understanding OBJ: Nursing Process Step: Assessment MSC: Patient Needs: Health Promotion and Maintenance 3. Which method of heat loss may occur if a newborn is placed on a cold scale or touched with cold hands? a. Radiation b. Conduction c. Convection d. Evaporation ANS: B Conduction occurs when the infant comes in contact with cold objects. Radiation is the transfer of heat to a cooler object that is not in direct contact with the infant. Convection occurs when heat is transferred to the air surrounding the infant. Evaporation can occur during birth or bathing as a result of wet linens or clothes, or insensible heat loss. DIF: Cognitive Level: Understanding OBJ: Nursing Process Step: Implementation MSC: Patient Needs: Physiologic Integrity 4. How can nurses prevent evaporative heat loss in the newborn? a. Placing the baby away from the outside wall and the windows b. Keeping the baby out of drafts and away from air conditioners c. Drying the baby after birth and wrapping the baby in a dry blanket d. Warming the stethoscope and nurse‘s hands before touching the baby ANS: CWet linens or wet clothes can cause heat loss by evaporation. Radiation heat loss is caused by placing the baby near cold surfaces or equipment. Heat loss by convection occurs when drafts come from open doors and air currents created by people moving around. Conduction heat loss occurs when the baby comes into contact with cold objects or surfaces. DIF: Cognitive Level: Application OBJ: Nursing Process Step: Implementation MSC: Patient Needs: Physiologic Integrity 5. The nurse is explaining how a newly delivered baby initiates respirations. Which statement explains this process most accurately? a. Drying off the infant b. Chemical, thermal, and mechanical factors c. An increase in the PO2 and a decrease in the PCO2 d. The continued functioning of the foramen ovale ANS: B A variety of these factors are responsible for initiation of respirations. Tactile stimuli aid in initiating respirations but are not the main cause. The PO2 decreases at birth and the PCO2 increases. The foramen ovale closes at birth. DIF: Cognitive Level: Application OBJ: Nursing Process Step: Implementation MSC: Patient Needs: Physiologic Integrity 6. During fetal circulation the pressure is greatest in the a. left atrium. b. right atrium. c. hepatic system. d. pulmonary veins. ANS: B Pressure in the fetal circulation is greatest in the right atrium, which allows right-to-left shunting that aids in bypassing the lungs during intrauterine life. The pressure increases in the left atrium after birth and will close the foramen ovale. The liver does not filter the blood during fetal life until the end. It is functioning by birth. Blood bypasses the pulmonary vein during fetal life. DIF: Cognitive Level: Remembering OBJ: Nursing Process Step: Assessment MSC: Patient Needs: Health Promotion and Maintenance 7. The infant‘s heat loss immediately at birth is predominantly from a. radiation. b. conduction. c. convection. d. evaporation. ANS: D Because the infant is wet with amniotic fluid and blood, heat loss by evaporation occurs quickly. Radiation occurs with the transfer of heat to a cooler object that is not in direct contact with the infant. Conduction occurs when the infant comes into contact with a cold surface. The cribshould be preheated to prevent this from occurring. Convection occurs when heat is transferred to the air surrounding the infant. DIF: Cognitive Level: Understanding OBJ: Nursing Process Step: Assessment MSC: Patient Needs: Physiologic Integrity 8. The nurse is explaining the risk of hypothermia in the newborn to a group of nursing students. Which statement best describes the manifestations of hypothermia in the newborn? a. Newborns shiver to generate heat. b. Newborns have decreased oxygen demands. c. Newborns have increased glucose demands. d. Newborns have a decreased metabolic rate. ANS: C In hypothermia, the basal metabolic rate (BMR) is increased in an attempt to compensate, thus requiring more glucose. Shivering is not an effective method of heat production for newborns. Oxygen demands increase with hypothermia. The metabolic rate increases with hypothermia. DIF: Cognitive Level: Application OBJ: Nursing Process Step: Implementation MSC: Patient Needs: Physiologic Integrity 9. Which infant has the lowest risk of developing high levels of bilirubin? a. The infant who developed a cephalohematoma b. The infant who was bruised during a difficult birth c. The infant who uses brown fat to maintain temperature d. The infant who is breastfed during the first hour of life ANS: D The infant who is fed early will be less likely to retain meconium and resorb bilirubin from the intestines back into the circulation. Cephalohematomas will release bilirubin into the system as the red blood cells die off. Bruising will release more bilirubin into the system. Brown fat is normally used to produce heat in the newborn. DIF: Cognitive Level: Analysis OBJ: Nursing Process Step: Evaluation MSC: Patient Needs: Health Promotion and Maintenance 10. The nurse is preparing to administer a vitamin K injection to the infant shortly after birth. Which statement is important to understand regarding the properties of vitamin K? a. It is necessary for the production of platelets. b. It is important for the production of red blood cells. c. It is not initially synthesized because of a sterile bowel at birth. d. It is responsible for the breakdown of bilirubin and the prevention of jaundice. ANS: C The bowel is initially sterile in the newborn, and vitamin K cannot be synthesized until food is introduced into the bowel. The platelet count in term of newborns is near adult levels. Vitamin K is necessary to activate prothrombin and other clotting factors. Vitamin K is important for blood clotting. Vitamin K is necessary to activate the clotting factors.DIF: Cognitive Level: Application OBJ: Nursing Process Step: Implementation MSC: Patient Needs: Physiologic Integrity 11. A meconium stool can be differentiated from a transitional stool in the newborn because the meconium stool is a. seen at 3 days of age. b. the residue of a milk curd. c. passed in the first 24 hours of life. d. lighter in color and looser in consistency. ANS: C Meconium should be passed in the first 24 hours of life. Meconium stool is the first stool of the newborn. Meconium stool is made up of matter in the intestines during intrauterine life. Meconium is dark in color and sticky. DIF: Cognitive Level: Understanding OBJ: Nursing Process Step: Assessment MSC: Patient Needs: Health Promotion and Maintenance 12. Which of the following is the most likely cause of regurgitation when a newborn is fed? a. The gastrocolic reflex b. A relaxed cardiac sphincter c. An underdeveloped pyloric sphincter d. Placing the infant in a prone position following a feeding ANS: B The underlying cause of newborn regurgitation is a relaxed cardiac sphincter. The gastrocolic reflex increases intestinal peristalsis after the stomach fills. The pyloric sphincter goes from the stomach to the intestines. The infant should be placed in a supine position. DIF: Cognitive Level: Understanding OBJ: Nursing Process Step: Implementation MSC: Patient Needs: Health Promotion and Maintenance 13. The process in which bilirubin is changed from a fat-soluble product to a water-soluble product is known as a. albumin binding. b. enterohepatic circuit. c. conjugation of bilirubin. d. deconjugation of bilirubin. ANS: C Conjugation of bilirubin is the process of changing the bilirubin from a fat-soluble to a watersoluble product. Albumin binding attaches something to a protein molecule. Enterohepatic circuit is the route whereby part of the bile produced by the liver enters the intestine, is resorbed by the liver, and then is recycled into the intestine. Unconjugated bilirubin is fat-soluble. DIF: Cognitive Level: Understanding OBJ: Nursing Process Step: Assessment MSC: Patient Needs: Physiologic Integrity14. A newborn is admitted to the special care nursery with hypothermia. Which complication should the nurse monitor for closely? a. Hyperglycemia b. Metabolic acidosis c. Respiratory acidosis d. Vasodilation of peripheral blood vessels ANS: B Cold stress can cause a significant rise in oxygen demands. Metabolism of glucose in the presence of insufficient oxygen causes increased production of acids. Metabolism of brown fat also releases fatty acids. The result can be metabolic acidosis, which can be a life-threatening condition. Cold stress causes hypoglycemia because glucose is being metabolized. Cold stress does not cause respiratory acidosis. As the infant‘s body attempts to conserve heat, vasoconstriction, not vasodilation, of the peripheral blood vessels occurs to reduce heat loss from the skin surface. DIF: Cognitive Level: Analysis OBJ: Nursing Process Step: Evaluation MSC: Patient Needs: Physiologic Integrity 15. Which action by the nurse can result in hyperthermia in the newborn? a. Placing a cap on the newborn b. Wrapping the newborn in a warm blanket c. Placing the newborn in a skin to skin position with the mother d. Placing the newborn in the radiant warmer without attaching the skin probe ANS: D Newborns may be overheated by poorly regulated equipment designed to keep them warm. When radiant warmers, warming lights, or warmed incubators are used, the temperature mechanism must be set to vary the heat according to the infant‘s skin temperature; this prevents too much or too little heat. Alarms to signal that the infant‘s temperature is too high or too low should be functioning properly. If the skin probe is not used, the alarms will not function properly. Putting a hat on the newborn, wrapping the newborn in a warm blanket, or placing the newborn skin to skin with the mother will not cause hyperthermia. DIF: Cognitive Level: Application OBJ: Nursing Process Step: Implementation MSC: Patient Needs: Safe and Effective Care Environment 16. A multiparous patient arrives to the labor unit and urgently states, ―The baby is coming RIGHT NOW!‖ The nurse assists the patient into a comfortable position and delivers the infant. To prevent infant heat loss from conduction, what is the priority nursing action? a. Dry the baby off. b. Turn up the temperature in the patient‘s room. c. Pour warmed water over the baby immediately after birth. d. Place the baby on the patient‘s abdomen after the cord is cut. ANS: DMovement of heat away from the body occurs when newborns have direct contact with objects that are cooler than their skin. Placing infants on cold surfaces or touching them with cool objects causes this type of heat loss. The reverse is also true; contact with warm objects increases body heat by conduction. Warming objects that will touch the infant or placing the unclothed infant against the mother‘s skin (skin to skin) helps prevent conductive heat loss. Drying the baby off helps prevent heat loss through evaporation. Adjusting the temperature in the patient‘s room helps with heat loss through convection. Pouring warm water over a baby occurs with the first bath, which is conducted after the baby‘s temperature has stabilized. Pouring warm water over the baby prior to that time will increase heat loss through evaporation. DIF: Cognitive Level: Application OBJ: Nursing Process Step: Implementation MSC: Patient Needs: Health Promotion and Maintenance 17. The nurse is planning to conduct the initial assessment of a full-term newborn. Included in the plan is providing a neutral thermal environment. To accomplish this plan, what is the desired environmental temperature to conduct the assessment? a. 24 to 27°C (75.2 to 80.6°F) b. 28 to 31.5°C (82.4 to 88.7°F) c. 32 to 33.5°C (89.6 to 92.3°F) d. 34 to 37.5°C (93.2 to 99.5°F) ANS: C A neutral thermal environment is one in which the infant can maintain a stable body temperature with minimal oxygen need and without an increase in metabolic rate. The range of environmental temperature that allows this stability is called the thermoneutral zone. In healthy, unclothed, full-term newborns, an environmental temperature of 32 to 33.5°C (89.6 to 92.3°F) provides a thermoneutral zone. When the infant is dressed, the thermoneutral range is 24 to 27°C (75.2 to 80.6°F). DIF: Cognitive Level: Understanding OBJ: Nursing Process Step: Implementation MSC: Patient Needs: Health Promotion and Maintenance 18. An infant at 39 weeks‘ gestation was just delivered; included in the protocol for a term infant is an initial blood glucose assessment. The nurse obtains the blood sample and the reading is 58 mg/dL. What is the priority nursing action based upon this reading? a. Document the finding in the newborn‘s chart. b. Double-wrap the newborn under a warming unit. c. Feed the newborn a 10% dextrose solution. d. Notify the neonatal intensive care unit (NICU) of the pending admission. ANS: A In the term infant, glucose levels should be 40 to 60 mg/dL on the first day and 50 to 90 mg/dL thereafter. There is no general consensus regarding the level of blood glucose that defines hypoglycemia; however, a level below 40 to 45 mg/dL in the term infant is often used. If an infant is placed in a warming unit, the skin needs to be exposed. Because the glucose level is normal, no supplemental feeding is necessary. Dextrose solution is only administered when the glucose levels are very low. There is no information in the stem indicating the need for admission to the NICU. DIF: Cognitive Level: Analysis OBJ: Nursing Process Step: AnalysisMSC: Patient Needs: Health Promotion and Maintenance 19. During the first few minutes after birth, which physiologic change occurs in the newborn as a response to vascular pressure changes in increased oxygen levels? a. Increased pulmonary vascular resistance b. Decreased systemic resistance c. Decreased pressure in the left heart d. Dilation of pulmonary vessels ANS: D Dilation of pulmonary vessels occurs in response to increased oxygen levels. Decrease in pulmonary vascular resistance occurs. Increase in systemic vascular resistance occurs. Increased pressure in the left heart occurs. DIF: Cognitive Level: Application OBJ: Nursing Process Step: Assessment MSC: Patient Needs: Physiologic Integrity/Physiologic Adaptation 20. Which infant is at greater risk to develop cold stress? a. Full-term infant delivered vaginally without complications. b. 36-week infant with an Apgar score of 7 to 9. c. 38-week female infant delivered via cesarean birth because of cephalopelvic disproportion. d. Term infant delivered vaginally with epidural anesthesia. ANS: B Preterm infants are at greater risk to develop cold stress because of thin skin, decreased subcutaneous fat, and poor muscle tone. DIF: Cognitive Level: Analysis OBJ: Nursing Process Step: Assessment MSC: Patient Needs: Health Promotion and Maintenance 21. A reported hematocrit level for a newborn delivered by vaginal birth is 75%. Based on this lab value, which complication is the newborn least likely to develop? a. Hypoglycemia b. Respiratory distress c. Infection d. Jaundice ANS: C The presence of polycythemia as indicated by this lab result could result in the infant being at risk to develop hypoglycemia, respiratory distress, and jaundice. Possible infection would be unrelated to this diagnostic value. DIF: Cognitive Level: Application OBJ: Nursing Process Step: Assessment MSC: Patient Needs: Physiologic Integrity/Reduction of Risk Potential MULTIPLE RESPONSE 1. In the newborn nursery, you are reviewing the maternal medication list to ascertain if there is any significant risk to the newborn. Which medications would pose a potential risk to the newborn in terms of clotting ability? (Select all that apply.)a. Carbamazepine b. Phenytoin (Dilantin) c. Phenobarbital d. INH (Isoniazid) e. Prenatal vitamins with iron ANS: A, B, C, D Carbamazepine, phenytoin (Dilantin), phenobarbital, and isoniazid (INH) when taken by the mother can affect the newborn‘s clotting ability. Anticonvulsant usage can cause bleeding problems. Prenatal vitamins with added iron should have no effect on the newborn‘s clotting ability. DIF: Cognitive Level: Application OBJ: Nursing Process Step: Assessment MSC: Patient Needs: Physiologic Integrity/Pharmacologic and Parenteral Therapies 2. The nurse is teaching the postpartum patient about newborn transitional stools. Which should the nurse include in the teaching session with regard to transitional stools? (Select all that apply.) a. They are a greenish brown color. b. They are of a looser consistency. c. They have a tarlike consistency. d. They have a consistency of mustard. e. They are seedy, with a sweet-sour smell. ANS: A, B Meconium stools are followed by transitional stools, a combination of meconium and milk stools. They are greenish brown and of a looser consistency than meconium. Stools that are tarlike are meconium stools. Infants fed with breast milk are seedy, with a sweet-sour smell; the meconium has the consistency of mustard. DIF: Cognitive Level: Application OBJ: Nursing Process Step: Planning MSC: Patient Needs: Health Promotion and Maintenance 3. Which newborn is at higher risk for developing hypoglycemia? (Select all that apply.) a. Post-term newborn b. 38 weeks‘ gestation newborn c. Small-for-gestational-age newborn d. Large-for-gestational-age newborn e. Term newborn delivered by cesarean birth ANS: A, C, D Many newborns are at increased risk for hypoglycemia. In the preterm, late preterm (born between 34 weeks and 36 6/7 weeks of gestation), and small-for-gestational-age infant,adequate stores of glycogen or even fat for metabolism may not have accumulated. Stores may be used up before birth in the post-term infant because of poor intrauterine nourishment from a deteriorating placenta. Large-for-gestational-age infants and those with diabetic mothers may produce excessive insulin that consumes available glucose quickly. The newborn born at 38 weeks and the newborn born by cesarean at term have lower risk for hypoglycemia. DIF: Cognitive Level: Analysis OBJ: Nursing Process Step: Evaluation MSC: Patient Needs: Physiologic Integrity COMPLETION 1. The postpartum nurse is administering vitamin K (phytonadione) to a newborn. The prescribed order is to administer one dose of 0.5 mg of vitamin K via the intramuscular (IM) route within 1 hour after birth. The ampule of vitamin K sent from the pharmacy is 1 mg/0.5 mL. How many milliliters does the nurse draw up to administer the correct dose? Record your answer to two decimal points. mL ANS: 0.25 Use the medication calculation formula to calculate the correct dose. Formula: Desired/available volume = milliliters per dose 0.5 mg/1 mg 0.5 mL = 0.25 mL DIF: Cognitive Level: Application OBJ: Nursing Process Step: Implementation MSC: Patient Needs: Safe and Effective Care Environment Chapter 20: Assessment of the Normal Newborn Foundations of Maternal-Newborn & Women's Health Nursing, 7th Edition MULTIPLE CHOICE 1. The hips of a newborn are examined for developmental dysplasia. Which clinical finding indicates an incomplete development of the acetabulum? a. Negative Barlow test b. Equal knee heights c. Negative Ortolani sign d. Thigh and gluteal creases are asymmetric. ANS: D Asymmetric thigh and gluteal creases may indicate potential dislocation of the hip. If the hip is dislocated, the knee on the affected side will be lower. A positive Ortolani sign yields a clunking sensation and indicates a dislocated femoral head moving into the acetabulum. During a positive Barlow test, the examiner can feel the femoral head move out of the acetabulum. DIF: Cognitive Level: Understanding OBJ: Nursing Process Step: Assessment MSC: Patient Needs: Health Promotion and Maintenance 2. Which newborn reflex is elicited by stroking the lateral sole of the infant‘s foot from the heel to the ball of the foot? a. Babinski b. Stepping c. Tonic neck d.Plantar grasp ANS: A The Babinski reflex causes the toes to flare outward and the big toe to dorsiflex. The stepping reflex occurs when infants are held upright, with their heel touching a solid surface, and the infant appears to be walking. The tonic neck reflex (also called the fencing reflex) refers to the posture assumed by newborns when in a supine position. Plantar grasp reflex is similar to the palmar grasp reflex; when the area below the toes is touched, the infant‘s toes curl over the nurse‘s finger. DIF: Cognitive Level: Application OBJ: Nursing Process Step: Assessment MSC: Patient Needs: Health Promotion and Maintenance 3. Infants who develop cephalohematoma are at an increased risk for a. infection. b. jaundice. c. caput succedaneum. d. erythema toxicum. ANS: B Cephalohematomas are characterized by bleeding between the bone and its covering, the periosteum. Because of the breakdown of the red blood cells within a hematoma, the infants are at greater risk for jaundice. Cephalohematomas do not increase the risk for infections. Caput is an edematous area on the head from pressure against the cervix. Erythema toxicum is a benign rash of unknown cause that is sometimes referred to as ―fleabite rash.‖ DIF: Cognitive Level: Understanding OBJ: Nursing Process Step: Assessment MSC: Patient Needs: Physiologic Integrity 4. Which statement best explains why a newborn with a congenital defect of the penis should not be circumcised? a. There is increased risk of infection. b. The foreskin might be needed for future repairs. c. A circumcision will make the defect more visible. d. There is no medical rationale for a circumcision. ANS: B The foreskin may be used to correct a defect. There is no significant increase in infection. A circumcision would not make the defect more noticeable. A circumcision is a decision made by the parents; however, in this case the foreskin might be needed to correct a defect. Such defects include epispadias and hypospadias. DIF: Cognitive Level: Understanding OBJ: Nursing Process Step: Planning MSC: Patient Needs: Physiologic Integrity 5. A maculopapular rash with a red base and a small white papule in the center is commonly known as a. milia. b. Mongolian spots. c.erythema toxicum. d. Café-au-lait spots. ANS: C A maculopapular rash with a red base and a small white papule in the center is a description of erythema toxicum, a normal rash in the newborn. Milia are minute epidermal cysts on the face of the newborn. Mongolian spots are bluish-black discolorations found on dark-skinned newborns, usually on the sacrum. Café-au-lait spots are pale tan (the color of coffee with milk) macules. Parents should be reassured that occasional spots occur normally in most newborns. DIF: Cognitive Level: Analysis OBJ: Nursing Process Step: Assessment MSC: Patient Needs: Health Promotion and Maintenance 6. A newborn that is a large-for-gestational-age (LGA) infant is in which percentile(s) for weight? a. Below the 90th b. Less than the 10th c. Greater than the 90th d. Between the 10th and 90th ANS: C The LGA rating is based on weight and is defined as greater than the 90th percentile in weight. An infant between the 10th and 90th percentiles is average for gestational age. An infant in less than the 10th percentile is small for gestational age. DIF: Cognitive Level: Analysis OBJ: Nursing Process Step: Assessment MSC: Patient Needs: Health Promotion and Maintenance 7. A new patient asks, ―Why are you doing a gestational age assessment on my baby?‖ The nurse‘s best response is a. ―It was ordered by your physician.‖ b. ―This must be done to meet insurance requirements.‖ c. ―It helps us identify infants who are at risk for any problems.‖ d. ―The gestational age determines how long the infant will be hospitalized.‖ ANS: C The nurse should provide the mother with accurate information about various procedures performed on the newborn. Assessing gestational age is a nursing assessment and does not have to be ordered. It is not necessary for insurance needs. Gestational age does not dictate hospital stays. Problems that occur because of gestational age may prolong the stay. DIF: Cognitive Level: Application OBJ: Nursing Process Step: Implementation MSC: Patient Needs: Health Promotion and Maintenance 8. Which nursing action is designed to avoid unnecessary heat loss in the newborn? a. Maintain room temperature at 21°C (70°F). b. Place a blanket over the scale before weighing the infant. c. Take the rectal temperature every hour to detect early changes. d.Undress the infant completely for assessments so that they can be finished quickly. ANS: B Padding the scale prevents heat loss from the infant to a cold surface by conduction. The room temperature should be appropriate to prevent heat loss from convection. Also, if the room is warm enough, radiation will assist in maintaining body heat. Hourly assessments are not necessary for a normal newborn with a stable temperature. Undressing the infant completely will expose the child to cooler room temperatures and cause a drop in body temperature by convection. DIF: Cognitive Level: Application OBJ: Nursing Process Step: Implementation MSC: Patient Needs: Physiologic Integrity 9. The nurse is performing a gestational age assessment on a newborn. Which characteristic indicates the greatest gestational maturity? a. The infant‘s arms and legs are extended. b. There is some peeling and cracking of the skin. c. There are few rugae on the scrotum and the testes are high in the scrotum. d. The arm can be positioned with the elbow beyond the midline of the chest. ANS: B Peeling, cracking, dryness, and a few visible veins in the skin are signs of maturity in the newborn. Extended arms and legs are a sign of preterm infants. Few rugae on the scrotum indicate a younger age in the newborn. The arm being able to be positioned with the elbow beyond the midline of the chest is a result of the scarf sign and indicates a newborn of a younger age. DIF: Cognitive Level: Application OBJ: Nursing Process Step: Assessment MSC: Patient Needs: Health Promotion and Maintenance 10. A new mother states, ―My baby is so thin and wrinkled. It looks like he has too much skin.‖ Which is the most therapeutic response by the nurse in response to the patient‘s statement? a. ―You sound disappointed about how your infant looks.‖ b. ―All mothers are concerned about how their babies look.‖ c. ―Don‘t worry. In no time he‘ll fill out his skin and look just fine.‖ d. ―You know, all the cigarettes you smoked interfered with the nourishment he needed.‖ ANS: A The nurse should clarify the patient‘s statement and allow her to verbalize her feelings. ―All mothers are concerned about how their babies look‖ generalizes her concerns and does not answer the mother‘s question. ―Don‘t worry. In no time he‘ll fill out his skin and look just fine‖ does not directly answer the mother‘s question and could leave her feeling like she asked an unacceptable question. ―You know, all the cigarettes you smoked interfered with the nourishment he needed‖ is condescending and hurtful and would not allow for further conversation between the nurse and mother. DIF: Cognitive Level: Application OBJ: Nursing Process Step: Implementation MSC: Patient Needs: Psychosocial Integrity 11. Which assessment finding of a newborn requires prompt action by the nurse? a. Respiratory rate of 50 breaths per minuteb. Cyanosis of the extremities c. Pause in breathing lasting 20 seconds d. Pause in breathing for 15 seconds followed by rapid respirations ANS: C Apnea is a pause in breathing lasting 20 seconds or more, or accompanied by cyanosis, pallor, bradycardia, and/or decreased muscle tone. Apnea is abnormal and requires prompt intervention. A respiratory rate of 50 breaths per minute is still within the normal range. Tachypnea is considered to be 60 breaths per minute or more. Cyanosis of the extremities or acrocyanosis is normal during the first day after birth and if the infant becomes cold. Periodic breathing is pauses in breathing lasting 5 to 10 seconds without other changes followed by rapid respirations for 10 to 15 seconds. This occurs in some full-term infants during the first few days but is more common in preterm infants. DIF: Cognitive Level: Application OBJ: Nursing Process Step: Assessment MSC: Patient Needs: Physiologic Integrity 12. The nurse is receiving shift report on her mother-baby couplet assignment. Which infant should the nurse evaluate first? a. 38-weeks‘ gestation female newborn with a blood sugar level of 60 mg/dL b. Term male newborn with a noted axillary temperature of 37.2°C (99°F) c. 40-weeks‘ gestation female newborn with reported poor feed at last attempt d. 39-weeks‘ gestation male newborn who has been crying prior to initial bath ANS: C Newborns who are poor feeds may be showing initial signs of hypoglycemia, so this newborn should be assessed first at the start of the shift. Although the newborn is term, and it is more likely to see hypoglycemia with preterm infants, sometimes hypoglycemia is asymptomatic. Blood sugar results are within normal range and the newborn is considered to be term. Temperature is within normal range and the newborn is term. This newborn is considered to be term, and crying alone does not increase risk stratification. DIF: Cognitive Level: Analysis OBJ: Nursing Process Step: Assessment MSC: Patient Needs: Safe and Effective Care Environment 13. Inspection of a newborn‘s head following birth reveals a hard ridged area and significant molding. The anterior and posterior fontanels show no sign of depression. Delivery history indicates that the mother was pushing for over 3 hours and had epidural anesthesia. A vacuum extraction was necessary. Based on this information the nurse would a. continue to monitor newborn and anticipate that molding will subside. b. inspect and document location of fontanels to complete the head assessment. c. contact the pediatric provider. d. note findings as being within normal limits as a result of the strenuous birth process. ANS: C Assessment data reveal a significant finding, and the nurse should suspect craniosynostosis (premature closing of sutures) and therefore should contact the pediatric provider immediately.Even though the birth process was difficult and vacuum extraction was used, this does not account for the physical findings. Continuing to monitor is not a prudent action at this time. Although it is important to note the presence of fontanels, the immediate action would be to make the appropriate referral for medical intervention. DIF: Cognitive Level: Analysis OBJ: Nursing Process Step: Evaluation MSC: Patient Needs: Safe and Effective Care Environment/Establishing Priorities 14. The nurse is performing the initial assessment of a newborn and notes retractions, nasal flaring, and tachypnea. The nurse will continue to perform a focused assessment on which system? a. Respiratory b. Cardiovascular c. Gastrointestinal d. Musculoskeletal ANS: A Tachypnea, a respiratory rate of more than 60 breaths per minute, is the most common sign of respiratory distress. Retractions occur when the soft tissue around the bones of the chest is drawn in with the effort of pulling air into the lungs. Xiphoid (substernal) retractions occur when the area under the sternum retracts each time the infant inhales. When the muscles between the ribs are drawn in so that each rib is outlined, intercostal retractions are present. A reflex widening of the nostrils occurs when the infant is receiving insufficient oxygen. Nasal flaring helps decrease airway resistance and increase the amount of air entering the lungs. DIF: Cognitive Level: Understanding OBJ: Nursing Process Step: Assessment MSC: Patient Needs: Health Promotion and Maintenance 15. The mother-baby nurse is providing care to a patient and her newborn 2 hours after delivery. On review of the newborn‘s chart, the nurse sees a notation of caput succedaneum. What will the nurse expect to find in the mother‘s chart? a. Race: non-White b. A longer than usual labor c. Administration of an epidural d. Delivery by cesarean birth ANS: B A caput succedaneum is an area of localized edema that appears over the vertex of the newborn‘s head as a result of pressure against the mother‘s cervix during labor. The pressure interferes with blood flow from the area, causing localized edema at birth. The edematous area crosses suture lines, is soft, and varies in size. The longer the labor, the more pronounced the caput. Mongolian spots are associated with infants born to non-White parents. An epidural may be a contributing factor to a prolonged labor, but it is the pressure of the head against the cervix that gives rise to the caput. If labor is prolonged without descent of the head, a cesarean birth may follow but is not the cause of the caput. DIF: Cognitive Level: Analysis OBJ: Nursing Process Step: Analysis MSC: Patient Needs: Health Promotion and Maintenance 16. The nurse is assessing a newborn delivered 24 hours ago for jaundice. What is the best way to evaluate for this finding?a. Depress the tip of the nose. b. Stroke the outer aspect of the foot. c. Place a finger in the palm of the hand. d. Rotate the hips in an upward and outward direction. ANS: A The nurse assesses for jaundice at least every 8 to 12 hours and is particularly watchful when infants are at increased risk for hyperbilirubinemia. Jaundice is identified by pressing the infant‘s skin over a firm surface, such as the end of the nose or the sternum. The skin blanches as the blood is pressed out of the tissues, making it easier to see the yellow color that remains. Jaundice is more obvious when the nurse assesses in natural light. Jaundice begins at the head and moves down the body, and the areas of the body involved should be documented. Jaundice becomes visible when the bilirubin level is greater than 5 mg/dL. The Babinski reflex is assessed by stroking the outer aspect of the foot. The grasp reflex is determined by placing a finger in the newborn‘s palm. The Barlow and Ortolani tests are methods of assessing for hip instability in the newborn period. Both legs should abduct equally in normal infants. Abducting the affected hip may be difficult. A hip click may be felt or heard but is usually normal and is different from the clunk of hip dysplasia when the femoral head moves in the hip socket. DIF: Cognitive Level: Application OBJ: Nursing Process Step: Assessment MSC: Patient Needs: Health Promotion and Maintenance 17. An infant at term was born at 0105 hours. The nurse is developing a plan of care for the newborn. During which time range will the nurse plan on performing the assessment to determine a Ballard score? a. 0115 to 0130 b. 0200 to 0600 c. 1400 to 1800 d. 2000 to 2300 ANS: B The new Ballard score is often used to determine gestational age based on neuromuscular and physical characteristics. It is designed to assess gestational age from 20 to 44 weeks. The assessment is most accurate when performed within 12 hours of birth. The Ballard score is accurate within a 2-week window of gestational age. DIF: Cognitive Level: Application OBJ: Nursing Process Step: Assessment MSC: Patient Needs: Health Promotion and Maintenance 18. The nurse is assessing a newborn and notes a nevus flammeus birthmark. Which of the following figures depicts this birthmark? a. b. c. d. ANS: C A nevus flammeus (port wine stain) is a permanent, flat, pink to dark reddish-purple mark that varies in size and location. Erythema toxicum is a red blotchy area that may have white or yellow papules or vesicles in the center; it is not a birthmark. Mongolian spots are bluish-blackmarks that resemble bruises. They usually occur in the sacral area but may appear on the buttocks, arms, shoulders, and other areas. A nevus simplex is also called salmon patch, stork bite, or telangiectatic nevus. It is a flat pink or reddish discoloration from dilated capillaries that occur over the eyelids, just above the bridge of the nose, or at the nape of the neck. DIF: Cognitive Level: Analysis OBJ: Nursing Process Step: Assessment MSC: Patient Needs: Physiologic Integrity MULTIPLE RESPONSE 1. The nurse is conducting a body system assessment of the newborn. Which are abnormal findings that the nurse should report? (Select all that apply.) a. Low-set ears b. Yellow sclera c. A doll‘s eye sign d. Edema of the eyelids e. Absence of the grasp reflex ANS: A, B, E Low-set ears may indicate chromosomal abnormalities. The sclera should be white or bluish white. A yellow color indicates jaundice. Absence of reflexes may indicate a serious neurologic problem. The doll‘s eye sign is a normal finding in the newborn; when the head is turned quickly to one side, the eyes move toward the other side. Edema of the eyelids and subconjunctival hemorrhages (reddened areas of the sclera) result from pressure on the head during birth, which causes capillary rupture in the sclera. DIF: Cognitive Level: Application OBJ: Nursing Process Step: Assessment MSC: Patient Needs: Physiologic Integrity 2. To differentiate between caput succedaneum and cephalohematoma in a newborn, the nurse would consider the following clinical information. (Select all that apply.) a. These are both normal presentations because of the birth process and will resolve within 24 to 48 hours. b. Cephalohematoma manifests as a localized area of swelling as compared with caput succedaneum, which appears as a general swelling of the head. c. A cephalohematoma can develop several hours or days after the birth event, whereas caput succedaneum is noted shortly before or immediately after the birth event. d. Edema that crosses suture lines is observed with caput succedaneum. e. With a cephalohematoma, bleeding occurs between the bone and skull. ANS: C, D, E Cephalohematoma can be detected up to 24 to 48 hours after the birth process. This clinical condition is caused by bleeding between the periosteum and skull and is a serious medical condition. Caput succedaneum occurs in the presence of pressure from the vaginal canal on the fetal head during the birth process. Swelling is localized and crosses the suture line, whereas with cephalohematoma the swelling is more generalized and crosses the suture line. Caput resolves within 12 to 48 hours after the birth event. DIF: Cognitive Level: Application OBJ: Nursing Process Step: AssessmentMSC: Patient Needs: Physiologic Integrity/Physiologic Adaptation 3. Which clinical findings are early signs of hypoglycemia in the newborn? (Select all that apply.) a. Jitteriness b. Poor feeding c. Respiratory difficulty d. An increase in temperature e. A capillary refill of 2 seconds ANS: A, B, C Early signs of hypoglycemia include jitteriness and other central nervous system signs and signs of respiratory difficulty, a decrease in temperature, and poor feeding. A capillary refill of 2 seconds is a normal finding in the newborn. DIF: Cognitive Level: Analysis OBJ: Nursing Process Step: Evaluation MSC: Patient Needs: Physiologic Integrity 4. The nurse is performing a gestational age assessment on a newborn. Which characteristics indicate a preterm newborn? (Select all that apply.) a. Translucent skin b. Extended limp arms and legs c. The ear springs back when folded d. Square window angle of 45 degrees or less e. Large clitoris and labia minora in the female newborn ANS: A, B, E The very preterm infant‘s skin is translucent because it is thin and has little subcutaneous fat beneath the surface. Preterm neonates have immature flexor muscles and little energy or muscle tone. Therefore they have extended and limp arms and legs that offer little resistance to movement by the examiner. In the preterm female infant, the labia majora are small and separated, and the clitoris and labia minora are large by comparison. In the term neonate, the ear springs back to its original position immediately. The more mature the neonate, the smaller the angle of the square window assessment until the palm folds flat against the forearm at term, the result of maternal hormones at the end of pregnancy. DIF: Cognitive Level: Application OBJ: Nursing Process Step: Assessment MSC: Patient Needs: Physiologic IntegrityChapter 21: Care of the Normal Newborn Foundations of Maternal-Newborn & Women‘s Health Nursing, 7th Edition MULTIPLE CHOICE 1. A yellow crust has formed over the circumcision site. The mother calls the hotline at the local hospital 5 days after her son was circumcised. She is very concerned. Which response by the nurse is most appropriate? a. The yellow crust should not be removed. b. This yellow crust is an early sign of infection. c. Discontinue the use of petroleum jelly to the tip of the penis. d. After circumcision, the diaper should be changed frequently and fastened snugly. ANS: A Crusting is a normal part of healing. The normal yellowish exudate that forms over the site should be differentiated from the purulent drainage of infection. The only contraindication for petroleum jelly is the use of a PlastiBell device. The diaper should be fastened loosely to prevent rubbing or pressure on the incision site. DIF: Cognitive Level: Application OBJ: Nursing Process Step: Implementation MSC: Patient Needs: Health Promotion and Maintenance 2. Most newborns receive a prophylactic injection of vitamin K soon after birth. Which site is optimal for the newborn? a. Deltoid muscle b. Gluteal muscle c. Rectus femoris muscle d. Vastus lateralis muscle ANS: D The vastus lateralis muscle is located away from the sciatic nerve and femoral blood vessels. Gluteal muscles are not used until a child has been walking for at least 1 year to develop these muscles. The rectus femoris is used only if absolutely necessary because this muscle is located closer to the sciatic nerve and blood vessels, which poses a greater danger. The deltoid is not a recommended site for newborn injections. DIF: Cognitive Level: Understanding OBJ: Nursing Process Step: Implementation MSC: Patient Needs: Health Promotion and Maintenance 3. Which information should the nurse teach to new parents regarding the use of a bulb syringe? a. Use it only once per day. b. Suction the back of the throat vigorously. c. Insert the syringe into the sides of the mouth. d. Always suction the mouth before suctioning the nose. ANS: CThe syringe should be inserted into the sides of the mouth rather than the back of the throat to avoid a vagal response and bradycardia. Suction can occur as needed. Vigorous suction of the back of the throat may stimulate the vagal nerve and produce bradycardia. The mouth should be suctioned first to prevent aspiration. DIF: Cognitive Level: Application OBJ: Nursing Process Step: Implementation MSC: Patient Needs: Health Promotion and Maintenance 4. In providing and teaching cord care, which guidance is most appropriate? a. Cord care is done only to control bleeding. b. Alcohol is the only agent used for cord care. c. It takes a minimum of 24 days for the cord to separate. d. Keeping the cord dry will decrease bacterial growth. ANS: D Bacterial growth increases in a moist environment; therefore keeping the umbilical cord dry impedes bacterial growth. Evidence-based practice guidelines show that cleaning the cord with water when necessary and keeping it clean and dry is the best method of care. No other agents are necessary to facilitate drying of the cord. The cord will fall off within 10 to 14 days. DIF: Cognitive Level: Understanding OBJ: Nursing Process Step: Implementation MSC: Patient Needs: Health Promotion and Maintenance 5. Which of the following guidelines should the nurse implement to prevent the abduction of a newborn from the hospital? a. Restricting the amount of time infants are out of the nursery b. Questioning anyone who is seen walking in the hallways carrying an infant c. Allowing no visitors in the maternity area except those who have identification bracelets d. Instructing the parents not to give the baby to anyone except the nurse assigned that day ANS: B Infants should be transported in the hallways only in their cribs. In many facilities babies are cared for in the mother‘s room, rather than a well-baby nursery. Infants need to spend time with the parents to facilitate the bonding process and facilitate learning. It is impossible for one nurse to be on call for one mother and baby for the entire shift; therefore the parents need to be able to identify all of the staff that will be caring for them. Most maternity units have special identification badges unique to that area. All patients should be oriented to these identification badges. DIF: Cognitive Level: Application OBJ: Nursing Process Step: Implementation MSC: Patient Needs: Safe and Effective Care Environment 6. A nursing student has been caring for a patient and newborn all morning. After taking the newborn to the nursery for hearing screening, the student is returning the infant to his mother. Which procedure is correct for identifying the newborn? a. Ask the mother to state her name and the name of her infant. b. Call out the mother‘s full name before leaving the infant with her. c. Have the mother read her printed band number and verify that it matches the infant‘s number.d. Return the infant with no special procedure because the student knows the mother and infant. ANS: C The mother and infant should have identifying armbands with matching numbers. Both of these bands should be reviewed to determine that the mother has the correct infant. The other actions do not adequately verify the identities of mother and infant. DIF: Cognitive Level: Application OBJ: Nursing Process Step: Implementation MSC: Patient Needs: Safe and Effective Care Environment 7. The nurse is explaining the procedure of newborn screening to parents prior to discharge. Which statement by the parents indicates a need for further teaching? a. ―We understand the tests are performed at 24 to 48 hours.‖ b. ―We‘re glad all the tests can be done on one blood sample.‖ c. ―We wish the tests would screen for congenital hypothyroidism, it runs in our family.‖ d. ―We know that if the tests are done before 24 hours, the tests will need to be repeated at 1 to 2 weeks.‖ ANS: C Common disorders often included in newborn screening are phenylketonuria (PKU), hypothyroidism, galactosemia, hemoglobinopathies such as sickle cell disease and thalassemia, and congenital adrenal hyperplasia. The parents require further teaching if they suggest that congenital hypothyroidism is not screened. The newborn screening tests are performed at 24 to 48 hours after birth. Newborn screening requires a blood sample taken from the infant‘s heel, and only one blood sample is needed for all tests. Tests performed within the first 24 hours of life are less sensitive than those performed after 24 hours. Infants tested before 12 to 24 hours of age should have repeat tests at 1 to 2 weeks of age so that disorders are not missed because of early testing. DIF: Cognitive Level: Application OBJ: Nursing Process Step: Evaluation MSC: Patient Needs: Health Promotion and Maintenance 8. Which newborn assessment finding requires the nurse to take immediate action? a. Glucose level of 40 mg/dL b. Axillary temperature of 37°C (98.6°F) c. Mild yellow tinge to skin at 32 hours of age d. Mild inflammation of conjunctiva after eye prophylaxis ANS: A A glucose level of 40 mg/dL requires an action. The nurse should follow agency policy and health care provider orders regarding feeding infants with low glucose levels. A common practice is to feed the newborn breast milk or formula if the glucose screening reveals a level of 40 to 45 mg/dL or less to prevent further depletion of glucose. Infants with severe hypoglycemia may need intravenous feedings to provide glucose more rapidly. A normal temperature for a newborn is 36.5 to 37.5°C (97.7 to 99.5°F). Mild jaundice at 32 hours of age is physiologic jaundice and does not need an action by the nurse, just further monitoring. Some infants develop a mild inflammation a few hours after prophylactic eye treatment. DIF: Cognitive Level: Application OBJ: Nursing Process Step: Implementation MSC: Patient Needs: Physiologic Integrity9. The nurse is evaluating a newborn‘s circumcision 30 minutes after the procedure. The nurse notes excessive bleeding coming from the circumcised area. Which priority intervention should the nurse implement at this time? a. Apply pressure to the site. b. Continue to observe for another 30 minutes. c. Apply the diaper tightly over the circumcised area. d. Apply petroleum jelly to the site with a small piece of gauze. ANS: A If excessive bleeding occurs after a circumcision, pressure is applied to the site. The nurse notifies the physician, who may apply Gelfoam or epinephrine or suture the small blood vessels. A small amount of blood loss may be significant in an infant, who has a small total blood volume. Continuing to observe could mean additional blood loss. Applying the diaper tightly will not stop the bleeding. Petroleum jelly is applied to keep the diaper from sticking to the circumcised area. It will not stop the bleeding. DIF: Cognitive Level: Application OBJ: Nursing Process Step: Implementation MSC: Patient Needs: Physiologic Integrity 10. In which position should the parents be instructed to place their newborn for sleep? a. On the back b. On the left side c. On the right side d. On the abdomen ANS: A The American Academy of Pediatrics (AAP) recommends that mothers and fathers be taught to place infants consistently on the back for sleep. This position is associated with the lowest rate of SIDS. The side-lying position is not advised because of the possibility that the infant might roll to the prone position. The newborn should not be placed on the abdomen except for short periods under supervision in order to prevent plagiocephaly. DIF: Cognitive Level: Application OBJ: Nursing Process Step: Planning MSC: Patient Needs: Health Promotion and Maintenance 11. A 38 weeks‘ gestation fetus is delivered via cesarean birth and transported to the newborn nursery in an isolette. Apgar scores were 8, 9, and 10. At this time, the infant is receiving an initial assessment in the newborn nursery. Which is the priority nursing diagnosis? a. Risk for injury related to potential equipment malfunction of radiant warmer b. Altered tissue perfusion related to use of medications during delivery process c. Ineffective airway clearance due to mode of delivery and use of anesthetics d. Risk for ineffective thermoregulation related to gestational age ANS: C Delivery via cesarean birth may affect the newborn‘s ability to remove excess fluid secretions because the infant did not move down the birth canal and thus may be at risk for airwayconcerns. There is no evidence to support that the equipment is malfunctioning. Although the use of medications may affect the newborn in terms of respiratory, cardiac, and neurologic depression, Apgar scores do not indicate any immediate deficit. The infant is at term based on reported gestational age and therefore is not a risk for ineffective thermoregulation because of this fact. DIF: Cognitive Level: Analysis OBJ: Nursing Process Step: Nursing Diagnosis MSC: Patient Needs: Health Promotion and Maintenance/Ante/Intra/Postpartum and Newborn Care 12. An infant‘s temperature is recorded at 36°C (96.8°F) during the morning assessment. Which action should the nurse take? a. Note the findings in the electronic health record (EHR). b. Unwrap the infant and inspect for abnormalities. c. Provide the infant with glucose water. d. Make sure that the infant is wrapped securely with a blanket and recheck temperature in 15 minutes. ANS: D This temperature potentially indicates hypothermia, so the infant should be wrapped securely in a blanket and reassessed after that intervention. Findings should be documented in the EHR; however, this is not the priority intervention. Unwrapping the infant would lead to further compromise and additional risk for the core temperature to drop. Feeding the infant with glucose water may eventually be used as an intervention if the infant shows additional signs of hypoglycemia, which may accompany hypothermia. DIF: Cognitive Level: Application OBJ: Nursing Process Step: Implementation MSC: Patient Needs: Safe and Effective Care Environment/Establishing Priorities 13. When an infant‘s temperature drops from (37 to 36.3°C) 98.7 to 97.4°F, the nurse should a. instruct parents on the risks of cold stress. b. determine the time and amount of last feeding. c. increase the temperature in the mother‘s room. d. evaluate infant for the presence of a blood sugar level higher than 50 mg/dL. ANS: B Temperature instability in the neonate may be caused by a decrease in blood glucose levels. Infants who do not maintain adequate intake will not have adequate energy to maintain temperature; instructing parents on cold stress and increasing the temperature in the room are interventions to maintain a stable temperature but will not correct the underlying problem. A blood sugar level higher than 50 mg/dL is a normal finding. DIF: Cognitive Level: Application OBJ: Nursing Process Step: Implementation MSC: Patient Needs: Physiologic Integrity 14. Administration of medications after birth is the topic of discussion during a prenatal education class. Which statement indicates to the nurse that the pregnant patient understands the primary indication for the administration of vitamin K? a. ―The nurse will draw blood to determine if vitamin K is needed.‖b. ―Vitamin K prevents the possibility of bleeding problems in my baby.‖ c. ―My baby will receive medication by mouth when the nurse administers the vitamin K.‖ d. ―Vitamin K will be administered shortly after birth, generally within the first hour.‖ ANS: B This indication is the reason for vitamin K administration. Vitamin K is given to neonates because they cannot synthesize it in the intestines without bacterial flora. This places them at risk for hemorrhagic disease of the newborn (vitamin K deficiency disease). One dose of vitamin K intramuscularly after birth prevents bleeding problems until the infant is able to produce vitamin K in sufficient amounts. Vitamin K is not routinely given by mouth. Although the injection is usually given within the first hour after birth, it can be delayed until the infant has finished breastfeeding shortly after birth. DIF: Cognitive Level: Application OBJ: Nursing Process Step: Evaluation MSC: Patient Needs: Health Promotion and Maintenance 15. An hour after birth, the nurse assesses a newborn‘s temperature and notes that it is 36.2°C (97.2°F). The next activity planned for the newborn is the bath, and the new mother and father are invited to participate in the procedure. What is the nurse‘s next action? a. Take the infant‘s temperature rectally. b. Ask the father to test the water to determine if it is too hot. c. Delay the bath until the newborn‘s temperature is above 36.7°C (98°F). d. Explain to the new parents that no soap should be used to cleanse the eyes. ANS: C A temperature of 36.7°C (98°F) or higher is often used to determine when to bathe the infant. The infant can lose heat in the bath through the process of evaporation. Rectal temperatures are avoided because they can traumatize the rectal mucosa. The water temperature should be approximately 38 to 40°C (100.4 to 104°F). The nurse should determine if the bath water is the correct temperature to avoid scalding the newborn. Explain the process of giving a bath during the procedure. Informing the parents before the procedure may result in loss of information. DIF: Cognitive Level: Application OBJ: Nursing Process Step: Implementation MSC: Patient Needs: Health Promotion and Maintenance 16. Which intervention will be most helpful to parents in identifying problems with an infant car seat? a. Questioning the parents about the instructions b. Providing the parents with current laws on infant and child safety c. Asking the parents to demonstrate how to secure the infant in the car seat d. Allowing the parents to ask questions and express feelings about infant restraint ANS: C If the nurse observes the parents demonstrating the use of the car seat, any problems or misunderstandings can be identified. Questioning the parents is not a helpful way to identify problems with a car seat; a return demonstration is preferable. Providing information without a return demonstration will not prove that the parents are comfortable with the car seat for theinfant. A return demonstration is the best way to ensure that the parents understand car seat safety. Parents should also be encouraged to attend a local car seat fitting station. DIF: Cognitive Level: Application OBJ: Nursing Process Step: Implementation MSC: Patient Needs: Safe and Effective Care Environment 17. Which statement made by a parent indicates a need for the nurse to provide instruction on safety and accident prevention? a. ―I always take the phone off the hook when I give my baby a bath so I won‘t be disturbed.‖ b. ―I‘m going to buy a backpack for my 2-week-old baby so I can carry her in it whenever she gets fussy.‖ c. ―I‘ve been reading about what new things my baby will be learning to do in the next month or two, so I‘ll know what to expect.‖ d. ―I make sure I always place the baby in her own crib after feeding her in my bed.‖ ANS: B Backpacks should be used only for infants old enough to support their heads well by themselves. Ideally parents should obtain an infant carrier designed specifically for carrying a baby. ―I always take the phone off the hook when I give my baby a bath so I won‘t be disturbed,‖ ―I‘m going to buy a backpack for my 2-week-old baby so I can carry her in it whenever she gets fussy,‖ and ―I make sure I always place the baby in her own crib after feeding her in my bed‖ are all appropriate statements regarding newborn safety. DIF: Cognitive Level: Analysis OBJ: Nursing Process Step: Evaluation MSC: Patient Needs: Safe and Effective Care Environment 18. Which statement made by a new mother should be a cause of concern to the nurse? a. ―I will start my baby on solid foods at 5 months.‖ b. ―I usually keep the temperature in my house at 22.2°C (72°F).‖ c. ―I plan to position my infant on his back when sleeping.‖ d. ―I don‘t intend to spoil my baby by picking him up every time he cries.‖ ANS: D Infant crying often indicates an unmet need. Parents should be cautioned about ignoring crying. Infants whose parents intervene appropriately for crying are less likely to cry excessively as they grow older. Solid foods should be started no earlier than 4 to 6 months. A house temperature of 22.2°C (72°F) is appropriate for a newborn. The appropriate position for a baby is on his or her back. DIF: Cognitive Level: Analysis OBJ: Nursing Process Step: Evaluation MSC: Patient Needs: Safe and Effective Care Environment 19. Which of the following is the appropriate treatment for miliaria? a. Application of oil b. Removal of wet clothing c. Removal of excess clothing d. Application of soothing lotionANS: C Miliaria (heat rash) develops in infants who are too warmly dressed. Infants should be dressed in the same amount of clothing as the parent plus a receiving blanket. Oils and ointments should be avoided. Wet clothing is not the cause of miliaria. Lotion should be avoided. Often a bath will assist in cooling the infant, especially in hot weather. DIF: Cognitive Level: Understanding OBJ: Nursing Process Step: Assessment MSC: Patient Needs: Health Promotion and Maintenance 20. An infant who eats very rapidly may experience problems with swallowing excessive air. What should the mother be instructed to do? a. Use a nipple with a smaller hole. b. Place the infant on the abdomen after feeding. c. Provide the infant with water between feedings. d. Begin the feeding before the infant becomes too hungry. ANS: D Infants eat rapidly when they are very hungry. If fed before becoming excessively hungry, the infant will eat at a slower rate. Using a nipple with a smaller hole will not prevent swallowing excessive air. Infants should be placed on their back; however, can be put on their ‗front to play‘ for short periods under supervision. Water should not be given in between feedings. All infants should be burped frequently throughout the feeding. DIF: Cognitive Level: Application OBJ: Nursing Process Step: Planning MSC: Patient Needs: Health Promotion and Maintenance 21. Which statement is true regarding growth and development during the first 6 months? a. The infant will grow 1 cm in length per month. b. The infant will gain about 2 lb per month. c. The infant will regain weight lost after birth within 1 week. d. The infant will have a 1-inch increase in head circumference per month. ANS: B Each month the average infant gains 2 lb. Infants grow about 3.5 cm each month. Birth weight is usually regained in 14 days. An infant‘s head circumference increases about 2 cm a month. DIF: Cognitive Level: Understanding OBJ: Nursing Process Step: Assessment MSC: Patient Needs: Health Promotion and Maintenance 22. Infant immunizations should begin at which age? a. Birth b. 2 months c. 3 months d. 4 months ANS: A The schedule of infant immunizations calls for the initial dose of hepatitis B vaccine at birth. The first set of immunizations is given at birth.DIF: Cognitive Level: Understanding OBJ: Nursing Process Step: Planning MSC: Patient Needs: Health Promotion and Maintenance 23. Which clinical finding indicates a sign of illness in the newborn? a. A yellow scaly lesion on the scalp b. More than two soft stools per day c. Regurgitating a small amount of feeding d. An axillary temperature greater than 38°C (100.4°F) ANS: D Infants commonly respond to a variety of illnesses with an elevation in temperature. Yellow scaly lesions on the scalp are normal findings and are probably cradle cap. More than two soft stools per day are appropriate for a newborn. Regurgitating a small amount of a feeding is a normal variance. DIF: Cognitive Level: Understanding OBJ: Nursing Process Step: Assessment MSC: Patient Needs: Health Promotion and Maintenance 24. During the first 6 months of life, the infant should have well-baby checkups at which interval? a. 1 to 2 weeks b. 2 to 4 weeks c. 1 to 2 months d. 3 to 4 months ANS: C Most pediatricians schedule well-baby checkups every 1 to 2 months (4 to 8 weeks) to assess the infant‘s growth and development, answer parental questions, observe for abnormalities, and give immunizations. Checkups are scheduled for every 1 to 2 months. Two to 4 weeks are too soon between visits, and 3 to 4 months are too long between checkups. DIF: Cognitive Level: Understanding OBJ: Nursing Process Step: Assessment MSC: Patient Needs: Health Promotion and Maintenance 25. As the nurse assists a newly discharged patient and her infant to the waiting car, the nurse notes that the infant seat is in the front seat of the car facing the front and secured by the seat belt. The nurse should explain to the parents that the car seat should be placed a. in an upright position. b. at a 30-degree angle. c. not secured by the seat belt. d. in the back seat facing the rear of the car. ANS: D A car seat in the back seat facing the rear of the car provides the best protection by keeping the infant from being hurled forward on impact. The car seat should be in the back seat, facing the rear of the car. New recommendations suggest a rear facing car seat at a 45-degree angle for up to 2 years of age.DIF: Cognitive Level: Application OBJ: Nursing Process Step: Planning MSC: Patient Needs: Safe and Effective Care Environment 26. Which statement by a parent suggests that the nurse intervene with further teaching? a. ―I put my newborn baby on her back when she goes to sleep. I understand this is the best position.‖ b. ―Jennifer‘s eyes sometimes cross, but I know that this is normal in 1-month-old babies.‖ c. ―My 5-month-old infant has been drooling, biting, and running a fever for the past few days. I think he‘s teething.‖ d. ―My neighbor has been giving her baby solids since he was 8 weeks old. I think I‘ll wait until my baby is about 5 months old.‖ ANS: C Although drooling and biting are signs of teething, a fever should always be considered a sign of illness. A back position is the appropriate position for an infant to sleep. Eye crossing at this age is a normal deviation. Infants should not be started on solids until they are 4 to 6 months old. DIF: Cognitive Level: Analysis OBJ: Nursing Process Step: Evaluation MSC: Patient Needs: Safe and Effective Care Environment 27. A new mother asks what she can do to help her infant sleep through the night. Which should the nurse suggest? a. Bring the infant into a well-lit room for the feeding. b. Avoid talking to the infant and keep the room quiet during night feedings. c. Play with the infant after the feeding before putting the infant back into the crib. d. Change the infant‘s diaper after the feeding to prevent waking the infant later in the night. ANS: B Decreasing stimulation of the infant during and after the bedtime feeding will assist the infant in establishing a normal sleep pattern. Keeping the baby in a quiet, dimly lit room is a better option for a feeding during the night. The baby should be put right back into the crib after a feeding; it is not the time to play with the infant. The infant‘s diaper should be changed before the feeding is started or can be skipped so as not to disturb the infant too much. DIF: Cognitive Level: Application OBJ: Nursing Process Step: Planning MSC: Patient Needs: Health Promotion and Maintenance 28. A new mother asks, ―Why should I bring my baby in for a checkup? He is not sick.‖ Which is the nurse‘s best response? a. ―Please ask your pediatrician to explain this to you.‖ b. ―He may have a problem that you haven‘t identified.‖ c. ―These visits are required by law to identify communicable diseases.‖ d. ―Well-baby visits allow the doctor to determine whether your baby is growing and developing normally.‖ ANS: DThe pediatric provider utilizing well-baby checkups to observe for abnormalities, answer parental questions, give immunizations, and observe the normal growth and development of the infant. Checkups are done to allow for the provider to identify problems, not for the mother to identify problems. The nurse can answer this question; it does not need to be answered by the provider. Checkups are not required by law. DIF: Cognitive Level: Application OBJ: Nursing Process Step: Planning MSC: Patient Needs: Health Promotion and Maintenance 29. Which infant should be seen immediately by a health care provider? a. A 1-week-old infant with a diaper rash b. A 1-month-old infant with an axillary temperature of 37.7°C (99.8°F) c. A 3-week-old breast-fed infant who has had two loose stools d. A 2-week-old infant with nasal congestion and respirations of 64 breaths per minute ANS: D Normal respiratory function is a high priority in the newborn. Any situation in which respiratory function in the infant is impaired should be evaluated immediately by a physician. Diaper rashes are a normal variant. A temperature of 37.7°C (99.8°F) is still within normal limits. Breast-fed infants have loose stools, this is a normal finding. DIF: Cognitive Level: Application OBJ: Nursing Process Step: Assessment MSC: Patient Needs: Physiologic Integrity 30. Which intervention should be included in the home care of a high-risk infant? a. Feeding the infant on a strict schedule b. Keeping the infant in the supine or prone position c. Providing continued respiratory support and oxygen d. Cleaning the umbilical cord several times daily with alcohol ANS: C High-risk infants may continue to need assistance with respiratory function after discharge. It is unnecessary for the infant does to be kept on a strict schedule so as not to disrupt the sleeping patterns of the infant. A high-risk infant should be placed on the side or back as appropriate positions. Cleaning the cord several times a day with alcohol prep is not necessary for any infant. DIF: Cognitive Level: Understanding OBJ: Nursing Process Step: Planning MSC: Patient Needs: Health Promotion and Maintenance 31. Which statement by the parents indicates the need for further education with regard to pacifier use? a. ―We will discard the pacifier if it becomes torn.‖ b. ―We will replace the pacifier every 1 to 2 months.‖ c. ―We will be sure to cleanse the pacifier frequently.‖ d. ―We will keep track of the pacifier by tying it to a string around the baby‘s neck.‖ ANS: DPacifiers should never be placed on a string around the infant‘s neck. The string could become tangled tightly around the neck and cause strangulation. If parents make this statement, they need further instruction. When infants use a pacifier, parents should be instructed to examine it often to see if it is in good condition. Cracked, torn, or sticky nipples or nipples that can be pulled away from the shield should be discarded. Pacifiers should be replaced every 1 or 2 months because they may come apart as they deteriorate and cause aspiration of parts. Pacifiers should be kept clean by frequent washing, and parents should buy several so that one is always clean when needed. DIF: Cognitive Level: Analysis OBJ: Nursing Process Step: Evaluation MSC: Patient Needs: Safe and Effective Care Environment 32. The nurse is calling a new mother to schedule a routine home visit planned for 48 to 72 hours after discharge. What is the nurse‘s priority question to help determine the best time for the visit? a. ―When will the baby‘s father be home?‖ b. ―Do you plan on having any visitors in the day or two?‖ c. ―At approximately what time do you think you will be nursing your baby?‖ d. ―When will your home be presentable enough for me to come and visit?‖ ANS: C A feeding session should be observed, especially if the mother is breastfeeding. Establishment of milk supply, adequacy of the breast milk, and general support are important topics to discuss for the mother who is breastfeeding for the first time. During the home visit, the nurse performs a physical examination of the mother and infant. Family adaptation to the addition of a new member and the adequacy of the mother‘s support system is also assessed. Cleanliness of the home environment is only a concern when the baby‘s health is at risk. DIF: Cognitive Level: Analysis OBJ: Nursing Process Step: Analysis MSC: Patient Needs: Health Promotion and Maintenance 33. A new mother is preparing for discharge from the birthing center and relays to the nurse her concerns about how she will handle the baby‘s episodes of crying. What is the nurse‘s best response? a. ―I hear your concern. Is there someone in the household who cannot tolerate hearing a baby cry?‖ b. ―It is okay to just let the baby cry from time to time. You don‘t want to risk spoiling the baby too soon.‖ c. ―Infants only cry when they are hungry or if they have gas. If you don‘t eat any gas-producing food, your baby will cry less.‖ d. ―Crying is the way your baby communicates with you. It is important for you to meet your baby‘s needs consistently and promptly.‖ ANS: D Infants cannot signal that they have unmet needs in any way other than crying and are not spoiled when parents meet their needs. In fact, their needs must be met in a consistent, warm, prompt manner for the development of trust to occur. Infants who are consistently held when in distress cry less at 1 year and are less aggressive at 2 years of age. Therefore parents should be taught the importance of consistently and quickly answering infant cries. The response to theassessment of intolerance of crying is a leading question and nontherapeutic communication. Infants cry for many reasons, including hunger, discomfort, fatigue, overstimulation, and boredom. Parents can often identify the problem based on the type of sound made during crying. Sometimes no specific cause can be determined. There is no mention in the stem of the question that the new mother is breastfeeding. DIF: Cognitive Level: Application OBJ: Nursing Process Step: Implementation MSC: Patient Needs: Health Promotion and Maintenance 34. During a prenatal education class regarding infant home care, the nurse is reviewing the simulated setting created by new mothers for putting the baby to bed. Which observation indicates to the nurse that the new mothers understood the nurse‘s teaching about infant safety? a. The crib is lined with a bumper pad. b. Stuffed animals are placed in the crib. c. The baby mannequin is in the supine position. d. The baby mannequin is covered with a handmade quilt. ANS: C Infants should be positioned on the back for sleep. The nurse should explain that the prone position has been associated with sudden infant death syndrome (SIDS). No pillows, blankets, or soft stuffed animals should be allowed in the crib because they could cause suffocation. Infants can be placed in a zippered blanket sleeper for warmth. DIF: Cognitive Level: Application OBJ: Nursing Process Step: Evaluation MSC: Patient Needs: Health Promotion and Maintenance MULTIPLE RESPONSE 1. The nurse is teaching new parents strategies to help with newborn colic. Which interventions should the nurse suggest? (Select all that apply.) a. Increase the number of feedings. b. Feed the infant in an upright position. c. Burp the infant frequently during feedings. d. Allow the infant to cry for a period of time. e. Increase carrying time by use of a front carrier pack. ANS: B, C, E Feeding the infant in an upright position and burping frequently may help relieve discomfort from swallowed air, which can cause colic. Increasing the time spent carrying the infant often produces some improvement for colic. Feeding techniques such as overfeeding may contribute to colic, so the number of feedings should not be increased. Allowing the infant to cry excessively will cause the infant to swallow more air and will exacerbate the colic. White noise such as a fan in the background or car rides may also help reduce crying episodes. DIF: Cognitive Level: Application OBJ: Nursing Process Step: Implementation MSC: Patient Needs: Health Promotion and Maintenance 2. Parents ask the nurse, ―How many wet diapers a day should we expect and how will we know the baby‘s stools are normal?‖ Which response should the nurse make if the infant is being formula fed? (Select all that apply.)a. The stools should be watery. b. The stools should be dry and hard. c. The infant should have at least one stool a day. d. The infant should have at least six wet diapers a day. e. The infant will only have a bowel movement every other day. ANS: C, D Formula-fed infants generally pass at least one stool each day. The infant should have at least six wet diapers by the fourth day of life. Stools that are dry, hard, and marble-like indicate constipation. Watery stools indicate diarrhea. DIF: Cognitive Level: Application OBJ: Nursing Process Step: Implementation MSC: Patient Needs: Health Promotion and Maintenance 3. The nurse is teaching new parents how to avoid and treat newborn diaper rash. Which should the nurse include in the teaching session? (Select all that apply.) a. Keep the diaper area clean and dry. b. Do not use talc-based powders in the diaper area. c. Cleanse the diaper area with a scrubbing motion. d. Apply a thick layer of zinc oxide to prevent further outbreaks. e. Remove the diaper and expose the perineum to warm air if a rash develops. ANS: A, B, E Diaper rash is primarily treated by keeping the diaper area clean and dry. Talc-based powders should not be used because they can cause pneumonia if they get into the infant‘s lungs. Removing the diapers and exposing the perineum to warm air help healing. Parents should gently wash the perineum with mild soap and warm water but should avoid excessive washing or scrubbing. Applying a thin layer of zinc oxide or petrolatum may speed healing and help prevent further outbreaks. The nurse should tell parents not to apply the ointment too thickly because it may be difficult to remove. DIF: Cognitive Level: Application OBJ: Nursing Process Step: Planning MSC: Patient Needs: Health Promotion and Maintenance 4. The nurse is preparing a newborn for a circumcision. Which prescribed interventions should the nurse implement to alleviate pain? (Select all that apply.) a. Oral sucrose during the procedure b. Bright lights after the procedure c. Adequate stimulation before and after the procedure d. Acetaminophen (Tylenol) postprocedure, as needed e. EMLA cream (eutectic mixture of local anesthetics) before the procedure ANS: A, D, ENonpharmacologic pain relief methods during and after the circumcision include pacifiers, oral sucrose, soothing music, recordings of intrauterine sounds, decreased lights, and talking softly to the infant. Acetaminophen may be given throughout the first day for postprocedure pain. EMLA cream (eutectic mixture of local anesthetics) may be applied to anesthetize the skin before the procedure. Bright lights and stimulation would not be methods to reduce circumcision pain. DIF: Cognitive Level: Application OBJ: Nursing Process Step: Implementation MSC: Patient Needs: Physiologic Integrity 5. The nurse has just completed discharge teaching to parents on newborn bathing. Which statement made by the parents indicates a further need for teaching? (Select all that apply.) a. ―We will clean the diaper area last.‖ b. ―We will use cotton-tipped swabs to clean the ears.‖ c. ―We will use an antibacterial soap during the sponge bath.‖ d. ―We can submerge the baby in a tub of water after the cord falls off.‖ e. ―We will shampoo the baby‘s head using a football hold before unwrapping.‖ ANS: B, C Soap is not necessary for the young infant but if used, it should be gentle and nonalkaline to protect the natural acids of the infant‘s skin. Do not use cotton-tipped swabs in the infant‘s ears or nose because injury may occur if the baby moves suddenly. Clean the diaper area last. The cord generally falls off in about 10 to 14 days. Some care providers suggest waiting for the cord to fall off before tub bathing. Before fully undressing the baby, use the football position to shampoo the baby‘s head. DIF: Cognitive Level: Analysis OBJ: Nursing Process Step: Evaluation MSC: Patient Needs: Health Promotion and Maintenance Chapter 22: Infant Feeding Foundations of Maternal-Newborn & Women‘s Health Nursing, 7th Edition MULTIPLE CHOICE 1. The breastfeeding patient should be taught a safe method to remove her breast from the baby‘s mouth. Which suggestion by the nurse is most appropriate? a. Break the suction by inserting your finger into the corner of the infant‘s mouth. b. A popping sound occurs when the breast is correctly removed from the infant‘s mouth. c. Slowly remove the breast from the baby‘s mouth when the infant has fallen asleep and the jaws are relaxed. d. Elicit the Moro reflex in the baby to wake the baby up, and remove the breast when the baby cries. ANS: A Inserting a finger into the corner of the baby‘s mouth between the gums to break the suction avoids trauma to the breast. A popping sound indicates improper removal of the breast from the baby‘s mouth and may cause cracks or fissures in the breast. The infant who is sleeping may lose grasp on the nipple and areola, resulting in chewing on the nipple, making it sore. Mostmothers prefer the infant to continue to sleep after the feeding. Gentle wake-up techniques are recommended. DIF: Cognitive Level: Application OBJ: Nursing Process Step: Implementation MSC: Patient Needs: Health Promotion and Maintenance 2. Which woman is most likely to continue breastfeeding beyond 6 months? a. A woman who avoids using bottles. b. A woman who uses formula for every other feeding. c. A woman who offers water or formula after breastfeeding. d. A woman whose infant is satisfied for 4 hours after the feeding. ANS: A Women who avoid using bottles and formula are more likely to continue breastfeeding. Use of formula decreases breastfeeding time and decreases the production of prolactin and, ultimately, the milk supply. Overfeeding after breastfeeding causes a sense of fullness in the infant, so the infant will not be hungry in 2 to 3 hours. Formula takes longer to digest. The new breastfeeding mother needs to nurse often to stimulate milk production. DIF: Cognitive Level: Analysis OBJ: Nursing Process Step: Assessment MSC: Patient Needs: Health Promotion and Maintenance 3. In which condition is breastfeeding contraindicated? a. Triplet birth b. Flat or inverted nipples c. Human immunodeficiency virus infection d. Inactive, previously treated tuberculosis ANS: C Human immunodeficiency virus is a serious illness that can be transmitted to the infant via body fluids. Because the amount of milk being produced depends on the amount of suckling of the breasts, providing enough milk should not be a problem. Nipple abnormality can begin to be treated during pregnancy but may begin after birth. Many methods help flat or inverted nipples to become more erect. Only active tuberculosis patients would be cautioned not to breastfeed. DIF: Cognitive Level: Understanding OBJ: Nursing Process Step: Assessment MSC: Patient Needs: Safe and Effective Care Environment 4. Which type of formula should not be diluted before being administered to an infant? a. Powdered b. Concentrated c. Ready to use d. Modified cow‘s milk ANS: C Ready to use formula can be poured directly from the can into the baby‘s bottle and is ideal (although expensive) when a proper water supply is not available. Formula should be well mixed to dissolve the powder and make it uniform. Improper dilution of concentrated formula maycause malnutrition or sodium imbalances. Cow‘s milk is more difficult for the infant to digest and is not recommended, even if it is diluted. DIF: Cognitive Level: Understanding OBJ: Nursing Process Step: Assessment MSC: Patient Needs: Health Promotion and Maintenance 5. How many kilocalories per kilogram (kcal/kg) of body weight does a full-term formula-fed infant need each day? a. 50 to 75 b. 100 to 110 c. 120 to 140 d. 150 to 200 ANS: B The term newborn being fed with formula requires 100 to 110 kcal/kg to meet nutritional needs each day. 50 to 75 kcal/kg is too little and 120 to 140 kcal/kg and 150 to 200 kcal/kg are too much. Requirements for breastfed infants are different. DIF: Cognitive Level: Understanding OBJ: Nursing Process Step: Assessment MSC: Patient Needs: Health Promotion and Maintenance 6. Which hormone is essential for milk production? a. Estrogen b. Prolactin c. Progesterone d. Lactogen ANS: B Prolactin, secreted by the anterior pituitary, is a hormone that causes the breasts to produce milk. Estrogen decreases the effectiveness of prolactin and prevents mature breast milk from being produced. Progesterone decreases the effectiveness of prolactin and prevents mature breast milk from being produced. Human placental lactogen decreases the effectiveness of prolactin and prevents mature breast milk from being produced. DIF: Cognitive Level: Understanding OBJ: Nursing Process Step: Assessment MSC: Patient Needs: Physiologic Integrity 7. Which recommendation should the nurse make to a patient to assist in initiating the milkejection reflex? a. Wear a well-fitting firm bra. b. Drink plenty of fluids. c. Place the infant to the breast. d. Apply cool packs to the breast. ANS: C Oxytocin, which causes the milk let-down reflex, increases in response to nipple stimulation. A firm bra is important to support the breast; however, will not initiate the let-down reflex. Drinking plenty of fluids is necessary for adequate milk production; but, will not initiate the let-downreflex. Cool packs to the breast will decrease the let-down reflex. For many mothers simply thinking of her infant will result in the let-down reflex. DIF: Cognitive Level: Application OBJ: Nursing Process Step: Implementation MSC: Patient Needs: Health Promotion and Maintenance 8. Which is the first step in assisting the breastfeeding mother to nurse her infant? a. Assess the woman‘s knowledge of breastfeeding. b. Provide instruction on the composition of breast milk. c. Discuss the hormonal changes that trigger the milk-ejection reflex. d. Help her obtain a comfortable position and place the infant to the breast. ANS: A The nurse should first evaluate the woman‘s knowledge and skill in breastfeeding to determine her learning needs. Assessment should occur prior to instruction. Discussing the hormonal changes and helping her obtain a comfortable position may be part of the instructional plan; however, assessment should occur first to determine what instruction is needed. DIF: Cognitive Level: Application OBJ: Nursing Process Step: Implementation MSC: Patient Needs: Health Promotion and Maintenance 9. Which of the following is an important consideration in positioning a newborn for breastfeeding? a. Placing the infant at nipple level facing the breast b. Keeping the infant‘s head slightly lower than the body c. Using the forefinger and middle finger to support the breast d. Limiting the amount of areola the infant takes into the mouth ANS: A Positioning the infant at nipple level will prevent downward pulling of the nipple and subsequent nipple trauma. Keeping the infant‘s head slightly lower will pull the nipple down and cause trauma. The forefinger and middle finger can be used to support the breast; however, this is not an important consideration in positioning the newborn. The infant should take in as much areola as possible to prevent trauma to the nipples. DIF: Cognitive Level: Analysis OBJ: Nursing Process Step: Evaluation MSC: Patient Needs: Physiologic Integrity 10. The patient should be taught that when her infant falls asleep after feeding for only a few minutes, she should do which of the following? a. Unwrap and gently arouse the infant. b. Wait an hour and attempt to feed again. c. Try offering a bottle at the next feeding. d. Put the infant in the crib and try again later. ANS: AThe infant who falls asleep during feeding may not have fed adequately and should be gently aroused to continue the feeding. Breastfeeding should continue. By offering a bottle, breast milk production will decrease. The infant should be aroused and feeding continued. DIF: Cognitive Level: Application OBJ: Nursing Process Step: Implementation MSC: Patient Needs: Health Promotion and Maintenance 11. To prevent breast engorgement, what should the new breastfeeding mother be instructed to do? a. Feed her infant no more than every 4 hours. b. Limit her intake of fluids for the first few days. c. Apply cold packs to the breast prior to feeding. d. Breast-feed frequently and for adequate lengths of time. ANS: D Engorgement occurs when the breasts are not adequately emptied at each feeding or if feedings are not frequent enough. Breast milk moves through the stomach within 1.5 to 2 hours, therefore waiting 4 hours to feed is too long. Frequent feedings are important to empty the breast and establish lactation. Fluid intake should not be limited with a breastfeeding mother; that would decrease the amount of breast milk produced. Warm packs should be applied to the breast before feedings. DIF: Cognitive Level: Application OBJ: Nursing Process Step: Implementation MSC: Patient Needs: Physiologic Integrity 12. As the nurse assists a new mother with breastfeeding, the mother asks, ―If formula is prepared to meet the nutritional needs of the newborn, what is in breast milk that makes it better?‖ The nurse‘s best response is that it contains a. more calcium. b. more calories. c. essential amino acids. d. important immunoglobulins. ANS: D Breast milk contains immunoglobulins that protect the newborn against infection. Calcium levels are higher in formula than breast milk. This higher level can cause an excessively high renal solute load if the formula is not diluted properly. The calorie counts of formula and breast milk are about the same. All the essential amino acids are in formula and breast milk. The concentrations may differ. DIF: Cognitive Level: Application OBJ: Nursing Process Step: Implementation MSC: Patient Needs: Physiologic Integrity 13. How should the nurse explain mild supply and demand when responding to the question, ―Will I produce enough milk for my baby as she grows and needs more milk at each feeding?‖ a. Early addition of baby food will meet the infant‘s needs. b. The breast milk will gradually become richer to supply additional calories. c. As the infant requires more milk, feedings can be supplemented with cow‘s milk.d. The mother‘s milk supply will increase as the infant demands more at each feeding. ANS: D The amount of milk produced depends on the amount of stimulation of the breast. Increased demand with more frequent and longer breastfeeding sessions results in more milk available for the infant. Solids should not be added until about 4 to 6 months, when the infant‘s immune system is more mature. This will decrease the chance of allergy formations. Mature breast milk will stay the same. The amounts will increase as the infant feeds for longer times. Supplementation will decrease the amount of stimulation of the breast and decrease the milk production. DIF: Cognitive Level: Application OBJ: Nursing Process Step: Implementation MSC: Patient Needs: Physiologic Integrity 14. Which technique should the nurse recommend to the postpartum patient in order to prevent nipple trauma? a. Assess the nipples before each feeding. b. Limit the feeding time to less than 5 minutes. c. Wash the nipples daily with mild soap and water. d. Position the infant so the nipple is far back in the mouth. ANS: D If the infant‘s mouth does not cover as much of the areola as possible, the pressure during sucking will be applied to the nipple, causing trauma to the area. Assessing the nipples for trauma is important; however, it will not prevent sore nipples. Stimulating the breast for less than 5 minutes will not produce the extra milk the infant may need. Soap can be drying to the nipples and should be avoided during breastfeeding. DIF: Cognitive Level: Application OBJ: Nursing Process Step: Implementation MSC: Patient Needs: Physiologic Integrity 15. A breastfeeding patient who was discharged yesterday calls to ask about a tender hard area on her right breast. What should the nurse‘s first response be? a. ―This is a normal response in breastfeeding mothers.‖ b. ―Notify your doctor so he can start you on antibiotics.‖ c. ―Stop breastfeeding because you probably have an infection.‖ d. ―Try massaging the area and apply heat; it is probably a plugged duct.‖ ANS: D A plugged lactiferous duct results in localized edema, tenderness, and a palpable hard area. Massage of the area followed by heat will cause the duct to open. This is a normal deviation but requires intervention to prevent further complications. Tender hard areas are not the signs of an infection, so antibiotics are not indicated. Fatigue, aching muscles, fever, chills, malaise, and headache are signs of mastitis. She may have a localized area of redness and inflammation. DIF: Cognitive Level: Application OBJ: Nursing Process Step: Implementation MSC: Patient Needs: Physiologic Integrity 16. Which is an important consideration regarding the storage of breast milk? a. Can be thawed and refrozenb. Can be frozen for up to 6 months c. Should be stored only in glass bottles d. Can be kept refrigerated for 72 hours ANS: D If used within 72 hours after being refrigerated, breast milk will maintain its full nutritional value. It should not be refrozen. Ideally frozen milk should be used within 6 months. Frozen milk should be kept at the back of the freezer. Milk can be stored in glass or rigid polypropylene plastic containers with a tight cap. Frozen milk should be thawed in the refrigerator and need used within 48 hours. DIF: Cognitive Level: Understanding OBJ: Nursing Process Step: Assessment MSC: Patient Needs: Safe and Effective Care Environment 17. What is the most serious consequence of propping an infant‘s bottle? a. Colic b. Aspiration c. Dental caries d. Ear infections ANS: B Propping the bottle increases the likelihood of choking and aspiration if regurgitation occurs. Colic can occur in any infant. Dental caries becomes a problem when milk stays on the gums for a long period of time. This may cause a buildup of bacteria that will alter the growing teeth buds. However, this is not the most serious consequence. Ear infections can occur when the warm formula runs into the ear and bacterial growth occurs. DIF: Cognitive Level: Understanding OBJ: Nursing Process Step: Assessment MSC: Patient Needs: Physiologic Integrity 18. A new mother asks why she has to open a new bottle of formula for each feeding. What is the nurse‘s best response? a. ―Formula may turn sour after it is opened.‖ b. ―Bacteria can grow rapidly in warm milk.‖ c. ―Formula loses some nutritional value once it is opened.‖ d. ―This makes it easier to keep track of how much the baby is taking.‖ ANS: B Formula should not be saved from one feeding to the next because of the danger of rapid growth of bacteria in warm milk. Formula will have bacterial growth before turning sour. This will cause problems in a newborn with an immature immune system. The loss of some nutritional value after the formula is opened is not the reason for using fresh bottles with each feeding. The danger of bacterial growth is the primary concern. DIF: Cognitive Level: Application OBJ: Nursing Process Step: Implementation MSC: Patient Needs: Safe and Effective Care Environment 19. A new mother asks whether she should feed her newborn colostrum because it is not ―real milk.‖ The nurse‘s best answer includes which information?a. Colostrum is unnecessary for newborns. b. Colostrum is high in antibodies, protein, vitamins, and minerals. c. Colostrum is lower in calories than milk and should be supplemented by formula. d. Giving colostrum is important in helping the mother learn how to breast-feed before she goes home. ANS: B Colostrum is important because it has high levels of the nutrients needed by the neonate and helps protect against infection. Colostrum provides immunity and enzymes necessary to clean the gastrointestinal system, among other things. Supplementation is not necessary. It will decrease stimulation to the breast and decrease the production of milk. It is important for the mother to feel comfortable in this role before discharge, but the importance of the colostrum to the infant is top priority. DIF: Cognitive Level: Application OBJ: Nursing Process Step: Implementation MSC: Patient Needs: Physiologic Integrity 20. A mother is breastfeeding her newborn son and is experiencing signs of her breasts feeling tender and full in between infant feedings. She asks if there are any suggestions that you can provide to help alleviate this physical complaint. The ideal nursing response would be to a. tell the patient to wear a bra at all times to provide more support to breast tissue. b. have the patient put the infant to her breast more frequently. c. place ice packs on breast tissue after infant feeding. d. explain that this is a normal finding and will resolve as her breast tissue becomes more used to nursing. ANS: B The patient may be experiencing the signs of engorgement. Intervention methods such as placing the infant to feed more frequently may help prevent physical complaints of tenderness to milk accumulation. Wearing a bra at all times will not help resolve engorgement issues but can provide comfort. Ice packs provide symptomatic relief but do not resolve engorgement issues. Warm water compresses are more likely to provide comfort. Engorgement is not a normal finding but is a common presentation in nursing mothers. These symptoms will not dissipate with continuation of breastfeeding. DIF: Cognitive Level: Application OBJ: Nursing Process Step: Evaluation MSC: Patient Needs: Health Promotion and Maintenance/Ante/Intra/Postpartum and Newborn Care 21. A mother is attempting to breastfeed her infant in the hospital setting. The infant is sleepy and displays some audible swallowing, the maternal nipples are flat, and the breasts are soft. The nurse has attempted to teach the mother positioning on one side, and now the mother wants to place the infant to the breast on the other side. Based on LATCH scores, what score would the nurse assign to this feeding session? a. 10 and document findings in the chart. b. 6 and further teach and assist the mother in feeding activities. c.5 and tell the mother to discontinue feeding attempts at this time because the infant is too sleepy. d. 8 and no further assistance is needed for feeding. ANS: B The LATCH assessment tool is used to identify whether mothers need additional instruction in the area of breastfeeding. The LATCH categories are latch, audible communication/swallowing, type of nipple, comfort of breasts, and holding position of infant. The assessment data reveal a score of 6 (0 + 2 + 1 + 2 + 1). The mother will need additional assistance during breastfeeding at this time. DIF: Cognitive Level: Analysis OBJ: Nursing Process Step: Evaluation MSC: Patient Needs: Health Promotion and Maintenance/Ante/Intra/Postpartum and Newborn Care 22. A mother conveys concern over the fact that she is not certain if her newborn is receiving enough nutrients from breastfeeding. This is the baby‘s first clinic visit after birth. What information can you provide that will help alleviate her fears regarding nutrient status for her newborn? a. Monitor the infant‘s output; as long as at least six or more diapers are changed in a 24-hour period, the baby is receiving sufficient intake. b. Tell the mother that if a baby is satisfied with feeding, she or he will be content and not fussy. c. Tell the mother that breast milk contains everything required for the infant and not to worry about nutrition. d. Provide nutrition information in the form of pamphlets for the mother to take home with her so that she uses them as a point of reference. ANS: A The presence of wet diapers confirms that the infant is receiving enough milk. Recording weight and seeing an increase in weight is also an objective finding that can be used to note nutritional status. Newborns may be fussy and still be receiving adequate nutrition. Although breast milk is potentially the perfect food for the newborn, not everyone‘s breast milk has the same nutrient quality, therefore recording of weight gain and output measurements (wet diapers and stool production) confirm nutritional status. Providing the mother with educational pamphlets may be advisable; however, does not address the immediate problem. DIF: Cognitive Level: Application OBJ: Nursing Process Step: Evaluation MSC: Patient Needs: Health Promotion and Maintenance/Ante/Intra/Postpartum and Newborn Care 23. A breastfeeding mother asks the postpartum nurse if any supplementation is necessary once her breast milk comes in. What is the nurse‘s most appropriate response? a. ―Are you concerned about your ability to adequately nurse your baby?‖ b. ―Do you eat a well-balanced diet, high in protein and carbohydrates?‖ c. ―Breast milk is low in vitamin D and supplementation with 400 IU is recommended.‖ d. ―Your breast milk has all the vitamins and will adequately meet your baby‘s needs.‖ ANS: CGenerally, nutrients provided in breast milk are present in amounts and proportions needed by the infant. However, recent studies have shown that the vitamin D content of breast milk is low, and daily supplementation with 400 IU of vitamin D is recommended within the first few days of life. Breastfeeding infants who are not exposed to the sun and those with dark skin are particularly at risk for insufficient vitamin D. Formula-fed infants who drink less than 1 quart of vitamin D–fortified milk per day should also be supplemented. Although the fatty acid content of breast milk is influenced by the mother‘s diet, malnourished mothers‘ milk has about the same proportions of total fat, protein, carbohydrates, and most minerals as milk from those who are well nourished. Levels of water-soluble vitamins in breast milk are determined by the mother‘s intake. It is important for breastfeeding women to eat a well-balanced diet to maintain their own health and energy levels. DIF: Cognitive Level: Application OBJ: Nursing Process Step: Implementation MSC: Patient Needs: Health Promotion and Maintenance 24. A new mother is preparing for discharge. She plans on bottle feeding her baby. Which statement indicates to the nurse that the mom needs more information about bottle feeding? a. ―I should encourage my baby to consume the entire amount of formula prepared for each feeding.‖ b. ―I can make up a 24-hour supply of formula and refrigerate the bottles so I am ready to feed my baby.‖ c. ―I will hold my baby in a cradle hold and alternate sides from left to right when I feed my baby.‖ d. ―I will generally feed my baby every 3 to 4 hours or more often as signs of hunger are displayed.‖ ANS: A Infants will stop suckling when they are full. Encouraging them to overeat may lead to problems with regurgitation and possible aspiration. The mother can prepare a single bottle or a 24-hour supply if adequate refrigeration is available. Show the parents how to position the infant in a semi-upright position, such as the cradle hold. This allows them to hold the infant close in a face-to-face position. The bottle is held with the nipple kept full of formula to prevent excessive swallowing of air. Placing the infant in the opposite arm for each feeding provides varied visual stimulation during feedings. Feed the infant every 3 to 4 hours but avoid rigid scheduling and take cues from the infant. DIF: Cognitive Level: Analysis OBJ: Nursing Process Step: Evaluation MSC: Patient Needs: Health Promotion and Maintenance 25. The nurse is teaching a postpartum patient different holds for breastfeeding. Which of the following figures depicts the football hold frequently used for patients who have had a cesarean birth? a. b. c. d. ANS: B For the football or clutch hold, the mother supports the infant‘s head and neck in her hand, with the infant‘s body resting on pillows next to her hip. This method allows the mother to see the position of the infant‘s mouth on the breast, helps her control the infant‘s head, and is especially helpful for mothers with heavy breasts. This hold also avoids pressure against an abdominal incision. For the cradle hold, the mother positions the infant‘s head at or near the antecubital space and level with her nipple, with her arm supporting the infant‘s body. Her other hand is freeto hold the breast. The cross-cradle or modified cradle hold is helpful for infants who are preterm or have a fractured clavicle. The mother holds the infant‘s head with the hand opposite the side on which the infant will feed and supports the infant‘s body across her lap with her arm. The other hand holds the breast. The side-lying position avoids pressure on the episiotomy or abdominal incision and allows the mother to rest while feeding. DIF: Cognitive Level: Analysis OBJ: Nursing Process Step: Implementation MSC: Patient Needs: Physiologic Integrity MULTIPLE RESPONSE 1. Late in pregnancy, the patient‘s breasts should be evaluated by the nurse to identify any potential concerns related to breastfeeding. Which of the following nipple conditions make it necessary to intervene prior to birth. (Select all that apply.) a. Flat nipples b. Cracked nipples c. Everted nipples d. Inverted nipples e. Nipples that contract when compressed ANS: A, D, E Flat nipples appear soft, like the areola, and do not stand erect unless stimulated by rolling them between the fingers. Inverted nipples are retracted into the breast tissue. These nipples appear normal; however, they will draw inward when the areola is compressed by the infant‘s mouth. Dome-shaped devices known as breast shells can be worn during the last weeks of pregnancy and between feedings after birth. The shells are placed inside the bra, with the opening over the nipple. The shells exert slight pressure against the areola to help the nipples protrude. The helpfulness of breast shells has been debated. A breast pump can be used to draw the nipples out before feedings after birth. Everted nipples protrude and are normal. No intervention will be required. Cracked, blistered, and bleeding nipples occur after breastfeeding has been initiated and are the result of improper latching on. The infant should be repositioned during feeding. The application of colostrum and breast milk after feedings will aid in healing. DIF: Cognitive Level: Application OBJ: Nursing Process Step: Assessment MSC: Patient Needs: Health Promotion and Maintenance 2. For which infant should the nurse anticipate the use of soy formula? (Select all that apply.) a. Preterm infant b. Infant with galactosemia c. Infant with phenylketonuria d. Infant with lactase deficiency e. Infant with a malabsorption disorder ANS: B, D, E Soy formula may be given to infants with galactosemia or lactase deficiency or those whose families are vegetarians. Soy milk is derived from the protein of soybeans and supplemented with amino acids. The formulas are also used for infants with malabsorption disorders. The preterm infant may require a more concentrated formula, with more calories in less liquid.Modifications of other nutrients are also made. Human milk fortifiers can be added to breast milk to adapt it for preterm infants. Low-phenylalanine formulas are needed for infants with phenylketonuria, a deficiency in the enzyme to digest phenylalanine found in standard formulas. DIF: Cognitive Level: Analysis OBJ: Nursing Process Step: Planning MSC: Patient Needs: Physiologic Integrity 3. A new mother asks the nurse, ―How will I know early signs of hunger in my baby?‖ The nurse‘s best response is which of the following? (Select all that apply.) a. Crying b. Rooting c. Lip smacking d. Decrease in activity e. Sucking on the hands ANS: B, C, E Early signs of hunger in a baby are rooting, lip smacking, and sucking on the hands. Crying is a late sign, and the baby‘s activity will increase, not decrease. DIF: Cognitive Level: Application OBJ: Nursing Process Step: Implementation MSC: Patient Needs: Health Promotion and Maintenance Chapter 23: High-Risk Newborn: Complications Associated with Gestational Age and Development Foundations of Maternal-Newborn & Women‘s Health Nursing, 7th Edition MULTIPLE CHOICE 1. Which is the most useful factor in preventing premature birth? a. High socioeconomic status b. Adequate prenatal care c. Aid to Families with Dependent Children d. Women, Infants, and Children (WIC) nutritional program ANS: B Prenatal care is vital for identifying possible problems. People with higher socioeconomic status are more likely to seek adequate prenatal care, which is the most helpful for prevention of premature births. Lower socioeconomic groups do not seek out health care, which puts them at risk for preterm labor. Aid to Families with Dependent Children and WIC assist in the nutritional status of the pregnant woman; however, the most helpful intervention for the prevention of premature births is adequate prenatal care. DIF: Cognitive Level: Understanding OBJ: Nursing Process Step: Assessment MSC: Patient Needs: Health Promotion and Maintenance 2. In comparison with the term infant, the preterm infant has a. more subcutaneous fat. b. well-developed flexor muscles. c.few blood vessels visible through the skin. d. greater surface area in proportion to weight. ANS: D Preterm infants have greater surface area in proportion to their weight. More subcutaneous fat, well-developed flexor muscles, and few blood vessels visible through the skin are features that are more characteristic of a term infant. DIF: Cognitive Level: Analysis OBJ: Nursing Process Step: Assessment MSC: Patient Needs: Physiologic Integrity 3. Decreased surfactant production in the preterm lung is a problem because a. surfactant keeps the alveoli open during expiration. b. surfactant causes increased permeability of the alveoli. c. surfactant dilates the bronchioles, decreasing airway resistance. d. surfactant provides transportation for oxygen to enter the blood supply. ANS: A Surfactant prevents the alveoli from collapsing each time the infant exhales, thus reducing the work of breathing. It does not affect the bronchioles. By keeping the alveoli open, surfactant permits enhanced oxygen exchange. Infants treated with surfactant have higher survival rates. DIF: Cognitive Level: Understanding OBJ: Nursing Process Step: Assessment MSC: Patient Needs: Physiologic Integrity 4. A preterm infant is on a ventilator, with intravenous lines and other medical equipment. When the parents come to visit for the first time, what is the most important action by the nurse? a. Encourage the parents to touch their infant. b. Reassure the parents that the infant is progressing well. c. Discuss the care they will give their infant when the infant goes home. d. Suggest that the parents visit for only a short time to reduce their anxiety. ANS: A Touching the infant will increase the development of attachment. As the infant‘s condition improves the parents should be encouraged to provide Kangaroo care. It is important to keep the parents informed regarding the infant‘s progress; however, the nurse needs to be honest with the explanations. Discussing home care is an important part of parent teaching, although is not the most important priority during the first visit. Parents should be encouraged to visit for as long as they are comfortable. DIF: Cognitive Level: Application OBJ: Nursing Process Step: Implementation MSC: Patient Needs: Psychosocial Integrity 5. Which preterm infant should receive gavage feedings instead of bottle feedings? a. Sucks on a pacifier during gavage feedings b. Sometimes gags when a feeding tube is inserted c. Has a sustained respiratory rate of 70 breaths per minute d.Has an axillary temperature of 36.9C (98.4F), an apical pulse of 149 beats/minute, and respirations of 54 breaths per minute ANS: C Infants less than 34 weeks of gestation or those who weigh less than 1500 g generally have difficulty with bottle-feeding. Gavage feedings should be initiated if the respiratory rate is above 60 breaths per minute. Providing a pacifier during gavage feedings gives positive oral stimulation and helps the infant associate the comfortable feeling of fullness with sucking. The presence of the gag reflex is important before initiating bottle-feeding. Axillary temperature of 36.9C (98.4F), an apical pulse of 149 beats/minute, and respirations of 54 breaths per minute are within expected limits and an indication that the infant is not having respiratory problems at that time. DIF: Cognitive Level: Analysis OBJ: Nursing Process Step: Evaluation MSC: Patient Needs: Physiologic Integrity 6. Overstimulation may cause increased oxygen use in a preterm infant. Which nursing intervention helps to avoid this problem? a. Group all care activities together to provide long periods of rest. b. Keep charts on top of the incubator so the nurses can write on them there. c. While giving a report to the next nurse, stand in front of the incubator and talk softly about how the infant responds to stimulation. d. Teach the parents signs of overstimulation, such as turning the face away or stiffening and extending the extremities and fingers. ANS: D Parents should be taught the signs of overstimulation so they will learn to adapt their care to the needs of their infant. Grouping care activities may under stimulate the infant during those long periods and overtire the infant during the procedures. Keeping charts on the incubator and giving the report in front of the incubator may cause overstimulation. Any clip boards or binders in use should be kept at the desk, never on top of the incubator. DIF: Cognitive Level: Application OBJ: Nursing Process Step: Implementation MSC: Patient Needs: Health Promotion and Maintenance 7. A characteristic of a post-term infant who weighs 7 lb, 12 oz, and who lost weight in utero, is a. soft and supple skin. b. a hematocrit level of 55%. c. lack of subcutaneous fat. d. an abundance of vernix caseosa. ANS: C This post-term infant has actually lost weight in utero, which is seen as loss of subcutaneous fat. The skin is normally wrinkled, cracked, and peeling. A hematocrit of 55% is within the expected range of all newborns. There is no vernix caseosa in a post-term infant. DIF: Cognitive Level: Understanding OBJ: Nursing Process Step: Assessment MSC: Patient Needs: Physiologic Integrity 8. In caring for the preterm infant, which complication is thought to be a result of high arterial blood oxygen level? a.Necrotizing enterocolitis (NEC) b. Retinopathy of prematurity (ROP) c. Intraventricular hemorrhage (IVH) d. Bronchopulmonary dysplasia (BPD) ANS: B ROP is thought to occur as a result of high levels of oxygen in the blood. NEC is due to the interference of blood supply to the intestinal mucosa. Necrotic lesions occur at that site. IVH is caused by rupture of the fragile blood vessels in the ventricles of the brain. It is most often associated with hypoxic injury, increased blood pressure, and fluctuating cerebral blood flow. BPD is caused by the use of positive-pressure ventilation against the immature lung tissue. DIF: Cognitive Level: Analysis OBJ: Nursing Process Step: Evaluation MSC: Patient Needs: Physiologic Integrity 9. In caring for the post-term infant, thermoregulation can be a concern, especially in an infant who also has a(n) a. hematocrit level of 58%. b. RBC count of 5 million/mcL. c. WBC count of 15,000 cells/mm3. d. blood glucose level of 25 mg/dL. ANS: D Because glucose is necessary to produce heat, the infant who is also hypoglycemic will not be able to produce enough body heat. A hematocrit level of 58% is within the expected range for newborns. WBC count may be as high as 30,000 cells/mm3. RBC count ranges from 3.9 to 5.5 million/mcL. DIF: Cognitive Level: Understanding OBJ: Nursing Process Step: Planning MSC: Patient Needs: Physiologic Integrity 10. Which statement regarding newborns classified as small for gestational age (SGA) is accurate? a. They weigh less than 2500 g. b. They are born before 38 weeks of gestation. c. They are below the tenth percentile on gestational growth charts. d. Placental malfunction is the only recognized cause of this condition. ANS: C SGA infants are defined as below the tenth percentile in growth when compared with other infants of the same gestational age. SGA is not defined by weight. Infants born before 38 weeks are classified as preterm. There are many factors that contribute to the development of an SGA infant, not just placental malfunction. DIF: Cognitive Level: Understanding OBJ: Nursing Process Step: Assessment MSC: Patient Needs: Health Promotion and Maintenance 11. Which nursing action is especially important for an SGA newborn? a.Promote bonding. b. Observe for and prevent dehydration. c. Observe for respiratory distress syndrome. d. Prevent hypoglycemia with early and frequent feedings. ANS: D The SGA infant has poor glycogen stores and is subject to hypoglycemia. Promoting bonding is a concern for all infants and is not specific to SGA infants. Dehydration is a concern for all infants and is not specific to SGA infants. Respiratory distress syndrome is most commonly seen in preterm infants. DIF: Cognitive Level: Understanding OBJ: Nursing Process Step: Implementation MSC: Patient Needs: Physiologic Integrity 12. What will the nurse note when assessing an infant with asymmetric intrauterine growth restriction? a. All body parts appear proportionate. b. The extremities are disproportionate to the trunk. c. The head seems large compared with the rest of the body. d. One side of the body appears slightly smaller than the other. ANS: C In asymmetric intrauterine growth restriction, the head is normal in size; but, appears large because the infant‘s body is long and thin due to lack of subcutaneous fat. The left and right side growth should be symmetric. With asymmetric intrauterine growth restrictions, the body appears smaller than normal compared to the head. The body parts are out of proportion, with the body looking smaller than anticipated. The body, arms, and legs have lost subcutaneous fat so they will look small compared with the head. DIF: Cognitive Level: Understanding OBJ: Nursing Process Step: Assessment MSC: Patient Needs: Physiologic Integrity 13. Which data should alert the nurse caring for an SGA infant that additional calories may be needed? a. The latest hematocrit was 53%. b. The infant‘s weight gain is 40 g/day. c. The infant is taking 120 mL/kg every 24 hours. d. Three successive temperature measurements were 36.1C, 35.5C, and 36.1C (97, 96, and 97F). ANS: D Low body temperature indicates that additional calories are needed to maintain body temperature. The hematocrit is within the expected range for a newborn. A weight gain of about 20 g/day is expected. Preterm SGA infants need about 120 kcal/kg/day. DIF: Cognitive Level: Analysis OBJ: Nursing Process Step: Evaluation MSC: Patient Needs: Physiologic Integrity 14. Which statement regarding large-for-gestational age (LGA) infants is most accurate? a.They weigh more than 3500 g. b. They are above the 80th percentile on gestational growth charts. c. They are prone to hypoglycemia, polycythemia, and birth injuries. d. Postmaturity syndrome is the most common complication. ANS: C Hypoglycemia, polycythemia, and birth injuries are all common in LGA infants. LGA infants are determined by their weight compared to their age. They are above the 90th percentile on gestational growth charts. Postmaturity syndrome is not an expected complication with LGA infants. DIF: Cognitive Level: Understanding OBJ: Nursing Process Step: Assessment MSC: Patient Needs: Physiologic Integrity 15. Following the vaginal birth of a macrosomic infant, the nurse should evaluate the infant for a. hyperglycemia. b. clavicle fractures. c. hyperthermia. d. an increase in red blood cells. ANS: B Macrosomic infants may have a complicated birth and are susceptible to birth injuries, such as fractured clavicles, cephalohematomas, and brachial palsy. A macrosomic infant would have the potential to become hypoglycemic and would be at risk for hypothermia. An increase in red blood cells would not be the priority assessment for a macrosomic infant. DIF: Cognitive Level: Application OBJ: Nursing Process Step: Assessment MSC: Patient Needs: Physiologic Integrity 16. An infant delivered prematurely at 28 weeks‘ gestation weighs 1200 g. Based on this information the infant is classified as a. SGA. b. VLBW. c. ELBW. d. low birth weight at term. ANS: B VLBW (very-low-birth-weight) infants weigh 1500 g or less at birth. SGA infants fall below the tenth percentile in growth charts. ELBW (extremely low-birth-weight) infants weigh 100 g or less at birth. Low birth weight pertains to an infant weighing 2500 g or less at birth. This option is incorrect because it specifies at term and the infant in question is designated as preterm at 28 weeks‘ gestation. DIF: Cognitive Level: Application OBJ: Nursing Process Step: Assessment MSC: Patient Needs: Health Promotion and Maintenance/Ante/Intra/Postpartum and Newborn Care17. The nurse is observing a 38-week gestation newborn in the nursery. Data reveals periods of apnea lasting approximately 10 seconds followed by a period of rapid respirations. The infant‘s color and heart rate remain unchanged. The nurse suspects that the infant a. is exhibiting signs of RDS. b. requires tactile stimulation around the clock to ensure that apneic periods do not progress further. c. is experiencing periodic breathing episodes and will require continuous monitoring while in the nursery unit. d. requires the use of CPAP to promote airway expansion. ANS: C Periodic breathing can occur in term or preterm infants; it consists of periods of breathing cessation (5 to 10 seconds) followed by a period of increased respirations (10 to 15 breaths per minute). It is not associated with any color or heart rate changes. Infants who exhibit this pattern should continue to be observed. There is no clinical evidence that the infant is exhibiting signs of respiratory distress syndrome (RDS). There is no indication that a pattern of tactile stimulation should be initiated. Continuous positive airway pressure (CPAP) and tactile stimulation would be indicated if the infant were to have apneic spells. DIF: Cognitive Level: Application OBJ: Nursing Process Step: Planning MSC: Patient Needs: Health Promotion and Maintenance/Ante/Intra/Postpartum and Newborn Care 18. Which nursing diagnosis would be considered a priority for a newborn infant who is receiving phototherapy in an isolette? a. Hypothermia because of phototherapy treatment b. Impaired skin integrity related to diarrhea as a result of phototherapy c. Fluid volume deficit related to phototherapy treatment d. Knowledge deficit (parents) related to initiation of medical therapy ANS: C Infants who undergo phototherapy as a result of the medical diagnosis of hyperbilirubinemia are at risk for hyperthermia, not hypothermia. Although impaired skin integrity can occur, the priority nursing diagnosis focuses on the physiologic effects of fluid volume deficit. The infant is losing fluid via insensible losses, increased output (in the form of diarrhea), and limited intake. Lack of knowledge is a pertinent nursing diagnosis for these parents; but, physiologic needs take precedence at this time. DIF: Cognitive Level: Analysis OBJ: Nursing Process Step: Nursing Diagnosis MSC: Patient Needs: Health Promotion and Maintenance/Ante/Intra/Postpartum and Newborn Care 19. An infant presents with lethargy in the newborn nursery on the second day of life. On further examination, vital signs are stable and muscle tone is slightly decreased, with sluggish reflexes noted. Other physical characteristics are noted as being normal. Lab tests reveal a decreased hematocrit and increased blood sugar. The nurse suspects that the infant may be exhibiting signs and symptoms of a. RDS.b. PIVH. c. BPD. d. ROP. ANS: B IVH or PIVH (intraventricular hemorrhage or periventricular hemorrhage) can be seen during the first week of life. Signs and symptoms are based on the extent of hemorrhage. Typically, one would see lethargy, decreased muscle tone and reflexes, decreased hematocrit, hyperglycemia, acidosis, and seizures. If the newborn had RDS or BPD, there would be more respiratory symptoms exhibited. If the infant had ROP, there would be signs and symptoms related to the eyes. Other physical characteristics are reported as being normal. DIF: Cognitive Level: Analysis OBJ: Nursing Process Step: Evaluation MSC: Patient Needs: Physiologic Integrity/Physiologic Adaptation 20. Following a traumatic birth of a 10-lb infant, the nurse should evaluate a. gestational age status. b. flexion of both upper extremities. c. infant‘s percentile on growth chart. d. blood sugar to detect hyperglycemia. ANS: B Large infants are at risk for shoulder dystocia, which may result in clavicle fracture or damage to the brachial plexus. Gestational age or the infant‘s growth chart percentile will not provide data about potential injuries from a traumatic birth. A large infant is at increased risk for hypoglycemia. DIF: Cognitive Level: Application OBJ: Nursing Process Step: Implementation MSC: Patient Needs: Health Promotion and Maintenance 21. A newborn assessment finding that would support the nursing diagnosis of postmaturity would be a. loose skin. b. ruddy skin color. c. presence of vernix. d. absence of lanugo. ANS: A Decreased placental function because of a prolonged pregnancy results in loss of subcutaneous tissue in the neonate, which is evidenced by loose skin. Ruddy skin color, presence of vernix, and absence of lanugo do not indicate a postmature infant. DIF: Cognitive Level: Analysis OBJ: Nursing Process Step: Assessment MSC: Patient Needs: Physiologic Integrity MULTIPLE RESPONSE 1. Because late preterm infants are more stable than early preterm infants, they may receive care that is much like that of a full-term baby. The mother-baby or nursery nurse knows that these infants are at increased risk for which of the following? (Select all that apply.)a. Sepsis b. Hyperglycemia c. Hyperbilirubinemia d. Cardiac distress e. Problems with thermoregulation ANS: A, C, E Sepsis, hyperbilirubinemia, and problems with thermoregulation are all conditions that are related to immaturity and warrant close observation. After discharge, the infant is at risk for rehospitalization related to these problems. The Association of Women‘s Health, Obstetric and Neonatal Nurses (AWHONN) has launched the Near-Term Infant Initiative to study the problem and determine ways to ensure that these infants receive adequate care. The nurse should ensure that this infant is feeding adequately before discharge and that parents are taught the signs and symptoms of these complications. These infants are at risk for respiratory distress and hypoglycemia. DIF: Cognitive Level: Analysis OBJ: Nursing Process Step: Evaluation MSC: Patient Needs: Health Promotion and Maintenance Chapter 24: High-Risk Newborn: Acquired and Congenital Conditions Foundations of Maternal-Newborn & Women‘s Health Nursing, 7th Edition MULTIPLE CHOICE 1. The nurse is responsible for monitoring the feedings of the infant with hyperbilirubinemia every 2 to 3 hours around the clock. If breastfeeding must be supplemented, formula should be used instead of water. The purpose of this plan is to a. prevent hyperglycemia. b. provide fluids and protein. c. decrease gastrointestinal motility. d. prevent rapid emptying of the bilirubin from the bowel. ANS: B Proteins help maintain the albumin level in the blood, and the extra fluids help eliminate the excess bilirubin from the infant‘s system. Feedings every 2 hours will help prevent hypoglycemia. Increased gastrointestinal motility can facilitate the prompt emptying of the bilirubin from the bowel. Breast milk or formula is more effective in promoting stooling and removal of bilirubin. DIF: Cognitive Level: Application OBJ: Nursing Process Step: Implementation MSC: Patient Needs: Physiologic Integrity 2. Four hours after the birth of a healthy neonate of an insulin-dependent (type 1) diabetic mot 9. When a cardiac defect causes the mixing of arterial and venous blood in the right side of the heart, the nurse might expect to find a. cyanosis. b. diuresis. c. signs of congestive heart failure. d. increased oxygenation of the tissues. ANS: C Mixing of the blood in the right side of the heart will cause excessive blood flow to the lungs and pulmonary congestion and congestive heart failure. Cyanosis is seen more frequently with rightto-left shunts. Diuresis is not a common finding with cardiac defects. Increased oxygenation of the tissues is not seen with this type of cardiac defect. DIF: Cognitive Level: Analysis OBJ: Nursing Process Step: Assessment MSC: Patient Needs: Physiologic Integrity 10. In an infant with cyanotic cardiac anomaly, the nurse should expect to see a. feedings taken eagerly. b. a consistent and rapid weight gain. c. a decrease in the heart rate with activity.d. little to no improvement in color with oxygen administration. ANS: D With a cyanotic cardiac defect, the shunting of blood is right to left, so there is little if any improvement in the oxygenation of the blood with the administration of oxygen. Infants with cardiac anomalies are usually difficult feeders, have difficulty gaining weight, and have an increase in the heart rate with activity. DIF: Cognitive Level: Application OBJ: Nursing Process Step: Assessment MSC: Patient Needs: Physiologic Integrity 11. The difference between nonphysiologic jaundice (pathologic jaundice) and physiologic jaundice is that nonphysiologic jaundice a. may result in kernicterus. b. appears during the first 24 hours of life. c. begins on the head and progresses down the body. d. results from the breakdown of excessive erythrocytes not needed after birth. ANS: B Nonphysiologic jaundice appears during the first 24 hours of life, whereas physiologic jaundice appears after the first 24 hours of life. This type of jaundice may lead to kernicterus; however, screening and appropriate treatment needs to take place in a time sensitive manner in order to prevent kernicterus. Jaundice proceeds from the head down. Both jaundices are the result of the breakdown of erythrocytes. Nonphysiologic jaundice is caused by an underlying condition, such as Rh incompatibility. DIF: Cognitive Level: Understanding OBJ: Nursing Process Step: Assessment MSC: Patient Needs: Health Promotion and Maintenance 12. Shortly after a cesarean birth, a newborn begins to exhibit difficulty breathing. Nasal flaring and slight retractions are noted. The newborn is admitted to the neonatal intensive care unit (NICU) for closer observation, with a diagnosis of transient tachypnea of the neonate (TTN). The parents are notified and become anxious because they have no understanding of what this means for their infant. The best action that the nurse can take at this time is to a. refer them to the neonatologist for more information. b. reassure them not to worry. The infant will be monitored closely by trained staff. c. explain to them that this often occurs following a birth and it will most likely resolve in the next 24 to 48 hours. d. tell them that they will be able to come and see their baby, which will help make calm their anxiety. ANS: C The clinical diagnosis of TTN has been established, and the nurse should provide factual information relative to the clinical condition. The RN should be able to provide information to clarify the parents‘ concern without referral to the pediatric provider. Telling parents not to worry usually has the opposite effect in terms of a medical crisis. Facilitating an interaction with the newborn and parents may help ease anxiety; however, this does not address the parents‘ knowledge deficit. DIF: Cognitive Level: Analysis OBJ: Nursing Process Step: EvaluationMSC: Patient Needs: Safe and Effective Care Environment/Establishing Priorities 13. While in utero, the fetus passes meconium. Based on this assessment, which nursing diagnosis takes priority for the newborn at birth? a. Risk for infection related to release of meconium b. Risk for injury related to high-risk birth interventions, such as amino infusion c. Risk for aspiration related to retained secretions d. Risk for thermoregulation because of high-risk labor status ANS: C Because the fetus has already passed meconium in utero, the labor and birth assume a highrisk management perspective. The likelihood that the infant will develop meconium aspiration syndrome (MAS) is increased, therefore airway complications take precedence in terms of nursing diagnosis and medical management. DIF: Cognitive Level: Application OBJ: Nursing Process Step: Nursing Diagnosis MSC: Patient Needs: Safe and Effective Care Environment/Establishing Priorities 14. Which diagnostic test is used to help confirmation of hyperbilirubinemia in an infant? a. Direct Coombs test based on maternal blood sample b. Indirect Coombs test based on infant cord blood sample c. Infant bilirubin level d. Maternal blood type ANS: C The direct Coombs test is based on cord blood drawn from the infant, whereas the indirect Coombs test is based on maternal blood samples. Although maternal blood type is important in determining whether there is a potential ABO incompatibility, the infant‘s bilirubin level provides the best evidence of whether the infant has hyperbilirubinemia or pathologic jaundice. DIF: Cognitive Level: Analysis OBJ: Nursing Process Step: Planning MSC: Patient Needs: Physiologic Integrity/Reduction of Risk Potential 15. Which of the following lab values indicates that an infant may have polycythemia? a. Hct 50% b. Hct 55% c. Hct 62% d. Hct 70% ANS: D The presence of polycythemia in an infant is characterized by a hematocrit value greater than 65%. DIF: Cognitive Level: Application OBJ: Nursing Process Step: Assessment MSC: Patient Needs: Physiologic Integrity/Reduction of Risk Potential 16. The nurse notes that the infant has been feeding poorly over the last 24 hours. The nurse should immediately assess for other signs of a.hyperglycemia. b. neonatal infection. c. hemolytic anemia. d. increased bilirubin levels. ANS: B Signs of neonatal infection (sepsis) in the newborn are subtle. Temperature instability, respiratory problems, and changes in feeding habits may be common. Hyperglycemia, hemolytic anemia, and increased bilirubin levels are not associated with poor infant feeding. DIF: Cognitive Level: Application OBJ: Nursing Process Step: Analysis MSC: Patient Needs: Health Promotion and Maintenance 17. The priority assessment for the Rh-negative infant whose mother‘s indirect Coombs test was positive at 36 weeks is a. skin color. b. temperature. c. respiratory rate. d. blood glucose level. ANS: A An Rh-negative infant whose mother was sensitized during the current pregnancy will have decreased red blood cells (RBCs) and exhibit skin pallor due to erythroblastosis fetalis. The temperature, respiratory rate, and blood glucose level are not assessments associated specifically to an infant with an Rh incompatibility issue. DIF: Cognitive Level: Application OBJ: Nursing Process Step: Analysis MSC: Patient Needs: Physiologic Integrity 18. The nurse should be alert to a blood group incompatibility if a. both mother and infant are O-positive. b. mother is A-positive and infant is A-negative. c. mother is O-positive and infant is B-negative. d. mother is B-positive and infant is O-negative. ANS: D Blood group incompatibilities occur because O-positive mothers who have natural antibodies to type A or B blood. When mother and infant both have blood group O or A, no incompatibility exists. The mother with blood group B does not have any antibodies to group O. DIF: Cognitive Level: Analysis OBJ: Nursing Process Step: Analysis MSC: Patient Needs: Health Promotion and Maintenance MULTIPLE RESPONSE 1. Infection can be transmitted to the neonate from mother during the pregnancy or birth or from the mother, family members, visitors, or agency staff after birth. Which viral infections are most likely to be transmitted during the birth process? (Select all that apply.) a. Hepatitis Bb. Rubella c. Herpes d. Varicella Zoster e. Cytomegalovirus ANS: A, C, E Hepatitis B, herpes, cytomegalovirus, and HIV are usually transmitted during birth; however, they can also be acquired through transplacental transfer or from breast milk. Rubella and varicella zoster (Chickenpox) are acquired in utero and may result in fetal death or significant abnormalities. DIF: Cognitive Level: Analysis OBJ: Nursing Process Step: Evaluation MSC: Patient Needs: Health Promotion and Maintenance 2. The drug-exposed infant often presents with irritability, frantic crying, and is difficult to console. Which nursing measures can be used to prevent this behavior in this high-risk infant? (Select all that apply.) a. Keep the room well lit. b. Swaddle the infant. c. Rock slowly and gently. d. Coo softly and gently. e. Avoid pacifier use. ANS: B, C, D Comfort measures that will assist in consoling this infant and prevent crying include: swaddling, providing a pacifier, slow and smooth rocking in a vertical or horizontal position, cooing, gently stroking the back, keeping the room fairly dark, and avoiding both auditory and visual stimulation. These infants are particularly sensitive to light and should be placed in a darker corner of the nursery or have the lights in their room kept low. Pacifier use will assist the baby in meeting non-nutritive sucking needs and provide a method to self soothe. DIF: Cognitive Level: Application OBJ: Nursing Process Step: Evaluation MSC: Patient Needs: Health Promotion and Maintenance Chapter 25: Family Planning Foundations of Maternal-Newborn & Women‘s Health Nursing, 7th Edition MULTIPLE CHOICE 1. Which contraceptive method provides protection against sexually transmitted diseases? a. Oral contraceptives b. Tubal ligation c. Male or female condoms d. Intrauterine device (IUD) ANS: CBecause latex condoms provide the best protection available, they should be used during any potential exposure to a sexually transmitted disease (STD). Only the barrier methods provide some protection from STDs. A tubal ligation is considered a permanent contraceptive method; however, does not offer any protection against sexually transmitted diseases. IUDs are inserted into the uterus and provide no protection from STDs. PTS: 1 DIF: Cognitive Level: Understanding OBJ: Nursing Process Step: Assessment MSC: Patient Needs: Health Promotion and Maintenance 2. A nurse is leading a discussion regarding options for birth control. Which of the following methods is considered the most reliable? a. Coitus interruptus b. Breastfeeding c. Natural family planning d. Intrauterine device ANS: D IUDs are 99% effective. Although coitus interruptus is considered 78% effective, it requires great control by the man and may be unsatisfying for both partners. Women who exclusively breast feed (without formula or solid food supplementation) may avoid ovulation. Another method of birth control should be used when the frequency of breastfeeding decreases. At 6 months postpartum ovulation commences in most women, even if exclusively breastfeeding. Couples must be highly motivated to use natural family planning because they must abstain from sex for as much as half of the menstrual cycle. Errors in the forbidden time carry a very high risk of pregnancy. PTS: 1 DIF: Cognitive Level: Understanding OBJ: Nursing Process Step: Assessment MSC: Patient Needs: Health Promotion and Maintenance 3. Which patient is a safe candidate for the use of oral contraceptives? a. 39-year-old with a history of thrombophlebitis. b. 16-year-old with a benign liver tumor. c. 20-year-old who suspects she may be pregnant. d. 43-year-old who does not smoke cigarettes. ANS: D Heavy cigarette smoking is a contraindication to oral contraceptive use due to risk of stroke. Oral contraceptives are contraindicated with a history of thrombophlebitis. Liver tumors, benign or malignant, preclude the use of oral contraceptives. Pregnancy is also a contraindication. PTS: 1 DIF: Cognitive Level: Understanding OBJ: Nursing Process Step: Assessment MSC: Patient Needs: Health Promotion and Maintenance 4. The role of the nurse in family planning is to a. refer the couple to a reliable physician. b. decide on the best method for the couple.c. advise couples on which contraceptive to use. d. educate couples on the various methods of contraception. ANS: D The nurse‘s role is to provide information to the couple so that they can make an informed decision about family planning. The nurse can assist the couple; they do not need to be sent to a physician for contraceptive education. The nurse‘s role is to evaluate the couple‘s knowledge base and educate regarding birth control options, not to decide which is the best contraceptive method for them to utilize. PTS: 1 DIF: Cognitive Level: Application OBJ: Nursing Process Step: Implementation MSC: Patient Needs: Health Promotion and Maintenance 5. Informed consent concerning contraceptive use is important since some of the methods a. may not be reliable. b. require a surgical procedure to insert. c. have potentially dangerous side effects. d. are invasive procedures that require hospitalization. ANS: C It is important for couples to be aware of potential side effects in order for them to make an informed decision regarding the use of contraceptives. Even if a method is less reliable, it still carries with it side effects that require informed consent. A written consent is required if the contraceptive choice involves a surgical procedure. Some contraceptive procedures are invasive; however, do not require hospitalization for insertion. PTS: 1 DIF: Cognitive Level: Understanding OBJ: Nursing Process Step: Assessment MSC: Patient Needs: Health Promotion and Maintenance 6. Which contraceptive method should be contraindicated in a patient with a history of toxic shock syndrome? a. Condom b. Spermicide c. Cervical cap d. Oral contraceptives ANS: C The cervical cap should not be used in women with a history of toxic shock syndrome. A condom, spermicide, or oral contraceptives are not contraindicated with a history of toxic shock syndrome. PTS: 1 DIF: Cognitive Level: Understanding OBJ: Nursing Process Step: Assessment MSC: Patient Needs: Health Promotion and Maintenance 7. When instructing a patient in the use of spermicidal foam or gel, it is important to include the information that a.effectiveness is about 85%. b. douching should be avoided for at least 6 hours. c. it should be inserted 1 to 2 hours before intercourse. d. one application is effective for several hours. ANS: B Douching within 6 hours of intercourse would remove the spermicide and increase the risk of pregnancy. Effectiveness of spermicidal foam or gel is only 72% when used alone. The spermicidal foam or gel should be inserted 15 minutes before intercourse. One application is effective for approximately 1 hour. PTS: 1 DIF: Cognitive Level: Application OBJ: Nursing Process Step: Assessment MSC: Patient Needs: Health Promotion and Maintenance 8. Which symptom in a patient using oral contraceptives should be reported to the physician immediately? a. 5-lb weight gain b. Leg pain and edema c. Decrease in menstrual flow d. Increased pigmentation of the face ANS: B Oral contraceptives increase clotting factors, which place the woman at risk for thrombophlebitis. Leg pain and edema are symptoms of thrombophlebitis. A 5-lb weight gain can be expected. A decrease in menstrual flow is an expected finding. Increased pigmentation of the face is a common finding. PTS: 1 DIF: Cognitive Level: Application OBJ: Nursing Process Step: Assessment MSC: Patient Needs: Health Promotion and Maintenance 9. When using the basal body temperature method of family planning, the woman should understand that a. she will remain fertile for 5 days after ovulation. b. she should take her temperature each night before going to bed. c. her temperature will increase about 0.2 to 0.4°C (0.4 to 0.8°F) after ovulation. d. her temperature is normally lower during the second half of her cycle. ANS: C The basal body temperature will increase approximately 0.2 to 0.4°C (0.4 to 0.8°F) at the same time ovulation occurs. The woman is fertile for up to 18 days. She should take her temperature at the same time upon rising in the morning. A woman‘s temperature is usually higher in the second half of her cycle. PTS: 1 DIF: Cognitive Level: Understanding OBJ: Nursing Process Step: Assessment MSC: Patient Needs: Health Promotion and Maintenance10. The major difference between the diaphragm and the cervical cap is that the diaphragm a. is more effective. b. requires spermicide. c. applies pressure on the urethra. d. has no contribution to toxic shock syndrome. ANS: C The diaphragm is made to fit snugly in the vaginal area and contains a hard rim that may exert pressure on the urethra. The cervical cap is smaller and fits around the cervix. The cervical cap is not more effective than a diaphragm. The diaphragm does not require spermicide. Both methods may contribute to the onset of toxic shock syndrome. PTS: 1 DIF: Cognitive Level: Analysis OBJ: Nursing Process Step: Evaluation MSC: Patient Needs: Health Promotion and Maintenance 11. The patient who has had an intrauterine device (IUD) inserted should be instructed to a. use a vinegar douche weekly for 4 weeks. b. have the IUD replaced every 2 to 4 years. c. check the placement of the string once a week for 4 weeks. d. use another method of contraception for 2 weeks after insertion. ANS: C Checking the placement of the string is necessary to determine whether the IUD is still correctly positioned. If the string is shorter or longer than when checked previously, the patient should contact her physician. A vinegar douche weekly for 4 weeks is not necessary. An IUD can be left in place for up to 10 years. A second method of contraception is not required after insertion of the IUD; it is effective immediately. PTS: 1 DIF: Cognitive Level: Application OBJ: Nursing Process Step: Implementation MSC: Patient Needs: Health Promotion and Maintenance 12. A male patient asks, ―Why do I have to use another contraceptive? I had a vasectomy last week.‖ The best response is a. ―A vasectomy is only 80% effective.‖ b. ―A vasectomy is not effective in all men.‖ c. ―Semen may contain sperm for 6 months following a vasectomy.‖ d. ―Complete sterilization doesn‘t occur until all sperm have left the system.‖ ANS: D It may take a month or longer for all sperm to be removed from the system. During that time, an additional method of contraception must be used. A vasectomy is 99% effective. Vasectomies have a high success rate; however, it may take 1 month for all the sperm to be removed from the system. PTS: 1 DIF: Cognitive Level: ApplicationOBJ: Nursing Process Step: Implementation MSC: Patient Needs: Health Promotion and Maintenance 13. A woman who has a successful career and a busy lifestyle will most likely look for which type of contraceptive? a. Requires extensive education to use b. Is the easiest and most convenient to use c. Costs the least d. Is permanent ANS: B A woman who has a busy lifestyle will probably have less time to devote to contraceptive use, therefore she requires something that is easy and convenient. Extra time for education would not be acceptable for this woman. Cost would probably not be a problem. There is no indication that a woman who has a successful career and a busy lifestyle wants contraception to be permanent, simply convenient. PTS: 1 DIF: Cognitive Level: Understanding OBJ: Nursing Process Step: Assessment MSC: Patient Needs: Health Promotion and Maintenance 14. The method of contraception that is considered the safest for women is a(n) a. IUD. b. diaphragm. c. male condom. d. oral contraceptive. ANS: C A male condom does not have any side effects or risk factors for the woman. Oral contraception, an IUD, and the diaphragm all have significant side effects or risk factors for the woman. PTS: 1 DIF: Cognitive Level: Understanding OBJ: Nursing Process Step: Assessment MSC: Patient Needs: Health Promotion and Maintenance 15. A patient is 27 years old and delivered her first baby yesterday. She and her husband do not want to have another baby for at least 3 to 4 years. The most appropriate method of birth control to meet their needs is a. withdrawal. b. fertility awareness method. c. combination of condoms and foam. d. vasectomy with a reversal in 3 years. ANS: C Of the methods listed, condoms and foam would be the best for this couple. Withdrawal is the least effective form of birth control. Fertility awareness is not that effective and an unwantedpregnancy could result. They want another child, therefore a vasectomy with a reversal would not be an appropriate option. PTS: 1 DIF: Cognitive Level: Application OBJ: Nursing Process Step: Planning MSC: Patient Needs: Health Promotion and Maintenance 16. The most appropriate statement for introducing the topic of family planning in the postpartum setting is a. ―What are your plans for future pregnancies?‖ b. ―Do you plan on being sexually active in the future?‖ c. ―Let‘s talk about birth control, because breastfeeding is not 100% effective for preventing pregnancy.‖ d. ―Here are some pamphlets on available methods of birth control. I‘ll come back later and discuss them with you.‖ ANS: A Discussing future pregnancy plans opens the conversation to ways of preventing pregnancy from occurring before the woman is ready to have another child. ―Do you plan on being sexually active in the future?‖ will only provide a yes or no answer and not allow for conversation. The family needs to be ready to talk about birth control; the effect of breastfeeding on birth control is applicable only to the woman. Pamphlets are not always the best form of teaching. The patient is usually too tired and overwhelmed to read more information in the immediate postpartum period. PTS: 1 DIF: Cognitive Level: Application OBJ: Nursing Process Step: Planning MSC: Patient Needs: Health Promotion and Maintenance 17. In reviewing information related to the occurrence of pregnancies using a focus group discussion with women, concern was expressed that many of them had problems using their respective type of contraception. As a result of noncompliance issues several women became pregnant. Based on this information, the nurse would incorporate which of the following in a teaching plan for group members? a. Provide information relative to product recalls of contraceptive devices. b. Have the patients keep a contraceptive diary related to the consistency of using methods because it is apparent that they forgot to use their preferred method as directed. c. Have the patients consider switching to a different form of contraception because the contraception did not prevent pregnancy for them. d. Plan for assessing the patients‘ knowledge related to the contraception methods and provide information to increase the knowledge base so that the effectiveness rate would improve. ANS: D A typical effectiveness rate refers to the occurrence of pregnancy while using a specific contraception method. If contraception is used correctly and consistently, pregnancy should not occur. A decreased effectiveness rate is associated with incorrect usage in terms of application or inconsistent use. Providing information relative to product recalls of contraceptive devices refers to an ideal effectiveness rate; the implication is that the contraception method, although used correctly, is at fault. There is no evidence to support this finding. Having the patient keep acontraceptive diary does not address the primary concerns related to the typical effectiveness rate. Having the patient switch methods may not be necessary because the primary focus is to determine the knowledge base and identify learning needs. PTS: 1 DIF: Cognitive Level: Synthesis OBJ: Nursing Process Step: Evaluation MSC: Patient Needs: Health Promotion and Maintenance 18. You are teaching a group of adolescents regarding myths and facts related to contraception. Which statement indicates that additional teaching is needed for this group? a. Adolescents are more likely to become pregnant even if they use available contraception methods correctly. b. The withdrawal technique provides a higher likelihood that a teen will not get pregnant. c. Pregnancy can occur in the presence or absence of orgasm. d. Pregnancy can occur even if a teen is menstruating at the time of coitus. ANS: B The withdrawal technique does not decrease the likelihood of becoming pregnant for any woman including a teen. Withdrawal is a highly unreliable method of birth control. Even without penetration, ejaculation may result in pregnancy. Based on current clinical evidence, adolescents are more likely to become pregnant even if they correctly use available contraception methods. Pregnancy can occur in the presence or absence of orgasm, and also if the teen is menstruating at the time of coitus. PTS: 1 DIF: Cognitive Level: Application OBJ: Nursing Process Step: Planning MSC: Patient Needs: Health Promotion and Maintenance 19. A patient presents to the Women‘s Health Clinic for continuation of her contraceptive method. She has been using Depo-Provera (medroxyprogesterone acetate) for 24 months. In preparation for instituting a plan of care, the nurse would consider which option as a priority? a. Schedule the patient for follow-up baseline diagnostic testing to confirm that the patient is not pregnant. b. Obtain information for an alternate contraception method. c. Ask the patient for additional information related to her menstrual cycle. d. Inspect the skin for site selection of contraceptive method. ANS: B According to WHO (World Health Organization) guidelines, women should not be on DepoProvera for more than 2 years due to loss of bone density. Therefore, the nurse should include assessments for other types of contraception methods for this patient. Although it may prove to be important to rule out possible pregnancy, based on the provided information, discontinuation of this method is the priority intervention. Depo-Provera can cause menstrual irregularities, but this is not the priority intervention. The nurse can follow up on this issue later. Because the current method of contraception should no longer be used, this option is not necessary. PTS: 1 DIF: Cognitive Level: Analysis OBJ: Nursing Process Step: Planning MSC: Patient Needs: Safe and Effective Care Environment20. Which of the following statements is correct regarding the use of contraception and the occurrence of sexually transmitted diseases (STDs)? a. As long as the oral contraception method is used correctly, there is no transmission of STDs during sexual activity. b. Oral contraceptives provide the greatest protection against getting STDs. c. Barrier methods, if used correctly, are more likely to protect individuals from STDs as compared with other contraceptive methods. d. It is less likely to see transmission of STDs if patients engage in oral sex as opposed to vaginal penetration. ANS: C The correct use of barrier methods helps protect against the transmission of STDs compared with other methods of contraception. The use of oral contraceptives has no effect on the transmission of STDs. The effectiveness of oral contraceptives is increased related to the prevention of pregnancy compared with other methods with the exception of abstinence. The method of sexual activity does not affect the transmission of STDs. PTS: 1 DIF: Cognitive Level: Application OBJ: Nursing Process Step: Evaluation MSC: Patient Needs: Health Promotion and Maintenance 21. Which of the following is a potential disadvantage for the patient who wishes to use an intrauterine device (IUD) as a method of birth control? a. Insertion of the device prior to coitus resulting in decreased spontaneity b. Ectopic pregnancy c. Protection against STDs d. Decrease in dysmenorrhea ANS: B The insertion of an IUD is performed in a health care provider‘s office. An ectopic pregnancy can occur as a possible complication of the IUD. An IUD does not offer protection against STDs. A decrease in dysmenorrhea would be an advantage of using an IUD. PTS: 1 DIF: Cognitive Level: Application OBJ: Nursing Process Step: Assessment MSC: Patient Needs: Health Promotion and Maintenance 22. A patient is using Depo-Provera as her method of birth control. Which clinical finding warrants immediate intervention by the nurse? a. Mid-cycle bleeding b. Nausea c. Temperature of 37.8°C (100°F) d. Irregular periods ANS: AWhen using Depo-Provera, the major side effect is irregular bleeding. The presence of midcycle bleeding warrants further investigation at this time. Nausea, fever, and irregular periods are not the result of Depo-Provera. PTS: 1 DIF: Cognitive Level: Application OBJ: Nursing Process Step: Assessment MSC: Patient Needs: Safe and Effective Care Environment 23. A patient has had a prior history of endometriosis and comes to the clinic asking about which method of birth control might be helpful to alleviate her symptoms. Which birth control method would provide the greatest benefit to this patient? a. Withdrawal method b. Oral contraceptives c. Depo-Provera d. Intrauterine device (IUD) ANS: B A patient who has a history of endometriosis may gain additional benefit from using an oral contraceptive as her birth control method because hormone levels will be more uniformly regulated with this type of treatment. The withdrawal method and Depo-Provera will not provide any additional benefit relative to a history of endometriosis. An IUD may cause further irritation to the endometrial lining so it would not be a prudent choice. PTS: 1 DIF: Cognitive Level: Application OBJ: Nursing Process Step: Planning MSC: Patient Needs: Health Promotion and Maintenance 24. You are evaluating a patient in the clinic setting who has been taking oral contraceptives for several years, without side effects. Vital signs are stable and the patient denies any pain or tenderness. On examination, you note a small erythematous area of approximately 2 cm on her right lower leg. She denies any traumatic injury and says this is a recent onset of a few days. Based on this information you would a. instruct the patient to use warm compresses for several days and keep the leg elevated as much as possible. b. have the patient wear flats rather than heels to modify her gait and help alleviate this issue. c. refer the patient to the health care provider for additional diagnostic work up. d. have the patient take an over-the-counter (OTC) nonsteroidal antiinflammatory drug (NSAID) and return to the clinic if the problem persists. ANS: C Because the patient has a history of oral contraceptive use, the nurse must assess and evaluate findings relative to ACHES (warning signs of oral contraceptives). Thus the patient should be worked up for the possibility of a superficial or deep vein thrombosis (DVT). Warm compresses and elevation of the leg, wearing flats, and taking an OTC NSAID may lead to further problems if there is an underlying clot that is not addressed promptly. PTS: 1 DIF: Cognitive Level: Application OBJ: Nursing Process Step: Assessment MSC: Patient Needs: Safe and Effective Care Environment Chapter 26: InfertilityFoundations of Maternal-Newborn & Women‘s Health Nursing, 7th Edition MULTIPLE CHOICE 1. Large amounts of leukocytes in the seminal fluid suggest a clinical finding of a. inadequate fructose. b. inflammation of the testes. c. an infection of the genital tract. d. an obstruction in the vas deferens. ANS: C The presence of large amounts of leukocytes suggests an infection. Adequate fructose must be present to supply energy for the sperm. An inflammatory process would be diagnosed by abnormal consistency or chemical composition. If an obstruction is present, the total amount of seminal fluid would be decreased. DIF: Cognitive Level: Understanding OBJ: Nursing Process Step: Assessment MSC: Patient Needs: Health Promotion and Maintenance 2. A couple who has not achieved a successful pregnancy is scheduled to meet with a fertility specialist. Which simple evaluation is usually the first test to be performed? a. Semen analysis b. Testicular biopsy c. Endometrial biopsy d. Hysterosalpingography ANS: A Semen analysis is usually the first test to be performed because it is least costly and noninvasive. Endometrial biopsy determines whether the endometrium is responding to ovarian stimulation. A testicular biopsy is an invasive examination using a local anesthetic. Hysterosalpingography uses a contrast medium to evaluate the structure and patency of the uterus and tubes. DIF: Cognitive Level: Understanding OBJ: Nursing Process Step: Assessment MSC: Patient Needs: Physiologic Integrity 3. Which situation best describes secondary infertility in a couple? a. Never conceived. b. Had repeated spontaneous abortions. c. Not conceived after 1 year of unprotected intercourse. d. Has one child but cannot conceive a second time. ANS: D Secondary infertility occurs in couples that have conceived before; but, are unable to conceive again. Primary infertility occurs when a couple has never conceived or who has not conceived after 1 year of unprotected intercourse (6 months if the women is over age 35). Repeated spontaneous abortions are considered primary infertility (pregnancy wastage). DIF: Cognitive Level: Understanding OBJ: Nursing Process Step: AssessmentMSC: Patient Needs: Health Promotion and Maintenance 4. A woman undergoing evaluation of infertility states, ―At least when we‘re through with all of these tests, we will know what is wrong.‖ The nurse‘s best response is a. ―I know the test will identify what is wrong.‖ b. ―I‘m sure that once you finish these tests, your problem will be resolved.‖ c. ―Even with diagnostic testing, infertility remains unexplained in about 20% of couples.‖ d. ―Once you‘ve identified your problem, you may want to look at the option of adoption.‖ ANS: C Problems with infertility must be approached realistically. Nurses should not make judgments or provide false reassurance. Providing accurate information to the couple is the optimal response. The nurse should not make statements indicating that problems will be resolved, because this gives a false impression. The tests are not always definitive; therefore, the nurse should not offer her view or opinion; rather, should state the facts. DIF: Cognitive Level: Application OBJ: Nursing Process Step: Planning MSC: Patient Needs: Physiologic Integrity 5. A newly married woman states, ―My friend told me I would never have a baby because I had pelvic inflammatory disease when I was younger. I don‘t understand how that can affect whether or not I get pregnant.‖ The nurse‘s most appropriate response is a. ―Your friend may be right. The disease may affect your ability to conceive.‖ b. ―Pelvic inflammatory disease may damage the ovaries and prevent ovulation.‖ c. ―Your friend has been misinformed. Fallopian tube damage occurs only following gonorrhea.‖ d. ―Infection may cause scarring and obstruction of the fallopian tubes, which can prevent the fertilized egg from reaching the uterus.‖ ANS: D Providing the patient with accurate complete information is the best response. If untreated, pelvic inflammatory disease produces scarring and obstruction of the fallopian tube. Tubal scarring and obstruction do not occur following gonorrhea. DIF: Cognitive Level: Application OBJ: Nursing Process Step: Planning MSC: Patient Needs: Health Promotion and Maintenance 6. The procedure in which ova are removed by laparoscopy, mixed with sperm, and the embryo(s) returned to the woman‘s uterus is a. in vitro fertilization (IVF). b. tubal embryo transfer (TET). c. therapeutic insemination (IUI). d. gamete intrafallopian transfer (GIFT). ANS: A In vitro fertilization is a procedure used to bypass blocked or absent fallopian tubes. Tubal embryo transfer places the conceptus into the fallopian tube. Therapeutic insemination, also known as intrauterine insemination, uses the partner‘s sperm or that of a donor and places itdirectly into the uterus. Gamete intrafallopian transfer is when the sperm and ova are placed in the fallopian tube. DIF: Cognitive Level: Understanding OBJ: Nursing Process Step: Assessment MSC: Patient Needs: Physiologic Integrity 7. Chromosome analysis is a diagnostic test that should be offered to which couple? a. Never conceived b. Has long-standing infertility c. Has had repeated pregnancy losses d. Has a normal child but has not conceived again ANS: C Repeated failures to carry a pregnancy to term may indicate genetic defects in the fetus that are incompatible with life. A couple that has never conceived would not be offered chromosome analysis. Long-standing infertility is not an indicator for chromosome analysis. Secondary infertility with an existing normal child would not be an indicator for chromosome analysis. DIF: Cognitive Level: Understanding OBJ: Nursing Process Step: Assessment MSC: Patient Needs: Physiologic Integrity 8. A woman who is undergoing infertility testing states, ―My husband won‘t discuss this with me. I don‘t think he cares about or wants a baby.‖ The nurse‘s ideal response is a. ―You should confront him about this.‖ b. ―He probably doesn‘t understand your concern.‖ c. ―Men are sometimes less eager to have children.‖ d. ―It may be harder for him to express his feelings.‖ ANS: D Men often internalize their feelings, which may appear to women as lack of concern or interest. Suggesting that the woman confront her husband suggests that the woman is at fault and not communicating with her husband. ―He probably doesn‘t understand your concern‖ does not explain to the woman why her husband won‘t discuss the problem; it passes judgment on the husband. ―Men are sometimes less eager to have children‖ does not allow the woman to express her feelings; it offers the nurse‘s opinion, which is not appropriate. DIF: Cognitive Level: Application OBJ: Nursing Process Step: Planning MSC: Patient Needs: Psychosocial Integrity 9. Which of the following medical conditions could possible affect a woman‘s fertility status? a. Past medical history of asthma during childhood that is presently under control with the use of an inhaler b. Recently diagnosed with polycystic ovarian syndrome c. Past surgical history of removal of external polyps on labial tissue d. History of frequent sinus headaches that is seasonal in nature treated with over-the-counter medication ANS: BWomen with polycystic ovarian syndrome (PCOS) often have challenges conceiving in addition to other problems due to abnormal hormones and ovarian function. The use of inhaler therapy for the treatment of asthma should not affect the patient‘s fertility status. Removal of external polyps on the labia should not affect the patient‘s fertility. A history of sinus headaches should not affect the patient‘s fertility. DIF: Cognitive Level: Analysis OBJ: Nursing Process Step: Evaluation MSC: Patient Needs: Health Promotion and Maintenance 10. A patient has been diagnosed with an incompetent cervix (the cervix will not remain closed). What treatment option will be incorporated into the plan of care for this patient? a. Bed rest throughout the pregnancy b. Wait and see approach to determine if the patient goes into preterm labor c. Preparation for cerclage procedure at 32 weeks‘ gestation d. More frequent ultrasounds to assess progression of pregnancy ANS: D An incompetent cervix would place the patient in a high-risk category, and more frequent ultrasound monitoring would be included in her care plan. Although bed rest may be ordered, there is conflicting evidence regarding the merits of this intervention. An incompetent cervix is considered a clinical abnormality, therefore the standard of care requires appropriate surgical intervention. A cerclage procedure is typically done much earlier in the pregnancy period. DIF: Cognitive Level: Analysis OBJ: Nursing Process Step: Planning MSC: Patient Needs: Physiologic Integrity/Physiologic Adaptation MULTIPLE RESPONSE 1. Which factors would contribute to abnormalities of the fallopian tube associated with the development of infertility? (Select all that apply.) a. History of conization of the cervix b. History of pelvic surgical procedures c. Incompetent cervix d. Past treatments of STD with follow-up test of cure e. Endometriosis ANS: B, D, E Surgical procedures related to the cervix, along with an incompetent cervix, would not affect the fallopian tubes in terms of infertility. It would affect fertility issues related to the cervix as a result of potential scarring (conization) and an inability to maintain the pregnancy in the presence of an incompetent cervix. A history of pelvic surgical procedures could result in the development of pelvic adhesions, which would affect the fallopian tube. Also, the presence of STDs, even with effective treatment, along with the clinical diagnosis, would affect the fallopian tube and possibly result in infertility. DIF: Cognitive Level: Analysis OBJ: Nursing Process Step: Assessment MSC: Patient Needs: Physiologic Integrity/Reduction of Risk Potential 2. Which adverse reactions are associated with the administration of clomiphene citrate (Clomid)? (Select all that apply.) a.Abdominal bloating b. Diarrhea c. Oliguria d. Nausea and vomiting e. Abnormal uterine bleeding ANS: A, D, E Some adverse reactions associated with Clomid are abdominal distension, frequent urination, nausea and vomiting, and abnormal uterine bleeding. Diarrhea is not a common presentation. DIF: Cognitive Level: Application OBJ: Nursing Process Step: Assessment MSC: Patient Needs: Physiologic Integrity/Pharmacologic and Parenteral Therapies Chapter 27: Women‘s Health Foundations of Maternal-Newborn & Women‘s Health Nursing, 7th Edition MULTIPLE CHOICE 1. Which piece of the usual equipment setup for a pelvic examination is omitted with a Pap test? a. Lubricant b. Speculum c. Fixative agent d. Gloves and eye protectors ANS: A Lubricants other than water or water-based lubricants on the cervix interfere with the accuracy of the cytology report. A speculum is necessary to visualize the cervix. A fixative agent is applied to the slide to prevent drying or disruption of the specimen. The examiner should always use standard precautions including gloves and eye protection. DIF: Cognitive Level: Application OBJ: Nursing Process Step: Implementation MSC: Patient Needs: Health Promotion and Maintenance 2. A 45-year-old patient asks how often she should have a mammogram. The most appropriate answer is a. whenever she feels a lump. b. every year beginning at age 40. c. they are unnecessary until age 50. d. every year if you have risk factors. ANS: B The American Cancer Society recommends that women have an annual mammogram after 40 years of age. Mammography should be done routinely following the American Cancer Society guidelines. Mammograms are necessary when a woman is in her 40s. Women with high-risk factors may require them more often. DIF: Cognitive Level: Understanding OBJ: Nursing Process Step: Assessment MSC: Patient Needs: Health Promotion and Maintenance3. While performing a self-breast exam, the patient notes an area on the right breast that is nodular, with some associated tenderness. This is a new onset finding because the exams were not problematic in the past. The left breast examination is unremarkable. The patient calls to report her findings to the clinical nurse because this is not her typical result. What action should the nurse perform next? a. Refer the patient to an oncologist because the results are suspicious. b. Ask the patient to come in for an office visit so that the findings can be validated but tell her that this information is within the normal range of presentation. c. Have the patient wear a tight-fitting bra and tell her that the tenderness is associated with ovulation and will pass. d. Have the patient repeat the self-breast exam in 2 weeks and call back with findings to provide a basis for comparison. ANS: B Although these findings are within the normal range of presentation for breast tissue, they are not in the normal presentation for this patient. The patient has called to express concern; therefore, the nurse should have the patient schedule an appointment for assessment and evaluation. There is no need for referral to a specialist at this time. Wearing a tight-fitting bra may help provide support; however, does not address the physical findings and concern of the patient. Repeating the self-breast exam may be required; however, it does not address the patient‘s current concerns. DIF: Cognitive Level: Application OBJ: Nursing Process Step: Implementation MSC: Patient Needs: Health Promotion and Maintenance 4. Findings of a Pap smear exam denote atypical cells of undetermined significance (ASCUS). The Pap test is repeated at 6 months and the same finding of ASCUS is reported. Which therapeutic treatment option would the nurse expect the practitioner to order? a. Mammography b. Bone scan c. Transvaginal ultrasound d. Biopsy ANS: D Based on the standard of care, a colposcopy or biopsy of the cervix is indicated. A Pap smear is performed to evaluate the cervix. There is no indication that mammography, which is used to assess and evaluate breast tissue, is required. There is no evidence to warrant a bone scan. Although a transvaginal ultrasound might be included in the treatment plan, the Pap smear indicates cervical pathology so a colposcopy or biopsy is indicated. DIF: Cognitive Level: Analysis OBJ: Nursing Process Step: Planning MSC: Patient Needs: Health Promotion and Maintenance 5. Which concern is included in the plan of care for the patient who receives HPV (human papillomavirus) vaccine? a. It is available in oral form. b. It involves a series of two injections.c. Injections should be given over a 3-month period. d. The vaccine (Gardasil) should not be given to any patient with a sensitivity to yeast. ANS: D The vaccine should not be administered to any woman who has a sensitivity to any component of the yeast family. It is available only in injection form. It is given as a series of three injections. The series of three injections should be given over a 6-month period according to Centers for Disease Control and Prevention (CDC) recommendations. Side effects of the vaccine include: headache, fever, nausea, and dizziness. DIF: Cognitive Level: Analysis OBJ: Nursing Process Step: Planning MSC: Patient Needs: Physiologic Integrity/Pharmacologic and Parenteral Therapies 6. Which are the most common sites of breast cancer metastasis? a. Kidneys b. Bones and liver c. Heart and blood vessels d. Skin ANS: B Metastasis occurs when the cancer cells spread by both blood and lymph system to distant organs and to vascular sites, commonly the lungs, liver, bones, and brain. Kidney metastasis is uncommon. Metastasis to the heart and blood vessels is uncommon. Skin cancer is not associated with breast cancer metastasis. DIF: Cognitive Level: Understanding OBJ: Nursing Process Step: Assessment MSC: Patient Needs: Physiologic Integrity 7. Which sexually transmitted disease can be treated and eradicated? a. Herpes b. AIDS c. Chlamydia d. Venereal warts ANS: C Treatment options chlamydial bacterial infection include: azithromycin, doxycycline, ofloxacin, levofloxacin, or erythromycin. Concurrent treatment of all sexual partners is necessary to prevent recurrence. Because no cure is known for herpes, treatment focuses on pain relief and preventing secondary infections. Because no cure is known for AIDS, prevention and early detection are the main focus. Condylomata acuminata is caused by the human papillomavirus (HPV). No treatment eradicates the virus; however, there is a vaccine available. DIF: Cognitive Level: Understanding OBJ: Nursing Process Step: Assessment MSC: Patient Needs: Physiologic Integrity 8. A benign breast condition that includes dilation and inflammation of the collecting ducts is known as a. fibroadenoma. b.ductal ectasia. c. intraductal papilloma. d. chronic cystic disease. ANS: B Generally occurring in women approaching menopause, ductal ectasia results in a firm irregular mass in the breast, enlarged axillary nodes, and nipple discharge. Fibroadenoma is evidenced by fibrous and glandular tissues. They are felt as firm, rubbery, and freely mobile nodules. Intraductal papillomas develop in the epithelium of the ducts of the breasts; as the mass grows, it causes trauma or erosion within the ducts. Chronic cystic disease causes pain and tenderness. The cysts that form are multiple, smooth, and well delineated. DIF: Cognitive Level: Understanding OBJ: Nursing Process Step: Assessment MSC: Patient Needs: Physiologic Integrity 9. Which statement regarding primary dysmenorrhea is most accurate? a. Primary dysmenorrhea is experienced by all women. b. It is unaffected by oral contraceptives. c. It occurs in young multiparous women. d. It may be caused by excessive endometrial prostaglandin. ANS: D Primary dysmenorrhea is menstrual pain without identified pathology. Some women produce excessive endometrial prostaglandin during the luteal phase of the menstrual cycle. Prostaglandin diffuses into endometrial tissue and causes uterine cramping. Contrary to popular belief, primary dysmenorrhea is not experienced by all women. Oral contraceptives can be a treatment choice if cramps associated with primary dysmenorrhea are debilitating. It most often occurs in young nulliparous women. DIF: Cognitive Level: Understanding OBJ: Nursing Process Step: Assessment MSC: Patient Needs: Physiologic Integrity 10. Which information should the nurse stress in teaching a patient how best to relieve the symptoms of premenstrual syndrome (PMS)? a. Decrease her consumption of caffeine. b. Drink a small glass of wine with her evening meal. c. Decrease her fluid intake to prevent fluid retention. d. Eat three large meals a day to maintain glucose levels. ANS: A Caffeine increases irritability, insomnia, anxiety, and nervousness. Alcohol aggravates depression and should be avoided in the patient with PMS. The patient should drink at least 2000 mL of water per day. Six smaller meals a day will help maintain glucose levels and reduce symptoms. DIF: Cognitive Level: Application OBJ: Nursing Process Step: Implementation MSC: Patient Needs: Health Promotion and Maintenance11. A patient, age 49, confides in the nurse that she has started experiencing pain with intercourse. The patient asks, ―Is there anything I can do about this?‖ The nurse‘s best response is a. ―No, it is part of the aging process.‖ b. ―Water-soluble vaginal lubricants may provide relief.‖ c. ―You need to be evaluated for a sexually transmitted disease.‖ d. ―You may have vaginal scar tissue that is producing the discomfort.‖ ANS: B Loss of lubrication, with resulting discomfort in intercourse, is a symptom of estrogen deficiency. It is part of the aging process; however, the use of lubrication will help relieve the symptoms. This is a normal occurrence with the aging process and does not indicate an STD. It is caused by loss of lubrication with the decrease in estrogen. Scar tissue problems would have occurred earlier. DIF: Cognitive Level: Application OBJ: Nursing Process Step: Implementation MSC: Patient Needs: Health Promotion and Maintenance 12. Which patient is most likely to develop osteoporosis? a. A 50-year-old patient on estrogen therapy b. A 55-year-old patient with a sedentary lifestyle c. A 65-year-old patient who walks 2 miles each day d. A 60-year-old patient who takes supplemental calcium ANS: B Risk factors for the development of osteoporosis include smoking, alcohol consumption, sedentary lifestyle, family history of the disease, and a high-fat diet. A number of drug therapies are available to reduce the progression of osteoporosis. Weight-bearing exercises have been shown to increase bone density. Supplemental calcium will help prevent bone loss, especially when combined with vitamin D. DIF: Cognitive Level: Understanding OBJ: Nursing Process Step: Assessment MSC: Patient Needs: Physiologic Integrity 13. A patient with a history of a cystocele should contact the physician if she experiences a. backache. b. constipation. c. urinary frequency and burning. d. involuntary loss of urine when she coughs. ANS: C Urinary frequency and burning are symptoms of cystitis, a common problem associated with cystocele. Back pain is a symptom of uterine prolapse. Constipation may be a problem for the patient with a rectocele. Involuntary loss of urine during coughing is referred to as stress incontinence and is not an emergency. DIF: Cognitive Level: Application OBJ: Nursing Process Step: ImplementationMSC: Patient Needs: Physiologic Integrity 14. Which specific instruction should the nurse teach to assist a patient to regain control of her urinary sphincter? a. Perform Kegel exercises. b. Void every hour while awake. c. Drink 8 to 10 glasses of water each day. d. Allow the bladder to become distended before voiding. ANS: A Kegel exercises, tightening and relaxing the pubococcygeal muscle, will improve control of the urinary sphincter. A prescribed schedule may help; however, every hour is too frequent. Restricting fluids will cause bladder irritation, which exacerbates the problem. Drinking adequate fluids will decrease the concentration of urine; however, this intervention will not improve sphincter control. Overdistention of the bladder will result in further stress incontinence. DIF: Cognitive Level: Application OBJ: Nursing Process Step: Implementation MSC: Patient Needs: Health Promotion and Maintenance 15. The physician diagnoses a 3-cm cyst in the ovary of a 28-year-old patient. You expect the initial treatment to include a. initiating hormone therapy. b. scheduling a laparoscopy to remove the cyst. c. examining the patient after her next menstrual period. d. aspirating the cyst and sending the fluid to pathology. ANS: C Most ovarian cysts regress spontaneously. Cysts in women of childbearing age may decrease within one cycle, so treatment is not necessary at this point. It is too early to anticipate removal of the cysts. Most ovarian cysts regress spontaneously within one cycle. A transvaginal ultrasound examination will help determine if the cyst is fluid-filled or solid. The cyst can then be removed if warranted. DIF: Cognitive Level: Understanding OBJ: Nursing Process Step: Planning MSC: Patient Needs: Physiologic Integrity 16. The drug of choice to treat a gonorrhea infection is a. penicillin G (Pfizerpen). b. tetracycline (Achromycin). c. ceftriaxone (Rocephin). d. acyclovir (Zovirax). ANS: C Additional drugs include cefixime (Suprate) or ciprofloxacin (Cipro). Ceftriaxone is effective for treatment of all gonococcal infections. Penicillin G is used most commonly to treat syphilis. Tetracycline is prescribed to treat chlamydial infections. Acyclovir is most commonly used to treat herpes genitalis.DIF: Cognitive Level: Understanding OBJ: Nursing Process Step: Planning MSC: Patient Needs: Physiologic Integrity 17. Which option could be used for the treatment and management of a patient who reports mild pain associated with a clinical diagnosis of fibrocystic breast disease? a. Chamomile tea as a relaxant therapy b. Danazol (Danocrine) c. Tamoxifen (Nolvadex) d. Over-the-counter nonsteroidal antiinflammatory drug (NSAID) therapy ANS: D Because the patient is reporting mild pain, NSAIDs may provide adequate pain relief and comfort. It is recommended that tea, coffee, and/or other stimulants be limited or restricted for patients with fibrocystic breast disease. Danazol is typically used for moderate to severe pain for patients with fibrocystic breast disease because its use is associated with more serious side effects. Danazol should not be used longer than 4 to 6 months. The patient reports mild pain so this would not be warranted. Tamoxifen is a selective estrogen receptor modulator (SERM) used for the treatment of breast cancers and also for moderate to severe pain in fibrocystic breast disease. The patient reports mild pain, so this would not be warranted. DIF: Cognitive Level: Application OBJ: Nursing Process Step: Evaluation MSC: Patient Needs: Physiologic Integrity/Pharmacologic and Parenteral Therapies 18. Which treatment option minimizes the development of lymphedema in the surgical management of a patient with breast cancer? a. Radical mastectomy procedure b. Radiation therapy c. Sentinel lymph node mapping d. Ultrasound ANS: C The use of sentinel lymph node mapping identifies only those affected lymph node tissues that require surgical removal and helps minimize the development of lymphedema in the surgical management of a patient with breast cancer. Radical mastectomy is associated with lymphedema in the postsurgical breast cancer patient because of the removal of lymph node tissue. Radiation therapy is not associated with a decrease in lymphedema for the breast cancer patient. Ultrasound as an intervention does not affect the development of lymphedema. DIF: Cognitive Level: Analysis OBJ: Nursing Process Step: Planning MSC: Patient Needs: Physiologic Integrity/Physiologic Adaptation 19. You are taking care of a patient who has had a colporrhaphy. Which option would indicate a priority assessment during the postoperative period? a. Documentation of a pessary in the operative procedure notes by the physician b. Removal of vaginal packing as ordered by the physician c. Use of a cell saver for transfusion therapy in the postoperative period d.Order for removal of staples 2 to 3 days post-procedure ANS: B Vaginal packing is typically used in this type of pelvic surgery. The removal of the packing should be verified and documented. This is the priority assessment. A pessary would be utilized as a nonsurgical intervention for a patient who has had uterine prolapse and was not a surgical candidate based on medical history. A cell saver is used in orthopedic surgeries that are at risk for blood loss so that the patient‘s own blood can be re-infused based on established protocol. There are no staples used in this type of surgical procedure, which is also known as an A & P (anterior and posterior) repair. DIF: Cognitive Level: Analysis OBJ: Nursing Process Step: Implementation MSC: Patient Needs: Safe and Effective Care Environment/Establishing Priorities 20. In reviewing genetic testing for a female patient, you note the presence of BRCA1, BRCA2, and CHEK2. How should these findings be interpreted? a. There is no increased likelihood that the patient will develop breast or ovarian cancer. b. There is an increased likelihood only for the development of breast cancer in a woman. c. More information is needed to interpret these findings based on the patient‘s family history and the patient‘s current and past medical history. d. A radical bilateral mastectomy is required as soon as possible, since the cancer may have already undergone sub-metastasis. ANS: C The presence of genetic markers (BRCA1, BRCA2, and CHEK2) provides strong indicators of the increased risk for the development of breast cancer in males and females as well as ovarian cancer. It is important to obtain additional information in order that a treatment plan can be developed and implemented to improve patient outcomes. There is an increased likelihood that the patient will develop breast or ovarian cancer. Stating that there is an increased likelihood only for the development of breast cancer in a woman fails to include that men are also at risk of developing breast cancer. At this point, surgical intervention is speculative because the presence of biomarkers does not indicate that sub-metastasis has occurred or that the cancer has even developed. DIF: Cognitive Level: Analysis OBJ: Nursing Process Step: Evaluation MSC: Patient Needs: Physiologic Integrity/Reduction of Risk Potential MULTIPLE RESPONSE 1. While interviewing a 48-year-old patient during her annual physical examination, the nurse learns that she has never had a mammogram. The American Cancer Society recommends annual mammography screening starting at age 40. Before the nurse encourages this patient to begin annual screening, it is important for her to understand the reasons why women avoid testing. These reasons include which of the following? (Select all that apply.) a. Fear of x-ray exposure b. Expense of the procedure c. Reluctance to hear bad news d. Having heard that the test is painful e. Belief that lack of family history makes this test unnecessaryANS: A, B, C, D Fear of x-ray exposure, expense, reluctance to hear bad news, and fear of pain are reasons women avoid having a mammogram done. Although the test is expensive, it is usually covered by health insurance. Many communities offer low-cost or free screening to women without insurance. It is important to acknowledge that some discomfort occurs with screening. Scheduling the test immediately at the end of a period makes it less painful. The risk of radiation exposure is minimal to none. Nurses play a vital role in providing information and reassurance to help women overcome these fears. Even patients with no family history should have a regular screening done. The nurse should emphasize that a combination of breast self-examination and mammography needs to be performed at regular intervals. Women with a family history may need to begin screening at a younger age and have additional testing such as ultrasound performed. DIF: Cognitive Level: Analysis OBJ: Nursing Process Step: Evaluation MSC: Patient Needs: Health Promotion and Maintenance 2. Healthy People 2020 goals directed at women‘s health issues focus on which areas? (Select all that apply.) a. Increased screening for cervical and colorectal cancers b. Reduction of cancer survivor rate based on clinical management treatment c. Decreased morbidity and mortality related to breast cancer d. Reduction in hospitalization for hip fractures in the older female population e. Reduction in deaths associated with cardiovascular causes such as stroke and coronary artery disease (CAD) ANS: A, C, D, E Healthy People 2020 goals directed at women‘s health focus on increased access to screening for cervical and colorectal cancers, decreased deaths occurring from breast cancer and heart disease, and decreased hospitalization for hip fractures. A reduction of the cancer survival rate would reflect increased morbidity and mortality. DIF: Cognitive Level: Application OBJ: Nursing Process Step: Planning MSC: Patient Needs: Health Promotion and Maintenance 3. A 38-year-old patient presents to the clinic office complaining of increased bilateral tenderness of her breasts prior to the onset of menses. On questioning the patient, this presentation has occurred off and on for several years; however, the pain has increased. Physical examination reveals lumpy areas bilaterally on the upper outer quadrants of each breast tissue. The areas of concern are approximately 2 cm in size. Based on this assessment, which diagnostic testing would be necessary? (Select all that apply.) a. Ultrasound examination b. Open biopsy c. Fine-needle aspiration (FNA) biopsy d. CBC with differential e. Mammogram ANS: A, C, EBased on the clinical presentation, the patient may have fibrocystic breast disease. Although this condition is typically benign, the fact that the patient has noted a change in tenderness should be evaluated. Ultrasound, FNA, and mammography may be indicated to provide a baseline for comparison and rule out any malignancy. An open or surgical biopsy is not indicated at the present time but may be needed if the other test results indicate any pathology. Blood work is not indicated at this time relative to the diagnosis. DIF: Cognitive Level: Analysis OBJ: Nursing Process Step: Planning MSC: Patient Needs: Physiologic Integrity/Reduction of Risk Potential [Show More]

Last updated: 1 year ago

Preview 1 out of 292 pages

Add to cart

Instant download

We Accept:

We Accept
document-preview

Buy this document to get the full access instantly

Instant Download Access after purchase

Add to cart

Instant download

We Accept:

We Accept

Reviews( 0 )

$12.00

Add to cart

We Accept:

We Accept

Instant download

Can't find what you want? Try our AI powered Search

OR

REQUEST DOCUMENT
53
0

Document information


Connected school, study & course


About the document


Uploaded On

May 09, 2022

Number of pages

292

Written in

Seller


seller-icon
Bobweiss

Member since 4 years

39 Documents Sold


Additional information

This document has been written for:

Uploaded

May 09, 2022

Downloads

 0

Views

 53

Document Keyword Tags

Recommended For You

Get more on TEST BANK »

$12.00
What is Browsegrades

In Browsegrades, a student can earn by offering help to other student. Students can help other students with materials by upploading their notes and earn money.

We are here to help

We're available through e-mail, Twitter, Facebook, and live chat.
 FAQ
 Questions? Leave a message!

Follow us on
 Twitter

Copyright © Browsegrades · High quality services·